You are on page 1of 130

Chapter 1: Onco-Nephrology: Growth of the

KidneyCancer Connection
Mark A. Perazella, MD,* and Mitchell H. Rosner, MD
*Section of Nephrology, Department of Medicine, Yale University School of Medicine, New Haven, Connecticut; and Division
of Nephrology, Department of Medicine, University of Virginia Health System, Charlottesville, Virginia

The question has been asked of many of us in- Emerging kidney toxicities associated with drugs tar-
terested in the kidneycancer connection; Why geting VEGF and TKIs and other signaling pathways,
onco-nephrology? Nephrologists have tradition- tumor lysis syndrome, cytotoxic chemotherapy-induced
ally treated cancer patients with various forms of kidney toxicities, kidney problems in myeloma, tumor or
kidney disease. However, although typical AKI and treatment-related microangiopathies and glomerulo-
electrolyte/acidbase disturbances can be handled nephritis, stem cell transplantassociated acute and
by the practicing clinical nephrologist, increasingly chronic kidney injuries, obstructive uropathies, severe
it has become clear that many of the issues are more uid and electrolytes abnormalities, and dosing and
complex and highly specialized. For example, timing of chemotherapy in CKD and ESRD patients:
many nephrologists were not trained in the era of these and other complex problems, and their increasing
bone marrow/hematopoietic stem cell transplant, frequency and severity, provide a unique and un-
which has a number of unusual and complicated precedented opportunity for nephrologists to improve
forms of kidney injury. In addition, the number of treatment for cancer patients worldwide.
anticancer drugs with various types of nephrotox-
icity has increased dramatically, and their entry
into clinical practice continues at a fast pace. These Onco-nephrologists help cancer care teams prevent
and other issues have led to a burgeoning interest kidney problems or resolve them as they arise and im-
in a more specic focus of nephrologists on the prove patient outcomes. Research in cancer nephrology is
patient with cancer. already improving kidney care in cancer patients. A
Recognizing this changing landscape, nephrol- more focused approach to cancer nephrology may also
ogists at various large cancer centers began a help address challenges like renal cell carcinoma in
discussion about the need to address this rapidly ESRD.
growing area of nephrology. In 2009/2010, Abdulla
Salahudeen recruited leaders from these cancer
centers and asked the American Society of Ne- The American Society of Nephrology believes onco-
phrology (ASN) to sponsor a forum where this nephrology represents an emerging frontier in the ght
emerging area of nephrology could be discussed. In against kidney disease.
2012, the ASN had its rst ofcial meeting of what
The ASN Onco-Nephrology Forum has worked
was later named the Onco-Nephrology Forum.
hard to spread the word and metastasize into
Dr. Salahudeen chaired the group, pushed forward
multiple journal, meetings, and conferences. One
the kidneycancer agenda with the forum mem-
very helpful contribution to this endeavor included
bers, and put the edgling area of onco-nephrology
the appearance of publications on the onco-
on the nephrology map. The description of the
nephrology topic in several high-level journals visible
Onco-Nephrology Forum on the ASN website is
to many nephrologists. Entire issues dedicated to
noted below:
Why onco-nephrology? While all nephrologists address
nephrology problems in cancer patients, many of these Correspondence: Mark A. Perazella, Section of Nephrology,
problems are increasingly complex. To provide the best Department of Medicine, Yale University School of Medicine, BB
114, 330 Cedar Street, New Haven, Connecticut.
nephrology care for cancer patients, we must understand
rapidly changing protocols and therapies. Copyright 2016 by the American Society of Nephrology

American Society of Nephrology Onco-Nephrology Curriculum 1


Table 1. Onco-Nephrology Curriculum committee members
Mark A. Perazella (ONF Chair, Lead Editor)
Mitchell H. Rosner (Lead Editor)
Kevin W. Finkel (Section Editor)
Ilya Glezerman (Section Editor)
Susie L. Hu (Section Editor)
Kenar D. Jhaveri (Section Editor)
Amit Lahoti (Section Editor)
Anushree C. Shirali (Section Editor)
Ala Abudayyeh
Joseph R. Angelo
Joseph V. Bonventre
Anthony Chang
Eric P. Cohen
Farhad R. Danesh
Figure 1. Timeline of the birth and growth of onco-nephrology. Mona D. Doshi
Formation of the ASN Onco-Nephrology Forum, numerous con- Amaka Edeani
ference publications, and dedicated journal publications charac- Carlos Flombaum
terize and highlight the process. ASN, American Society of Sangeeta R. Hingorani
Nephrology; ONF, Onco-Nephrology Forum; NKF, National Benjamin Humphreys
Kidney Foundation; JCO, Journal of Clinical Oncology; ON, Onco- Divya Monga
Nephrology; ACKD, Advances in Chronic Kidney Disease; CJASN, Abdulla K. Salahudeen
Clinical Journal of the American Society of Nephrology; KI, Kidney
International; JASN, Journal of the American Society of Nephrology;
Sem Nephrol, Seminar in Nephrology. Nephrology Forum Chair continued to forge ahead and felt the
time was ripe for the creation of an Onco-Nephrology
Curriculum. After creation of an outline of topics and
onco-nephrology appeared in Seminars in Nephrology and Ad- discussion by the advisory group (Table 1), the core curricu-
vances in Chronic Kidney Disease, while a series of articles on this lum was submitted to the ASN Education Committee for re-
subject was published in the Clinical Journal of the American view. The curriculum was subsequently approved and the
Society of Nephrology Moving Points in Nephrology feature. Onco-Nephrology Forum group, under the direction of the
In 2011, the ASN had its rst Kidney Week Early Program ded- curriculum committee co-chairs (Perazella and Rosner), put
icated to onco-nephrology. This Early Program continues on an together the ultimate plan for creation of the curriculum doc-
every other year schedule. Many of the Kidney Week Clinical ument. The lead editors, section editors, and chapter authors
Nephrology Conferences included sessions covering various (Tables 1 and 2) were identied and the writing began. The
onco-nephrology topics. The National Kidney Foundation chapters are truly an example of outstanding contributions by
Annual Spring Meeting similarly dedicated a session to onco- experts in the subeld of onco-nephrology. All of the authors
nephrology (Figure 1). In addition, editorials describing the im- are to be congratulated on their ne work and keeping to the
portance of onco-nephrology, some suggesting the need for a originally planned timeline for completion. The product of
new subspecialty appeared in the Journal of the American So- this work will appear on the ASNs website and will be avail-
ciety of Nephrology, Kidney International, and the Journal of Clin- able to the ASN membership, the nephrology training pro-
ical Oncology, authored by members of the ASN Onco-Nephrology grams, and all other interested health care providers. We are
Forum. condent that this curriculum will strengthen the teaching of
With these important accomplishments, the Onco- onco-nephrology and further expand all practitioners knowl-
Nephrology Forum with Mark Perazella as the new Onco- edge of the subject. We hope the readers enjoy the document.

2 American Society of Nephrology Onco-Nephrology Curriculum


Table 2. Onco-Nephrology Curriculum chapters and authors
1) Onco-Nephrology: Growth of the Kidney-Cancer Connection Mark Perazella, Mitchell Rosner
2) Why do we need an Onco-Nephrology Curriculum? Mark Perazella, Mitchell Rosner
3) AKI associated with Malignancies Amit Lahoti, Benjamin Humphreys
4) Tumor Lysis Syndrome Amaka Edeani, Anushree Shirali
5) Electrolyte and Acid-Base Disorders and Cancer Anushree Shirali
6) Glomerular Disease and Cancer Divya Monga, Kenar Jhaveri
7) Hematologic Diseases and Kidney Disease Ala Abudayyeh, Kevin Finkel
8) Clinical tests for Monoclonal Proteins Nelson Leung
9) Hematopoietic Stem Cell Transplant-related Kidney Disease Sangeeta Hingorani, Joseph Angelo
10) Radiation-associated Kidney Injury Amaka Edeani, Eric Cohen
11) Chemotherapy and Kidney injury Ilya Glezerman, Edgar Jaimes
12) Pharmacokinetics of Chemotherapeutic Agents in Kidney Disease Sheron Latcha
13) CKD as a Complication of Cancer Maurizio Gallieni, Camillo Porta, and Laura Cosmai
14) Hereditary Renal Cancer Syndromes Katherine Nathanson
15) Work-up and Management of Small Renal Masses Susie Hu, Anthony Chang
16) Cancer in Solid Organ Transplantation Mona Doshi
17) Cancer Screening in ESRD Jean Holley
18) Ethics of RRT, Initiation and Withdrawal, in Cancer Patients Michael Germain
19) Palliative Care in Patients with Kidney Disease and Cancer Alvin Moss

American Society of Nephrology Onco-Nephrology Curriculum 3


AUTHOR QUERIES

AUTHOR PLEASE ANSWER ALL QUERIES

There are no queries in this article.


Chapter 2: Why Do We Need an Onco-Nephrology
Curriculum?
Mark A. Perazella, MD,* and Mitchell H. Rosner, MD
*Section of Nephrology, Department of Medicine, Yale University School of Medicine, New Haven, Connecticut; and

Division of Nephrology, Department of Medicine, University of Virginia Health System, Charlottesville, Virginia

As health care providers, we are acutely aware of the effective chemotherapeutic agents, including bio-
National Vital Statistics Report describing the logics, and stem cell therapies. However, this has
signicant toll cancer, as the second leading cause led to an increase in the number of cancer survivors
of death, has on our patients (1). Importantly, can- that often develop acute and/or CKD due to their
cer incidence rates are highest in the elderly (2). At malignancy and/or its associated treatment. The
the same time, the US Renal Data System (USRDS) best example of the bidirectionality of cancer and
notes that AKI rates are increasing in the elderly, with kidney disease is seen between renal cancer and
rates 10-fold higher than the nonelderly population CKD (Figure 3 ).
(3). Importantly, both AKI and CKD are highly Cancer can directly injure the kidneys through
prevalent in cancer patients, in particular renal cell tumor inltration or production of nephrotoxic
cancer, liver cancer, multiple myeloma, leukemias, (paraneoplastic) substances. Any one of the growing
and lymphomas (4,5). The Belgian Renal Insuf- numbers of therapeutic agents that extend patient
ciency and Anticancer Medication (BIRMA) study lives can cause various types of acute or CKD, along
noted the frequent occurrence of kidney disease in with serious electrolyte and acidbase abnormalities.
ve major cancers (Figure 1) (6). Most concerning is In addition, patients may develop multiorgan illness
the increased mortality noted in patients with AKI/ requiring ICU-level care and RRT. Certain malig-
CKD compared with those without kidney disease. nancies are more likely to cause this severe form of
For instance, the development of AKI can be asso- multiorgan dysfunction and may be associated with
ciated with cessation of effective chemotherapeutic higher mortality rates. When this type of critical
regimens, or the presence of preexisting CKD may illness occurs in the setting of advanced malignancy,
limit the use of otherwise active regimens that may it raises questions about the appropriateness of ag-
be curative. This combination of cancer, kidney dis- gressive care in futile situations and the role of
ease, and mortality has led to the recognition that palliation. Thus, care for oncology patients has be-
nephrology and oncology are intricately linked and come more specialized and complicated, requiring
require our full attention as a subspecialty (Figure collaboration between nephrologists, oncologists,
2). Hence, onco-nephrology was born in a few intensivists, and palliative care specialists.
large centers but has steadily grown to include The remarkable advances in cancer management
many medical centers, hospitals, and clinics. present both new opportunities and complex chal-
What exactly is onco-nephrology? It is a rapidly lenges for the oncology and nephrology communi-
growing area of nephrology where kidney disease in ties. It is essential for nephrologists to be informed
cancer patients has become an important source of and actively involved in certain facets of cancer care.
consultations, with the trend occurring over the last A better understanding of the rapidly evolving eld
1015 years. Oncology patients now make up a sig- of cancer biology and its therapy is required for
nificant number of the patients that nephrologists see nephrologists to become valuable members of the
for kidney-related problems in the outpatient clinic,
on the inpatient oors, and in the medical intensive
care unit (ICU). There is an increase in the number of Correspondence: Mark A. Perazella, Section of Nephrology,
patients with kidney disease, in part related to high Department of Medicine, Yale University School of Medicine, BB
114, 330 Cedar Street, New Haven, Connecticut.
incidence rates for many malignancies, as well as im-
provement in the cancer death rates due to more Copyright 2016 by the American Society of Nephrology

American Society of Nephrology Onco-Nephrology Curriculum 1


Figure 1. Kidney injury associated with ve different cancers in the BIRMA study. The percentage of patients with kidney injury as
dened by SCR, GFR ,90, or GFR ,60 is noted both for the individual cancers and all cancers lumped together. BIRMA, Belgian Renal
Insufciency and Anticancer Medication study; SCR, serum creatinine; aMDRD, abbreviated MDRD. Adapted with permission from
reference 6.

cancer care team and to provide the best nephrology care and chronic kidney injury. Furthermore, the ever-evolving
possible. The goal of this American Society of Nephrology eld of cancer therapy demands a comprehensive team ap-
(ASN) sponsored Onco-Nephrology core curriculum is to proach with the nephrologist as one of the critically important
provide the ASN membership including veteran nephrologists, care providers. As such, it is essential for nephrologists to
newly minted nephro-clinicians, and fellowship trainees with develop expertise in the practice of onco-nephrology. We
the building blocks on which further information can be added hope this curriculum provides the initial framework to
as technology advances. This educational venue will be avail- achieve this goal.
able outside the ASN membership as well.
Nephrologists must be well prepared to care for patients
with cancer and its associated renal complications. The renal TAKE HOME POINTS
manifestations of cancer have many unique features, and these
conditions often require specialized approaches to manage c Kidney disease is a frequent and increasing complication of cancer.
uid, electrolyte, and acidbase disturbances, as well as acute c There is a bidirectional relationship between cancer and kidney disease.

Figure 2. The relationship between cancer and AKI and CKD. Cancer, AKI, and CKD are linked by various exposures and
pathways.

2 Onco-Nephrology Curriculum American Society of Nephrology


Figure 3. The bidirectionality between renal cancer and CKD. Common exposures that can cause both renal cell cancer and CKD
are noted in the middle bidirectional arrow.

c Onco-nephrology is a growing area of nephrology that requires clinicians 3. USRDS. Percent of Medicare patients aged 661 (a) with at least one AKI
to have a better understanding of the renal complications of cancer in- hospitalization, and (b) with an AKI hospitalization that had dialysis by
cluding electrolyte/acidbase disturbances, AKI, and CKD. year, 20032012 (Figure 5.1). Chapter 5: Acute kidney injury. Available
at: http://www.usrds.org/2014/view/v1_05.aspx. Accessed March 1,
c The Onco-Nephrology Curriculum is an educational tool created
2015
by ASN Onco-Nephrology Forum members and other expert ne- 4. Christiansen CF, Johansen MB, Langeberg WJ, Fryzek JP, Srensen HT.
phrologists. Incidence of acute kidney injury in cancer patients: A Danish population-
based cohort study. Eur J Intern Med 22: 399406, 2011
REFERENCES 5. Schmid M, Abd-El-Barr AE, Gandaglia G, Sood A, Olugbade K Jr,
Ruhotina N, Sammon JD, Varda B, Chang SL, Kibel AS, Chun FK, Menon
1. Hoybert DL, Xu J. Deaths: Preliminary data for 2011. Natl Vital Stat Rep M, Fisch M, Trinh QD. Predictors of 30-day acute kidney injury following
61: 2012 radical and partial nephrectomy for renal cell carcinoma. Urol Cancer 32:
2. National Cancer Institute. Age-adjusted SEER incidence rates, 2007 12851291, 2014
2011 (Table 2.7). SEER cancer statistics review (CSR) 19752011. 6. Janus N, Launay-Vacher V, Byloos E, Machiels JP, Duck L, Kerger J,
Surveillance, epidemiology, and end results program. Available at: Wynendaele W, Canon JL, Lybaert W, Nortier J, Deray G, Wildiers H.
http://seer.cancer.gov/csr/1975_2011/browse_csr.php?sectionSEL52& Cancer and renal insufciency results of the BIRMA study. Br J Cancer
pageSEL5sect_02_table.07.html. Accessed March 1, 2015 103: 18151821, 2010

American Society of Nephrology Onco-Nephrology Curriculum 3


REVIEW QUESTIONS 2. In a patient with a recent diagnosis of cancer, which of the
following complications are increased in the setting of the
1. Which of the following malignancies has the highest 1-year cancer diagnosis?
risk for AKI?
a. AKI
a. Multiple myeloma b. CKD
b. Lymphoma c. Mortality
c. Renal cell cancer d. All of the above
d. Liver cancer
e. Leukemia Answer: d is correct. Cancer is associated with an increased
incidence of AKI, CKD, and overall mortality. These compli-
Answer: c is correct. Although all of these cancers are cations are the result of the tumor itself (inltration or tumor
associated with increased AKI risk, renal cell cancer was products), drug nephrotoxicity, comorbid diseases, or all of
found to have the highest 1-year risk in a cohort study ex- the above.
amining the incidence of AKI in cancer patients (4).

4 Onco-Nephrology Curriculum American Society of Nephrology


Chapter 3: AKI Associated With Malignancies
Amit Lahoti, MD,* and Benjamin D. Humphreys, MD, PhD
*Division of Internal Medicine, Section of Nephrology, The University of Texas MD Anderson Cancer Center, Houston,
Texas; and Division of Nephrology, Washington University School of Medicine, St. Louis, Missouri

INTRODUCTION mild degrees of renal injury on mortality. Signi-


cant renal injury may occur without elevation in
Advances in treatment, risk stratication, and serum creatinine, and an elevation of 0.3 mg/dL
supportive care have improved survival of patients has been associated with increased mortality in
with cancer over the last two decades (1). AKI may hospitalized patients.
result from the cancer itself (e.g., inltration or ob- The Acute Kidney Injury Network (AKIN) pro-
struction), the treatment of cancer (e.g., chemo- posed modications to the RIFLE criteria with three
therapy toxicity), or associated complications stages of AKI corresponding to the risk, injury, and
(e.g., sepsis). Cancer, by itself, is not a contraindica- failure categories (8). Patients with an absolute rise in
tion for starting RRT, even in the setting of multi- serum creatinine of 0.3 mg/dL are included into the
organ failure (24). However, decision-making is least severe category (stage 1). The loss and ESRD
complex and requires a multidisciplinary approach categories were eliminated, and all patients requiring
between the oncologist, intensivist, and nephrologist. dialysis were classied into the most severe category
The development of AKI may lead to longer length of (stage 3). Last, a time constraint of 48 hours to reach
hospital stay, decreased functional status and quality stage 1 was also included in the AKIN denition.
of life, and exclusion from further cancer therapy. Whether the AKIN modications to the RIFLE
AKI and RRTmay lead to unpredictable levels of che- criteria have led to improvements in classication
motherapeutic agents and anti-infective drugs. AKI has yet to be determined (9). Recently, The Kidney
may also increase inammatory cytokines in the Disease Improving Global Outcomes (KDIGO)
lung, leading to increased vascular permeability (5) work group combined elements of the RIFLE and
and the need for mechanical ventilation (6). There- AKIN classications to dene AKI as 1) an increase
fore, early detection and prevention of AKI is crucial in serum creatinine (SCr) $0.3 mg/dL within 48
in patients with cancer. hours, 2) an increase in SCr to $1.5 times baseline
within the prior 7 days, or 3) a urine volume of
,0.5 mL/kg/h for 6 hours. Severity of AKI is staged
DEFINITION similar to the AKIN criteria. Several studies have
correlated AKI as dened by these criteria with in-
More than 35 different denitions for AKI have been creased mortality, length of stay, and hospital costs
used in the literature, which has made cross- in patients with cancer (1013).
comparisons between studies difcult. This led to
the development of the RIFLE classication, which
dened three stages of AKI (risk, injury, and failure) EPIDEMIOLOGY AND PROGNOSIS
and two stages of renal failure requiring dialysis (loss
and ESRD) (7). Stages for AKI are determined by AKI is common in hospitalized patients with cancer
the percent rise in serum creatinine relative to base- and is associated with increased length of stay and
line, decreased urine output, or the need for dialy- hospital costs. In a Danish population-based study of
sis. It is unclear whether the criteria are well 1.2 million cancer patients, the incidence of AKI
balanced in respect to urine output and serum dened by the RIFLE criteria was highest in patients
creatinine, as most studies have not utilized the
urine output component. The RIFLE classication Correspondence: Amit Lahoti, UT MD Anderson, Cancer Center,
Unit 1468, PO Box 301402, Houston, Texas 77230.
has been validated in numerous patient popula-
tions and has highlighted the signicant effect of Copyright 2016 by the American Society of Nephrology

American Society of Nephrology Onco-Nephrology Curriculum 1


with renal cell cancer (44%), multiple myeloma (33%), liver kidney injury molecule 1 (KIM-1), neutrophil gelatinase-
cancer (32%), and leukemia (28%) (14). Compared with pa- associated lipocalin (NGAL), N-acetyl-b-D-glucosaminidase
tients without cancer, critically ill patients with cancer have a (NAG), interleukin 18 (IL-18), and matrix metalloproteinase
higher incidence of AKI requiring RRT. Depending on the def- 9 (MMP-9). The accuracy and reliability of these markers var-
inition of AKI and the underlying case mix, it has been reported ies across individual studies. An assay for serum and urinary
that 13%42% of critically ill patients with cancer develop AKI NGAL levels has become recently available but is not routinely
and 8%60% require RRT (15). The incidence is highest used in the clinical setting at this time.
for those patients with hematologic malignancies,
multiple myeloma, and septic shock.
The 28-day mortality of patients with cancer who require RRT EPIDEMIOLOGY OF AKI IN CANCER PATIENTS
is 66%88% (16). In one study of critically ill patients with
cancer, the odds ratio for 30-day mortality was increased two- The overall incidence of AKI among cancer patients was recently
fold in patients with AKI. However, approximately one-half of dened in a large Danish study. Among 1.2 million people
the patients with AKI survived to day 30 after admission (17). In followed between 1999 and 2006, there were 37,267 incident
one study of AKI in critically ill patients, there was complete cancer patients with a baseline creatinine measurement. The
recovery of renal function in 82% and partial recovery in 12%, 1-year risk of AKI in this population (dened as a .50% rise in
and chronic dialysis was needed in only 6% of patients (18). serum creatinine) was 17.5%, with a 27% risk over 5 years (14).
Overall severity of illness, age, and functional status may have Patients with distant metastases were at the highest risk of AKI.
more of an impact on prognosis than underlying malignancy, More severe AKI, dened as a doubling of serum creatinine
and the presence of cancer may not be an absolute exclusion (injury in the RIFLE criteria) (20), had an 8.8% and 14.6%
criterion for withholding RRT. However, the prognosis of crit- risk at 1 and 5 years, respectively. Even more severe AKI, corre-
ically ill recipients of stem cell transplants who develop AKI sponding to failure in RIFLE criteria and reecting a tripling of
remains poor, with mortality exceeding 80%. A team-based serum creatinine or absolute rise .4 mg/dL, was seen in 4.5%
approach between the oncologist, critical care physician, and and 7.6% of patients at 1 and 5 years, respectively. Among cancer
nephrologist is necessary to identify patients who are most suit- patients with any stage of AKI (9,613 total), 5.1% required
able for initiation of RRT. dialysis within 1 year of AKI onset. Older patients were most
heavily represented in this analysis.

ASSESSMENT OF KIDNEY FUNCTION Cancers with highest AKI risk


Certain cancers carry a much higher risk of AKI than others. In
The ideal marker of kidney function would be a substance that is the Danish study above, kidney cancer, multiple myeloma, and
freely ltered, neither secreted nor reabsorbed, and is solely liver cancer had the highest 1-year risk of AKI at 44.0%, 33.0%,
eliminated by the kidney. Although inulin and radiolabeled EDTA and 31.8%, respectively. After diagnosis of renal cell carcinoma,
and iothalomate demonstrate many of these characteristics, their many patients still undergo radical nephrectomy, and this
complexity and cost of measurement have precluded use in daily procedure itself is associated with a 33.7% risk of AKI and
practice. Serum creatinine has been traditionally used as a marker predicts the future development of CKD at 1 year (21).
of kidney function, but when used in isolation, it is not an adequate Patients with acute lymphoma or leukemia undergoing
measure. Serum creatinine values are altered by many other factors induction chemotherapy are also at an especially high risk of
including muscle mass, diet, sex, and tubular secretion. Patients AKI. In a series of 537 patients with either acute myelogenous
with cancer may present with spuriously low serum creatinine leukemia or high-risk myelodysplastic syndrome undergoing
levels secondary to cachexia. However, estimating equations for induction, 36% developed AKI. Even among patients with mild
GFR, which factor other variables such as age, sex, and race along AKI (dened as RIFLE risk), 8-week mortality was 13.6% (95%
with serum creatinine, provide a reasonable estimate of renal condence interval, 7.8%23%) compared with patients with no
function in most patients. The most commonly used estimating AKI whose 8-week mortality was 3.8% (95% condence interval,
equations are the Cockcroft-Gault, the Modication of Diet in 2.2%6.4%). Patients requiring RRT experienced mortality of
Renal Disease (MDRD), and the Chronic Kidney Disease Epide- 61.7% (95% condence interval, 50%74%) over the same
miology Collaboration (CKD-EPI) formulas. Among patients time frame (12).
with cancer who have serum creatinine values within the normal AKI is common in hospitalized cancer patients and also
range, 20% of patients have unsuspected CKD when the GFR is correlates with increased length of stay, cost, and mortality.
estimated by Cockcroft-Gault formula (19). Candrilli and colleagues analyzed the 2004 Nationwide Inpatient
It is well understood that elevation in serum creatinine is a Sample for patients with hematologic malignancies. They
relatively late marker of renal injury, as a signicant amount of identied 350,601 patients without AKI, 27,654 patients with
kidney function may be lost before a rise in serum creatinine is mild or moderate AKI (not requiring dialysis), and 5,148 patients
apparent. Several urinary biomarkers of AKI that have greater with severe AKI (requiring dialysis). The average length of stay
sensitivity for acute renal injury have been proposed, including and costs among these groups were 7.4, 12.2, and 17.6 days, and

2 Onco-Nephrology Curriculum American Society of Nephrology


Table 1. Cancer-specic risk factors for AKI Table 2. Common causes of AKI in patients with cancer
Age .65 years Prerenal azotemia
Congestive heart failure (i.e., exposure to anthracyclines, trastuzumab) Volume depletion
CKD Nausea, vomiting, diarrhea
Hypovolemia (i.e., chemotherapy-related nausea and vomiting, acute Decreased oral intake owing to mucositis (5-uorouracil,
graft-versus-host disease) methotrexate, taxanes)
Distant metastases Polyuria caused by hyperglycemia (steroids) or diabetes
Multiple myeloma insipidus (pituitary tumor)
Liver cancer Third spacing (hypoalbuminemia, liver or peritoneal
Nephrectomy for renal cell carcinoma metastases, interleukin-2)
Induction chemotherapy for acute lymphoma or leukemia Insensible loss of uid from skin lesions (mycosis fungoides)
Hemodynamic-mediated
Sepsis
Renal arteriolar vasoconstriction (nonsteroidal anti-
$13,947, $25,638, and $44,619, respectively (22). Cancer-specic inammatory drugs [NSAIDs], calcineurin inhibitors,
risk factors for AKI are summarized in Table 1. hypercalcemia)
Congestive heart failure
Hepatorenal syndrome/hepatic sinusoidal obstruction
ETIOLOGY OF AKI syndrome
Budd-Chiari syndrome
Intrahepatic inferior vena cava compression or thrombosis
The causes of AKI in patients with cancer are numerous (Table
caused by hepatomegaly or a tumor
2). The sites along the nephron at which some of these syn-
Intravenous iodinated contrast agent
dromes act are depicted in Figure 1. The specic diagnoses Abdominal compartment syndrome
will be discussed in detail elsewhere in the core curriculum, Intrinsic renal disease
but some notable causes are highlighted in this chapter. Acute tubular necrosis
Chemotherapy (cisplatin, ifosfamide)
Sepsis Anti-infectives (amphotericin B, foscarnet, cidofovir,
Sepsis is the most common cause of AKI in patients with cancer. aminoglycosides, vancomycin)
In population-based studies, approximately 15% of critically ill Bisphosphonates
patients with sepsis have underlying cancer (23). Acute tubular Sepsis
necrosis secondary to sepsis remains the leading cause of AKI in Prolonged prerenal azotemia
Allergic interstitial nephritis (penicillins, cephalosporins,
critically ill patients with cancer. Patients with hematologic ma-
uoroquinolones, NSAIDs)
lignancies are especially prone to the development of bacterial
Crystal nephropathy (methotrexate, acyclovir, ciprooxacin,
infections and sepsis secondary to prolonged neutropenia. sulfonamides, rifampin)
Nearly half of patients admitted to the intensive care unit Osmotic nephrosis (IV immunoglobulin, mannitol, starch)
(ICU) with hematologic malignancies have underlying sepsis Thrombotic microangiopathy (post-hematopoietic stem cell
compared with 12%25% of patients with solid tumors (24). transplant, gemcitabine, prior
Studies have demonstrated improved survival of cancer pa- radiation therapy)
tients with sepsis over the last decade, except in patients that Myeloma-related kidney disease
require RRT, where hospital mortality approaches 80% Postrenal obstruction
(25,26). Bladder outlet obstruction (malignancy of cervix, prostate,
Sepsis causes AKI by systemic vasodilation, leading to decreased bladder, or uterus)
Retroperitoneal disease (metastasis, lymphadenopathy, brosis)
effective circulating volume, cytokine activation, endothelial
Hemorrhagic cystitis (cyclophosphamide, BK virus)
damage, and microthrombi formation. The use of vasoconstrict-
Ureteral strictures (prior radiation therapy, BK virus)
ing pressor agents further exacerbates an effective prerenal state.

Anti-infectives
The high incidence of sepsis in critically ill cancer patients toxicity and vasoconstriction, leading to nonoliguric AKI,
necessitates the use of nephrotoxic antibacterial and antifungal hypokalemia, hypomagnesemia, and distal renal tubular aci-
agents. Aminoglycosides may cause nephrotoxicity after 57 dosis. Newer liposomal and lipid formulations are less neph-
days of therapy, and patients present with nonoliguric AKI, rotoxic with comparable efcacy. Other novel antifungal
hypokalemia, hypomagnesemia, and hypocalcemia. The risk agents, caspofungin and voriconizole, are also less nephro-
of renal toxicity may be minimized with once daily dosing. toxic and are often used as rst-line therapy. Several studies
Several alternative drugs to aminoglycosides that do not cause have reported on the nephrotoxicity of vancomycin, although
AKI have become available in the treatment of neutropenic the biological mechanism remains undened. Reported risk
fever. Amphotericin B deoxycholate may cause tubular factors for AKI are higher trough levels (.15 mg/dL) and

American Society of Nephrology Onco-Nephrology Curriculum 3


Figure 1. Sites of injury in AKI syndromes. TMA, thrombotic microangiopathy; ATN, acute tubular necrosis.

higher daily doses (.4 g/day) (27,28). Patients present with from metabolites excreted in the urine, which helps prevent
nonoliguric AKI and bland urine sediment, and most patients hemorrhagic cystitis.
recover renal function after discontinuation of the drug. Methotrexate is an antifolate and antimetabolite commonly
used in the treatment of leukemia, lymphoma, and sarcoma.
Chemotherapy High-dose methotrexate (.1 g/m2) may cause AKI by forming
Cisplatin is a DNA alkylating agent used to treat a variety of intratubular crystals leading to obstruction and direct tubular
tumors including sarcomas, small cell lung cancer, ovarian cell toxicity. Patients generally present with nonoliguric AKI
cancer, and germ cell tumors. It is directly tubular toxic with a subsequent rapid rise in serum creatinine. Intravenous
and leads to salt wasting, hyponatremia, hypomagnesemia, and hydration and urinary alkalinization prevent the precipitation
AKI. A low chloride environment enhances toxicity, and of methotrexate crystals. In the setting of AKI, methotrexate
concurrent saline administration to achieve urine output may accumulate and cause neutropenia, hepatitis, mucositis,
.3 L/day is the mainstay of prevention. Approximately one- and neurologic impairment. Folinic acid may be given con-
third of patients will experience AKI within days after treatment, currently to replete folic acid stores and minimize toxicities.
and episodes worsen with repeated dosing. Tubular injury Dialysis can acutely clear methotrexate from the blood, but
may be permanent with doses .100 mg/m2. Amifostine, a levels quickly rebound after discontinuation of treatment.
free radical scavenger, has been shown to ameliorate cisplatin Carboxypeptidase G2 can rapidly convert methotrexate to
nephrotoxicity. Newer platinum agents such as carboplatin an inactive metabolite and recently became commercially
and oxaliplatin appear to cause less tubular injury. Ifosfamide available. This therapy also suffers from rebound in plasma
is an alkylating agent commonly used in treating sarcomas and levels, but to a lesser degree than high-ux dialysis.
metastatic germ cell turmors, which may cause AKI in up to
30% of patients. Proximal tubular injury may also lead to Targeted therapy
glucosuria, hypokalemia, hypophophatemia, and proximal Targeted therapy against vascular endothelial growth factor
renal tubular acidosis. Severe cases may present with Fanconis (VEGF) has advanced the treatment of certain tumors including
syndrome. Cumulative doses .100 g/m2 are associated with colorectal and renal cell carcinoma. Monoclonal antibody to
moderate to severe tubular injury. Risk factors for AKI include VEGF (bevacizumab) and tyrosine kinase inhibitors of the VEGF
prior cisplatin therapy, tumor inltration of the kidney, and pathway (sunitinib, sorafenib, pazopanib, axitinib, and regorafenib)
underlying CKD. Mesna protects against bladder toxicity have been associated with the development of hypertension and

4 Onco-Nephrology Curriculum American Society of Nephrology


proteinuria (29). Rare cases of thrombotic microangiopathy hepatic sinusoidal endothelium from the pretransplant con-
(TMA) have also been reported (30). Symptoms generally ditioning regimen leads to sloughing of the endothelium,
resolve with discontinuation of the drug. collagen deposition, brosis, and liver failure. In severe cases,
patients may subsequently develop AKI from hepatorenal
Multiple myeloma syndrome. Presentation includes right upper quadrant ab-
Multiple myeloma involves the clonal proliferation of malig- dominal pain, ascites, edema, and elevated bilirubin. Treat-
nant plasma cells and is the second most common hematologic ment includes salt restriction, diuretics, and RRT if needed.
malignancy after non-Hodgkin lymphoma. Approximately Severe HSOS, dened as severe liver injury unresponsive to
one-half of patients with multiple myeloma present with AKI, supportive care, often requires ICU admission and is histor-
and 10% require dialysis on initial presentation (31). The ically associated with near 100% mortality. Debrotide, an
common mechanisms of injury include cast nephropathy, oligonucleotide that has antithrombotic and probrinolytic
light chain deposition disease, light chain amyloidosis, hyper- properties with minimal anticoagulant effects, has shown
calcemia, and acute tubular necrosis (ATN) from sepsis. Sup- promise in patients with severe HSOS. Several clinical trials
pression of normal hematopoiesis predisposes patients to using debrotide for treatment of severe HSOS have demon-
infections and sepsis, which often requires ICU admission. strated improvement in complete response rates and overall
Initial management consists of saline hydration, correction survival, and the drug is currently commercially available in
of hypercalcemia, alkalinization of urine, and avoidance of Europe (35,36). A new drug application (NDA) for debrotide
nonsteroidal anti-inammatory drugs and iodinated contrast. was submitted to the Food and Drug Administration in 2014
Renal recovery occurs in up to one-half of patients, except in and has been granted Fast Track Designation.
patients who require dialysis, where recovery rates are ,25%. TMA occurs in approximately 2%21% of patients after
In a randomized controlled trial, the use of plasma exchange allogeneic stem cell transplant (37). In one study, 3% of all
did not signicantly decrease the composite end point of cancer patients admitted with AKI to the ICU had underlying
death, dialysis dependence, or GFR ,30 mL/min (32). With TMA (4). Patients often present with progressive AKI, anemia
concurrent chemotherapy, the use of high cut-off lters with out of proportion to underlying renal function, and hyperten-
extended daily dialysis may help to decrease circulating mono- sion. Risk factors for transplant-associated TMA (TA-TMA)
clonal light chains. Multicenter randomized controlled trials are acute GVHD, recipient/donor mismatch, total body
studying the utility of high cut-off hemolters are currently irradiation .1,200 cGy, and adenovirus infection (37).
ongoing. TA-TMA is not associated with ADAMTS-13 deciency and is
poorly responsive to plasmapheresis. Calcineurin inhibitors are
Hematopoietic cell transplant also associated with TMA and should be withheld or decreased
The number of hematopoietic cell transplants (HCTs) per- in dose if possible.
formed has dramatically increased over the last three decades.
Renement in techniques has permitted transplants in older Contrast-induced nephropathy
patients with more comorbidities. All patients, regardless of the Intravascular administration of iodinated contrast is associated
type of transplant, are susceptible to infection after transplant with contrast-induced nephropathy (CIN). Risk factors in-
until engraftment is complete. During this period, patients are clude underlying CKD, diabetes mellitus, volume depletion,
at most risk of developing AKI from ischemic and toxic ATN in and coadministration of other nephrotoxins. Intra-arterial
the setting of sepsis. Patients who receive allogeneic transplants injection is considered to be more nephrotoxic compared
require calcineurin inhibitors to prevent graft-versus-host with intravenous administration. In addition, high osmolar
disease (GVHD), which further increases the risk of AKI. The (.1400 mOsm/kg) and low osmolar (600800 mOsm/kg)
need for RRT after HCT increases mortality more than 70% contrast agents are associated with a higher incidence of
(33,34). AKI in comparison to iso-osmolar (300 mOsm/kg) contrast.
Engraftment syndrome may occur within days after autol- Preventive measures should be taken in patients with
ogous HCT and is a common reason for ICU admission. It is GFR ,60 mL/min including limiting contrast volume, using
associated with cytokine release in association with rapid iso-osmolar contrast, prehydration with normal saline, and
neutrophil recovery after HCT. Patients develop fever, non- discontinuation of concurrent nephrotoxic agents. Several
cardiogenic pulmonary edema, erythrodermatous skin rash, meta-analyses have examined the use of N-acetylcysteine in
and peripheral edema. Often these patients develop non- the prevention of CIN but results remain inconclusive, as is
oliguric AKI with relatively bland urine sediment. The the use of bicarbonate (38). There is insufcient evidence to
mainstay of treatment is corticosteroids and diuretics, and recommend hemodialysis or hemoltration for the preven-
most patients will recover renal function without the need for tion or treatment of CIN.
RRT.
Hepatic sinusoidal obstruction syndrome (HSOS), for- Abdominal compartment syndrome
merly termed veno-occlusive disease, is associated with AKI Abdominal compartment syndrome (ACS) is most commonly
within the rst month after allogeneic HCT. Damage to the dened as an intra-abdominal pressure (IAP) .10 and clearly

American Society of Nephrology Onco-Nephrology Curriculum 5


.20 mmHg with evidence of organ dysfunction that improves 4. Darmon M, Thiery G, Ciroldi M, Porcher R, Schlemmer B, Azoulay E.
with abdominal decompression. Patients may present with Should dialysis be offered to cancer patients with acute kidney injury?
Intensive Care Med 33: 765772, 2007
tachypnea with high ventilatory pressures, liver dysfunction, in-
5. Kramer AA, Postler G, Salhab KF, Mendez C, Carey LC, Rabb H. Renal
testinal ischemia, and oliguric AKI. In patients with cancer, com- ischemia/reperfusion leads to macrophage-mediated increase in pul-
mon causes include malignant ascites, urinary leak from a recent monary vascular permeability. Kidney Int 55: 23622367, 1999
urologic procedure, and colonic dilatation. The IAP, which is 6. Vieira JM, Jr., Castro I, Curvello-Neto A, Demarzo S, Caruso P, Pastore
measured by transducing a foley catheter lled with saline with a L, Jr., Imanishe MH, Abdulkader RC, Deheinzelin D. Effect of acute
pressure monitoring system, is normally 010 mmHg. Values kidney injury on weaning from mechanical ventilation in critically ill
patients. Crit Care Med 35: 184191, 2007
between 12 and 20 mmHg are classied as intra-abdominal 7. Bellomo R, Ronco C, Kellum JA, Mehta RL, Palevsky P. Acute renal
hypertension and are not generally associated with organ dys- failure: Denition, outcome measures, animal models, uid therapy and
function. Depending on the etiology, treatment may involve information technology needs: the Second International Consensus
diuretics, paracentesis, colonic decompression with nasogas- Conference of the Acute Dialysis Quality Initiative (ADQI) Group. Crit
tric suction, and decompression laparotomy. Generally, urine Care 8: R204R212, 2004
8. Mehta RL, Kellum JA, Shah SV, Molitoris BA, Ronco C, Warnock
output and renal function markedly improve with therapy.
DG, Levin A. Acute Kidney Injury Network: Report of an initiative to
improve outcomes in acute kidney injury. Crit Care 11: R31, 2007
9. Kellum JA. Dening and classifying AKI: One set of criteria. Nephrol
CONCLUSION Dial Transplant 23: 14711472, 2008
10. Lahoti A, Nates JL, Wakeeld CD, Price KJ, Salahudeen AK. Costs and
AKI is a common complication of cancer or its treatment. outcomes of acute kidney injury in critically ill patients with cancer. J
Support Oncol 9: 149155, 2011
Advances in supportive care including RRT have improved
11. Salahudeen AK, Doshi SM, Pawar T, Nowshad G, Lahoti A, Shah P. In-
outcomes in critically ill patients with cancer, with the cidence rate, clinical correlates, and outcomes of AKI in patients admitted
exception of patients with allogeneic stem cell transplants. A to a comprehensive cancer center. Clin J Am Soc Nephrol 8: 347354, 2013
joint decision-making process between the oncologist, inten- 12. Lahoti A, Kantarjian H, Salahudeen AK, Ravandi F, Cortes JE, Faderl S,
sivist, and nephrologist is vital to determine which patients are OBrien S, Wierda W, Mattiuzzi GN. Predictors and outcome of acute
best suited for RRT. Identication of risk factors for AKI, as well kidney injury in patients with acute myelogenous leukemia or high-risk
myelodysplastic syndrome. Cancer 116: 40634068, 2010
as the development of biomarkers of kidney injury, may lead to
13. Kim CS, Oak CY, Kim HY, Kang YU, Choi JS, Bae EH, Ma SK, Kweon SS, Kim
earlier intervention. SW. Incidence, predictive factors, and clinical outcomes of acute kidney
injury after gastric surgery for gastric cancer. PLoS One 8: e82289, 2013
14. Christiansen CF, Johansen MB, Langeberg WJ, Fryzek JP, Sorensen
TAKE HOME POINTS HT. Incidence of acute kidney injury in cancer patients: A Danish
population-based cohort study. Eur J Intern Med 22: 399406, 2011
15. Darmon M, Ciroldi M, Thiery G, Schlemmer B, Azoulay E. Clinical re-
c The selection of patients best suited for RRT requires a team-based
view: Specic aspects of acute renal failure in cancer patients. Crit Care
approach between the oncologist, intensivist, and nephrologist.
10: 211, 2006
c Manifestations of kidney disease from chemotherapy and targeted 16. Benoit DD, Hoste EA. Acute kidney injury in critically ill patients with
therapy include AKI, proteinuria, electrolytes derangements, and TMA. cancer. Crit Care Clin 26: 151179, 2009
c Nearly one-half of patients with multiple myeloma have evidence of AKI 17. Darmon M, Thiery G, Ciroldi M, de Miranda S, Galicier L, Raffoux E, Le
on initial presentation, and 10% require dialysis. Gall JR, Schlemmer B, Azoulay E. Intensive care in patients with newly
diagnosed malignancies and a need for cancer chemotherapy. Crit
c Engraftment syndrome, HSOS, and TMA are unique causes of AKI in
Care Med 33: 24882493, 2005
patients after stem cell transplant. The mortality of patients that require 18. Soares M, Salluh JI, Carvalho MS, Darmon M, Rocco JR, Spector N.
dialysis after stem cell transplant remains high. Prognosis of critically ill patients with cancer and acute renal dysfunc-
tion. J Clin Oncol 24: 40034010, 2006
19. Dogan E, Izmirli M, Ceylan K, Erkoc R, Sayarlioglu H, Begenik H, Alici S.
Incidence of renal insufciency in cancer patients. Adv Ther 22: 357
362, 2005
REFERENCES
20. Eheman C, Henley SJ, Ballard-Barbash R, Jacobs EJ, Schymura MJ,
Noone AM, Pan L, Anderson RN, Fulton JE, Kohler BA, Jemal A, Ward
1. Brenner H. Long-term survival rates of cancer patients achieved by the E, Plescia M, Ries LA, Edwards BK. Annual Report to the Nation on the
end of the 20th century: A period analysis. Lancet 360: 11311135, status of cancer, 1975-2008, featuring cancers associated with excess
2002 weight and lack of sufcient physical activity. Cancer 118: 23382366,
2. Benoit DD, Hoste EA, Depuydt PO, Offner FC, Lameire NH, 2012
Vandewoude KH, Dhondt AW, Noens LA, Decruyenaere JM. Outcome 21. Cho A, Lee JE, Kwon GY, Huh W, Lee HM, Kim YG, Kim DJ, Oh HY, Choi
in critically ill medical patients treated with renal replacement therapy HY. Post-operative acute kidney injury in patients with renal cell carci-
for acute renal failure: comparison between patients with and those noma is a potent risk factor for new-onset chronic kidney disease after
without haematological malignancies. Nephrol Dial Transplant 20: radical nephrectomy. Nephrol Dial Transplant 26: 34963501, 2011
552558, 2005 22. Candrilli S, Bell T, Irish W, Morris E, Goldman S, Cairo MS. A comparison
3. Berghmans T, Meert AP, Markiewicz E, Sculier JP. Continuous veno- of inpatient length of stay and costs among patients with hematologic
venous haemoltration in cancer patients with renal failure: A single- malignancies (excluding hodgkin disease) associated with and without
centre experience. Support Care Cancer 12: 306311, 2004 acute renal failure. Clin Lymphoma Myeloma 8: 4451, 2008

6 Onco-Nephrology Curriculum American Society of Nephrology


23. Lameire N, Van Biesen W, Vanholder R. Acute renal problems in the 32. Clark WF, Stewart AK, Rock GA, Sternbach M, Sutton DM, Barrett BJ,
critically ill cancer patient. Curr Opin Crit Care 14: 635646, 2008 Heidenheim AP, Garg AX, Churchill DN. Plasma exchange when my-
24. Taccone FS, Artigas AA, Sprung CL, Moreno R, Sakr Y, Vincent JL. eloma presents as acute renal failure: A randomized, controlled trial.
Characteristics and outcomes of cancer patients in European ICUs. Crit Ann Intern Med 143: 777784, 2005
Care (London, England) 13: R15, 2009 33. Lopes JA, Jorge S. Acute kidney injury following HCT: Incidence, risk
25. Larche J, Azoulay E, Fieux F, Mesnard L, Moreau D, Thiery G, Darmon factors and outcome. Bone Marrow Transplantation 46: 13991408,
M, Le Gall JR, Schlemmer B. Improved survival of critically ill cancer 2011
patients with septic shock. Intensive Care Med 29: 16881695, 2003 34. Humphreys BD. Onco-nephrology: Kidney disease in the cancer pa-
26. Pene F, Percheron S, Lemiale V, Viallon V, Claessens YE, Marque S, tient: Introduction. Semin Nephrol 30: 531533, 2010
Charpentier J, Angus DC, Cariou A, Chiche JD, Mira JP. Temporal 35. Richardson PG, Murakami C, Jin Z, Warren D, Momtaz P, Hoppensteadt
changes in management and outcome of septic shock in patients with D, Elias AD, Antin JH, Soiffer R, Spitzer T, Avigan D, Bearman SI, Martin
malignancies in the intensive care unit. Critical Care Med 36: 690696, PL, Kurtzberg J, Vredenburgh J, Chen AR, Arai S, Vogelsang G,
2008 McDonald GB, Guinan EC. Multi-institutional use of debrotide in 88
27. Hidayat LK, Hsu DI, Quist R, Shriner KA, Wong-Beringer A. High-dose patients after stem cell transplantation with severe veno-occlusive
vancomycin therapy for methicillin-resistant Staphylococcus aureus disease and multisystem organ failure: response without signicant
infections: Efcacy and toxicity. Arch Intern Med 166: 21382144, toxicity in a high-risk population and factors predictive of outcome.
2006 Blood 100: 43374343, 2002
28. Lodise TP, Lomaestro B, Graves J, Drusano GL. Larger vancomycin 36. Richardson PG, Soiffer RJ, Antin JH, Uno H, Jin Z, Kurtzberg J, Martin
doses (at least four grams per day) are associated with an increased PL, Steinbach G, Murray KF, Vogelsang GB, Chen AR, Krishnan A,
incidence of nephrotoxicity. Antimicrob Agents Chemother 52: 1330 Kernan NA, Avigan DE, Spitzer TR, Shulman HM, Di Salvo DN, Revta C,
1336, 2008 Warren D, Momtaz P, Bradwin G, Wei LJ, Iacobelli M, McDonald GB,
29. Izzedine H, Rixe O, Billemont B, Baumelou A, Deray G. Angiogenesis Guinan EC. Debrotide for the treatment of severe hepatic veno-
inhibitor therapies: Focus on kidney toxicity and hypertension. Am J occlusive disease and multiorgan failure after stem cell transplantation:
Kidney Dis 50: 203218, 2007 a multicenter, randomized, dose-nding trial. Biol Blood Marrow
30. Eremina V, Jefferson JA, Kowalewska J, Hochster H, Haas M, Weisstuch Transplant 16: 10051017, 2010
J, Richardson C, Kopp JB, Kabir MG, Backx PH, Gerber HP, Ferrara N, 37. Changsirikulchai S, Myerson D, Guthrie KA, McDonald GB, Alpers
Barisoni L, Alpers CE, Quaggin SE. VEGF inhibition and renal throm- CE, Hingorani SR. Renal thrombotic microangiopathy after hemato-
botic microangiopathy. N Engl J Med 358: 11291136, 2008 poietic cell transplant: Role of GVHD in pathogenesis. Clin J Am Soc
31. Kyle RA, Gertz MA, Witzig TE, Lust JA, Lacy MQ, Dispenzieri A, Fonseca Nephrol 4: 345353, 2009
R, Rajkumar SV, Offord JR, Larson DR, Plevak ME, Therneau TM, Greipp 38. Vaitkus PT, Brar C. N-acetylcysteine in the prevention of contrast-
PR. Review of 1027 patients with newly diagnosed multiple myeloma. induced nephropathy: Publication bias perpetuated by meta-analyses.
Mayo Clin Proc 78: 2133, 2003 Am Heart J 153: 275280, 2007

American Society of Nephrology Onco-Nephrology Curriculum 7


REVIEW QUESTIONS Answer: c is correct. The three most common mani-
festations of myeloma-related kidney disease include cast
1. Criteria for AKI as dened by the KDIGO classication nephropathy, light chain deposition disease, and light
include the following except: chain amyloidosis. Other less common manifestations
include heavy chain deposition disease, membranopro-
a. A rise in SCr $0.3 mg/dL within 48 hours liferative glomerulonephritis from cryoglobulinemia,
b. An increase in SCr to $1.5 times baseline within the prior and brillary glomerulonephritis. TMA is not a common
7 days presentation.
c. A urine volume of ,0.5 mL/kg/h for 6 hours
d. An increase in SCr to $1.5 times the upper limit of the
3. Which of the following therapies has shown efcacy in the
normal range as listed in the laboratory reference values
treatment of HSOS after stem cell transplant?
Answer: d is correct. The KDIGO classication denes AKI
a. Heparin
as 1) an increase in SCr $0.3 mg/dL within 48 hours; 2) an
b. Debrotide
increase in SCr to $1.5 times baseline within the prior 7 days,
c. Tissue plasminogen activator (tPA)
or 3) a urine volume of ,0.5 mL/kg/h for 6 hours. The upper d. Plasmapheresis
limit of normal from a reference range should not be used in
diagnosing AKI if the patients baseline SCr level is known.
Answer: b is correct. Heparin has been used for prophylaxis
2. Common manifestations of myeloma-related kidney dis- of HSOS with mixed results. Both heparin and tPA have un-
ease include all of the following except: acceptable bleeding risks when used for treatment of HSOS.
Debrotide, an oligonucleotide that has antithrombotic and
a. Cast nephropathy probrinolytic properties with minimal anticoagulant ef-
b. Light chain deposition disease fects, has shown promise in the treatment of patients with
c. Thrombotic microangiopathy (TMA) severe HSOS. Plasmapheresis has no role in the treatment
d. Light chain amyloidosis of HSOS.

8 Onco-Nephrology Curriculum American Society of Nephrology


Chapter 4: Tumor Lysis Syndrome
Amaka Edeani, MD,* and Anushree Shirali, MD
*Kidney Diseases Branch, National Institute of Diabetes and Digestive and Kidney Diseases, National Institutes of
Health, Bethesda, Maryland; and Section of Nephrology, Yale University School of Medicine, New Haven, Connecticut

INTRODUCTION class species patients with normal laboratory and


clinical parameters as having no LTLS or CTLS.
Tumor lysis syndrome (TLS) is a constellation of Cairo and Bishop also proposed a grading system
metabolic abnormalities resulting from either spon- combining the denitions of no TLS, LTLS, and CTLS,
taneous or chemotherapy-induced tumor cell death. with the maximal clinical manifestations in each
Tumor cytotoxicity releases intracellular contents, affected organ dening the grade of TLS (1). Although
including nucleic acids, proteins, and electrolytes this grading system attempts to provide uniform def-
into the systemic circulation and may lead to devel- initions to TLS severity, it is not widely used in clinical
opment of hyperuricemia, hyperphosphatemia, hy- practice.
pocalcemia, and hyperkalemia. Clinically, this results The Cairo-Bishop classication is not immune to
in multiorgan effects such as AKI, cardiac arrhyth- critique despite its common use. Specically, patients
mias, and seizures (1,2). TLS is the most common with TLS may not always have two or more abnor-
oncologic emergency (3), and without prompt rec- malities present at once, but one metabolic derange-
ognition and early therapeutic intervention, mor- ment may precede another (2). Furthermore, a 25%
bidity and mortality is high. change from baseline may not always be signicant if
it does not result in a value outside the normal range
(2). From a renal standpoint, Wilson and Berns (5)
DEFINITION have noted that dening AKI on the basis of a creat-
inine value .1.5 times the upper limit of normal
Hande and Garrow (4) rst initiated a denition of does not clearly distinguish CKD from AKI. Thus,
the clinical and pathologic characteristics of patients they propose using established denitions of AKI in
at risk for developing TLS. Based on a retrospective CTLS such as an absolute 0.3 mg/dL increase or rel-
analysis of 102 patients with non-Hodgkin lym- ative 50% increase in creatinine over baseline. Finally,
phoma (NHL), they classied TLS as laboratory they point out that the Cairo-Bishop classications
TLS (LTLS) or clinical TLS (CTLS). Cairo and Bishop cannot be applied to spontaneous TLS, which is com-
(1) modied these criteria to formulate a commonly mon with high-risk malignancies, as chemotherapy
used classication system for TLS. This system (Table is a required criterion for LTLS and CTLS.
1) denes LTLS when two or more of the following
abnormalities are met within 3 days before or 7 days
after the initiation of chemotherapy: 1) 25% decrease EPIDEMIOLOGY AND RISK FACTORS
from baseline in serum calcium, and/or 2) 25% in-
crease from baseline in the serum values of uric acid, TLS is most commonly described in NHL, particularly
potassium, or phosphorous. Burkitt-type lymphoma (BTL), as well as other he-
The Cairo and Bishop denition assumes adequate matologic malignancies, such as acute lymphocytic
volume expansion and prophylaxis with a hypour- and lymphoblastic leukemia (ALL) and acute myeloid
icemic agent. LTLS is dened as CTLS (Table 1) when leukemia (AML) (68), and less commonly in chronic
LTLS is accompanied by one or more clinical mani-
festations such as cardiac arrhythmia, death, seizure,
or AKI with an elevated serum creatinine .1.5 times Correspondence: Anushree Shirali, Section of Nephrology, Yale
upper limit of normal. Additionally, this denition of University School of Medicine, PO Box 208029, New Haven,
Connecticut 06520-8029.
CTLS assumes that the clinical manifestations are not
caused directly by the therapeutic agent. Last, a third Copyright 2016 by the American Society of Nephrology

American Society of Nephrology Onco-Nephrology Curriculum 1


Table 1. Cairo-Bishop denition of laboratory tumor lysis syndrome and clinical tumor lysis syndrome
Laboratory Tumor Lysis Syndrome
Metabolite or electrolyte Criterion for diagnosis
Uric acid $8 mg/dL or 25% increase from baseline
Potassium $6 mEq/L or 25% increase from baseline
Phosphorus $6.5 mg/dL (children), $4.5 mg/dL (adults), or 25% increase from baseline
Calcium $25% decrease from baseline
Clinical Tumor Lysis Syndrome
LTLS and one or more of the following: 1) creatinine 3 $1.5 ULN (age .12 years of age or age adjusted); 2) cardiac arrhythmia or sudden death;
3) seizure
LTLS, laboratory tumor lysis syndrome; ULN, upper limit of normal.

leukemias (911) and multiple myeloma (12,13). More rarely, of cancer cells, and cell sensitivity to cytotoxic therapy. Pa-
TLS has also been described with solid malignancies (14,15) with tient-related factors include age, volume depletion, preexist-
particular features, including large tumor burden, metastatic dis- ing CKD, hyperuricemia, and hyponatremia. Recognition of
ease, specically in the liver, short doubling time, increased che- these high-risk factors is an important step in the management
mosensitivity, and elevated uric acid and lactate dehydrogenase of TLS. In 2008, an expert panel (7) developed a TLS risk
(LDH) (15). Among solid tumors, small-cell carcinoma of the classication system, based on published evidence and expert
lung, germ cell tumors, neuroblastoma, and breast carcinoma opinion, in which malignancies as were described as low
have all been linked to development of TLS (8). TLS is usually (,1% chance), intermediate (1%5% chance), or high risk
associated with cytotoxic chemotherapy but reports have also (.5% chance) for developing TLS. Classication into these
linked it to the use of imatinib (11), bortezomib (12), corticoste- risk groups incorporates type of histology, extent of disease,
roids (16,17), rituximab (18), methotrexate (19), and thalido- renal involvement or dysfunction, and type of induction ther-
mide (13,20). There are also case reports of TLS following total apy (Table 2).
body irradiation (21) and chemoembolization (22). Last, TLS Other factors that have been shown to be predictive of TLS
may also be spontaneous, i.e., not requiring initiation of cytotoxic include male sex and presence of splenomegaly (26,28,30). Cer-
therapy. This has been most frequently described in BTL (2325). tain cytogenetic shifts may also portend greater risk for TLS.
The incidence of TLS varies based on the underlying Specically, MYCN gene mutation in neuroblastoma (31), t
malignancy and the denition of TLS. Most incidence data (8;14)(q24;q32) in L3 type of acute lymphoblastic leukemia
are from older, retrospective studies that precede the Cairo- (32), and inv(16)(p13;q22) in acute myelocytic leukemia (33)
Bishop classication, so there is considerable heterogeneity in are all linked to more aggressive disease and greater risk for TLS.
the data. In a review of 102 patients with high-grade NHL and
using the Hande-Garrow classication, LTLS was seen in 42%
of patients, with CTLS occurring only in 6% (4). In BTL, PATHOPHYSIOLOGY AND CLINICAL
however, 56% and 11% of patients met criteria for LTLS and MANIFESTATIONS
CTLS, respectively. Mato et al. (26) studied 194 patients re-
ceiving induction therapy for AML and found a TLS incidence TLS is a direct consequence of cell lysis and release of
of 9.8%. In a mixed adult and pediatric study of 788 European intracellular products. When clearance of these products, by
patients with acute leukemia or NHL (27), the overall inci- excretion (renal or hepatic excretion or phagocytosis by the
dence of LTLS and CTLS was 18.9% and 5%, respectively. reticuloendothelial system) (23), is impaired and their serum
When classied by tumor type, LTLS and CTLS incidence rates burden increases, the clinical sequelae of TLS may occur. Of
of 14.7% and 3.4% were seen in AML patients, respectively; these cellular products, nucleic acids (converted to uric acid),
21.4% and 5.2% in ALL patients, respectively; and 19.6% and potassium, and phosphorus are particularly important in the
6.1% in patients with NHL, respectively (27). Wssman et al. pathophysiology of TLS.
(28) reviewed the incidence and complications of 1,791 chil-
dren with NHL and reported an overall incidence of 4.4%, of Hyperuricemia
which 26% had B-cell ALL (B-ALL). The nucleic acids adenine and guanine are metabolized to
xanthine, which is further metabolized by xanthine oxidase to
Risk stratication the water-insoluble uric acid (5) (Figure 1). Because humans
Risk factors (2,29) for TLS include cancer and patient-specic lack a functional gene for urate oxidase (uricase), which fur-
factors. Increased tumor burden is the most cancer-specic ther metabolizes uric acid to the freely soluble and excretable
risk factor and is demonstrated by elevated LDH (28), white allantoin, patients with high-risk malignancy are susceptible
blood cell count .50,000/mm3, massive liver metastasis (14), to rapid increases in serum uric acid. Uric acid is freely ltered
bone marrow involvement (2), cancer stage, proliferation rate at the glomerulus, and handling in the renal proximal tubule

2 Onco-Nephrology Curriculum American Society of Nephrology


Table 2. Risk classication of TLS according to type of malignancy, extent of disease, and presence or absence of renal
dysfunction
Type of malignancy Risk
Solid tumor Low
Myeloma Low
Chronic leukemia CML: low
CLL w/alkylating agents: low
CLL w/targeted or biological agents: intermediate
Lymphoma: Burkitt type Early stage and LDH ,2 3 ULN: intermediate
Early stage and LDH .2 3 ULN: high
Advanced stage: high
Lymphoma: non-Burkitt type
Anaplastic large cell Child with stage III/IV disease: intermediate
All others: low
Lymphoblastic lymphoma Early stage and LDH ,2 3 ULN: intermediate
Early stage and LDH .2 3 ULN: high
Advanced stage: high
Hodgkin, small lymphocytic, follicular, marginal zone B cell, Low
MALT, nonblastoid mantle cell, cutaneous T cell
Adult T-cell lymphoma, diffuse large B cell, peripheral T cell, Adult with normal LDH: low
transformed, or blastoid mantle cell Child with stage I/II disease: low
Adult with LDH . ULN and nonbulky disease: intermediate
Adult with LDH . ULN and bulky disease: high
Child with stage III/IV disease and LDH ,2 3 ULN: intermediate
Child with stage III/IV disease and LDH .2 3 ULN: high
Leukemia:-Burkitt type High
Leukemia: non-Burkitt type; acute myeloid leukemia (AML); AML with WBC ,25 3 109/L and LDH ,2 3 ULN: low
acute lymphoblastic Lleukemia (ALL) AML with WBC ,25 3 109/L and LDH .2 3 ULN: intermediate
AML with WBC 5 25100 3 109/L: intermediate
ALL with WBC ,100 3 109/L and LDH ,2 3 ULN: intermediate
ALL with WBC ,100 3 109/L and LDH .2 3 ULN: high
ALL with WBC .100 3 109/L: high
Renal dysfunction Risk
Absent If low risk disease, no change
If intermediate risk disease and normal UA, phosphorus,
and potassium, no change
If UA, phosphorus, or potassium . ULN, intermediate risk disease
becomes high risk
Present Low risk disease become intermediate risk
Intermediate risk disease becomes high risk
CML, chronic myeloid leukemia; CLL, chronic lymphocytic leukemia; MALT, mucosa-associated lymphoid tissue; LDH, lactate dehydrogenase; AML, acute myeloid
leukemia; WBC, white blood cell count; ALL, acute lymphocytic and lymphoblastic leukemia; UA, urinalysis; ULN, upper limit of normal.

is a combination of reabsorption and secretion via the luminal crystal-independent mechanisms which target hemodynamics.
urate/anion exchanger urate transporter 1 (URAT-1) and the These include increased peritubular capillary pressures, in-
basolateral organic anion transporter (OAT) (34). URAT-1 is creased vasoconstriction, and decreased blood ow (5,3739).
an apical membrane transporter and exchanges anions for urate Uric acid may also prevent recovery from AKI in TLS, as it has
absorption from the tubular lumen. It is critical in regulating been shown to inhibit proximal tubule cell proliferation (38).
urate levels and is targeted by uricosuric and antiuricosuric These diverse mechanisms are united in their propensity to
agents (34). When the capacity to transport luminal uric acid cause AKI. Clinically, hyperuricemia is unlikely to cause symp-
is overwhelmed, there is potential for uric acid to crystallize toms because urinary crystallization of uric acid does not result
within the tubular lumen. An acidic urine pH favors this process. in the renal colic, which is typical of uric acid nephrolithiasis.
Uric acid crystals can cause direct tubular injury by
obstruction, but other pathways for injury include induction Hyperkalemia
of chemokine-mediated inammation from monocyte Massive tumor cell lysis releases potassium into the extracel-
chemoattractant protein-1 (MCP-1) (35) and macrophage lular environment, leading to severe hyperkalemia when
migration inhibition factor (MIF) (36). There are also uptake capacity by muscle and liver is exceeded, especially in

American Society of Nephrology Onco-Nephrology Curriculum 3


Figure 1. Schematic of purine metabolism. Allopurinol acts an inhibitor of xanthine oxidase via its active metabolite, oxypurinol.
Dashed arrow and box indicate arm of metabolism not constitutively present in humans; this conversion of uric acid to water-soluble
allantoin is stimulated clinically by the administration of rasburicase (recombinant urate oxidase). Black arrows denote enzyme stim-
ulation; red lines denote inhibition.

the setting of CKD or AKI. Muscle weakness may be the initial population (42). Although the data were not broken down into
symptom, but cardiac arrhythmia, manifested initially by cause of AKI, the incidence of TLS was similar at 17%, suggesting
peaked Twaves, widened QRS complexes, and sine waves, is the that AKI and TLS were also linked in this population. AKI due to
feared complication. TLS may be asymptomatic or include symptoms of uremia, in-
cluding nausea, vomiting, and lethargy.
Hyperphosphatemia and hypocalcemia
Because phosphate is an intracellular electrolyte, cell lysis releases
signicant amounts of it. However, malignant hematologic cells MANAGEMENT
may contain four times more intracellular phosphate in com-
parison to normal mature lymphoid cells (3), making Prophylaxis and monitoring
hyperphosphatemia a particular issue with tumor cell lysis. Be- Prevention of TLS begins with recognition of risk factors and
cause phosphorus excretion is tied to kidney function, hyper- close laboratory and clinical monitoring. Patients at highest
phosphatemia occurs when the kidneys excretory capacity is risk of developing TLS (Table 2) require intensied monitoring
overwhelmed. Thus, preexisting CKD or AKI enhances risk with more frequent electrolyte checks. Patients with high-risk
for hyperphosphatemia with TLS. Spontaneous tumor lysis, disease may be prone to lactic acidosis from massive tumor cell
however, is less commonly associated with hyperphosphatemia necrosis. Because acidosis inhibits uric acid excretion (43),
and may be due to rapid uptake of extracellular phosphate by prompt recognition and correct of acidosis may prevent or
residual highly metabolically active tumor cells (5). Hyperphos- ameliorate uric acid nephropathy. Additionally, nonsteroidal
phatemia may cause nausea, vomiting, diarrhea, or lethargy, but anti-inammatory drugs, iodinated radiocontrast dye, and
it exerts its predominant toxicity by binding to calcium cations. other potentially nephrotoxic therapeutic agents should be
This results in secondary hypocalcemia and its downstream neu- avoided to abrogate the risk of AKI from TLS.
romuscular and cardiovascular effects such as cramps, hypoten-
sion, tetany, and arrhythmias. Additionally, calciumphosphate Volume expansion
precipitates may deposit in tissues, as seen in nephrocalcinosis, Delivery of crystalloid intravenous uids (IVFs) is recommended
including the renal interstitium. for all patients and is essential for those with higher TLS risk.
Volume expansion supports adequate intravascular volume and
AKI renal blood ow, which maintain glomerular ltration. This is the
AKI in TLS may be either due to the aforementioned effects of cornerstone of uric acid, potassium, and phosphate excretion and
acute urate nephropathy or hyperphosphatemic nephrocalci- may delay and prevent the need for renal replacement measures
nosis affecting the renal tubulointerstitium or a combination of (2,6,44). High-dose IVFs up to 3 L have been recommended (2),
the two. Some studies have suggested that a urine uric acid to for a target urine output of $2 mL/kg/h. Diuretics may be nec-
creatinine ratio of .1 may be specic to uric acid nephropathy essary if patients develop volume overload, but routine use is not
(40), but another study has noted high uric acid to creatinine recommended to avoid volume depletion.
ratios in AKI from other etiologies (41).
The association between AKI and TLS has been demonstrated Urinary alkalinization
across various populations and tumor subtypes (5). Annemans Alkalinization makes physiologic sense, as increasing urine pH
et al. (27) found that in patients with leukemia and NHL who from 5 to 7 can increase the solubility of uric acid .10-fold
had TLS, 45% had AKI. A smaller pediatric cohort of B-cell NHL (28). However, urinary alkalinization decreases calcium
or ALL noted renal insufciency in 20% percent of the study phosphate solubility (2), thereby exacerbating its precipitation

4 Onco-Nephrology Curriculum American Society of Nephrology


and deposition. Furthermore, if urinary alkalinization results to FDA approval, 1,069 adult and pediatric patients received
in rising serum pH, free calcium may bind albumin more rasburicase on a compassionate use basis (53). Decreased serum
avidly and further exacerbate hypocalcemia (45). Thus, uri- uric acid levels were observed in 99% of children and 100% of
nary alkalinization is not recommended in the management of adults. Hemodialysis was performed in only 2.8% of patients.
TLS (2,6,45). In a study of 131 patients with newly diagnosed leukemia or
lymphoma, Pui et al. (54) reported a decrease in plasma uric
Allopurinol acid concentrations from 9.7 to 1 mg/dL (P 5 0.0001) in 65
Allopurinol is converted in vivo to oxypurinol and as a xanthine patients who presented with hyperuricemia and a decrease
analog acts as a competitive inhibitor of xanthine oxidase and from 4.3 to 0.5 mg/dL (P 5 0.0001) in the remaining patients.
blocks the conversion of purines to uric acid (6,46) (Figure 1). There was negligible toxicity, and no patients required dialysis.
This prevents hyperuricemia but does not treat preexisting hy- Cortes et al. (55) compared response rates in dosing rasburi-
peruricemia (6). Furthermore, because oxypurinol also inhibits case alone versus rasburicase followed by allopurinol versus al-
the conversion of xanthine to uric acid, serum and urine xan- lopurinol alone. They reported a plasma uric acid response rate
thine levels may rise and precipitate xanthine crystal deposition of 87% in the rasburicase group, 78% in the rasburicase followed
in the renal tubules and xanthine-induced obstructive nephrop- by allopurinol group, and 66% in the allopurinol group, with a
athy (47). Administration of allopurinol is recommended for signicantly greater response for rasburicase compared with al-
prophylaxis in patients with low and intermediate risk of devel- lopurinol in the overall study population (P 5 0.001), in patients
oping TLS (2,6). Smalley et al. (48) studied 1,172 patients to at high risk for TLS (89% versus 68%; P 5 0.012), and in those
evaluate the efcacy and safety of intravenous allopurinol in with baseline hyperuricemia (90% versus 53%; P 5 0.015). Of
patients with hyperuricemia. They noted reduced uric acid levels note, there are no prospective studies to date that have examined
in 57% of adults and 88% of children. When used as prophylactic the impact of rasburicase on relevant clinical end points such as
therapy, allopurinol prevented an increase in uric acid levels in morbidity from AKI. Nonetheless, rasburicase should be used
93% of adults and 92% of children. for prophylaxis in patients with high risk of developing TLS (7).
Because oxypurinol excretion is by the kidney, dose The FDA-approved dosing guidelines recommend 0.2 mg/kg in
adjustments are necessary for patients with CKD and AKI. 50 mL normal saline as a 30-minute intravenous infusion once
Allopurinol has been associated with a hypersensitivity syn- daily for up to 5 days (51). Length of treatment is related to
drome with rash, acute hepatitis, and eosinophilia (45,49). control of plasma uric acid levels, but use of rasburicase for
Allopurinol reduces the clearance of purine-based chemother- .5 days is rarely needed (6,51). In comparison with generically
apeutic agents such as 6-mercaptopurine and azathioprine available allopurinol, rasburicase is signicantly more expensive
(6). It may also interact with azathioprine and cyclophospha- (up to $3,600 per 7.5-mg vial) (45), and in most published stud-
mide in potentiating severe bone marrow suppression (6,45). ies, one-time dosing was sufcient to suppress hyperuricemia.
Rasburicase does not require dosing adjustment for GFR
Febuxostat and is not known to have any known clinically relevant drug
Febuxostat is a novel xanthine oxidase inhibitor lacking the drug interactions (51,56). Adverse reactions are rare but may
hypersensitivity prole of allopurinol. Because it is metabolized to include rash, increased liver enzyme levels, headaches, fever,
inactive metabolites by the liver, adjustment for reduced GFR is not vomiting, and nausea (56).
necessary. It has been proposed as a viable alternative to allopurinol Rasburicase is active ex vivo, so blood samples for serum
in TLS prophylaxis for patients with allopurinol hypersensitivity or uric acid levels must be stored on ice to avoid erroneously low
renal dysfunction (45). A recently completed phase III study of results (45). Patients with glucose 6-phosphate dehydrogenase
febuxostat versus allopurinol in TLS prevention found signi- (G6PD) deciency can develop signicant methemoglobine-
cantly lower serum uric acid in the febuxostat but found no sig- mia and hemolysis due to oxidative stress triggered by hydro-
nicant difference in serum creatinine change compared with gen peroxide (57,58). Accordingly, patients should have G6PD
allopurinol (50). Febuxostat use has been limited by its signicant status tested prior to starting rasburicase.
cost compared with generically available allopurinol (45).
RRT
Rasburicase The need for renal replacement has signicantly reduced since
Rasburicase (Elitek) is an Aspergillus-derived recombinant urate the advent of rasburicase, but about 1.5% of children and 5% of
oxidase approved by the US Food and Drug Administration adults require dialysis during induction therapies (53). Indi-
(FDA) in 2002 for the initial management of hyperuricemia in cations for RRT are similar to those for AKI from other causes,
pediatric patients with leukemia, lymphoma, and solid tumor but due to the rapid onset of the clinical manifestations of TLS,
malignancies receiving anticancer therapy (51). It was subse- the threshold for initiating dialytic therapies is lower than in
quently approved for use in adults in 2009 (51). Rasburicase other situations. Although intermittent hemodialysis (IHD)
catalyzes the conversion of uric acid to allantoin, carbon dioxide, may be sufcient for most patients, continuous RRT (CRRT)
and hydrogen peroxide (Figure 1). Allantoin is 5- to 10-fold at high dialysate or replacement uid ow rates (.3-4 L/h)
more soluble than uric acid (52) and is readily excreted. Prior may be necessary in those patients with severe TLS who

American Society of Nephrology Onco-Nephrology Curriculum 5


experience rebound in serum potassium and phosphorous 5. Perry Wilson F, Berns JS. Onco-nephrology: Tumor lysis syndrome. Clin
levels with IHD (45,59,60). J Am Soc Nephrol 7: 17301739, 2012
6. Coifer B, Altman A, Pui C, Younes A, Cairo MS. Guidelines for the
Management of Pediatric and Adult Tumor Lysis Syndrome: An
evidence-based review. J Clin Oncol 26: 27672778, 2008
PROGNOSIS
7. Cairo MS, Coifer B, Reiter A, Younes A, Baruchel A, Bosly A, Goldman
SC, Leverger G, Ohyashiki K, Panagiotidis P, Pession A, Pui CH, Ribera
There are many confounding factors that impact clinical JM, Rosti G, Rule S, Tsukimoto I, Zinzani PL. Recommendations for the
outcomes in patients with malignancies, particularly in those evaluation of risk and prophylaxis of tumour lysis syndrome (TLS) in
who have TLS, but AKI appears to be a signicant predictor of adults and children with malignant diseases: An expert TLS panel
consensus. Br J Haematol 149: 578586, 2010
short- and long-term mortality from TLS. A study comparing 8. Baeksgaard L, Srenson JB. Acute tumor lysis syndrome in solid tumors:
hematologic cancer patients without AKI to patients with AKI A case report and review of the literature. Cancer Chemother Phar-
(61) showed signicantly lower hospital mortality (7% and macol 51: 187192, 2003
21%, respectively) and 6-month mortality (51% and 66%, re- 9. Jensen M, Winkler U, Manzke O, Diehl V, Engert A. Rapid tumor lysis
spectively) in patients without AKI. TLS is most common in a patient with B-cell chronic lymphocytic leukemia and lymphocytosis
treated with an anti-CD20 monoclonal antibody (IDEC-C2B8, ritux-
during initial presentation of disease because relapsed malig- imab). Ann Hematol 77: 8991, 1998
nancies are signicantly more chemoresistant (5). There are 10. Cech P, Block JB, Cone LA, Stone A. Tumor lysis syndrome after ta-
fewer case reports of TLS in recurrent disease (62). moxifen are. N Engl J Med 315: 263264, 1986
11. Al-Kali A, Farooq S, Tfayli A. Tumor lysis syndrome after starting
treatment with Gleevec in a patient with chronic myelogenous leuke-
CONCLUSIONS mia. J Clin Pharm Ther 34: 607610, 2009
12. Sezer O, Vesole DH, Singhal S, Richardson P, Stadtmauer E, Jakob C, Boral
AL, Esseltine DL, Mehta J. Bortezomib-induced tumor lysis syndrome in
TLS is a common oncologic emergency that requires imme- multiple myeloma. Clin Lymphoma Myeloma 7: 233235, 2006
diate diagnosis and prompt treatment to avoid morbidity and 13. Fuente N, Mane JM, Barcelo R, Muoz A, Perez-Hoyos T, Lopez-
mortality. Understanding the diagnostic criteria for TLS, Vivanco G. Tumor lysis syndrome in a multiple myeloma patient treated
knowing the tumor types at high risk for TLS, and instituting with thalidomide. Ann Oncol 15: 537, 2004
prophylactic and treatment measures are essential for the 14. Mirrakhimov AE, Ali AM, Khan MK, Barbaryan A. Tumor lysis syndrome
in solid tumors: An up to date review of the literature. Rare Tumors 6:
nephrologist who treats patients with malignant diseases. 6876, 2014
15. Gemici C. Tumour lysis syndrome in solid tumors. Clin Oncol 18: 773
780, 2006
TAKE HOME POINTS 16. Lerza R, Botta M, Barsotti B, Schenone E, Mencoboni M, Bogliolo G,
Pannacciulli I, Arboscello E. Dexamethasone-induced acute tumor lysis
c TLS is the most common oncologic emergency. syndrome in a T-cell malignant lymphoma. Leuk Lymphoma 43: 1129
1132, 2002
c The risk of TLS depends on tumor type but is also inuenced by other
17. Tiley C, Grimwade D, Findlay M, Treleaven J, Height S, Catalano J,
factors.
Powles R. Tumour lysis following hydrocortisone prior to a blood
c There is a high burden of AKI in patients with TLS. product transfusion in T-cell acute lymphoblastic leukemia. Leuk
c Prophylaxis with volume expansion is the mainstay of preventing TLS in Lymphoma 8: 143146, 1992
any patient-risk category. 18. Jabr F. Acute tumor lysis syndrome induced by rituximab in diffuse
large B-cell lymphoma. Int J Hematol 82: 312314, 2005
c Patients at high risk for TLS should receive rasburicase for initial treat-
19. Simmons ED, Somberg KA. Acute tumor lysis syndrome after in-
ment of hyperuricemia.
trathecal methotrexate administration. Cancer 67: 20622065, 1991
20. Lee CC, Wu YH, Chung SH, et al. Acute tumor lysis syndrome after
ACKNOWLEDGMENTS thalidomide therapy in advanced hepatocellular carcinoma. Oncolo-
gist 11: 8788, 2006
21. Linck D, Basara N, Tran V, Chen WJ. Peracute onset of severe tumor
Dr. Edeanis work is supported by the intramural Research Program lysis syndrome immediately after 4 Gy fractionated TBI as part of re-
of the National Institute of Diabetes and Digestive and Kidney Dis- duced intensity preparative regimen in a patient with T-ALL with high
eases, National Institutes of Health. tumor burden. Bone Marrow Transplant. 31: 935937, 2003
22. Hsieh PM, Hung KC, Chen YS. Tumor lysis syndrome after transarterial
chemoembolization of hepatocellular carcinoma: Case reports and
literature review. World J Gastroenterol 15: 47264728, 2009
REFERENCES 23. Cohen LF, Balow JE, Magrath IT, Poplack DG, Zeigler JL. Acute tumor
lysis syndrome: A review of 37 patients with Burkitts Lymphoma. Am J
1. Cairo MS, Bishop M. Tumor lysis syndrome: New therapeutic strategies Med 68: 486491, 1980
and classication. Br J Haematol 127: 311, 2004 24. Iversen U, Iversen OH, Bluming AZ, Zeigler JL, Kyalwasi S. Cell kinetics
2. Howard SC, Jones DP, Pui C. The tumor lysis syndrome. N Engl J Med of African cases of Burkitt lymphoma. A preliminary report. Eur J Cancer
364: 18441854, 2011 8: 305308, 1972
3. Flombaum CD. Metabolic emergencies in the cancer patient. Semin 25. Jasek AM, Day HJ. Acute spontaneous tumor lysis syndrome. Am J
Oncol 27: 322334, 2000 Hematol 47: 129131, 1994
4. Hande KR, Garrow GC. Acute tumor lysis syndrome in patients with 26. Mato AR, Riccio BE, Qin L, Heitjan DF, Carroll M, Loren A, Porter DL,
high-grade non-Hodgkins lymphoma. Am J Med 94: 133139, 1993 Perl A, Stadtmauer E, Tsai D, Gewirtz A, Luger SM. A predictive model

6 Onco-Nephrology Curriculum American Society of Nephrology


for the detection of tumor lysis syndrome during AML induction ther- 43. Lieber C, Jones D, Losowsky M, Davidson CS. Interrelation of uric acid
apy. Leuk Lymphoma 47: 877883, 2006 and ethanol metabolism in man. J Clin Invest 41: 18631870, 1962
27. Annemans L, Moeremans K, Lamotte M, Garcia Conde J, van den Berg 44. Jones DP, Mahmoud H, Chesney W. Tumor lysis syndrome: Patho-
H, Myint H, Pieters R, Uyttebroeck A. Incidence, medical resource uti- genesis and management. Pediatr Nephrol 9: 206212, 1995
lization and costs of hyperuricemia and tumour lysis syndrome in pa- 45. Perry Wilson F, Berns JS. Tumor lysis syndrome: New challenges and
tients with acute leukemia and non-Hogkins lymphoma in four recent advances. Adv Chronic Kidney Dis 21: 1826, 2014
European countries. Leuk Lymphoma 44: 7783, 2003 46. Krakoff IH, Meyer RL. Prevention of hyperuricemia in leukemia and
28. Wssman W, Schrappe M, Meyer U, Zimmermann M, Reiter A. In- lymphoma: Use of allopurinol, a xanthine oxidase inhibitor. JAMA 193:
cidence of tumor lysis syndrome in children with advanced stage 16, 1965
Burkitts lymphoma/leukemia before and after introduction of pro- 47. LaRosa C, McMullen L, Bakdash S, Ellis D, Krishnamurti L, Wu HY, Moritz
phylactic use of urate oxidase. Ann Hematol 82: 160165, 2003 ML. Acute renal failure from xanthine nephropathy during manage-
29. Tosi P, Barosi G, Lazzaro C, Lis V, Marchetti M, Morra E, Pession A, ment of acute leukemia. Pediatr Nephrol 22: 132135, 2007
Rosti G, Santoro A, Zinzani PL, Tura S. Consensus conference on the 48. Smalley RV, Guaspari A, Haase-Statz S, Anderson SA, Cederberg D,
management of tumor lysis syndrome. Haematologica 93: 1877 Hohneker JA. Allopurinol: Intravenous use for prevention and treat-
1885, 2008 ment of hyperuricemia. J Clin Oncol 18: 17581763, 2000
30. Montesinos P, Lorenzo I, Martn G, Sanz J, Prez-Sirvent ML, Martnez 49. Arellano F, Sacristan JA. Allopurinol hypersensitivity syndrome: A re-
D, Ort G, Algarra L, Martnez J, Moscard F, de la Rubia J, Jarque I, view. Ann Pharmacother 27: 337343, 1993
Sanz G, Sanz MA. Tumor lysis syndrome in patients with acute myeloid 50. Spina M, Nagy Z, Ribera JM, Spina M, Nagy Z, Ribera FM, Federico M,
leukemia: Identication of risk factors and development of a predictive Aurer I, Jordan K, Borsaru G, Pristupa AS, Bosi A, Grosicki S, Glushko
model. Haematologica 93: 6774, 2008 NL, Ristic D, Jakucs J, Montesinos P, Mayer J, Rego EM, Baldini S,
31. Kushner BH, LaQuaglia MP, Modak S, Cheung NK. Tumor lysis Scartoni S, Capriati A, Maggi CA, Simonelli C. A randomized double-
syndrome, neuroblastoma, and correlation between lactate dehy- blind phase III pivotal study of febuxostat versus allopurinol in the pre-
drogenase levels and MYCN-amplication. Med Pediatr Oncol 41: vention of tumor lysis syndrome: Florence study. J Clin Oncol 32: 5s, 2014
8082, 2003 51. Elitek (Rasburicase) Package Label. Available at: http://www.accessdata.
32. Fenaux P, Lai JL, Miaux O, Zandecki M, Jouet JP, Bauters F. Burkitt cell fda.gov/drugsatfda_docs/label/2009/103946s5083lbl.pdf. Accessed
acute leukemia (L3 ALL) in adults: A report of 18 cases. Br J Haematol April 30, 2015
71: 371376, 1989 52. Brogard JM, Coumaros D, Franckhauser J, Stahl A, Stahl J. Enzymatic
33. Seftel MD, Bruyere H, Copland M, Hogge DE, Horsman DE, Nantel SH, uricolysis: A study of the effect of a fungal urate-oxydase. Rev Eur
Shepherd JD, Lavoie JC, Le A, Sutherland HJ, Toze CL, Nevill TJ. Etudes Clin Biol 17: 890895, 1972
Fulminant tumour lysis syndrome in acute myelogenous leukemia with 53. Jeha S, Kantarjian H, Irwin D, Shen V, Shenoy S, Blaney S, Camitta B, Pui
inv(16)(p13;q22). Eur J Haematol 69: 193199, 2002 CH. Efcacy and safety of rasburicase (ElitekTM), in the management of a
34. Enomoto A, Kimura H, Chairoungdua A, Shigeta Y, Jutabha P, Cha SH, malignancy-associated hyperuricemia in pediatric and adult patients: Final
Takeda M, Sekine T, Igarashi T, Matsuo H, Kikuchi Y, Oda T, Ichida K, results of a multicenter compassionate use trial. Leukemia 19: 3438, 2005
Hosoya T, Shimokata K, Niwa T, Kanai Y, Endou H. Molecular identi- 54. Pui C, Mahmoud HH, Wiley JM, Woods GM, Leverger G, Camitta B,
cation of a renal urate-anion exchanger that regulates blood urate Hastings C, Blaney SM, Relling MV, Reaman GH. Recombinant urate
levels. Nature 417: 447452, 2002 oxidase for the prophylaxis or treatment of hyperuricemia in patients
35. Umekawa T, Chegini N, Khan SR. Increased expression of monocyte with leukemia or lymphoma. J Clin Oncol 19: 697704, 2001
chemoattractant protein-1 (MCP-1) by renal epithelial cells in culture on 55. Cortes J, Moore JO, Maziarz RT, Wetzler M, Craig M, Matous J, Luger S,
exposure to calcium oxalate, phosphate and uric acid crystals. Nephrol Dey BR, Schiller GJ, Pham D, Abboud CN, Krishnamurthy M, Brown A Jr,
Dial Transplant 18: 664669, 2003 Laadem A, Seiter K. Control of plasma uric acid in adults at risk for tumor
36. Kim YG, Huang XR, Suga S, Mazzali M, Tang D, Metz C, Bucala R, lysis syndrome: Efcacy and safety of rasburicase alone and rasburicase
Kivlighn S, Johnson RJ, Lan HY. Involvement of macrophage migration followed by allopurinol compared with allopurinol aloneResults of a
inhibitory factor (MIF) in experimental uric acid nephropathy. Mol Med multicenter phase III study. J Clin Oncol 28: 42074213, 2010
6: 837848, 2000 56. Sood AR, Burry LD, Cheng DKF. Clarifying the role of rasburicase in
37. Kang DH, Park SK, Lee IK, Johnson RJ. Uric acid-induced C-reactive tumor lysis syndrome. Pharmacotherapy 27: 111121, 2007
protein expression: Implication on cell proliferation and nitric acid pro- 57. Sonbol MS, Yadav H, Vaidya R, Rana V, Witzig TE. Methemoglobinemia
duction of human vascular cells. J Am Soc Nephrol 16: 35533562, 2005 and hemolysis in a patient with G6PD deciency treated with rasburi-
38. Han HJ, Lim MJ, Lee YJ, Lee JH, Yang IS, Taub M. Uric acid inhibits renal case. Am J Hematol 88: 152154, 2013
proximal tubule cell proliferation via at least two signaling pathways 58. Bontant T, Le Garrac S, Avran D, Dauger S. Methaemoglobinaemia in a
involving PKC, MAPK, cPLA2 and NF-kappaB. Am J Physiol Renal G6PD-decient child treated with rasburicase. BMJ Case Rep 2014
Physiol 292: F373F381, 2007 59. Agha-Razii M, Amyot SL, Pichette V, Cardinal J, Ouimet D, Leblanc M.
39. Feig DI, Kang DH, Johnson RJ. Uric acid and cardiovascular risk. N Engl Continuous veno-venous hemodialtration for the treatment of spon-
J Med 359: 18111821, 2008 taneous tumor lysis syndrome complicated by acute renal failure and
40. Davidson MB, Thakkar S, Hix JK, Bhandarkar ND, Wong A, Schreiber severe hyperuricemia. Clin Nephrol 54: 5963, 2000
MJ. Pathophysiology, clinical consequences, and treatment of tumor 60. Sakarcan A, Quigley R. Hyperphosphatemia in tumor lysis syndrome:
lysis syndrome. Am J Med 116: 546554, 2004 The role of hemodialysis and continuous veno-venous hemoltration.
41. Tungsanga K, Boonwichit D, Lekhakula A, Setprija V. Urine uric acid and Pediatr Nephrol 3: 351353, 1994
urine creatinine ratio in acute renal failure. Arch Intern Med 144: 934 61. Darmon M, Guichard I, Vincent F, Schlemmer B, Azoulay E. Prognostic
937, 1984 signicance of acute renal injury in acute tumor lysis syndrome. Leuk
42. Cairo MS, Gerrard M, Sposto R, Auperin A, Pinkerton CR, Michon J, Lymphoma 51: 221227, 2010
Weston C, Perkins SL, Raphael M, McCarthy K, Patte C; FAB LMB96 62. Hummel M, Buchheidt D, Reiter S, Bergmann J, Adam K, Hehlmann R.
International Study Committee. Results of a randomized international Recurrent chemotherapy-induced tumor lysis lysis syndrome (TLS) with
study of high-risk central nervous system B non-Hodgkin lymphoma renal failure in a patient with chronic lymphocytic leukemia: Successful
and B acute lymphoblastic leukemia in children and adolescents. Blood treatment and prevention of TLS with low-dose rasburicase. Eur J
109: 27362743, 2007 Haematol 75: 518521, 2005

American Society of Nephrology Onco-Nephrology Curriculum 7


REVIEW QUESTIONS d. Hypernatremia
e. Hypocalcemia
1. Which of the following cancers are considered high risk for
tumor lysis syndrome? Answer: e is correct. As show in Table 2, laboratory TLS is
dened by two or more abnormalities in serum electrolytes.
a. Lung cancer
These include a 25% increase from baseline in phosphorous,
b. Lung cancer and patient has AKI
potassium, or uric acid or a 25% decrease from baseline in cal-
c. Burkitt-type lymphoma, advanced stage
cium. Thus, answers a, b, and c are incorrect. Serum sodium
d. Adult T-cell lymphoma and normal LDH
concentration is not directly affected in TLS; therefore, answer
e. ALL with WBC ,100 3 109/L and LDH ,2 3 ULN
d is incorrect.
Answer: c is correct. As shown in Table 2, the risk of TLS
3. Rasburicase is part of the treatment regimen for tumor lysis
depends on type of malignancy, stage or extent of disease, and
syndrome because
presence/absence of renal disease. Burkitt-type lymphoma
that is in an advanced stage confers a high risk of TLS. Solid a. It increases urinary alkalinization
tumors such as lung cancer are considered low risk, and the b. It improves the ability of proximal tubular cells to recover
presence of renal failure raises that to intermediate risk. Thus, from AKI
answers a and b are incorrect. Adult T-cell lymphoma is con- c. It stimulates the URAT1 transporter to increase uptake of
sidered low risk if LDH is normal and acute lymphoblastic uric acid from the tubular lumen
leukemia with WBC ,100 3 109/L is considered intermediate d. It catalyzes the conversion of uric acid into allantoin
risk if LDH ,2 3 ULN. e. It prevents xanthine crystal deposition in tubular lumens

2. Which of the following electrolyte abnormalities dene Answer: d is correct. Rasburicase, as shown in Figure 3, is
laboratory TLS? recombinant urate oxidase that enzymatically transforms uric
acid into allantoin. It has no known effect on urine pH, renal
a. Hypokalemia
tubular cells, URAT1 transporters, or xanthine crystals. Thus,
b. Hypercalcemia
answers ac and e are incorrect.
c. Hypophosphatemia

8 Onco-Nephrology Curriculum American Society of Nephrology


Chapter 5: Electrolyte and AcidBase Disorders in
Malignancy
Anushree C. Shirali, MD
Section of Nephrology, Yale University School of Medicine, New Haven, Connecticut

INTRODUCTION leukemia, the prevalence of hyponatremia is only


10%, whereas the prevalence of hypokalemia
Renal complications in cancer patients include AKI, ranges between 43% and 64% (3). This suggests
hypertension, or electrolyte and acidbase disorders. that differences in pathophysiologic mechanisms
Of the latter, there are various types that share the may drive unique electrolyte disorders in different
ability to increase morbidity and mortality, delay malignancies. In the next sections, the clinical fea-
treatment, and decrease quality of life. Understand- tures, pathophysiology, and treatment of the most
ing the etiology of electrolyte and acidbase abnor- common electrolyte and acidbase disorders in can-
malities in cancer patients is critical to prompt cer patients will be considered.
recognition and appropriate treatment so that these
complications may be avoided. This section of the Hyponatremia
Onco-Nephrology Curriculum will review the path- Cancer is a common etiology for hyponatremia in the
ophysiology, clinical presentation, and management hospitalized patient, accounting for 14% of cases in a
of electrolyte and acidbase abnormalities in patients prospective observational cohort (4). Similar to re-
with malignancies. Specically, disturbances in the ports on hyponatremia in the general population,
following will be reviewed: 1) electrolytes: disorders lower serum sodium concentration is associated
of sodium, potassium, calcium, magnesium, and with increased hospital length of stay and 90-day
phosphorous; and 2) acidbase: metabolic acidosis. mortality (1). In patients with small-cell lung cancer
(SCLC), in those who had hyponatremia prior to che-
motherapy initiation, failure to achieve normonatremia
BACKGROUND, EPIDEMIOLOGY, AND
within the rst two cycles of chemotherapy was a
SPECIFIC DISORDERS
predictive marker for decreased survival (5).
Hyponatremia associated with cancer may have
Electrolyte and acidbase disturbances are com-
several potential etiologies (Table 1). Regardless of the
mon in cancer patients, either due to the malig-
etiology, patients may be asymptomatic with mild to
nancy or treatment of the malignancy. For
example, a patient may develop metabolic acidosis moderate disease but may experience headache, fa-
tigue, and mental status changes with moderate to
from lactate produced by disseminated lympho-
severe hyponatremia. Examination ndings such as
ma or from chemotherapy-induced diarrhea.
frank or orthostatic hypotension in volume depletion
Published statistics are not robust for each type of
or edema in the third-spacing states of cirrhosis may
electrolyte or acidbase disorder, but there are data
point to potential causes. In conjunction with exam-
on those associated with greater morbidity or mor-
ination data, urine studies are indispensable, with
tality, such as hyponatremia. In one such analysis of
urine sodium ,20 mEq/L reecting the sodium avid-
cancer-related admissions, 47% of patients with
mostly solid tumors had hyponatremia, and of ity of volume depletion and urine sodium .40 mEq/L
suggesting the syndrome of inappropriate antidiuretic
these, 11% had moderate (sodium [Na] 120129
mEq/L) to severe (Na , 120 mEq/L) hyponatremia
(1). This is disproportionately higher than the
hyponatremia prevalence rates of 15%30% Correspondence: Anushree Shirali, Section of Nephrology, Yale
reported for general medicine admissions (2). In University School of Medicine, PO Box 208029, New Haven,
Connecticut 06520-8029.
cancer patients with nonsolid tumors, rates of
hyponatremia are less. For example, in acute Copyright 2016 by the American Society of Nephrology

American Society of Nephrology Onco-Nephrology Curriculum 1


Table 1. Common mechanisms for hyponatremia in the cancer patient
Etiology of hyponatremia Clinical examples specic to the cancer patient
Pseudohyponatremia Paraproteinemias
Reduced water excretion Underlying CKD or AKI
Decreased circulating volume Nausea, vomiting, nasogastric suctioning, and diarrhea;
hematemesis or hematochezia (gastrointestinal malignancies
or steroid-induced ulcer disease)
Decreased effective circulating volume Underlying or new onset of CHF, cirrhosis, ascites, severe
hypoalbuminemia, veno-occlusive disease
SIADH Tumor release of ADH: SCLC and head and neck cancer
Chemotherapy: cyclophosphamide, cisplatin/carboplatin, vincristine,
vinblastine
Other drugs: SSRIs, NSAIDs
Nonosmotic stimuli for ADH Pain, nausea, vomiting
Salt wasting Cisplatin
ADH, antidiuretic hormone; CHF, congestive heart failure; SIADH, syndrome of inappropriate ADH secretion; NSAIDs, nonsteroidal anti-inammatory drugs; SCLC,
small cell lung cancer; SSRI, selective serotonin reuptake inhibitors.

hormone secretion (SIADH) in a euvolemic patient or rarely salt or improve cognitive testing (9). In addition, chronic tolvaptan
wasting due to cisplatin therapy. use may be limited by expense and cumulative dose-dependent
Although patients with cancer have hyponatremia due to hepatotoxicity (10).
many etiologies (Table 1), SIADH is the most common etiol-
ogy that is directly attributable to cancer. This is because can- Hypokalemia
cer patients have nonvolume and nonosmotic stimuli for Similar to hyponatremia, hypokalemia is commonly encoun-
antidiuretic hormone (ADH) release, such as nausea/vomiting tered in cancer patients, resulting from cancer-distinct and
and pain or medications such as cyclosphosphamide. An ad- cancer-specic causes (Table 2) and, more commonly, from a
ditional factor is the paraneoplastic release of ADH from tu- combination of the two. Proper diagnosis starts with excluding
mor subtypes, notably SCLC and head and neck cancer (6). pseudohypokalemia from postphlebotomy transcellular
Finally, several chemotherapy drugs have been linked to shifts, which is seen in patients with profound leukocytosis
hyponatremia via potentiation of ADH release or action, in- whose blood samples are not refrigerated or immediately an-
cluding vinblastine, vincristine, and cyclophosphamide (7). alyzed. Once true hypokalemia is conrmed, measurement of
Cisplatin is another antineoplastic agent linked to urine potassium and the trans-tubular potassium gradient can
hyponatremia through a mechanism that involves salt wasting be helpful in analyzing renal potassium wasting (11).
at the loop of Henle (8). Ten percent of patients on cisplatin In cancer-distinct causes, chemotherapy leads to hypoka-
therapy at a single center developed hyponatremia with a high lemia either indirectly via side effects of decreased appetite/
urine sodium concentration but with profound volume de- intake, vomiting, and diarrhea or directly via renal tubular
pletion. Thus, these patients required volume expansion with effects. For example, ifosfamide causes renal potassium
saline to correct their hyponatremia, in contrast to patients wasting, either as an isolated proximal tubulopathy or Fanconi
with SIADH who are volume replete. This underscores the syndrome (FS), which may persist after treatment. Fifteen
need for correct etiologic diagnosis of hyponatremia to pro- percent of pediatric cancer patients who received ifosfamide
vide appropriate therapy. Therapy must be tailored for the therapy exhibited persistent hypokalemia months to years after
patient, the underlying diagnosis, and the severity of hypona- the end of treatment (12).
tremia. Severe hyponatremia with serum Na concentration Cancer-specic causes of hypokalemia include tumors that
,110 mEq/L and neurologic symptoms may need 3% hyper- secrete ectopic adrenocorticotropin hormone (ACTH) such as
tonic saline for acute management. Fluid restriction is the SCLC, thymus or bronchial carcinoid, thyroid medullary
mainstay of treatment for SIADH, with salt tablets and loop carcinoma, or neuroendocrine tumors (13). Although un-
diuretics as adjunctive therapy. However, these measures can common, these tumors stimulate renal potassium wasting
hinder quality of life in the cancer patient, and thus, aquaretics via excessive cortisol release that activates the mineralocorticoid
that inhibit the vasopressin type 2 receptor (V2-R) to inhibit pathway. Accordingly, other features of hypercortisolemia
water reabsorption in the collecting duct are suggested for are also present including pigmented skin, diabetes, and hy-
management of hyponatremia secondary to SIADH in cancer pertension (13). Another cancer-specic etiology for hypoka-
patients if other therapies are not feasible or effective. How- lemia is evident in acute myeloid leukemia (AML), M4 and M5
ever, the data on their clinical utility in cancer patients are sparse. subtypes, which has been long associated with hypokalemia
In a small, single center safety and efcacy study, tolvaptan, a (14,15). These malignancies increase serum lysozyme
V2-R antagonist, was superior to placebo in the correction of and lysozymuria, leading to the hypothesis that lysozyme-
hyponatremia but did not decrease hospital length of stay (LOS) mediated tubular injury leaks potassium (and other electrolytes)

2 Onco-Nephrology Curriculum American Society of Nephrology


Table 2. Cancer-distinct and cancer-specic causes of hypokalemia in the patient with malignancy
Etiology of hyponatremia Cancer distinct Cancer specic
Pseuodohypokalemia Phlebotomy error with tight tourniquet Clonal leukocytosis with leukemias
Redistribution into cells Use of GM-CSF, vitamin B12 Blast crisis with leukemias
Poor intake Anorexia, nausea, mucositis Tumor-induced dysphagia
Extrarenal losses Vomiting, diarrhea from chemotherapy or radiation enteritis VIPoma, villous adenoma (rare)
Renal losses Hypomagnesemia, Fanconi syndrome with chemotherapy Lysozymuria, Fanconi syndrome from light chain
injury (myeloma), ectopic ACTH production
ACTH, adrenocorticotropin hormone; GM-CSF, granulocyte macrophage colony stimulating factor; VIP, vasoactive intestinal peptide.

into urine. Another putative mechanism may be renin-like panitumumab, display tumoricidal activity against a variety of
activity by AML blast cells stimulating the mineralocorticoid cancers, but they also prevent the insertion of a magnesium
pathway (16). channel, transient receptor potential M6 (TRPM6), into the
The potassium losses in these cases may be profound and apical membrane of distal tubular cells (Figure 1) (18). As a re-
require aggressive replacement. The choices for replacement sult, magnesium cannot be reabsorbed from the tubular lumen
are the same as those utilized for hypokalemia in the noncancer and serum magnesium levels fall, affecting 10%36% of patients
patient (11), but it should be noted that given the difculty in early clinical trials of cetuximab (7). A fractional excretion of
cancer patients may have with oral intake due to nausea, magnesium .15% in a hypomagnesemic patient indicates renal
mucositis, etc., intravenous dosing is often necessary. Hypo- wasting. Treatment involves replacing magnesium, and intrave-
kalemia treatment is also ineffective if hypomagnesemia re- nous dosing is usually needed because diarrhea is a dose-limiting
mains uncorrected, due to unchecked potassium losses via the adverse effect of oral magnesium. Fortunately, renal magnesium
renal outer medullary K1 channel (ROMK) channel in distal wasting subsides over time following discontinuation of the
nephron tubular cells (17). EGFR antagonist. However, this is not the case with the platin
drugs, where renal magnesium wasting can be permanent.
Hypomagnesemia in cancer patients
Hypomagnesemia in the cancer patient may be due to decreased Hypercalcemia
intake or from renal magnesium wasting. Renal losses of Twenty percent to 30% of cancer patients experience hypercal-
magnesium are principally due to chemotherapy-mediated cemia during the course of their malignancy (19), and this is
injury to the distal nephron, the site of active magnesium predictive of poor prognosis (20). Hypercalcemia of malignancy
reabsorption in the nephron. This has been noted with cisplatin, uses one of two mechanisms: 1) osteolytic release of local cal-
but a rising number of cases are being attributed to drugs that cium from bone directly involved by cancer cells or 2) stim-
target the epidermal growth factor receptor (EGFR) pathway. ulation of osteoclast activity by release of the tumor-derived
Monoclonal antibodies against EGFR, such as cetuximab and endocrine factors. Although these mechanisms are distinct,

Figure 1. Absorption of magnesium from the tubular lumen is via an EGFR-dependent apical channel, TRPM6. This pathway is
inhibited by use of anti-EGFR monoclonal antibodies such cetuximab. EGF, epidermal growth factor; EGFR, epidermal growth factor
receptor; TRPM6, transient receptor potential M6. Red line denotes inhibition of interaction.

American Society of Nephrology Onco-Nephrology Curriculum 3


the resultant hypercalcemia in either case may be mild and malnutrition from anorexia or malnutrition causing poor intake,
asymptomatic, moderate and accompanied by nausea/vomiting, or it may be the result of renal phosphate wasting from drug-
constipation, bone pain, and fatigue, or severe and manifested by induced proximal tubulopathy and FS. As mentioned previously,
confusion and coma (21). It is important to correct serum cal- FS is common with ifosfamide, but has also been associated with
cium concentration for hypoalbuminemia so that hypercalcemia cisplatin and imatinib use (32,33). A fractional excretion of phos-
levels can be properly graded. phate that is .5% in the setting of hypophosphatemia is diagnos-
Among solid tumors, primary bone cancers and metastatic tic of renal phosphate wasting. Treatment of hypophosphatemia
breast or prostate cancer stimulate osteolysis, which correlates centers on phosphate replacement, which may approach several
with the overall tumor burden. Although metastases do not grams per day in cases of renal phosphate wasting.
occur in nonsolid tumors, osteolysis may be stimulated by a A rare cause of hypophosphatemia is tumor-induced osteo-
variety of immune and nonimmune pathways in multiple malacia, whose mechanism is dependent on the phosphatonin,
myeloma. Both result in release of sequestered calcium from broblast growth factor 23 (FGF-23). The role of the FGF-23
bone with the common pathway centered on the interaction pathway has been detailed previously (6,34). Briey, FGF-23 is a
between receptor activator of nuclear factor-kB (RANK), phosphaturic agent whose expression is tightly regulated by
which is present on osteoclasts and their precursors, and phosphate, 1,25(OH)2D, and other factors. In tumor-induced
RANK ligand (RANKL), which is present on osteoblast and osteomalacia, constitutive release of FGF-23 without usual
bone marrow stromal cell surfaces (22). The putative mecha- regulatory checkpoints leads to persistent FGF-23 activation,
nism involves RANKL binding to its cognate receptor RANK resulting in severe phosphaturia, hypophosphatemia, and oste-
through the inuence of parathyroid hormone (PTH) and omalacia. Several malignancies are associated with this syndrome
PTH-related peptide (PTHRP), which subsequently increases including hemangiopericytomas, giant cell tumors, and osteo-
osteoclastic activity and release of local calcium (21). blastomas (34). Denitive treatment is surgical resection, as the
Tumor-derived endocrine factors are responsible for the phosphate wasting may be so profound that medical manage-
humoral hypercalcemia of malignancy, including PTHRP and ment may not be sufcient. Thus, functional imaging such as
1,25-dihydroxyvitamin D [1,25(OH)2D]. More rarely, there is F-18 uorodeoxyglucose positron emission tomography is
PTH release from primary parathyroid carcinoma (23) or ovarian suggested for diagnosis, which has high sensitivity for these
cancer (24). PTHRP is most commonly secreted by squamous cell tumors but may not be specic (34).
carcinoma of the lung or head and neck, but renal cell, ovarian,
breast, and esophageal cancers have all been associated with hy- Metabolic acidosis in cancer patients
percalcemia from PTHRP release (21). 1,25(OH)2D, however, is Anion gap (AG) acidosis and nonanion gap (NAG) acidosis is
more likely to be secreted by lymphoma cells or tumor-associated prevalent in cancer patients. Among the various AG acidosis
macrophages that possess inherent 1-a-hydroxylase activity that disorders, lactic acidosis (LA) is the most cancer specic. Can-
is not subject to regulation by PTH (2527). cer patients may have type A LA due to tissue hypoxia from
Treatment of hypercalcemia of malignancy is focused on sepsis or cardiac failure, but they may also have type B LA with
increasing urinary calcium excretion and suppression of the no evidence for tissue ischemia. Type B LA is well described in
calcium source. The rst objective is achieved by volume leukemias and lymphomas (35), but other reported cases in-
expansion with saline to drive urinary calcium excretion. clude multiple myeloma, gastric cancer, and breast cancer
Furosemide, once routinely touted as an adjunct to saline, has (3638). The pathophysiology of malignancy-associated LA
no proven benet and should only be reserved for cases of volume is unclear, but speculative mechanisms include anaerobic
overload (28). The second objective may be fullled by suppressing glycolysis by tumor cells, stimulation of lactate production by
osteoclast activity through use of bisphosphonates such as tumor-derived cytokines, and thiamine deciency (36). Treat-
zoledronate or pamidronate. The former causes acute renal ment involves control of tumor burden. Bicarbonate infusion
tubular injury, and the latter has been linked to a collapsing may be necessary for critical drops in serum pH, but paradox-
variant of FSGS, and high intravenous (IV) dosing should be ically may stimulate more lactate production. Dialysis is often
used with caution, particularly with preexisting CKD. An requested for lactic acidosis, but clearance with either inter-
emerging option that directly targets a pathway in hypercalce- mittent or continuous dialysis modalities is insufcient to
mia of malignancy is the use of RANKL inhibitors such as overcome ongoing production.
denosumab. These agents have shown to be superior to Non-AG acidosis in cancer patients is most likely related
bisphosphonates in the treatment of skeletal related events in to infection or therapy-related diarrhea, but renal tubular
cancers with bony metastases (29,30), and early evidence sug- acidosis (RTA) should be considered. Tubular injury from
gests they are useful in the treatment of hypercalcemia of chemotherapy can cause RTA either in isolation or as part of
malignancy, particularly in bisphosphonate-resistant cases (31). the FS. Light chainassociated tubular injury in multiple my-
eloma is another cause of FS and can present with RTA. Bicar-
Hypophosphatemia in cancer patients bonate supplementation is sometimes necessary in patients
Cancer patients usually experience hypophosphatemia as a con- with RTAs, and its success depends on the degree of renal bi-
sequence of chemotherapy. This may be due to generalized carbonate wasting.

4 Onco-Nephrology Curriculum American Society of Nephrology


Other disorders placebo-controlled clinical trial on efcacy and safety. Cancer 120:
Cancer patients may have electrolyte and acidbase abnor- 744751, 2014
10. Administration USFaD. FDA Drug Safety Communication: FDA limits
malities beyond those reviewed in this chapter. In particular,
duration and usage of Samsca (tolvaptan) due to possible liver injury
hyperkalemia, hyperphosphatemia, and hypocalcemia are di- leading to organ transplant or death 2013. Available from: http://www.
agnostic criteria for tumor lysis syndrome, which is detailed fda.gov/Drugs/DrugSafety/ucm350062.htm
in Chapter 4 of the ASN Onco-Nephrology Curriculum. In 11. Unwin RJ, Luft FC, Shirley DG. Pathophysiology and management of
acidbase disorders, metabolic alkalosis may accompany the hypokalemia: a clinical perspective. Nat Rev Nephrol 7: 7584, 2011
12. Skinner R, Cotterill SJ, Stevens MC. Risk factors for nephrotoxicity after
rare renin-producing tumor but is more common with per-
ifosfamide treatment in children: a UKCCSG Late Effects Group study.
sistent vomiting or diuretic use. Respiratory alkalosis may United Kingdom Childrens Cancer Study Group. Br J Cancer 82:
occur with pontine tumor or infection-associated stimulation 16361645, 2000
of central respiratory centers. 13. Alexandraki KI, Grossman AB. The ectopic ACTH syndrome. Rev
Endocr Metab Disord 11: 117126, 2010
14. Perazella MA, Eisen RN, Frederick WG, Brown E. Renal failure and se-
vere hypokalemia associated with acute myelomonocytic leukemia.
CONCLUSION Am J Kidney Dis 22: 462467, 1993
15. Muggia FM, Heinemann HO, Farhangi M, Osserman EF. Lysozymuria
A myriad of cancer and chemotherapy-related electrolyte and renal tubular dysfunction in monocytic and myelomonocytic
and acid-base disorders can affect cancer patients. Although leukemia. Am J Med 47: 351366, 1969
16. Wulf GG, Jahns-Streubel G, Strutz F, Basenau D, Hufner M, Buske C,
most patients develop mild disease, some patients may
Wormann B, Hiddemann W. Paraneoplastic hypokalemia in acute myeloid
experience signicant morbidity. Diagnosing electrolyte leukemia: a case of renin activity in AML blast cells. Ann Hematol 73:
and acidbase abnormalities and initiating treatment quickly 139141, 1996
is essential for the nephrologist seeking to improve the out- 17. Huang CL, Kuo E. Mechanism of hypokalemia in magnesium de-
comes of cancer patients. ciency. JASN 18: 26492652, 2007
18. Groenestege WM, Thebault S, van der Wijst J, van den Berg D,
Janssen R, Tejpar S, van den Heuvel LP, van Cutsem E, Hoenderop JG,
TAKE HOME POINTS Knoers NV, Bindels RJ. Impaired basolateral sorting of pro-EGF causes
isolated recessive renal hypomagnesemia. J Clin Invest 117:
c Electrolyte and acidbase abnormalities occur frequently in the cancer pa- 22602270, 2007
tient and contribute to poor quality of life. 19. Stewart AF. Clinical practice. Hypercalcemia associated with cancer.
c Disturbances in electrolyte and acidbase homeostasis may occur due to the N Engl J Med 352: 373379, 2005
cancer itself or due to adverse effects of therapy. 20. Ralston SH, Gallacher SJ, Patel U, Campbell J, Boyle IT. Cancer-
c Treatment of electrolyte and acidbase disorders in cancer should be etiology associated hypercalcemia: morbidity and mortality. Clinical experience
specic and patient centered. in 126 treated patients. Ann Intern Med 112: 499504, 1990
21. Rosner MH, Dalkin AC. Onco-nephrology: the pathophysiology
and treatment of malignancy-associated hypercalcemia. CJASN 7:
17221729, 2012
REFERENCES
22. Papadopoulou EC, Batzios SP, Dimitriadou M, Perifanis V, Garipidou V.
1. Doshi SM, Shah P, Lei X, Lahoti A, Salahudeen AK. Hyponatremia in Multiple myeloma and bone disease: pathogenesis and current ther-
Q:1 hospitalized cancer patients and its impact on clinical outcomes. Am J apeutic approaches. Hippokratia 14: 7681, 2010
Kidney Dis 59: 222228, 2012 23. Collins MT, Skarulis MC, Bilezikian JP, Silverberg SJ, Spiegel AM, Marx
2. Verbalis JG, Goldsmith SR, Greenberg A, Korzelius C, Schrier RW, SJ. Treatment of hypercalcemia secondary to parathyroid carcinoma
Sterns RH, Thompson CJ. Diagnosis, evaluation, and treatment of with a novel calcimimetic agent. J Clin Endocrinol Metab 83:
hyponatremia: expert panel recommendations. Am J Med 126[Suppl 1]: 10831088, 1998
S1S42, 2013 24. Nussbaum SR, Gaz RD, Arnold A. Hypercalcemia and ectopic secretion
3. Filippatos TD, Milionis HJ, Elisaf MS. Alterations in electrolyte equi- of parathyroid hormone by an ovarian carcinoma with rearrangement
librium in patients with acute leukemia. Eur J Haematol 75: 449460, of the gene for parathyroid hormone. N Engl J Med 323: 13241328,
2005 1990
4. Gill G, Huda B, Boyd A, Skagen K, Wile D, Watson I, van Heyningen C. 25. Maletkovic J, Isorena JP, Palma Diaz MF, Korenman SG, Yeh MW.
Characteristics and mortality of severe hyponatraemiaa hospital- Multifactorial hypercalcemia and literature review on primary hyper-
based study. Clin Endocrinol 65: 246249, 2006 parathyroidism associated with lymphoma. Case Rep Endocrinol 2014:
5. Hansen O, Sorensen P, Hansen KH. The occurrence of hyponatremia in SCLC 893134, 2014
and the inuence on prognosis: a retrospective study of 453 patients treated 26. Evans KN, Taylor H, Zehnder D, Kilby MD, Bulmer JN, Shah F, Adams
in a single institution in a 10-year period. Lung cancer 68: 111114, 2010 JS, Hewison M. Increased expression of 25-hydroxyvitamin D-1alpha-
6. Rosner MH, Dalkin AC. Electrolyte disorders associated with cancer. hydroxylase in dysgerminomas: a novel form of humoral hypercalcemia
Adv Chronic Kidney Dis 21: 717, 2014 of malignancy. Am J Pathol 165: 807813, 2004
7. Shirali AC, Perazella MA. Tubulointerstitial injury associated with che- 27. Hewison M, Kantorovich V, Liker HR, Van Herle AJ, Cohan P, Zehnder
motherapeutic agents. Adv Chronic Kidney Dis 21: 5663, 2014 D, Adams JS. Vitamin D-mediated hypercalcemia in lymphoma: evi-
8. Hutchison FN, Perez EA, Gandara DR, Lawrence HJ, Kaysen GA. Renal dence for hormone production by tumor-adjacent macrophages.
salt wasting in patients treated with cisplatin. Ann Intern Med 108: J Bone Miner Res 18: 579582, 2003
2125, 1988 28. LeGrand SB, Leskuski D, Zama I. Narrative review: furosemide for hy-
9. Salahudeen AK, Ali N, George M, Lahoti A, Palla S. Tolvaptan in hos- percalcemia: an unproven yet common practice. Ann Intern Med 149:
pitalized cancer patients with hyponatremia: a double-blind, randomized, 259263, 2008

American Society of Nephrology Onco-Nephrology Curriculum 5


29. Fizazi K, Carducci M, Smith M, Damiao R, Brown J, Karsh L, Milecki P, 33. Francois H, Coppo P, Hayman JP, Fouqueray B, Mougenot B, Ronco P.
Shore N, Rader M, Wang H, Tadros S, Dansey R, Goessl C. Denosumab Partial fanconi syndrome induced by imatinib therapy: a novel cause of
versus zoledronic acid for treatment of bone metastases in men with urinary phosphate loss. Am J Kidney Dis 51: 298301, 2008
castration-resistant prostate cancer: a randomised, double-blind study. 34. Chong WH, Molinolo AA, Chen CC, Collins MT. Tumor-induced
Lancet 377: 1813822, 2011 osteomalacia. Endocr Relat Cancer 18: R53R77, 2011
30. Stopeck AT, Lipton A, Body JJ, Steger GG, Tonkin K, de Boer RH, 35. Friedenberg AS, Brandoff DE, Schiffman FJ. Type B lactic acidosis as a
Lichinitser M, Fujiwara Y, Yardley DA, Viniegra M, Fan M, Jiang Q, severe metabolic complication in lymphoma and leukemia: a case series
Dansey R, Jun S, Braun A. Denosumab compared with zoledronic acid from a single institution and literature review. Medicine 86: 225232, 2007
for the treatment of bone metastases in patients with advanced breast 36. Sia P, Plumb TJ, Fillaus JA. Type B lactic acidosis associated with
cancer: a randomized, double-blind study. J Clin Oncol 28: 51325739, multiple myeloma. Am J Kidney Dis 62: 633637, 2013
2010 37. de Groot R, Sprenger RA, Imholz AL, Gerding MN. Type B lactic acidosis as a
31. Hu MI, Glezerman I, Leboulleux S, Insogna K, Gucalp R, Misiorowski W, severe metabolic complication in lymphoma and leukemia: a case series
Yu B, Ying W, Jain RK. Denosumab for patients with persistent or re- from a single institution and literature review.nger RA, Imholz AL, Gerding
lapsed hypercalcemia of malignancy despite recent bisphosphonate MN. Type B lactic acidosis in solid malignancies. Neth J Med 69: 120123,
treatment. J Natl Cancer Inst 105: 14171420, 2013 2011
32. Hall AM, Bass P, Unwin RJ. Drug-induced renal Fanconi syndrome. QJM 38. Hashemi-Sadraei N, Machicado JD, Gupta R, Huapaya JA. Lactic acidosis
107: 261269, 2014 in gastric cancer. J Gastrointest Cancer 45[Suppl 1]: 192194, 2014

6 Onco-Nephrology Curriculum American Society of Nephrology


REVIEW QUESTIONS reabsorption of bicarbonate, ammonia, amino acids, glucose,
and most electrolytes including sodium, potassium, and phos-
1. Which of the following tumors is most likely to be asso- phorus, this syndrome typically results in potassium, phospho-
ciated with SIADH? rus, and bicarbonate wasting. Patients may exhibit hypokalemia,
hypophosphatemia, metabolic acidosis due to renal tubular ac-
a. Nonsmall-cell lung cancer idosis, and glucosuria. Because active magnesium reabsorption
b. Acute myeloid leukemia, M4 type takes place at the distal tubule, magnesium levels are not affected.
c. Small-cell lung cancer
d. Breast cancer
e. Bronchial carcinoid 3. Which of the following is true of the lactic acidosis of
malignancy?
Answer: c is correct. Hyponatremia in cancer patients from
persistent ADH stimulation or potentiation may be cancer a. Levels of lactic acid decrease with bicarbonate infusion
distinct, e.g., from pain, nausea, or chemotherapy. Certain b. Continuous RRT is recommended for control of lactic
cancers, however, are known to release ectopic ADH, includ- acid levels
ing small cell lung cancer and head and neck cancer. Non c. Measurement of the urinary anion gap can aid in di-
small cell lung cancer and breast cancer are not associated agnosis
with SIADH from ectopic ADH release. AML-M4 is associ- d. Control of tumor burden is not necessary in improving
ated with lysozyme-mediated renal potassium wasting, lactic acid levels
whereas bronchial carcinoid secretes ectopic adrenocortico- e. It may be the result of anaerobic glycolysis and lactate
tropin hormone. production by tumor cells

2. Which of the following laboratory abnormalities are seen Answer: e is correct. Type B lactic acidosis is the production
in Fanconi syndrome? of lactic acid in the absence of ischemia. It is linked with cer-
tain malignancies, particularly lymphoma, and hypotheses
a. Hyperkalemia for its pathophysiology include tumor-induced anaerobic gly-
b. Hypomagnesemia colysis and lactate production. Lactic acidosis of malignancy
c. Hypophosphatemia correlates well with tumor burden, and levels improve with
d. Metabolic alkalosis control of that burden. The urinary anion gap is not useful in
e. Hyperglycemia the diagnosis of this disorder. Although acute treatment of aci-
dosis may require bicarbonate, levels of lactic acid may rise with
Answer: c is correct. Fanconi syndrome is a constellation of sustained bicarbonate infusion. Last, hemodialysis is an inef-
metabolic abnormalities, which result following proximal tubule cient modality for lactic acid clearance, as production is higher
injury. As the proximal tubule is the primary site for than clearance rates, even in continuous RRT.

American Society of Nephrology Onco-Nephrology Curriculum 7


AUTHOR QUERIES

AUTHOR PLEASE ANSWER ALL QUERIES

Q1: Please verify corrections, especially to references. The end of the references were changed to match
style, so please make sure the edits are correct.
Chapter 6: Glomerular Diseases and Cancer
Divya Monga* and Kenar D. Jhaveri
*Nephrology Division, University of Mississippi Medical Center, Jackson, Mississippi; and Nephrology Division,
Northwell Health, Hofstra Northwell School of Medicine, Great Neck, New York

INTRODUCTION should be high in a patient with known cancer who


has presence of proteinuria or nephrotic syndrome.
Glomerular diseases are associated with many solid Also development of proteinuria within a year of
and hematologic malignancies. Additionally, many che- diagnosis of cancer should raise the suspicion of
motherapeutic agents and poststem cell transplant secondary form of glomerulopathy. Studies have
associated glomerular lesions have been described. reported risk factors like age .65 years and history
These glomerular lesions are most likely due to of smoking for .20 pack-years for paraneoplastic
abnormal products produced by tumor cells, al- MN (3). Review of relevant studies (37) has sug-
though the exact pathogenesis is unclear. The treat- gested certain parameters, which can help differen-
ment of these cancer-associated glomerulopathies tiate primary from secondary MN, the latter being
is primarily targeted at treating the underlying associated with cancer. These features are summa-
malignancy. This chapter will review glomerular rized in Table 1 (8).
diseases associated with cancer, chemotherapy, In addition to these ndings, one should have a
and hematopoietic stem cell transplantation high index of suspicion for malignancy when a
(HSCT). patient with MN is evaluated. It is reasonable to
perform routine age- and sex-appropriate screen-
ing for malignancy, once other known causes of
SOLID TUMORASSOCIATED secondary MN have been excluded. In patients with
GLOMERULAR DISEASES high risk of lung cancer, low-dose chest computed
tomography should be considered. The risk of
Membranous nephropathy cancer persists for $5 years from the time of kid-
Membranous nephropathy (MN) is the most com- ney biopsy (9). This is most likely due to slow-
mon glomerular pathology (Figure 1, A and B) growing malignancy, use of cytotoxic therapy for
described in patients with solid tumors (1,2). MN, or increased surveillance. Therefore, close
In a review of 240 patients with biopsy proven medical follow-up is needed even if the cancer is
MN, Lefaucheur et al. (3) reported a prevalence not detected on initial screening at the time of
of malignancy of 10%. Only about half of these MN diagnosis.
patients had symptoms related to cancer at the The possible mechanisms by which solid tumors
time of their kidney biopsy. Also, most of these may be associated with MN include the following
patients were diagnosed with malignancy within (10):
a year of MN diagnoses. Review of case series
shows a reported prevalence of as low as 1% to as 1) In situ immune complex formation: Antibodies
high as 22% (2). are formed against a tumor antigen, which is
Classically, the solid tumors most commonly localized in the sub epithelial location or to a
associated with MN are lung, bronchus, and gastric podocyte antigen that is identical or similar to
cancers, followed by renal cell, prostate, and the tumor antigen.
thymoma (2). Other cancers reported with MN
are colorectal, pancreatic, esophageal, and hepatic
carcinomas. Correspondence: Divya Monga, Division of Nephrology, Uni-
Differentiating primary MN from secondary versity of Mississippi Medical Center, 2500 N. State St., Jackson,
Mississippi 39216.
MN associated with malignancy can be difcult.
Our suspicion for a secondary glomerular disease Copyright 2016 by the American Society of Nephrology

American Society of Nephrology Onco-Nephrology Curriculum 1


Figure 1. Membranous nephropathy. (A) Light microscopy showing immune complex deposits. Note the thickened basement
membrane, which stains black while deposits within it stain pink, giving a variegated appearance to the capillary wall. Silver [periodic
acid silver methamine (PASM)] stain; 603, original magnication. (B) Electron microscopy showing immune complex deposits in a
subepithelial location, between effaced podocyte foot processes (top) and the basement membrane (bottom).

2) Tumor antigens may form circulating immune com- Older age and male sex were identied risk factors for cancer-
plexes that are subsequently trapped in glomerular associated HSP (14).
capillaries. Crescentic glomerulonephritis (CGN) has been associated
3) External factors such as infections with oncogenic viruses or with renal cell, gastric, and lung cancers (2).
altered immune function that can cause both the malig- Thrombotic microangiopathy (TMA) has been associated
nancy and MN. with mucin-producing gastric, lung, and breast cancers (16).
In these patients, ADAMTS13 activity is not impaired, and
Other glomerular diseases they respond poorly to plasmapharesis (17).
Minimal change disease (MCD) has been reported in associ- The exact mechanism of these solid tumor malignancy
ation with solid tumors like lung, colorectal, renal cell cancers, associations with glomerular disease is poorly understood.
and thymoma. Rarely, pancreatic, bladder, breast, and ovarian There have been animal studies (18) done to help us under-
cancers have also been associated (2). Focal segmental glomer- stand the pathomechanisms involved.
ular sclerosis (FSGS) has been observed with renal cell carci- This animal study suggested that T-cell response might be
noma, thymoma, and rarely with lung, breast, and esophageal critical in the development of paraneoplastic glomerular
cancers (2). A membranoproliferative glomerular nephritis disease. Th1 (T-helper type 1)-predominant responses have
(MPGN) pattern of injury has been described with lung, kid- been associated with proliferative and crescentic forms of GN
ney, and stomach cancer (2). and Th2 (T-helper type 2) type responses with a membranous
Mustonen et al. (11) reported the rst known association pattern of injury (19). Cancer-associated MCD might be related
between IgA nephropathy and solid tumors of the respiratory to vascular endothelial growth factor (VEGF) production
tract, buccal mucosa, and nasopharynx. Renal cell carcinoma (20). Overexpression of VEGF leads to a collapsing variant
is the most frequently reported solid malignancy associated of FSGS, and underexpression is associated with a TMA pattern
with IgA nephropathy (12). Treatment of underlying cancer of injury (21,22).
improved the IgA nephropathy (11).
Rarely, both solid and hematologic malignancies have been Thymoma-associated glomerular disease
associated with adult Henoch-Schnlein purpura (HSP) MCD is the most common glomerular disease associated
(13,14). Endocapillary glomerulonephritis is the most com- with thymoma (23). The prevalence of thymoma associated
monly seen lesion on kidney biopsy in adults with HSP (15). glomerulopathy is ;2% (23). Other glomerular lesions

Table 1. Differences between primary and tumor-associated secondary MN


Compared feature Primary MN Solid tumorassociated MN
History Younger age, no history of smoking Age .65 years, smoking .20 pack-years
Serologic markers Presence of circulating anti-PLA2R autoantibodies Absence of anti-PLA2R autoantibodies
in serum
Histopathologic clues on kidney Predominance of glomerular IgG4 deposition Predominance of glomerular IgG1/IgG2 deposition
biopsy Enhanced glomerular PLA2R staining Normal glomerular PLA2R staining
Presence of less than eight inammatory cells Presence of greater than eight inammatory cells
per glomeruli per glomeruli
IgG, immunoglobulin G; MN, membranous nephropathy; PLA2 R phospholipase A2. Reprinted with permission from reference 73.

2 Onco-Nephrology Curriculum American Society of Nephrology


Table 2. Glomerular diseases associated with solid tumors HEMATOLOGIC MALIGNANCIESASSOCIATED
and hematologic malignancies (23) GLOMERULAR DISEASES
Malignancy Glomerular diseases reported
Lung cancer a
MN, MCD, MPGN, IgAN, FSGS, CGN, Minimal change disease
TMA MCD is classically associated with Hodgkin lymphoma (HL),
Colon cancer MN, MCD, CGN more so in the mixed cellularity and nodular sclerosing types.
Stomach cancer MN MCD usually presents around the time of diagnosis of the
Pancreas cancer MN, MCD, IgAN malignancy (28). One case series does report diagnosis of
Bladder cancer MCD MCD preceding the diagnosis of lymphoma by several months
Renal cell cancer AAA, CGN, IgAN, MCD, FSGS, MPGN (29). A poor response to the treatment of MCD with cortico-
Prostate cancer MN, CGN
steroids should raise suspicion of underlying lymphoma. In
Breast cancer MN, FSGS, MPGN, TMA
the case series by Audard et al., (29), the simultaneous diag-
Esophageal cancer MPGN, FSGS
Gastrointestinal stromal AAA
nosis of HL and MCD was associated with the remission of
tumor proteinuria in response to chemotherapy.
Gastric cancer MPGN, CGN, TMA Th2-related cytokines such as interleukin (IL)-13 have been
Spleen sarcoma AAA reported to cause inammatory response in Hodgkin disease
Head and neck cancer MN, IgAN (30), and rat studies have shown that overexpression of IL-13
Wilms tumor MN, MPGN induces proteinuria, hypoalbuminemia, and hypercholester-
Teratoma MN olemia (31). These kidney biopsies showed fusion of foot
Ovarian cancer MN, MCD processes, suggesting MCD like pathology.
Cervical cancer MN
Endometrial cancer MN Membranoproliferative glomerulonephritis
Tongue cancer IgAN
Daas et al. (32) reviewed 42 cases of glomerular disease with
Mesothelioma MCD
chronic lymphocytic leukemia (CLL); of these, 36 had ne-
Melanoma MN, MPGN
Skin cancers (basal and MN
phrotic syndrome. The most common glomerular lesion was
squamous cell) MPGN, followed by MN. Another case series of 13 patients
Pheochromocytoma MN with glomerular disease and either CLL or well-differentiated
Thymoma MCD, FSGS, CGN, MPGN lymphocytic lymphoma (33) showed that the majority had an
Hodgkin disease MCD, MN, MPGN, IgAN, FSGS, CGN, MPGN pattern of injury. Most MPGN patients had an asso-
AAA, Anti-GBM ciated cryoglobulinemia.
Non-Hodgkins disease MN, MCD, MPGN, IgAN, FSGS An MPGN pattern on kidney biopsy (Figure 2) may also be a clue
CLL MN, MCD, MPGN, FSGS, CGN to a developing of undiagnosed lymphoplasmacytic malignancy
AML MN, FSGS (8). Sethi et al. (34) reported an association between MPGN and
CML MN, MCD, MPGN
monoclonal gammopathy of uncertain signicance. They showed
MGUS MPGN
that patients with monoclonal gammopathy with normal bone
T-cell leukemia FSGS
a
marrow biopsies had granular immune deposits on their kidney
Includes small-cell, nonsmall-cell, squamous cell, and bronchogenic cancers.
AAA, AA amyloidosis; AML, acute myelogenous leukemia; CGN, Crescentic biopsy, which correlated with their serum and urine monoclonal
glomerulonephritis; CLL, chronic lymphocytic leukemia; CML, chronic mye- proteins. This study (34) also demonstrated that monoclonal
logenous leukemia; FSGS, focal segmental global sclerosis; GBM, glomer- gammopathy can be seen in the setting of other lymphoplasmacytic
ular basement membrane; IgAN, IgA nephropathy; MCD, minimal change
disease; MGUS, monoclonal gammopathy of unclear signicance; MN, diseases, including low-grade B-cell lymphoma, CLL, and multiple
membranous nephropathy; MPGN, membranoproliferative glomerular ne- myeloma. Although this direct relationship is not proven, the cur-
phritis; TMA, thrombotic microangiopathy. Reprinted with permission from rent observation suggests this possibility (34). More of this is dis-
reference 23.
cussed in the paraproteinemia chapter of the curriculum.

described are MN, FSGS, CGN, and lupus-like nephritis Glomerular diseases associated with myeloproliferative
(24). MN is associated with thymoma of epithelial origin. disorders
MCD is associated with thymoma with lymphocyte predom- Myeloproliferative disorders include chronic myelogenous
inance. The pathogenesis of thymoma-associated MN seems leukemia (CML), polycythemia Vera (PCV), and essential
to be similar to solid tumorassociated MN, and thymoma- thrombocythemia. A recent study (35) of 11 patients with
associated MN is likely related to T-cell dysfunction (24). myeloproliferative disorders with proteinuria and renal failure
Studies (2527) have suggested a major role of T cells, espe- showed mesangial sclerosis with hypercellularity in all pa-
cially the Th2 subtype, in thymoma-associated nephrotic tients, segmental sclerosis in eight patients, features of TMA
syndromes. in eight patients, and intracapillary hematopoietic cells in four
Table 2 summarizes the various solid tumors seen with patients. Glomerular disorders associated with myeloprolifer-
solid tumors. ative disorders are usually late complications and tend to

American Society of Nephrology Onco-Nephrology Curriculum 3


Fibrillary glomerulonephritis (FGN) and Immunotactoid
glomerulopathy (ITG) are rare groups of disorders character-
ized by formation of organized glomerular deposits (Figure 3,
A and B). These diseases can either occur as primary condition
or be secondary to systemic diseases, mainly lymphoproliferative
disorders. ITG is more strongly associated with neoplasms,
typically paraproteinemias and CLL, compared with FGN
(38). ITG on kidney biopsy should warrant an investigation
of an underlying hematologic malignancy. Treatment is di-
rected toward underlying malignancy.
Glomerular diseases have also been associated with hemo-
phagocytic syndrome. This syndrome is most commonly as-
sociated with Epstein-Barr virus; however, it has also been
described with T-cell lymphoma (39,40). Thaunat et al. (40)
described 11 patients with hemophagocytic syndrome who
Figure 2. Membranoproliferative glomerulonephritis. Light developed nephrotic syndrome. Renal biopsy showed glomer-
microscopy showing basement membrane duplication and in-
ular lesions consisting of MCD, FSGS, and TMA. In the ab-
creased cells in capillary lumens. Silver (PASM) stain; 603, original
sence of a causative viral infection, hemophagocytic syndrome
magnication.
is often treated with immunosuppressive therapy with uncer-
tain renal outcomes.
have a poor renal prognosis, with progressive kidney injury
occurring in most patients (35).
Essential thrombocythemia and PCV have been asso- HEMATOPOIETIC STEM CELL TRANSPLANT
ciated with FSGS and mesangial proliferative glomerular RELATED GLOMERULAR DISEASES
disease. The prevalence of glomerular disease in PCV and
essential thrombocythemia is approximately 3%4% (36). In HSCT patients, the kidney biopsy ndings in patients
CML is least likely to have an association with glomerular with nephrotic range proteinuria include MN, MCD, and
pathology (36). FSGS (41). Although we discuss briey here, an entire
FSGS has also been reported with Hodgkins lymphoma chapter is devoted to HSCT-related kidney disease in the
(28) with good response to chemotherapy. Curriculum.

Other glomerular diseases associated with Chronic graft-versus-host disease


lymphoproliferative disorders MN accounts for a majority of cases of HSCT-associated
MN has also been reported with CLL, but less commonly glomerular disease, followed by MCD (41). When MCD occurs
compared with MPGN (32). A case of IgA nephropathy has in such patients, it is prudent to rule out recurrence of the
been described with cutaneous T-cell lymphoma (37). primary hematologic malignancy.

Figure 3. Fibrillary and immunotactoid glomerulonephritis. (A) Electron microscopy view of brillary glomerulonephritis. (B) Electron
microscopy view of immunotactoid glomerulonephritis.

4 Onco-Nephrology Curriculum American Society of Nephrology


A review of literature by Brukamp et al. (41) showed a close
temporal relationship between development of nephrotic syn-
drome shortly after cessation of immunosuppression and the
diagnosis of chronic graft-versus-host disease (GVHD). Luo
et al. (42) investigated the etiology and pathogenesis of ne-
phrotic syndrome after allogenic HSCT. Nephrotic syndrome
after allogenic SCT was associated with the occurrence of
chronic GVHD.
Autologous HSCT can also develop glomerular diseases
(43), although in these patients, GVHD does not occur. It is
possible that there is an immune dysregulation that might be
causing nephrotic syndrome secondary to induction agents or
that these glomerular diseases are de novo. T celldepleted
HSCT recipients are highly unlikely to develop glomerular
diseases. However, our knowledge about glomerular diseases
Figure 4. Thrombotic microangiopathy. Light microscopy view
in HSCT patients is incomplete, and more research is needed
showing red cell thrombi in the afferent arteriole and two glo-
for complete understanding.
merular capillaries. Some basement membrane duplication, but
without increased intracapillary cells, is also visible. Silver (PASM)
Thrombotic microangiopathy after HSCT stain; 603, original magnication.
TMA after HSCT is also known as bone marrow transplant
nephropathy or, in some specic cases, radiation nephropathy.
A diagnosis criteria for HSCT-related TMA included .4% reported by Kwa et al. (52) in patients receiving years of
schistocytes in blood, de novo prolonged or progressive pegylated liposomal doxorubicin for recurrent ovarian cancer.
thrombocytopenia, sudden persistent increase in lactate de-
hydrogenase, and a decrease in serum haptoglobin (44). Stud- Bisphosphonate-induced glomerular injury
ies have suggested that acute GVHD grade 24, hepatic Pamidronate is used to treat malignancy associated bone
GVHD, veno-occlusive disease, adenovirus infection, older disorders in myeloma. Markowitz et al. (53) showed that
age, being female, and total body irradiation of .12 Gy are pamidronate causes biopsy-proven collapsing FSGS. MCD
risk factors for the development of TMA (45,46). TMA can also has also been reported with this agent (54).
occur in T celldepleted group of patients where calcineurin
inhibitors (CNIs) and GVHD do not exist (47). Treatment Interferon-induced glomerular injury
of HSCT-related TMA is usually supportive, with control of Interferons (IFN)-a, -b, and -g have been associated with mod-
hypertension and proteinuria. Plasma exchange has not proven erate proteinuria (55). Markowitz et al. (56) reported 11 cases of
to be effective. collapsing FSGS that developed during treatment with IFN. IFN-
a developed signicant proteinuria and renal failure after a short
duration of treatment. Patients treated with IFN-b developed
CHEMOTHERAPY-ASSOCIATED GLOMERULAR proteinuria after a prolonged course of treatment. The authors
DISEASE (56) also reviewed 21 additional cases of IFN-associated glomer-
ular disease. Thirteen of these patients had FSGS, and the rest had
Thrombotic microangiopathy MCD. The mechanism of this injury is not fully understood.
Mitomycin C, an alkylating agent, used to treat breast, gastric, There is a direct effect of IFN on the podocyte by altering the
and pancreatic cancer, can cause TMA-like syndrome. Its cellular proliferation and cell metabolism (56). The indirect effects
nephrotoxicity is dose dependent and usually appears after a of IFN might be due to adaptive immune mechanism that increase
cumulative dose of .4060 mg/m2 given over a period of macrophage activation or via 1L-6 or 1L-13 production (56).
several months (48). IFN-a, when used for treatment of CML, has been reported
Gemcitabine, commonly used for pancreas, urothelial, and to be associated with TMA (57,58).
ovarian cancers, has been shown to cause TMA (49). Cessation
of these medications is shown to improve TMA. Carlzomib Calcineurin and mammalian target of rapamycin
is a second-generation proteasome inhibitor used for the inhibitors
treatment of relapsed or refractory multiple myeloma. There CNIs can cause a rare manifestation of TMA with glomerular
has been recent case reports (50,51) that reported TMA asso- changes. The histology is indistinguishable from other causes
ciated with the use of this agent. One of them (51) had kidney of TMA (59). The only consensus on treatment is to withdraw
biopsy evidence of TMA (Figure 4). Treatment options include the CNIs (60).
cessation of the drug with uncertain importance of therapeutic Mammalian target of rapamycin (mTOR) inhibitors such
plasma exchange. Kidney biopsyproven renal TMA has been as sirolimus, tensirolimus, and everolimus can develop

American Society of Nephrology Onco-Nephrology Curriculum 5


Table 3. Glomerular toxicity associated with chemotherapeutic agents
Type of cells involved Chemotherapy agents
Glomerular epithelial cells (podocytes)
Minimal change disease Interferon-a and -b, pamidronate, tyrosine kinase inhibitors,
anthracyclines, mTOR inhibitors
Focal segmental glomerular sclerosis Interferon-a and -g, pamidronate, tyrosine kinase inhibitors, clofarabine,
anthracyclines, mTOR inhibitors
Other glomerular diseases Ipilimumab, mTOR inhibitors
Glomerular endothelial cells
Thrombotic microangiopathy Mitomycin-c, gemcitabine, cisplatin, carboplatin, cytarabine, lomustine,
tamoxifen, bleomycin, bortezomib, carlzomib, anthracyclines, hydroxyurea

complications including TMA and FSGS in renal transplant Administration (FDA) approved for unresectable or metastatic
patients (6163). MCD, MN, FSGS, MPGN, and IgA ne- melanoma. Renal biopsy in a patient with ipilimumab-
phropathy have also been associated with sirolimus in the associated AKI with nephrotic range proteinuria revealed
kidney transplant literature (6466). There is speculation that lupus nephritis with positive antidouble-stranded DNA an-
sirolimus-induced proteinuria is related to collapsing FSGS as- tibodies (74). There are also case reports of acute granuloma-
sociated with VEGF overexpression in podocytes. tous interstitial nephritis by this agent (75).
Anthracyclines like daunorubicin and doxorubicin have been
Antiangiogenesis agents known to cause nephrotic syndrome with renal lesions consis-
Antiangiogenic agents are used primarily for advanced stage solid tent with MCD, FSGS not otherwise specied (NOS), or col-
tumors, including renal cell carcinoma, nonsmall cell lung car- lapsing glomerulopathy (76).
cinoma, colorectal carcinoma, and gastrointestinal stromal tu- Table 3 summarizes the glomerular toxicities associated
mors. Monoclonal antibodies against VEGF and tyrosine kinase with chemotherapy agents.
inhibitors (TKIs) (67,68) have been observed to cause hyperten- Ongoing education and heightened physician awareness
sion, proteinuria, and renal vascular injury, manifested by pro- regarding these negative associations is central to early
teinuria and TMA (69). VEGF maintains normal functioning of recognition and their successful management.
glomerular endothelial cells, podocytes, mesangium, and
peritubular capillaries. Hence, inhibition of VEGF can lead to
dose-dependent proteinuria, swelling and detachment of glo- CONCLUSION
merular endothelial cells, vacuolization of endothelial cells,
disruption of slit diaphragms, and down-regulation of nephrin Several cancers are associated with various glomerular diseases.
(70). Examples of anti-VEGF therapy include bevacizumab, Membranous nephropathy remains the most common glo-
and TKIs include sunitinib and sorafenib. In the majority of merular pathology reported in patients with solid tumors.
cases, proteinuria and hypertension resolve or signicantly im- Although MCD disease has been classically associated with HL,
prove with cessation of this therapy (69). VEGF inhibitors are MPGN has been recognized in patients with CLL. Several
more likely to present as TMA or renal-limited TMA and TKIs reports and studies in the literature suggest that treating the
as MCD or MCD/FSGS on kidney biopsy (71). cancer leads to resolution of the glomerular disease.

New chemotherapeutic agentassociated glomerular


disease TAKE HOME POINTS
Several new chemotherapies are now available in clinical prac-
tice. Renal toxicity of these novel agents has been increasingly c Many solid and hematologic malignancies are associated with different
reported in the last decade. Clofaribine is a purine nucleoside glomerular diseases.
analog used to treat relapsed or refractory pediatric acute lym- c Several case reports and case series of cancer-associated glomerular
phoblastic leukemia and adult acute myelogenous leukemia. diseases have shown that treating the cancer may lead to resolution of
Nephrotoxicity most commonly manifests as elevation in serum the glomerular process.
creatinine. Kintzel et al. (72) reported AKI following exposure of c Although membranous nephropathy has been classically associated with
clofaribine along with nephrotic range proteinuria. Unfortunately solid malignancies, minimal change disease has been commonly described
a kidney biopsy was not available. Extrapolating from animal with hematologic malignancies, especially Hodgkin lymphoma.
studies, Jhaveri et al. (73) postulated that inhibition of ribonu- c Membranoproliferative glomerulonephritis is increasingly being
cleotide reductase by clofarabine might be the cause of collapsing recognized to be associated with chronic hematologic malignancies
glomerulopathy and/or kidney injury seen with this agent. such as chronic lymphocytic leukemia.
Ipilimumab is a monoclonal antibody against human c Chemotherapy agents can also lead to glomerular diseases, the most
cytotoxic T-lymphocyte antigen 4. It is US Food and Drug common being TMA associated with targeted therapies.

6 Onco-Nephrology Curriculum American Society of Nephrology


ACKNOWLEDGMENTS paraneoplastic glomerulopathy. Nephrol Dialysis Transplant 27: 1786
1792, 2012
19. Holdsworth SR, Kitching AR, Tipping PG. Th1 and Th2 T helper cell
Pathology images are courtesy of James Pullman, Albert Einstein
subsets affect patterns of injury and outcomes in glomerulonephritis.
Medical Center, NY. Kidney Int 55: 11981216, 1999
20. Taniguchi K, Fujioka H, Torashima Y, Yamaguchi J, Izawa K, Kanematsu
T. Rectal cancer with paraneoplastic nephropathy: Association of vas-
cular endothelial growth factor. Digestive Surg 21: 455457, 2004
REFERENCES 21. Eremina V, Sood M, Haigh J, Nagy A, Lajoie G, Ferrara N, Gerber HP,
Kikkawa Y, Miner JH, Quaggin SE. Glomerular-specic alterations of
1. Ronco PM. Paraneoplastic glomerulopathies: New insights into an old VEGF-A expression lead to distinct congenital and acquired renal dis-
entity. Kidney Int 56: 355377, 1999 eases. J Clin Invest 111: 707716, 2003
2. Bacchetta J, Juillard L, Cochat P, Droz JP. Paraneoplastic glomerular 22. Eremina V, Jefferson JA, Kowalewska J, Hochster H, Haas M, Weisstuch J,
diseases and malignancies. Crit Rev Oncol Hematol 70: 3958, 2009 Richardson C, Kopp JB, Kabir MG, Backx PH. VEGF inhibition and renal
3. Lefaucheur C, Stengel B, Nochy D, Martel P, Hill GS, Jacquot C, Rossert thrombotic microangiopathy. New Engl J Med 358: 11291136, 2008
J, Group G-PS. Membranous nephropathy and cancer: Epidemiologic 23. Jhaveri KD, Shah HH, Calderon K, Campenot ES, Radhakrishnan J.
evidence and determinants of high-risk cancer association. Kidney Int Glomerular diseases seen with cancer and chemotherapy: A narrative
70: 15101517, 2006 review. Kidney Int 84: 3444, 2013
4. Beck LH, Jr., Bonegio RG, Lambeau G, Beck DM, Powell DW, Cummins 24. Karras A, de Montpreville V, Fakhouri F, Grunfeld JP, Lesavre P, Groupe
TD, Klein JB, Salant DJ. M-type phospholipase A2 receptor as target dEtudes des Nephropathies Associees aux T. Renal and thymic pathol-
antigen in idiopathic membranous nephropathy. New Engl J Med 361: ogy in thymoma-associated nephropathy: Report of 21 cases and review
1121, 2009 of the literature. Nephrol Dialysis Transplant 20: 10751082, 2005
5. Qin W, Beck LH, Jr., Zeng C, Chen Z, Li S, Zuo K, Salant DJ, Liu Z. Anti- 25. Hirokawa K, Utsuyama M, Kasai M, Konno A, Kurashima C, Moriizumi E.
phospholipase A2 receptor antibody in membranous nephropathy. J Age-related hyperplasia of the thymus and T-cell system in the Buffalo
Am Soc Nephrol 22: 11371143, 2011 rat. Immunological and immunohistological studies. Virchows Arch B
6. Hoxha E, Kneissler U, Stege G, Zahner G, Thiele I, Panzer U, Harendza S, 59: 3847, 1990
Helmchen UM, Stahl RA. Enhanced expression of the M-type phos- 26. Le Berre L, Herve C, Buzelin F, Usal C, Soulillou JP, Dantal J. Renal
pholipase A2 receptor in glomeruli correlates with serum receptor anti- macrophage activation and Th2 polarization precedes the develop-
bodies in primary membranous nephropathy. Kidney Int 82: 797804, 2012 ment of nephrotic syndrome in Buffalo/Mna rats. Kidney Int 68: 2079
7. Ohtani H, Wakui H, Komatsuda A, Okuyama S, Masai R, Maki N, Kigawa 2090, 2005
A, Sawada K, Imai H. Distribution of glomerular IgG subclass deposits in 27. Le Berre L, Bruneau S, Naulet J, Renaudin K, Buzelin F, Usal C, Smit H,
malignancy-associated membranous nephropathy. Nephrol Dialysis Condamine T, Soulillou JP, Dantal J. Induction of T regulatory cells
Transplant 19: 574579, 2004 attenuates idiopathic nephrotic syndrome. J Am Soc Nephrol 20: 57
8. Jhaveri KD, Shah HH, Patel C, Kadiyala A, Stokes MB, Radhakrishnan J. 67, 2009
Glomerular diseases associated with cancer, chemotherapy, and he- 28. Mallouk A, Pham PT, Pham PC. Concurrent FSGS and Hodgkins
matopoietic stem cell transplantation. Adv Chronic Kidney Dis 21: 48 lymphoma: Case report and literature review on the link between ne-
55, 2014 phrotic glomerulopathies and hematological malignancies. Clin Exper
9. Bjorneklett R, Vikse BE, Svarstad E, Aasarod K, Bostad L, Langmark F, Nephrol 10: 284289, 2006
Iversen BM. Long-term risk of cancer in membranous nephropathy pa- 29. Audard V, Larousserie F, Grimbert P, Abtahi M, Sotto JJ, Delmer A,
tients. Am J Kidney Dis 50: 396403, 2007 Boue F, Nochy D, Brousse N, Delarue R. Minimal change nephrotic
10. Beck LH Jr. Membranous nephropathy and malignancy. Semin Nephrol syndrome and classical Hodgkins lymphoma: Report of 21 cases and
30: 635644, 2010 review of the literature. Kidney Int 69: 22512260, 2006
11. Mustonen J, Pasternack A, Helin H. IgA mesangial nephropathy in 30. Kuppers R, Schwering I, Brauninger A, Rajewsky K, Hansmann ML.
neoplastic diseases. Contribut Nephrol 40: 283291, 1984 Biology of Hodgkins lymphoma. Ann Oncol 13[Suppl 1]: 1118, 2002
12. Magyarlaki T, Kiss B, Buzogany I, Fazekas A, Sukosd F, Nagy J. Renal 31. Lai KW, Wei CL, Tan LK, Tan PH, Chiang GS, Lee CG, Jordan SC, Yap
cell carcinoma and paraneoplastic IgA nephropathy. Nephron 82: 127 HK. Overexpression of interleukin-13 induces minimal-change-like
130, 1999 nephropathy in rats. J Am Soc Nephrol 18: 14761485, 2007
13. Pertuiset E, Liote F, Launay-Russ E, Kemiche F, Cerf-Payrastre I, 32. Daas N, Polliack A, Cohen Y, Amir G, Darmon D, Kleinman Y, Goldfarb
Chesneau AM. Adult Henoch-Schonlein purpura associated with ma- AW, Ben-Yehuda D. Kidney involvement and renal manifestations in non-
lignancy. Semin Arthrit Rheum 29: 360367, 2000 Hodgkins lymphoma and lymphocytic leukemia: A retrospective study in
14. Zurada JM, Ward KM, Grossman ME. Henoch-Schonlein purpura as- 700 patients. Eur J Haematol 67: 158164, 2001
sociated with malignancy in adults. J Am Acad Dermatol 55[5 Suppl]: 33. Moulin B, Ronco PM, Mougenot B, Francois A, Fillastre JP, Mignon F.
S65S70, 2006 Glomerulonephritis in chronic lymphocytic leukemia and related B-cell
15. Pillebout E, Thervet E, Hill G, Alberti C, Vanhille P, Nochy D. Henoch- lymphomas. Kidney Int 42: 127135, 1992
Schonlein Purpura in adults: Outcome and prognostic factors. J Am Soc 34. Sethi S, Zand L, Leung N, Smith RJ, Jevremonic D, Herrmann SS,
Nephrol 13: 12711278, 2001 Fervenza FC. Membranoproliferative glomerulonephritis secondary to
16. Werner TL, Agarwal N, Carney HM, Rodgers GM. Management of monoclonal gammopathy. Clin J Am Soc Nephrol 5: 770782, 2010
cancer-associated thrombotic microangiopathy: What is the right ap- 35. Said SM, Leung N, Sethi S, Cornell LD, Fidler ME, Grande JP, Herrmann
proach? Am J Hematol 82: 295298, 2007 S, Tefferi A, DAgati VD, Nasr SH. Myeloproliferative neoplasms cause
17. Francis KK, Kalyanam N, Terrell DR, Vesely SK, George JN. Dissemi- glomerulopathy. Kidney Int 80: 753759, 2011
nated malignancy misdiagnosed as thrombotic thrombocytopenic 36. Au WY, Chan KW, Lui SL, Lam CC, Kwong YL. Focal segmental
purpura: A report of 10 patients and a systematic review of published glomerulosclerosis and mesangial sclerosis associated with myelo-
cases. Oncologist 12: 1119, 2007 proliferative disorders. Am J Kidney Dis 34: 889893, 1999
18. Takeda S, Chinda J, Murakami T, Numata A, Iwazu Y, Akimoto T, 37. Bajel A, Yin Lin M, Hill PA, Goodman D, McCormack C, Foley P, Davison
Hamano Y, Muto S, Takahashi M, Kusano E. Development of features J, Honemann D, Kenealy M, Lade S. IgA nephropathy associated with
of glomerulopathy in tumor-bearing rats: A potential model for cutaneous T cell lymphoma. Leukemia lymphoma 50: 20832085, 2009

American Society of Nephrology Onco-Nephrology Curriculum 7


38. Rosenstock JL, Markowitz GS, Valeri AM, Sacchi G, Appel GB, DAgati 55. Quesada JR, Talpaz M, Rios A, Kurzrock R, Gutterman JU. Clinical toxicity
VD. Fibrillary and immunotactoid glomerulonephritis: Distinct entities of interferons in cancer patients: a review. J Clin Oncol 4: 234243, 1986
with different clinical and pathologic features. Kidney Int 63: 1450 56. Markowitz GS, Nasr SH, Stokes MB, DAgati VD. Treatment with
1461, 2003 IFN-{alpha}, -{beta}, or -{gamma} is associated with collapsing focal
39. Chang CS, Wang CH, Su IJ, Chen YC, Shen MC. Hematophagic segmental glomerulosclerosis. Clin J Am Soc Nephrol 5: 607615, 2010
histiocytosis: A clinicopathologic analysis of 23 cases with special ref- 57. Badid C, McGregor B, Faivre JM, Guerard A, Juillard L, Fouque D,
erence to the association with peripheral T-cell lymphoma. J. Formosan Laville M. Renal thrombotic microangiopathy induced by interferon-
Med Assoc 93: 421428, 1994 alpha. Nephrol Dialysis Transplant 16: 846848, 2001
40. Thaunat O, Delahousse M, Fakhouri F, Martinez F, Stephan JL, Noel LH, 58. Ohashi N, Yonemura K, Sugiura T, Isozaki T, Togawa A, Fujigaki Y,
Karras A. Nephrotic syndrome associated with hemophagocytic syn- Yamamoto T, Hishida A. Withdrawal of interferon-alpha results in
drome. Kidney Int 69: 18921898, 2006 prompt resolution of thrombocytopenia and hemolysis but not renal
41. Brukamp K, Doyle AM, Bloom RD, Bunin N, Tomaszewski JE, Cizman B. failure in hemolytic uremic syndrome caused by interferon-alpha. Am J
Nephrotic syndrome after hematopoietic cell transplantation: Do glo- Kidney Dis 41: E10, 2003
merular lesions represent renal graft-versus-host disease? Clin J Am 59. Liptak P, Ivanyi B. Primer: Histopathology of calcineurin-inhibitor tox-
Soc Nephrol 1: 685694, 2006 icity in renal allografts. Nature Clin Pract Nephrol 2: 398404, 2006
42. Luo XD, Liu QF, Zhang Y, Sun J, Wang GB, Fan ZP, Yi ZS, Ling YW, Wei 60. Ho VT, Cutler C, Carter S, Martin P, Adams R, Horowitz M, Ferrara J,
YQ, Liu XL. Nephrotic syndrome after allogeneic hematopoietic stem Soiffer R, Giralt S. Blood and marrow transplant clinical trials network
cell transplantation: Etiology and pathogenesis. Blood Cells Molecules toxicity committee consensus summary: Thrombotic microangiopathy
Dis 46: 182187, 2011 after hematopoietic stem cell transplantation. Biol Blood Marrow
43. Troxell ML, Pilapil M, Miklos DB, Higgins JP, Kambham N. Renal Transplant 11: 571575, 2005
pathology in hematopoietic cell transplantation recipients. Modern 61. Jhaveri KD, Schatz JH, Young JW, Flombaum C. Sirolimus (rapamycin)
Pathol 21: 396406, 2008 induced proteinuria in a patient undergoing allogeneic hematopoietic
44. Ruutu T, Barosi G, Benjamin RJ, Clark RE, George JN, Gratwohl A, stem cell transplant. Transplantation 86: 180181, 2008
Holler E, Iacobelli M, Kentouche K, Lammle B. Diagnostic criteria for 62. Hochegger K, Wurz E, Nachbaur D, Rosenkranz AR, Clausen J. Rapamycin-
hematopoietic stem cell transplant-associated microangiopathy: Re- induced proteinuria following allogeneic hematopoietic stem cell trans-
sults of a consensus process by an International Working Group. plantation. Bone Marrow Transplant 44: 6365, 2009
Haematologica 92: 95100, 2007 63. Izzedine H, Boostandoot E, Spano JP, Bardier A, Khayat D. Temsirolimus-
45. Chang A, Hingorani S, Kowalewska J, Flowers ME, Aneja T, Smith KD, induced glomerulopathy. Oncology 76: 170172, 2009
Meehan SM, Nicosia RF, Alpers CE. Spectrum of renal pathology in 64. Mainra R, Mulay A, Bell R, Karpinski J, Hoar S, Knoll G, Robertson S,
hematopoietic cell transplantation: A series of 20 patients and review of Wang D. Sirolimus use and de novo minimal change nephropathy fol-
the literature. Clin J Am Soc Nephrol 2: 10141023, 2007 lowing renal transplantation. Transplantation 80:1816, 2005
46. Changsirikulchai S, Myerson D, Guthrie KA, McDonald GB, Alpers CE, 65. Franco AF, Martini D, Abensur H, Noronha IL. Proteinuria in transplant
Hingorani SR. Renal thrombotic microangiopathy after hematopoietic patients associated with sirolimus. Transplant Proc 39: 449452, 2007
cell transplant: Role of GVHD in pathogenesis. Clin J Am Soc Nephrol 66. Letavernier E, Bruneval P, Mandet C, Duong Van Huyen JP, Peraldi MN,
4: 345353, 2009 Helal I, Noel LH, Legendre C. High sirolimus levels may induce focal
47. Glezerman IG, Jhaveri KD, Watson TH, Edwards AM, Papadopoulos segmental glomerulosclerosis de novo. Clin J Am Soc Nephrol 2: 326
EB, Young JW, Flombaum CD, Jakubowski AA. Chronic kidney disease, 333, 2007
thrombotic microangiopathy, and hypertension following T cell- 67. Jhaveri KD, Flombaum CD, Kroog G, Glezerman IG. Nephrotoxicities
depleted hematopoietic stem cell transplantation. Biol Blood Marrow associated with the use of tyrosine kinase inhibitors: A single-center
Transplant 16: 976984, 2010 experience and review of the literature. Nephron Clin Pract 117: c312
48. Lesesne JB, Rothschild N, Erickson B, Korec S, Sisk R, Keller J, Arbus M, c319, 2011
Woolley PV, Chiazze L, Schein PS. Cancer-associated hemolytic-uremic 68. Bollee G, Patey N, Cazajous G, Robert C, Goujon JM, Fakhouri F,
syndrome: analysis of 85 cases from a national registry. J Clin Oncol 7: Bruneval P, Noel LH, Knebelmann B. Thrombotic microangiopathy
781789, 1989 secondary to VEGF pathway inhibition by sunitinib. Nephrol Dialysis
49. Glezerman I, Kris MG, Miller V, Seshan S, Flombaum CD. Gemcitabine Transplant 24: 682685, 2009
nephrotoxicity and hemolytic uremic syndrome: Report of 29 cases 69. Hayman SR, Leung N, Grande JP, Garovic VD. VEGF inhibition, hy-
from a single institution. Clin Nephrol 71: 130139, 2009 pertension, and renal toxicity. Curr Oncol Rep 14: 285294, 2012
50. Sullivan MR, Danilov AV, Lansigan F, Dunbar NM. Carlzomib associ- 70. Kelly RJ, Billemont B, Rixe O. Renal toxicity of targeted therapies.
ated thrombotic microangiopathy initially treated with therapeutic Target Oncol 4: 121133, 2009
plasma exchange. J Clin Apheresis 2014 71. Izzedine H, Escudier B, Lhomme C, Pautier P, Rouvier P, Gueutin V,
51. Hobeika L, Self SE, Velez JC. Renal thrombotic microangiopathy and Baumelou A, Derosa L, Bahleda R, Hollebecque A. Kidney diseases as-
podocytopathy associated with the use of carlzomib in a patient with sociated with anti-vascular endothelial growth factor (VEGF): An 8-year
multiple myeloma. BMC Nephrol 15: 156, 2014 observational study at a single center. Medicine 93: 333339, 2014
52. Kwa M, Baumgartner R, Shavit L, Barash I, Michael J, Puzanov I, 72. Kintzel PE, Visser JA, Campbell AD. Clofarabine-associated acute
Kopolovic J, Rosengarten O, Blank S, Curtin JP. Is renal thrombotic kidney injury and proteinuria. Pharmacotherapy 31: 923, 2011
angiopathy an emerging problem in the treatment of ovarian cancer 73. Jhaveri KD, Chidella S, Allen SL, Fishbane S. Clofarabine-induced
recurrences? Oncologist 17: 15341540, 2012 kidney toxicity. J Oncol Pharmacy Pract 20: 305308, 2013
53. Markowitz GS, Appel GB, Fine PL, Fenves AZ, Loon NR, Jagannath S, 74. Fadel F, El Karoui K, Knebelmann B. Anti-CTLA4 antibody-induced
Kuhn JA, Dratch AD, DAgati VD. Collapsing focal segmental glomer- lupus nephritis. New Engl J Med 361: 211212, 2009
ulosclerosis following treatment with high-dose pamidronate. J Am Soc 75. Izzedine H, Gueutin V, Gharbi C, Mateus C, Robert C, Routier E, Thomas
Nephrol 12: 11641172, 2001 M, Baumelou A, Rouvier P. Kidney injuries related to ipilimumab. Invest
54. Barri YM, Munshi NC, Sukumalchantra S, Abulezz SR, Bonsib SM, Wallach New Drugs 32: 769773, 2014
J, Walker PD. Podocyte injury associated glomerulopathies induced by 76. Mohamed N, Goldstein J, Schiff J, John R. Collapsing glomerulopathy
pamidronate. Kidney Int 65: 634641, 2004 following anthracycline therapy. Am J Kidney Dis 61: 778781, 2013

8 Onco-Nephrology Curriculum American Society of Nephrology


REVIEW QUESTIONS a. Melanoma-induced proliferative glomerulonephritis
b. Ipilimumab-induced lupus-like nephritis
1. Which of the following statements regarding glomerular c. De novo seronegative lupus nephritis
diseases seen with cancer is true? d. Membranoproliferative glomerulonephritis

a. The most common glomerular pathology seen with Answer: b is correct. Ipilimumab is a monoclonal antibody
solid tumors is minimal change disease against human cytotoxic T-lymphocyte antigen 4, which is
b. The most common glomerular pathology seen with he- FDA approved for unresectable or metastatic melanoma. Ne-
matologic malignancies is membranous nephropathy phrotic range proteinuria with a lupus nephritislike picture
c. The most common associated glomerular disease with on renal biopsy has been reported.
GVHD is membranous nephropathy
d. Thymoma has not been associated glomerular diseases 3. A 62-year-old white man with a long-standing history of
hypertension and recent history of CLL was referred by his
Answer: c is correct. The most common glomerular pathol- oncologist for evaluation of proteinuria and elevated serum
ogy seen with solid tumors is membranous nephropathy creatinine. He denied any history of diabetes, hepatitis, or
(MN). Minimal change disease (MCD) is commonly seen blood transfusion. There was no recent infection or travel
with hematologic malignancies such as Hodgkin lymphoma. history. Review of systems was signicant for bilateral in-
MN accounts for the majority of the cases of HSCT-associated termittent lower extremity swelling over the last 4 months.
glomerular disease. Thymoma has been associated with MCD He denied fever, chills, dyspnea, gross hematuria, arthral-
(lymphocyte predominant) and MN (epithelial origin). gia, or rash. His current medication included amlodipine
for hypertension management.
2. A primary care physician refers a 60-year-old white On physical examination, his BP was elevated at 160/94 mmHg.
woman for evaluation of nephrotic range proteinuria. She There was mild edema of his lower extremities. The rest
presented with a 1-month history of worsening bilateral of the examination was unremarkable. At the time of pre-
lower extremity edema. She has a history of refractory ma- sentation, serum creatinine was 1.5 mg/dL, and serum al-
lignant melanoma. She was recently started on ipilimumab bumin was 3.5 g/dL. Complete blood count, liver function
after failing standard chemotherapy. Her melanoma has tests, and a lipid prole were normal. Urinalysis was signif-
responded to therapy. icant for 1020 RBC/hpf and 21 proteinuria. A 24-hour
On physical examination, her BP was normal at 120/80 mmHg, urine collection revealed 1.8 g protein. A workup for sec-
and there was 31 pitting edema of his lower extremities. ondary causes of proteinuria revealed low C3 and C4 levels.
The rest of the examination was unremarkable. At the time Hepatitis B surface antigen, hepatitis C antibody, antinuclear
of presentation, serum creatinine was 0.9 mg/dL, serum antibody, cryoglobulins, and human immunodeciency
albumin was 2.8 g/dL, total cholesterol was 290 mg/dL, virus (HIV) antibody were negative. Serum and urine
and low-density lipoprotein (LDL) cholesterol was 197 mg/dL. immunoxation did not reveal any monoclonal immu-
Liver function tests and complete blood count were noglobulin. Sonogram revealed normal-sized kidneys. A
normal. A 24-hour urine collection revealed 8.5 g protein. A kidney biopsy was subsequently performed.
workup for secondary causes of nephrotic syndrome revealed What is the most likely kidney biopsy diagnosis?
normal complement levels. Hepatitis B surface antigen,
hepatitis C antibody, antinuclear antibody, cryoglobulins, a. Membranous nephropathy
and human immunodeciency virus (HIV) antibody were b. Membranoproliferative glomerulonephritis
negative. Serum free light chains were within the normal ratio. c. Focal segmental glomerulosclerosis
Sonogram revealed normal-sized kidneys. A kidney biopsy d. Acute interstitial nephritis
reveals a proliferative glomerular disease with immunouo-
rescence suggestive of a full house pattern and electron mi- Answer: b is correct. A membranoproliferative glomerulo-
croscopy showing mesangial and subendothelial deposits. nephritis (MPGN) pattern of injury on renal biopsy has been
What is the most likely diagnosis? most commonly associated with CLL, followed by MN.

American Society of Nephrology Onco-Nephrology Curriculum 9


Chapter 7: Hematologic Disorders and Kidney Disease
Ala Abudayyeh, MD,* and Kevin Finkel, MD, FACP, FASN, FCCM*
*Division of General Internal Medicine, Section of Nephrology, University of Texas MD Anderson Cancer Center,
Houston, Texas; and UTHealth Science Center at Houston Medical School, Department of Medicine, Division of
Renal Diseases and Hypertension, Houston, Texas

MULIPLE MYELOMA chains precipitate with Tamm-Horsfall protein


(THP) secreted by the thick ascending limb of the
Pathogenesis loop of Henle and produce casts in the distal tubule.
Multiple myeloma (MM) is a hematologic malig- Decreased GFR may increase the concentration of
nancy involving the pathologic proliferation of light chains in the distal tubule and enhance the
terminally differentiated plasma cells. It is the formation of casts. Therefore, hypercalcemia, vol-
second most common hematologic malignancy ume depletion, diuretics, and nonsteroidal anti-
behind non-Hodgkin lymphoma, with an annual inammatory drugs can exacerbate renal injury.
incidence of 47 cases per 100,000 in the United In some cases of AKI associated with MM, cast
States. Clinical symptoms are due to osteolysis of formation is rare on renal biopsy. Instead, renal
the bone, suppression of normal hematopoiesis, injury is attributed to the direct toxic effects of
and the overproduction of monoclonal immuno- urinary free light chains (FLCs) on proximal tubule
globulins that deposit in organ tissues. Clinical cells (5,6). After reabsorption, lysosomal degrada-
symptoms include bone pain and fractures, anemia, tion of FLCs can activate the NF-kB pathway lead-
infections, hypercalcemia, edema, heart failure, and ing to oxidative stress with an inammatory
renal disease. response, apoptosis, and brosis. This lesion is
characterized histologically by loss of brush border
Kidney involvement and pathology and cell vacuolization and necrosis (7).
More than one-half of patients with MM initially The classic presentation is an elderly patient with
present with varying degrees of AKI. Nearly 20% unexplained renal failure, anemia, and bone pain
of patients present with a serum creatinine .2.0 or fractures. Proteinuria, when quantitatively mea-
mg/dL, and 10% of patients require dialysis on sured with a 24-hour urine collection, is usually
presentation (1). AKI is associated with higher subnephrotic and primarily composed of mono-
mortality, but this may be reective of patients clonal light chains (Bence-Jones proteins). The
with more advanced disease (2). qualitative measurement of proteinuria using a
The major diseases in the spectrum of myeloma- urine test strip, which mainly detects albumin, is
related kidney disease include cast nephropathy, light generally minimally reactive.
chain deposition disease (LCDD), and AL-amyloidosis. Most patients with myeloma cast nephropathy
Renal biopsy demonstrates the presence of mono- are diagnosed without kidney biopsy using serum
typic light chains on immunouorescence exam, as and urine immunoxation and serum FLC analysis
well as characteristic ultrastructural features of (see Chapter 9). When biopsied, casts are eosin
deposits on electron microscopy. Less common positive, fractured, and waxy in appearance on light
forms of renal injury include light chaininduced microscopy (Figure 1) (8). Multinucleated giant
Fanconi syndrome, cryoglobulinemia, prolifera- cells may surround casts, and an interstitial
tive glomerulonephritis, heavy chain deposition
disease, and immunotactoid glomerulonephritis
(Table 1) (3).3 Correspondence: Kevin W. Finkel, MD, FACP, FASN, FCCM,
Division of Renal Diseases & Hypertension, UTHealth Science
Cast nephropathy Center at Houston-McGovern Medical School, 6431 Fannin St.,
Houston, Texas 77030. Email: kevin.w.nkel@uth.tmc.edu
Cast nephropathy has been diagnosed in 41% of
patients with MM and renal disease (4). Excess light Copyright 2016 by the American Society of Nephrology

American Society of Nephrology Onco-Nephrology Curriculum 1


Table 1. Kidney manifestations of multiple myeloma heart, liver, spleen, and peripheral nervous system may remain
Myeloma cast nephropathy asymptomatic. The hallmark of the disease is the development
AL amyloidosis of mesangial nodules from mesangial matrix expansion
Light chain deposition disease secondary to the up-regulation of platelet derived growth
Heavy chain deposition disease factor-b and transforming growth factor-b. The nodules can
Immunotactoid glomerulopathy occasionally be confused with the Kimmestiel-Wilson lesion
Fibrillary glomerulopathy of diabetic nephropathy.
Light chain Fanconi syndrome Clinically, patients present with proteinuria, renal insuf-
Plasma cell inltration
ciency, and a nodular sclerosing glomerulopathy. Several
Cryoglobulinemia
retrospective reviews have reported on the clinical character-
Membranoproliferative glomerulonephritis
Membranous glomerulonephritis
istics of these patients (9,10). The mean age was 58 years with
no signicant preference with respect to sex. Marked renal
insufciency was common on presentation, with a median
inammatory inltrate composed of lymphocytes and mono- serum creatinine .4 mg/dL, and renal function rapidly de-
cytes may also be present. Widespread tubular atrophy and clined thereafter. Nephrotic range proteinuria was detected
interstitial brosis eventually develops. Immunouorescence in 26%40% of patients and correlated with the degree of
staining generally demonstrates light chain restriction within glomerular involvement. Hypertension and microscopic he-
the casts, although patterns may be mixed or nondiagnostic as maturia were also present in the majority of patients.
well. Casts have a lattice-like appearance and may contain Light chain deposition stimulates mesangial and matrix
needle-shaped crystals on electron microscopy. The glomeruli expansion leading to nodule formation. On light microscopy,
and vessels appear normal, unless LCDD is concurrently pre- mesangial nodules are more uniform in distribution and size in
sent. LCDD compared with diabetic nephropathy (Figure 2) (8).
Irregular thickening and double contours of the glomerular
Light chain deposition disease basement membrane may also be present. Eosin-positive de-
LCDD has been diagnosed at autopsy in 19% of patients with posits may be seen diffusely throughout the tubular basement
MM and renal disease (4). The renal manifestations are most membranes. Immunouorescence demonstrates a character-
apparent clinically, whereas light chain deposits within the istic linear staining of basement membranes with monotypic

Figure 1. Pathology of myeloma cast nephropathy. (A) Large atypical casts are seen within distal tubular lumina. The casts appear
hypereosinophilic with fractured texture and sharp edges. They are associated with giant cell (arrows) and mononuclear cell reaction,
acute proximal tubular cell injury, and interstitial inammation (hematoxylin and eosin, 3,2003). (B) The casts characteristically are
periodic acidSchiff (PAS) negative (3,2003). (C) Myeloma casts almost always stain for just k or l light chain. This gure shows bright
staining of casts for k (immunouorescence, 3,4003). The casts were negative for l (data not shown). (D) Occasionally, myeloma casts
are composed of small rod- or needle-shaped crystals that ll the distal tubular lumina and, as shown, appear highly electron dense on
electron microscopy (31,8503). Reprinted from reference 8, with permission of the Elsevier Science and Technology Journals.

2 Onco-Nephrology Curriculum American Society of Nephrology


Figure 2. Pathology of light-chain proximal tubulopathy. (A) Large rod- and rhomboid-shaped hypereosinophilic crystals are seen
within proximal tubular cells. (hematoxylin and eosin, 3,6003). (B) In this patient with smoldering myeloma who has 20% k-restricted
plasma cells in the bone marrow, the proximal tubular crystals stain strongly for k (as shown) with negative l (not shown) by immu-
nouorescence performed on pronase-digested, parafn-embedded tissue. The intracellular crystals failed to stain for k or l on
standard immunouorescence on frozen tissue (not shown; 3,4003). (C) Ultrastructurally, the proximal tubular cells are lled with
electron-dense light-chain crystals with rod, rhomboid, or rectangular shapes. The proximal tubule brush border appears preserved
(31,8503). (D) In this case of light-chain proximal tubulopathy, the proximal tubular cells are loaded with large election-dense
phagolysosomes without crystals (electron microscopy, 32,5003). Reprinted from reference 8, with permission of the Elsevier Science
and Technology Journals.

light chains, which are most commonly k restricted. On elec- protein loss of 7 g/day. RRT was eventually required in 42% of
tron microscopy, granular-powdery deposits are distributed patients, and median survival after starting dialysis was less than
within the mesangium and midportion of the glomerular, 1 year. In general, cardiac involvement occurs in nearly one-third
tubular, and vessel wall basement membranes. of patients and portends a poor prognosis.
Therapy of LCDD is directed at the underlying myeloma. AL amyloid presents as an amorphic hyaline substance
Cytotoxic chemotherapy followed by hematopoietic stem cell within the mesangium, glomerular basement membranes, and
transplantation (HCT) has met with good success (11,12). vessel walls. Mesangial involvement may be diffuse or nodular.
Amyloid stains positive for Congo red and reveals a charac-
AL amyloidosis teristic apple-green birefringence under polarized light. Im-
AL amyloidosis occurs when pathogenic light chains unfold munouorescence staining reveals the underlying monotypic
and deposit as insoluble brils extracellularly within tissues. It light chain, which has a l: k ratio of 6:1. Electron microscopy
is found in up to 15% of patients with MM on autopsy (13,14). demonstrates nonbranching randomly oriented 8- to 10-nm
In 40% of patients with AL amyloidosis, the bone marrow will brils (Figure 3) (8). Amyloid deposits may appear as sub-
have .10% plasma cells, although only 10% will meet other epithelial spikes along the basement membrane similar to
criteria for MM (15). Amyloid brils may deposit within any membranous nephropathy.
organ, but most commonly affect the kidneys, heart, liver, and Treatment with high-dose melphalan followed by HCT
peripheral nervous system. increases hematologic response and overall median survival
Patients often present with fatigue, weight loss, and nephrotic (16). Improvement in renal function highly correlates with
syndrome. The clinical characteristics of patients with biopsy- increased survival (17).
proven renal amyloidosis were described in a retrospective review
of 84 patients at the Mayo Clinic (15). The median age at di- Treatment of cast nephropathy
agnosis was 61 years, and 62% were male. The median serum General measures
creatinine on presentation was 1.1 mg/dL. The majority of pa- Volume resuscitation to assure optimum hemodynamic
tients had nephrotic syndrome (86%) with a median 24-hour support and adequate urine output (;3 L/day) are of critical

American Society of Nephrology Onco-Nephrology Curriculum 3


Figure 3. Pathology of kidney AL amyloidosis. (A) There is extensive global mesangial and segmental glomerular capillary wall
deposition of acellular, PAS-negative amyloid deposits. For comparison, the hyaline casts depicted are PAS positive (3,4003). (B) By
denition, amyloid deposits should be Congo red positive (i.e., stain red). The gure shows global glomerular and extensive interstitial
Congo redpositive amyloid. (3,2003). (C) Congophilic amyloid deposits characteristically show an apple-green birefringence when
viewed under polarized light (3,2003). (D) On immunouorescence, amyloid deposits appear smudgy and only stain for one of the light
chains. The gure shows global mesangial and segmental glomerular capillary wall staining for l (3,4003). Staining for k was negative
(not shown). (E) A distinctive feature of kidney AL amyloidosis is glomerular amyloid spicules, which result from parallel alignment of
amyloid brils in the subepithelial space perpendicular to the glomerular basement membrane (electron microscopy, 320,0003). (F) On
high magnication, amyloid brils appear haphazardly oriented and measure between 7 and 12 nm in diameter (electron microscopy,
326,0003). AL, immunoglobulin light chain; PAS, periodic acidSchiff. Reprinted from reference 8, with permission of the Elsevier
Science and Technology Journals.

importance in the initial management. Based on experimen- alkalinization cannot be recommended. Colchicine was
tal evidence that furosemide promotes intratubular cast shown to reduce cast formation through decreasing THP
formation by increasing sodium delivery to the distal tubule, secretion and binding in rats, but human studies have
the use of loop diuretics should be avoided unless there is been disappointing (19,20).
volume overload. Hypercalcemia should be aggressively
treated because it can lead to renal vasoconstriction, volume Chemotherapy and stem cell transplantation
depletion, and enhanced cast formation. It has been sugges- The key to treating myeloma cast nephropathy is rapid
ted that urinary alkalinization decreases cast formation by reduction in FLC concentrations. An early decrease in FLC
reducing the net positive charge of FLCs and the interaction levels is associated with the highest rate of renal recovery. In
with THP (18). However, there is no clinical data supporting severe AKI due to cast nephropathy, a 60% reduction in FLC
this approach. Given the risk of causing renal calcium pre- levels by day 21 after diagnosis is associated with renal recovery
cipitation in the setting of hypercalcemia, urinar y in 80% of cases (21). Previous studies with conventional

4 Onco-Nephrology Curriculum American Society of Nephrology


chemotherapy protocols demonstrated that high-dose dexa- 14 patients with presumed myeloma kidney treated with bortezomib
methasone rapidly reduced FLCs. Newer, novel agents such as and TPE, 12 had complete or partial renal response by 6 months;
thalidomide and the proteasome inhibitor, bortezomib, also however, there were no control patients (33). Although there is still
rapidly lower FLC concentrations; this has been referred to as interest in TPE as a therapy for cast nephropathy, its routine use
renoprotective chemotherapy. cannot be recommended based on the current evidence.
Signicant improvement in renal dysfunction has been B. High cutoff hemodialysis (HCO-HD):
reported for MM patients treated with bortezomib-based
regimens (2224). Reversal of renal dysfunction with borte- More recently interest has developed for another method of ex-
zomib may be more frequent and rapid than with other agents, tracorporeal removal of FLCs: HCO-HD. In this technique, a
based on observational analysis. No dose adjustment for renal hemolter with a large pore size (45 kDa) is used for extended
function is necessary for bortezomib. periods of time to remove FLCs.
Thalidomide and lenalidomide are two related chemo- In the largest study of dialysis dependent renal failure secondary to
therapeutic agents commonly used in the treatment of MM. MM, 67 patients were treated with HCO-HD and chemotherapy
Lenalidomide dose must be adjusted for renal dysfunction (34). Only 57% of patients had a renal biopsy, of which 87% had
(25). Thalidomide is not dependent on renal function for cast nephropathy. Most patients (85%) received combination
clearance and does not require dose adjustment for renal chemotherapy with dexamethasone and either bortezomib or
function; however, it may predispose to hyperkalemia in the thalidomide. The median number of HCO-HD sessions was 11,
setting of renal failure (26,27). Regimens with thalidomide or and all patients had extended (.4 hour) treatments. Overall, 63%
lenalidomide have shown superior effectiveness to traditional of the patients became dialysis independent. The factors that
therapy with alkylating agents in terms of reversing renal dys- predicted renal recovery were the degree of FLC reduction at days
function in MM; these agents may be nearly as effective as 12 and 21 and the time to initiating HCO-HD. Unfortunately, this
bortezomib regimens (28). Their effects are likely due to rapid trial did not have a control group to assess the benet of HCO-HD
lowering of serum FLC levels. compared with renoprotective chemotherapy alone.
Hematopoietic stem cell transplantation is an important
and potentially curative therapy in MM; however, patient It is not known whether HCO-HD offers any additional benet
selection criteria are stringent, and signicant renal dysfunc- over current chemotherapeutic regimens. Randomized controlled
tion has traditionally excluded patients from transplantation. trials proving the benet of adding HCO-HD to patients with cast
Recent studies have shown that HCT may be safe and effective nephropathy treated with current chemotherapy will be necessary
in highly selected patients with renal failure (29). before its routine use can be recommended.

Extracorporeal removal of free light chains


Light chains are small-molecular-weight proteins. k light LEUKEMIA AND LYMPHOMA
chains usually circulate as monomers with a molecular weight
of 22.5 kDa, whereas l light chains are typically dimeric with a The development of kidney disease is common in patients with
molecular weight of 45 kDa (30). Because of their size, there lymphoma and leukemia. As with all hospitalized patients,
has been a keen interest in the use of extracorporeal therapy those with lymphoma and leukemia are at risk for developing
as a means of FLC removal. AKI from hypotension, sepsis, or administration of radio-
A. Therapeutic plasma exchange (TPE):
contrast, antifungal, and antibacterial agents. With the pres-
ence of cancer, renal injury can also result from chemotherapy,
Several small trials initially suggested that TPE was effective in rapidly immunosuppressive drugs, hematopoietic stem cell trans-
lowering FLC concentrations and improving renal function. How- plantation, or tumor lysis syndrome. Furthermore, patients
ever, these studies were small, single center, and underpowered. The are at risk for renal syndromes specic to the presence of
largest randomized controlled trial of TPE did not demonstrate any lymphoma or leukemia including various forms of paraneo-
benet in patients with cast nephropathy (31). This study assessed the plastic glomerulopathies, electrolyte disorders, urinary tract
benet of ve to seven TPE sessions in 104 patients (30% were di- obstruction, lysozymuria, leukostasis, and inltration of renal
alysis requiring) with presumed cast nephropathy (not all patients parenchyma (Table 2). Several of these manifestations are
had biopsy conrmation). There was no difference in the two groups discussed elsewhere in the Curriculum.
with respect to the composite outcome of death, dialysis, or reduced
renal function at 6 months. This lack of benet may be related to the Lymphomatous inltration
volume of distribution of FLCs. Based on their molecular weights, Background
85% of light chains are conned to the extravascular space (32). Renal involvement in lymphoma is often clinically silent so
Therefore, a traditional 2-hour TPE session would be ineffective in patients can present with slowly progressive CKD attributed to
removing signicant amounts of FLCs because of the excessive re- other etiologies. Therefore, a high index of suspicion is needed
bound effect. Most of the previous trials were performed prior to the to make a diagnosis. Patients may present with AKI, but this is
availability of bortezomib-containing regimens. In a recent study of rare and is most commonly seen in highly malignant and

American Society of Nephrology Onco-Nephrology Curriculum 5


Table 2. Kidney manifestations of lymphoma and leukemia
Comorbid factors
Sepsis and shock
Hypotension
Volume depletion
Radiocontrast administration
Chemotherapeutic agents
Anti-infective agents
Drug- induced tubulointerstitial nephritis
Hematopoietic stem cell transplantation
Radiotherapy
Disease-related factors
Tumor lysis syndrome
Thrombotic microangiopathy
Malignancy-associated hypercalcemia
Paraneoplastic glomerulopathies Figure 4. Computed tomography scan of abdomen and pelvis
Urinary tract obstruction without radiocontrast showing bilateral kidney enlargement.
Parenchymal inltration Reprinted from reference 43, with permission of the Elsevier
Lysozymuria Science and Technology Journals.
Leukostasis

non-Hodgkin lymphoma were found to have kidney abnor-


disseminated disease (3538). Other presentations include malities (44). Both diffuse enlargement and solitary lesions
proteinuria in both the nephrotic and nonnephrotic range, were detected. This discrepancy between radiologic and autopsy/
as well as a variety of glomerular lesions including pauci- histopathologic results may due to the fact that renal involve-
immune crescentic glomerulonephritis (39). Patients may also ment is often indolent and only detectable in histopathologic
present with ank pain and hematuria. examination (Figure 5) (43). Due to increased metabolic activity
Although a variety of cancers can metastasize to the kidneys within lymphomatous deposits, positron emission tomography
and invade the parenchyma, the most common malignancies to may be a more sensitive imaging technique (45). Although de-
do so are lymphomas and leukemia. The true incidence of renal nitive diagnosis depends on renal biopsy, this procedure often
involvement is unknown because it is usually a silent disease and is impossible because of contraindications. In such cases, the
only occasionally causes renal impairment. Autopsy studies following criteria support the diagnosis of kidney disease due
suggest renal involvement occurs in 90% of patients with to lymphomatous inltration: 1) renal enlargement without ob-
lymphoma, whereas radiographic evidence is signicantly lower. struction; 2) absence of other causes of kidney disease; and 3)
The cause of impaired renal function from lymphomatous rapid improvement of kidney function after radiotherapy or
inltration is poorly understood. Based on biopsy series, pa- systemic chemotherapy.
tients who present with AKI have predominantly bilateral
interstitial inltration of the kidneys with lymphoma cells and
Treatment
uniformly have increased renal size on radiographic imaging The treatment of lymphomatous involvement of the kidney is
(40). These ndings suggest that increased interstitial pressure directed at the underlying malignancy. There are numerous
results in reduced intrarenal blood ow with subsequent renal case reports of improvement in renal function after initiation of
tubular injury. Patients who present with proteinuria, on the antitumor therapy. In indolent malignant disease that is usually
other hand, often have intraglomerular inltration with lym- treated by observation alone, kidney involvement is an in-
phoma (40). It is not known how proteinuria develops in these dication for starting systemic therapy.
cases, but the local release of permeability factors and cyto-
kines has been suggested (41,42).
Leukemic inltration
Background
Diagnosis Leukemia cells can inltrate any organ, and the kidneys are the
The diagnosis of lymphomatous inltration is necessarily one most frequent extramedullary site of inltration. Autopsy
of exclusion because more common explanations are often studies reveal that 60%90% of patients have renal involve-
present. Renal ultrasonography or computed tomography ment (46). On biopsy, cells are usually located in the renal
(CT) scan may reveal diffusely enlarged kidneys sometimes interstitium, although occasional glomerular lesions are noted
with multiple focal lesions (Figure 4) (43). However, many (47). Increased interstitial pressure leads to vascular and tu-
times, radiology will be unrevealing. In a study of 668 con- bular compression and subsequent tubular injury. Occasional
secutive patients with lymphoproliferative disease who un- nodular lesions are found, but this is more common with
derwent diagnostic imaging with a CT scan, only 3% with lymphoma.

6 Onco-Nephrology Curriculum American Society of Nephrology


Figure 5. (A) Kidney biopsy stained with hematoxylin and eosin at 6033 magnication. (B) Tissue stained with B cellspecic
marker anti-CD38 at 20033 magnication. Reprinted from reference 43, with permission of the Elsevier Science and Technology
Journals.

Clinical Features Lysozymuria


Leukemic inltration of the kidneys is often an indolent and Lysozyme is a cationic protein produced by macrophages and
clinically silent disease. Most often it is incidentally noted after monocytes and released in response to bacterial infection. It is
autopsy or by detection of renal enlargement on ultrasound or freely ltered by the glomerulus and reabsorbed by the
CTscan. Although uncommon, many cases of AKI attributable proximal tubule. In certain leukemias, clonal expansion leads
to leukemic inltration have been described (4850). Patients to an excessive production of lysozyme and subsequent
may also experience hematuria or proteinuria. Occasionally proximal tubular injury and AKI (52). Damage to the proxi-
renal enlargement is accompanied by ank pain or fullness. mal tubule reduces reabsorption and can result in Fanconi
There are also reports of patients with chronic lymphocytic syndrome and nephrotic range proteinuria. The presence of
leukemia who develop AKI from leukemic inltration and are lysozymuria can be conrmed by detection of an increased g
infected with polyomavirus (BK) (51). Urine from patients globulin level on serum and urine protein electrophoresis with
demonstrates viral inclusions in tubular cells (decoy cells) immunoxation negative for monoclonal gammopathy (53).
and blood is positive for BK viral DNA. Therefore, in leukemia Treatment is directed at the underlying malignancy.
patients with AKI considered due to leukemic inltration,
evidence for coexisting BK virus infection should be sought. Leukostasis
Patients with myeloid leukemia and exceedingly high white
Diagnosis blood cell counts (usually in excess of 100,000 cells/mm3) can
The diagnosis of leukemia inltration as a cause of AKI develop organ dysfunction due to intravascular aggregation of
requires a high level of vigilance because it is often clinically leukemic cells. The pulmonary and cerebral circulations are
silent, and leukemic patients usually have multiple alternative the most severely affected, although there are case reports of
explanations for renal injury. A presumptive diagnosis can be patients developing AKI (54,55). Leukemic cells occlude the
made if there is no other obvious cause of AKI, bilateral renal peritubular and glomerular capillaries, thereby reducing GFR.
enlargement is demonstrated radiographically, and there is Patients may be oliguric, but their renal function often im-
prompt improvement in renal function after chemotherapy. proves with therapeutic leukopheresis or chemotherapy.
Screening for leukemic inltration with radiographic imaging Leukostasis is thought to result from the abnormal morphology
is not revealing. In a study of 668 consecutive patients with of blast cells and the hyperviscocity of the serum. It has been
lymphoproliferative disease who underwent diagnostic imag- described in both acute and chronic leukemia. Treatment is
ing with a CT scan, only 5% with leukemia were found to have directed at treatment of the malignancy with appropriate che-
kidney abnormalities (45). As with lymphoma, this discrep- motherapeutic regimens; in severe cases, therapeutic leuko-
ancy between radiologic and autopsy/histopathologic results pheresis can rapidly lower cell counts.
may be due to the fact that renal involvement is often indolent
and only detectable during histopathologic examination. CONCLUSION

Treatment Numerous kidney diseases are associated with hematologic


Treatment is directed by the type of leukemia. Although some malignancies unique to this population (Tables 1 and 2). The
patients do not recover, in the majority of cases, renal function most common cause of kidney injury in MM patients is cast
does improve as the leukemia responds to systemic treatment. nephropathy (myeloma kidney). Rapid reduction of

American Society of Nephrology Onco-Nephrology Curriculum 7


circulating free light chains with newer renoprotective che- High-dose melphalan and auto-SCT in patients with monoclonal Ig
motherapy can reverse renal failure in the majority of cases. deposition disease. Bone Marrow Transplant 42: 405412, 2008
13. Ivanyi B. Frequency of light chain deposition nephropathy relative to
Although inltration of the kidneys by leukemia or lymphoma renal amyloidosis and Bence Jones cast nephropathy in a necropsy
is almost universal histologically, clinical renal disease is un- study of patients with myeloma. Arch Pathol Lab Med 114: 986987,
common. Given the myriad of potential kidney insults in these 1990
patients, a high index of suspicion is necessary to diagnose 14. Oshima K, Kanda Y, Nannya Y, Kaneko M, Hamak T, Suguro M,
inltration as the cause of renal dysfunction. Yamamoto R, Chizuka A, Matsuyama T, Takezako N, Miwa A, Togawa A,
Niino H, Nasu M, Saito K, Morita T. Clinical and pathologic ndings
in 52 consecutively autopsied cases with multiple myeloma. Am J
TAKE HOME POINTS Hematol 67: 15, 2001
15. Kyle RA, Gertz MA. Primary systemic amyloidosis: Clinical and labora-
c Cast nephropathy is the most common cause of renal failure in patients tory features in 474 cases. Semin Hematol 32: 4559, 1995
16. Cibeira MT, Sanchorawala V, Seldin DC, Quillen K, Berk JL, Dember
with multiple myeloma.
LM, Segal A, Ruberg F, Meier-Ewert H, Andrea NT, Sloan JM, Finn KT,
c Rapid lowering of free light chains with renoprotective chemotherapy
Doros G, Blade J, Skinner M. Outcome of AL amyloidosis after high-
can reverse renal failure from cast nephropathy in the majority of cases. dose melphalan and autologous stem cell transplantation: Long-term
c Based on current evidence, extracorporeal removal of free light chains results in a series of 421 patients. Blood 118: 43464352, 2011
with either therapeutic plasmapheresis or HCO hemodialysis cannot be 17. Leung N, Dispenzieri A, Lacy MQ, Kumar SK, Hayman SR, Fervenza FC,
Cha SS, Gertz MA. Severity of baseline proteinuria predicts renal
recommended.
response in immunoglobulin light chain-associated amyloidosis after
c Leukemic and lymphomatous inltration of the kidneys is common on
autologous stem cell transplantation. Clin J Am Soc Nephrol 2: 440
autopsy, although it is usually not clinically apparent. 444, 2007
c Enlarged kidneys on imaging and resolution of AKI after therapy with 18. Holland MD, Galla JH, Sanders PW, Luke RG. Effect of urinary pH and
systemic chemotherapy or radiation support the diagnosis of kidney diatrizoate on Bence Jones protein nephrotoxicity in the rat. Kidney Int
27: 4650, 1985
inltration.
19. Sanders PW, Booker BB, Bishop JB, Cheung HC. Mechanisms of in-
tranephronal proteinaceous cast formation by low molecular weight
proteins. J Clin Invest 85: 570576, 1990
REFERENCES 20. Cairns HS, Dawnay A, Woolfson RG, Unwin RJ. Evaluation of therapy for
cast nephropathy: Failure of colchicine to alter urinary Tamm Horsfall
1. Kyle RA, Gertz MA, Witzig TE, Lust JA, Lacy MQ, Dispenzieri A, Fonseca glycoprotein excretion in normal subjects. Exper Nephrol 2:257258,
R, Rajkumar SV, Offord JR, Larson DR, Plevak ME, Therneau TM, Greipp 1994
PR. Review of 1027 patients with newly diagnosed multiple myeloma. 21. Hutchison CA, Cockwell P, Stringer S, Bradwell A, Cook M, Gertz MA,
Mayo Clinic Proc 78: 2133, 2003 Dispenzieri A, Winters JL, Kumar S, Rajkumar SV, Kyle RA, Leung N.
2. Eleutherakis-Papaiakovou V, Bamias A, Gika D, et al. Renal failure in Early reduction of serum-free light chains associates with renal recovery
multiple myeloma: Incidence, correlations, and prognostic signi- in myeloma kidney. J Am Soc Nephrol 22: 11291136, 2011
cance. Leukemia Lymphoma 48: 337341, 2007 22. Moreau P, Richardson PG, Cavo M, Orlowski RZ, San Miguel JF,
3. Markowitz GS. Dysproteinemia and the kidney. Adv Anatomic Pathol Palumbo A, Harousseau JL. Proteasome inhibitors in multiple myeloma:
11: 4963, 2004 10 years later. Blood 120: 947959, 2012
4. Montseny JJ, Kleinknecht D, Meyrier A, Vanhille P, Simon P, Pruna A, 23. Ludwig H, Adam Z, Hajek R, Greil R, Tothova E, Keil F, Autzinger EM,
Eladari D. Long-term outcome according to renal histological lesions Thaler J, Gisslinger H, Lang A, Egyed M, Womastek I, Zojer N. Light
in 118 patients with monoclonal gammopathies. Nephrol Dialysis chain-induced acute renal failure can be reversed by bortezomib-
Transplant 13:14381445, 1998 doxorubicin-dexamethasone in multiple myeloma: Results of a phase II
5. Sanders PW. Mechanisms of light chain injury along the tubular neph- study. J Clin Oncol 28: 46354641, 2010
ron. J Am Soc Nephrol 23: 17771781, 2012 24. Dimopoulos MA, Roussou M, Gkotzamanidou M, Nikitas N, Psimenou E,
6. Batuman V. Proximal tubular injury in myeloma. Contribut Nephrol 153: Mparmparoussi D, Matsouka C, Spyropoulou-Vlachou M, Terpos E,
87104, 2007 Kastritis E. The role of novel agents on the reversibility of renal impairment
7. Sanders PW, Herrera GA, Lott RL, Galla JH. Morphologic alterations of in newly diagnosed symptomatic patients with multiple myeloma.
the proximal tubules in light chain-related renal disease. Kidney Int 33: Leukemia 27: 423429, 2013
881889, 1988 25. Chen N, Lau H, Kong L, Kumar G, Zeldis JB, Knight R, Laskin OL.
8. Leung N, Nasr SH. Myeloma-related kidney disease. Adv Chronic Pharmacokinetics of lenalidomide in subjects with various degrees of
Kidney Dis 21: 3647, 2014 renal impairment and in subjects on hemodialysis. J Clin Pharmacol 47:
9. Lin J, Markowitz GS, Valeri AM, Kambham N, Sherman WH, Appel GB, 14661475, 2007
DAgati V D. Renal monoclonal immunoglobulin deposition disease: 26. Eriksson T, Hoglund P, Turesson I, Waage A, Don BR, Vu J, Schefer M,
The disease spectrum. J Am Soc Nephrol 12:14821492, 2001 Kaysen GA. Pharmacokinetics of thalidomide in patients with impaired
10. Pozzi C, DAmico M, Fogazzi GB, Curioni S, Ferrario F, Pasquali S, renal function and while on and off dialysis. J Pharm Pharmacol 55:
Quattrocchio G, Rollino C, Segagni S, Locatelli F. Light chain de- 17011706, 2003
position disease with renal involvement: Clinical characteristics and 27. Harris E, Behrens J, Samson D, Rahemtulla A, Russell NH, Byrne JL. Use
prognostic factors. Am J Kidney Dis 42: 11541163, 2003 of thalidomide in patients with myeloma and renal failure may be as-
11. Girnius S, Seldin DC, Quillen K, Dember LM, Segal A, Sanchorawala V. sociated with unexplained hyperkalaemia. Br J Haematol 122: 160
Long-term outcome of patients with monoclonal Ig deposition disease 161, 2003
treated with high-dose melphalan and stem cell transplantation. Bone 28. Roussou M, Kastritis E, Christoulas D, Migkou M, Gavriatopoulou M,
Marrow Transplant 46: 161162, 2011 Grapsa I, Psimenou E, Gika D, Terpos E, Dimopoulos MA. Reversibility
12. Hassoun H, Flombaum C, DAgati VD, Rafferty BT, Cohen A, Klimek of renal failure in newly diagnosed patients with multiple myeloma and
VM, Boruchov A, Kewalramani T, Reich L, Nimer SD, Comenzo RL. the role of novel agents. Leukemia Res 34: 13951397, 2010

8 Onco-Nephrology Curriculum American Society of Nephrology


29. Parikh GC, Amjad AI, Saliba RM, Kazmi SM, Khan ZU, Lahoti A, Hosing 41. DAgati V, Sablay LB, Knowles DM, Walter L. Angiotropic large cell
C, Mendoza F, Qureshi SR, Weber DM, Wang M, Popat U, Alousi lymphoma (intravascular malignant lymphomatosis) of the kidney:
AM, Champlin RE, Giralt SA, Qazilbash MH. Autologous hemato- Presentation as minimal change disease. Human Pathol 20: 263268,
poietic stem cell transplantation may reverse renal failure in pa- 1989
tients with multiple myeloma. Biol Blood Marrow Transplant 15: 42. Agar JW, Gates PC, Vaughan SL, Machet D. Renal biopsy in angiotropic
812816, 2009 large cell lymphoma. Am J Kidney Dis 24: 9296, 1994
30. Pratt G. The evolving use of serum free light chain assays in haema- 43. Luciano RL, Brewster UC. Kidney involvement in leukemia and lym-
tology. Br J Haematol 141: 413422, 2008 phoma. Adv Chronic Kidney Dis 21: 2735, 2014
31. Clark WF, Stewart AK, Rock GA, Sternbach M, Sutton DM, Barrett BJ, 44. Bach AG, Behrmann C, Holzhausen HJ, Katzer M, Arnold D, Spielmann
Heidenheim AP, Garg AX, Churchill DN, Canadian Apheresis G. Plasma RP, Surov A. Prevalence and patterns of renal involvement in imaging of
exchange when myeloma presents as acute renal failure: a randomized, malignant lymphoproliferative diseases. Acta Radiol 53: 343348,
controlled trial. Ann Intern Med 143: 777784, 2005 2012
32. Hutchison CA, Cockwell P, Reid S, Chandler K, Mead GP, Harrison J, 45. Sheth S, Ali S, Fishman E. Imaging of renal lymphoma: patterns of
Hattersley J, Evans ND, Chappell MJ, Cook M, Goehl H, Storr M, disease with pathologic correlation. Radiographics 26: 11511168,
Bradwell AR. Efcient removal of immunoglobulin free light chains by 2006
hemodialysis for multiple myeloma: In vitro and in vivo studies. J Am 46. Suh WM, Wainberg ZA, de Vos S, Cohen AH, Kurtz I, Nguyen MK. Acute
Soc Nephrol 18: 886895, 2007 lymphoblastic leukemia presenting as acute renal failure. Nature Clin
33. Burnette BL, Leung N, Rajkumar SV. Renal improvement in myeloma Pract Nephrol 3: 106110, 2007
with bortezomib plus plasma exchange. New Engl J Med 364: 2365 47. Kowalewska J, Nicosia RF, Smith KD, Kats A, Alpers CE. Patterns of
2366, 2011 glomerular injury in kidneys inltrated by lymphoplasmacytic neo-
34. Hutchison CA, Heyne N, Airia P, Schindler R, Zickler D, Cook M, plasms. Human Pathol 42: 896903, 2001
Cockwell P, Grima D. Immunoglobulin free light chain levels and 48. Comerma-Coma MI, Sans-Boix A, Tuset-Andujar E, Andreu-Navarro J,
recovery from myeloma kidney on treatment with chemotherapy and Perez-Ruiz A, Naval-Marcos I. Reversible renal failure due to specic
high cut-off haemodialysis. Nephrol Dialysis Transplant 27: 3823 inltration of the kidney in chronic lymphocytic leukaemia. Nephrol
3828, 2012 Dialysis Transplant 13: 15501552, 1998
35. Kanfer A, Vandewalle A, Morel-Maroger L, Feintuch MJ, Sraer JD, 49. Pagniez DC, Fenaux P, Delvallez L, Dequiedt P, Gosselin B, Tacquet A.
Roland J. Acute renal insufciency due to lymphomatous inltration of Reversible renal failure due to specic inltration in chronic lympho-
the kidneys: Report of six cases. Cancer 38: 25882592, 1976 cytic leukemia. Am J Med 85: 579580, 1988
36. Koolen MI, Schipper P, v Liebergen FJ, Kurstjens RM, v Unnik AJ, 50. Phillips JK, Bass PS, Majumdar G, Davies DR, Jones NF, Pearson TC.
Bogman MJ. Non-Hodgkin lymphoma with unique localization in the Renal failure caused by leukaemic inltration in chronic lymphocytic
kidneys presenting with acute renal failure. Clin Nephrol 29: 4146, leukaemia. J Clin Pathol 46: 11311133, 1993
1988 51. Boudville N, Latham B, Cordingly F, Warr K. Renal failure in a patient
37. Malbrain ML, Lambrecht GL, Daelemans R, Lins RL, Hermans P, Zachee with leukaemic inltration of the kidney and polyomavirus infection.
P. Acute renal failure due to bilateral lymphomatous inltrates. Primary Nephrol Dialysis Transplant 16: 10591061, 2001
extranodal non-Hodgkins lymphoma (p-EN-NHL) of the kidneys: Does 52. Patel TV, Rennke HG, Sloan JM, DeAngelo DJ, Charytan DM. A for-
it really exist? Clin Nephrol 42: 163169, 1994 gotten cause of kidney injury in chronic myelomonocytic leukemia. Am
38. Miyake JS, Fitterer S, Houghton DC. Diagnosis and characterization of J Kidney Dis 54: 159164, 2009
non-Hodgkins lymphoma in a patient with acute renal failure. Am J 53. Levinson SS, Elin RJ, Yam L. Light chain proteinuria and lysozymuria in a
Kidney Dis 16: 262263, 1990 patient with acute monocytic leukemia. Clin Chem 48: 11311132,
39. Henriksen KJ, Hong RB, Sobrero MI, Chang A. Rare association of 2002
chronic lymphocytic leukemia/small lymphocytic lymphoma, ANCAs, 54. McKee LC Jr, Collins RD. Intravascular leukocyte thrombi and aggre-
and pauci-immune crescentic glomerulonephritis. Am J Kidney Dis 57: gates as a cause of morbidity and mortality in leukemia. Medicine 53:
170174, 2011 463478, 1974
40. Tornroth T, Heiro M, Marcussen N, Franssila K. Lymphomas 55. Dietrich PY, Pedraza E, Casiraghi O, Bayle C, Hayat M, Pico JL. Cardiac
diagnosed by percutaneous kidney biopsy. Am J Kidney Dis 42: arrest due to leucostasis in a case of prolymphocytic leukaemia. Br J
960971, 2003 Haematol 78: 122123, 1991

American Society of Nephrology Onco-Nephrology Curriculum 9


REVIEW QUESTIONS a. Inltrative disease secondary to underlying leukemia
b. Membranous glomerulopathy secondary to neoplasm
1. An 80-year-old patient with multiple myeloma presents c. Renal vein thrombosis
with serum uric acid of 12.0 mg/dL, serum calcium of 11.0 d. Lysozyme-induced kidney injury
mg/dL, and a phosphate of 8.1 mg/dL. Serum creatinine is
4.0 mg/dL, and free l light chains are 3,700 mg/L. Which Answer: d is correct. Lysozyme-induced kidney injury has
ONE of the following therapies is contraindicated in this been underdiagnosed and underrecognized. Uncontrolled
patient? production of lysozyme secondary to underlying malignancy
predisposes the patient to the damage to the proximal tubule
a. Urinary alkalinization
reduces reabsorption of electrolytes and can result in Fanconi
b. Intravenous saline
syndrome and nephrotic range proteinuria. A clue to Fanconi
c. Rasburicase
syndrome is glycosuria with normal serum glucose level.
d. Allopurinol
On electron microscopy, the kidney biopsies have shown an
Answer: a is correct. There is a lack of evidence to support increase in number and size of lysosomes in the proximal
the use of urinary alkalinization to decrease cast formation by tubules.
reducing the net positive charge of FLCs. In addition, there is 4. A 56-year-old woman with a history of diabetes and hy-
an increased risk of renal calcium precipitation with the pres- pertension (HTN) presented to the hospital with AKI,
ence of hypercalcemia. calcium of 12.0 mg/dL, hemoglobin of 10.0 g/dL, and se-
rum albumin of 3.5 g/dL. Urinalysis revealed a specic
2. A 57-year-old man with no medical history is diagnosed
gravity 1.015 with trace protein on dipstick examination
with chronic lymphocytic leukemia with Richters trans-
and occasional granular cast on microscopic examination.
formation and presents to the emergency room with cre-
Renal ultrasonography revealed normal-sized kidneys
atinine of 3.61 mg/dL and white blood cell count of 20,000/
without hydronephrosis. Creatinine was noted to be at 5
mm3. The patient has undergone a renal ultrasound in-
mg/dL, and blood urea nitrogen was 82 mg/dL with a
dicating slightly enlarged kidneys 13 cm bilaterally. On
normal baseline 6 months ago. Because there was a high
examination he was noted to have hepatosplenomegaly.
suspicion for multiple myeloma, the patient had a serum
Urinalysis was negative for proteinuria and hematuria. He
free light chain (FLC) assay to determine monoclonality,
is planned for chemotherapy pending a renal consult for
and the free serum l light chains were 1,500 mg/L. Which
his rise in creatinine. Which one of the following likely
of the following treatments is indicated for AKI due to cast
explains his renal injury?
nephropathy?
a. Urinary obstruction related to retroperitoneal lymph- a. Chemotherapy
adenopathy b. Hemodialysis using dialyzers that have a high-molecular-
b. Tumor lysis syndrome weight cutoff
c. Leukemic inltration of the kidney c. Plasma exchange therapy
d. Membranoproliferative glomerulonephritis with C3 d. All of the above
and monoclonal immunoglobulin deposition
Answer: a is correct. Based on the current evidence, per-
Answer: c is correct. It is likely leukemic inltration in forming plasma exchange and high cutoff hemodialysis to
the setting of enlarged kidney and hepatosplenomegaly with treat cast nephropathy cannot be recommended. Signicant
the lack of other possible etiologies for the patients renal and rapid reductions in FLC concentrations using reno-
dysfunction. protective chemotherapy have been shown of benet in pre-
serving renal function.
3. A 74-year-old man with chronic myelomonocytic leuke-
mia with a long-standing history of hypertension and 5. A 63-year-old man with a medical history of well-controlled
hyperlipidemia is seen in the clinic. Peripheral blood diabetes and hypertension presents with a creatinine of 2.0
monocyte count is .1,000/mm3, and splenomegaly is mg/dL, weight loss, fatigue, edema, and worsening periph-
noted. His creatinine has started to rise in the last 6 months eral neuropathy. He was noted to have a serum albumin level
to 2.0 mg/dL from a baseline of 1.3 mg/dL. Urinalysis in- of 3.0 g/dL, and spot urine protein to creatinine ratio of 2 g,
dicated 31 protein and 31 glucose. His labs included the with no red blood cells in the urinalysis. Liver function
following: white blood cells, 50,000/mm3; hemoglobin, 9.0 studies were normal. He had a negative skeletal survey, and a
g/dL; platelets, 60,000/mm3; serum potassium, 3.0 mEq/L; bone marrow biopsy showed normal cellularity with 1%
calcium, 8.0 mg/dL; phosphorus, 1.8 mg/dL; serum glu- plasma cells. Serum protein electrophoresis indicated IgA l
cose, 80 mg/dL. Renal ultrasound showed normal size M-protein. Due to the possibility of amyloidosis, the patient
kidneys and no hydronephrosis. Which ONE of the fol- underwent a kidney biopsy which showed a mesangial area
lowing is the likely cause of this patients renal dysfunction? expanded by amyloid brils. If the patient had AL-type

10 Onco-Nephrology Curriculum American Society of Nephrology


amyloidosis, what is the most appropriate intervention(s) e. Chemotherapy plus extracorporeal removal of the Ig
that would improve overall survival? FLC

a. Chemotherapy to reduce monoclonal protein over- Answer: c is correct. Based on retrospective studies, pa-
production tients with AL amyloidosis whom underwent stem cell trans-
b. Plasma exchange to reduce circulating FLC levels plant have been shown to have improved overall survival and
c. High-dose melphalan followed by autologous hema- improved quality of life compared with those undergoing
topoietic stem cell transplant chemotherapy alone. There is no role for plasma exchange
d. Hemodialysis using dialyzers with high-molecular- or hemodialysis with a high cutoff lter in the treatment of
weight cutoff amyloidosis.

American Society of Nephrology Onco-Nephrology Curriculum 11


Chapter 8: Clinical Tests for Monoclonal Proteins
Nelson Leung, MD
Division of Nephrology and Hypertension, Hematology, Mayo Clinic, Rochester, Minnesotta

INTRODUCTION SERUM PROTEIN ELECTROPHORESIS

Monoclonal gammopathy is a hallmark of plasma Serum protein electrophoresis (SPEP) is the most
cell dyscrasias and some B-cell lymphoprolife- commonly used laboratory test for the detection of
rative disorders (1). They cover a wide spec- monoclonal proteins. Serum proteins are loaded
trum of diseases from the premalignant condition on to a gel or a capillary tube and are separated by
monoclonal gammopathy of undetermined sig- an electrical current based on charge and size. The
nicance (MGUS) to symptomatic multiple proteins are then stained for visualization. The
myeloma, malignant lymphomas, and chronic proteins migrate into ve zones or fractions. These
lymphocytic leukemia (CLL). The monoclonal are albumin, a1, a2, b, and g. The b fraction often
(M) proteins can be the entire immunoglobulin, has two peaks. Albumin is the most abundant protein
light chain only, or, rarely, heavy chain only. Their in the serum and should make up the largest peak in
ability to cause kidney disease is another char- normal serum. When a monoclonal (M) protein is
acteristic they have in common. In a disease present, a sharp band appears often in the g region.
such as multiple myeloma, the risk of AKI corre- However, M proteins can migrate to the b or even the
lates with the severity of disease and can be as high a fractions. This often occurs when the M protein is
as 50% (2,3). In one study, 87% of patients with comprised of an IgA or free light chain (FLC). In
AKI had the most advanced stage (III) of disease some cases, no band is detected but instead there
according to the Durie Salmon classication (4). is a decrease in the g peak (10). The hypogammaglob-
In fact, only 44% of the patients with stage III ulinemia is due to the monoclonal gammopathy.
disease had normal renal function. Other less Currently, SPEP is the most commonly used test
common causes of AKI in this population include for M proteins globally because of its ease of use and
interstitial nephritis and acute tubular necrosis relatively low cost. Fully automated systems are
(57). available for both the agarose gel and capillary tube
A number of glomerular and tubular lesions have methods, which have increased reproducibility and
also been described in myeloma patients; how- efciency. Because SPEP is quantitative, it is used in
ever, these lesions are actually more common in both diagnostic and response criteria in multiple
patients where the diagnostic criteria for multiple myeloma (11,12). Despite its utility, its detection limit
myeloma or lymphoma have not been met and are is not sensitive enough as a single screening test, espe-
diagnosed with monoclonal gammopathy of renal cially in low-burden diseases like MGRS. The detection
signicance (MGRS) (8). Patients with MGRS- limit for an M protein is 0.30.5 g/dL in the g region
related kidney disease are more likely to present and up to 0.7 g/dL in the a or b region (13). SPEP is
with proteinuria, hematuria, and mild renal im- positive in 87.6% of multiple myelomas but only
pairment than rapid-onset AKI as in cast nephro- 73.8% of immunoglobulin light chain (AL) amyloidosis
pathy. In either situation, the identication of a and 55.6% of light chain deposition disease (LCDD)
monoclonal protein changes the diagnosis, path- (14). In addition, the M-band on the SPEP only indi-
ophysiology, and prognosis and directs the cli- cates an M protein but it does not distinguish the iso-
nician toward a hematologic evaluation (9). types. To nd out the type, immunoxation is required.
Monoclonal protein testing should be a part of
any workup of AKI, as well as proteinuria with
mild reduction of renal function in adults. This Correspondence: Nelson Leung, Mayo Clinic, Rochester, Minnesotta: Q:1
Email: leung.nelson@mayo.edu
article will review the current available tests for
monoclonal proteins. Copyright 2016 by the American Society of Nephrology

American Society of Nephrology Onco-Nephrology Curriculum 1


Table 1. Screening panel
Condition SPEP SIFE SFLC UPEP/IFE
MGRS X X X X
AL/MIDD X X X X
WM X X
Multiple myeloma X X
Immunoxation should be performed if a screening test is positive to help
type the monoclonal protein. SPEP, serum protein electrophoresis; SIFE,
serum immunoxation; SFLC, serum free light chain; U, urine; MGRS,
monoclonal gammopathy of renal signicance; AL, AL amyloidosis; MIDD,
monoclonal immunoglobulin deposition disease; WM, Waldenstrm mac-
roglobulinemia.

URINE PROTEIN ELECTROPHORESIS

The rst person to recognize that monoclonal proteins have


special properties in the urine was Dr. William MacIntyre (15).
Urine of myeloma patients turns opaque when boiled, clears
with the addition of acid, and turns opaque again as it cools.
The property was attributed to a protein that was named after
the physician who misidentied it: Dr. Henry Bence Jones. It
was not until later that Bence-Jones protein was identied as
immunoglobulin FLC. Urine protein electrophoresis (UPEP)
uses the same principle as SPEP and has the same advantages
and disadvantages. The urine M-spike is used for diagnosis Figure 1. Serum protein electrophoresis (PEL) and immuno-
and response determination in multiple myeloma (16). How- xation (IFE). A monoclonal (M) spike was detected in the b
region of the protein electrophoresis. The immunoxation iden-
ever, because M proteins are not always present in the urine of
tied a band in the IgA and k lanes that corresponded to the
patients with monoclonal gammopathy, the sensitivity of
band in the b region of the protein electrophoresis. The M pro-
UPEP is the lowest of all the tests and should never be used tein in this case is a monoclonal IgA k. The blurry smudge in IgG
alone for screening. In a study with 2,799 patients of which lane indicated that the IgG was polyclonal.
4.4% had a plasma cell dyscrasia, SPEP was positive in 94.4%
of patients, whereas only 37.7% had a positive UPEP (17). This monoclonal protein. In multiple myeloma, serum IFE in-
has led some to suggest that the serum FLC assay should replace creased the detection rate from 87.6% to 94.4%. Similarly,
the UPEP for screening (see below). However, despite the low the sensitivity increases from 65.9% to 73.8% in AL amy-
sensitivity, UPEP does provide additional information. In pa- loidosis. In a mixed group of patients, the sensitivity
tients with renal impairment, the presence of an M protein and increases from 79% to 87%. Not only is the sensitivity im-
low albumin concentration on the UPEP is highly suggestive of proved with IFE, but the type of the monoclonal protein can
cast nephropathy, whereas a high albumin concentration is more be identied. IFE is qualitative and not quantitative. There-
likely the results of LCDD or AL amyloidosis (18). In addition, fore, for response determination in multiple myeloma and
the presence of a monoclonal light chain in the urine signi- AL amyloidosis, it is only used for assessment of complete
cantly increases the risk of renal injury in myeloma patients (19). response. The extra reagents add signicantly to the cost,
Finally, urine M-spike is still used in response determination in making IFE less affordable.
multiple myeloma (11). Therefore, UPEP remains a useful sup-
plemental test in patients with paraproteinemia.
SERUM FLC ASSAY

SERUM AND URINE IMMUNOFIXATION In the early 2000s, a new assay was introduced to aid in the detection
of monoclonal proteins. Using antibodies against epitopes that are
Samples are electrophoresed in parallel lanes in immunox- normally hidden in the intact immunoglobulin, the assay detects
ation (IFE). Antibodies against the heavy and light chains of the both k and l FLCs. It is quantitative and automated. The assay is
immunoglobulin are then applied to each lane separately. A not specic for monoclonal light chains but instead infers mono-
reaction forming a sharp band would suggest the presence of clonality when an abnormal ratio between the k and l FLCs is
monoclonal immunoglobulin component (Figure 1). The M detected. The normal ratio is between 0.26 and 1.65. A high ratio
protein composed of the entire monoclonal immunoglobulin suggests a k clone, whereas a ratio ,0.26 suggest a l clone.
would be positive for both a heavy chain and a light chain. The addition of the serum FLC assay to SPEP and serum IFE
The sensitivity of IFE has a detection limit of ;0.1 g/dL of has signicantly increased the sensitivity of monoclonal protein

2 Onco-Nephrology Curriculum American Society of Nephrology


testing. Prior to the introduction of the FLC assay, up to 5% of levels (27). Thus, urinary FLC levels do not appear to contrib-
multiple myelomas were thought to be nonsecretory. Using the ute to diagnostic sensitivity of the current monoclonal testing
FLC assay, 19 of 28 nonsecretory myeloma patients were found to regimen.
have abnormal k to l ratios, and 4 had suppression of one or
both FLCs (20). The FLC assay was able to identify abnormalities
SCREENING AND MONITORING
in 82% of patients classied as nonsecretory by serum and urine
PEP and IFN. Most of these were light chainonly myeloma.
Recently, the serum FLC ratio was added to the diagnostic criteria The screening test(s) for any disease must have sufcient sen-
sitivity to identify as many patients as possible but also cost
of multiple myeloma (12). In AL amyloidosis, the serum FLC
effective enough to apply to the general population. Curren-
assay increases the detection rate from 69% (with serum IFE
tly, no single test is capable of accomplishing these goals in
alone) to 99% (21). The addition of UPEP did not identify any
monoclonal gammopathy. This is especially true for diseases
additional patients. Another study found the combination of
with very low levels of monoclonal protein (28). However, in
serum IFE and serum FLC detected 100% of the patients with
diseases such as multiple myeloma and Waldenstrm macro-
multiple myeloma, Waldenstrm macroglobulinemia, and
globulinemia, where the monoclonal protein is usually abun-
smoldering multiple myeloma (14). The addition of UPEP did
not increase the sensitivity for the above diagnoses but did assist dant, the combination of serum IFE and serum FLC has been
shown to be nearly 100% sensitive (14). In diseases with lower
in the identication of MGUS, extramedullary plasmacytoma,
levels of monoclonal gammopathy, urine IFE can increase the
AL amyloidosis, and LCDD.
sensitivity but at a higher cost (Table 1).
In addition to diagnostic evaluation, serum FLC is also used
For monitoring, the goals are different. Typically, the type
in disease monitoring and assessment of response. The degree
of monoclonal protein is no longer important so IFE is not
and speed of serum FLC reduction have been found to be the
routinely required. The response is typically based on the
most important predictors of renal recovery in cast nephrop-
reduction of the M protein. Depending on the M protein, this
athy (22). In AL amyloidosis, the reduction of serum FLC has
been shown to be a better predictor of outcomes than the is done with SPEP, UPEP, and/or serum FLC assay. IFE is re-
quired when the M-spike is no longer detectable to evaluate
M-spike. To achieve stringent complete response in multiple
for complete response. In multiple myeloma, FLC should be
myeloma and complete response in AL amyloidosis, the serum
followed even when patient has a M-spike because of the
FLC ratio has to be normalized (16,23).
phenomenon light chain escape. This occurs in some heavily
Although the serum FLC assay increases the detection rate
treated patients where the clonal evolution produces a clone
of monoclonal gammopathy, clinicians should be aware of
that makes more FLC than the intact immunoglobulin (29). In
its limitations. First, the assay does not distinguish between
these patients, the M-spike would remain low, suggesting per-
polyclonal FLCs and monoclonal FLCs. The higher (or lower)
the k to l ratio, the more likely a monoclonal gammopathy sistent response but the involved FLC will rise rapidly.
exists. However, several conditions are known to cause minor
abnormalities. The most common is renal insufciency. Be- TAKE HOME POINTS
cause FLCs are mostly cleared by the kidney, a reduction in
glomerular ltration rate will cause a rise in the FLC levels. c Monoclonal protein testing is an important diagnostic tool for the

This rise, however, is not symmetric because k FLCs are evaluation of AKI and proteinuria.
cleared more readily than l. Thus, the asymmetric increase c The Serum free light chain assay signicantly increases the detection

in FLCs results in an increase in the ratio. In patients with rate of monoclonal protein when added to serum immunoxation.
severe renal impairment, a renal reference range for the k to c Urine protein electrophoresis can help distinguish between tubular or

l ratio (0.37 to 3.1) has been recommended (24). Patients with glomerular injury in patients with multiple myeloma.
autoimmune diseases can also have mildly abnormality k to l
ratios. Finally, look for a biclonal gammopathy if there is ele-
vation in both FLCs but the renal function is normal and REFERENCES
autoimmune disorders have been ruled out.
1. Kyle RA, Therneau TM, Rajkumar SV, Larson DR, Plevak MF, Offord JR,
Dispenzieri A, Katzmann JA, Melton LJ 3rd. Prevalence of monoclonal
gammopathy of undetermined signicance. N Engl J Med 354: 1362
URINARY FLC 1369, 2006
2. Knudsen LM, Hippe E, Hjorth M, Holmberg E, Westin J. Renal function
The measurement and quantication of FLC can also be per- in newly diagnosed multiple myeloma: A demographic study of 1353
formed in the urine. An elevated k to l ratio suggests a k clone, patients. The Nordic Myeloma Study Group. Eur J Haematol 53: 207
212, 1994
and a depressed ratio suggests a l clone (25). Studies suggest
3. Ivanyi B. Frequency of light chain deposition nephropathy relative to
that the urinary FLC can correlate with the serum FLC in an renal amyloidosis and Bence Jones cast nephropathy in a necropsy
individual patient (26). However, urinary FLC excretion does study of patients with myeloma. Arch Pathol Lab Med 114: 986987,
not always increase even in patients with elevated serum FLC 1990

American Society of Nephrology Onco-Nephrology Curriculum 3


4. Blade J, Fernandez-Llama P, Bosch F, Montoliu J, Lens XM, Montoto S, 17. McTaggart MP, Lindsay J, Kearney EM. Replacing urine protein electro-
Cases A, Darnell A, Rozman C, Montserrat E. Renal failure in multiple phoresis with serum free light chain analysis as a rst-line test for detecting
myeloma: Presenting features and predictors of outcome in 94 patients plasma cell disorders offers increased diagnostic accuracy and potential
from a single institution. Arch Intern Med 158: 18891893, 1998 health benet to patients. Am J Clin Pathol 140: 890897, 2013
5. Nasr SH, Valeri AM, Sethi S, Fidler ME, Cornell LD, Gertz MA, Lacy M, 18. Leung N, Gertz M, Kyle RA, Fervenza FC, Irazabal MV, Eirin A, Kumar S,
Dispenzieri A, Rajkumar SV, Kyle RA, Leung N. Clinicopathologic cor- Cha SS, Rajkumar SV, Lacy MQ, Zeldenrust SR, Buadi FK, Hayman SR,
relations in multiple myeloma: A case series of 190 patients with kidney Nasr SH, Sethi S, Ramirez-Alvarado M, Witzig TE, Herrmann SM,
biopsies. Am J Kidney Dis 59: 786794, 2012 Dispenzieri A. Urinary albumin excretion patterns of patients with cast
6. Pasquali S, Zucchelli P, Casanova S, Cagnoli L, Confalonieri R, Pozzi C, nephropathy and other monoclonal gammopathy-related kidney dis-
Ban G, Lupo A, Bertani T. Renal histological lesions and clinical syn- eases. Clin J Am Soc Nephrol 7: 19641968, 2012
dromes in multiple myeloma. Renal Immunopathology Group. Clin 19. Drayson M, Begum G, Basu S, Makkuni S, Dunn J, Barth N, Child JA.
Nephrol 27: 222228, 1987 Effects of paraprotein heavy and light chain types and free light chain
7. Rota S, Mougenot B, Baudouin B, De Meyer-Brasseur M, Lemaitre V, Michel load on survival in myeloma: An analysis of patients receiving con-
C, Mignon F, Rondeau E, Vanhille P, Verroust P, Ronco P. Multiple myeloma ventional-dose chemotherapy in Medical Research Council UK multiple
and severe renal failure: A clinicopathologic study of outcome and prog- myeloma trials. Blood 108: 20132019, 2006
nosis in 34 patients. Medicine (Baltimore) 66: 126137, 1987 20. Drayson M, Tang LX, Drew R, Mead GP, Carr-Smith H, Bradwell AR.
8. Leung N, Bridoux F, Hutchison CA, Nasr SH, Cockwell P, Fermand JP, Serum free light-chain measurements for identifying and monitoring
Dispenzieri A, Song KW, Kyle RA. Monoclonal gammopathy of renal patients with nonsecretory multiple myeloma. Blood 97: 29002902,
signicance: When MGUS is no longer undetermined or insignicant. 2001
Blood 120: 42924295, 2012 21. Katzmann JA, Abraham RS, Dispenzieri A, Lust JA, Kyle RA. Diagnostic
9. Bridoux F, Leung N, Hutchison CA, Touchard G, Sethi S, Fermand JP, performance of quantitative kappa and lambda free light chain assays
Picken MM, Herrera GA, Kastritis E, Merlini G, Roussel M, Fervenza FC, in clinical practice. Clin Chem 51: 878881, 2005
Dispenzieri A, Kyle RA, Nasr SH. Diagnosis of monoclonal gammopathy 22. Hutchison CA, Cockwell P, Stringer S, Bradwell A, Cook M, Gertz MA,
of renal signicance. Kidney Int 87: 698671, 2015 Dispenzieri A, Winters JL, Kumar S, Rajkumar SV, Kyle RA, Leung N.
10. OConnell TX, Horita TJ, Kasravi B. Understanding and interpreting Early reduction of serum-free light chains associates with renal recovery
serum protein electrophoresis. Am Fam Physician 71: 105112, 2005 in myeloma kidney. J Am Soc Nephrol 22: 11291136, 2011
11. Durie BG, Harousseau JL, Miguel JS, Blade J, Barlogie B, Anderson K, 23. Palladini G, Dispenzieri A, Gertz MA, Kumar S, Wechalekar A, Hawkins
Gertz M, Dimopoulos M, Westin J, Sonneveld P, Ludwig H, Gahrton G, PN, Schonland S, Hegenbart U, Comenzo R, Kastritis E, Dimopoulos
Beksac M, Crowley J, Belch A, Boccadaro M, Cavo M, Turesson I, MA, Jaccard A, Klersy C, Merlini G. New criteria for response to treat-
Joshua D, Vesole D, Kyle R, Alexanian R, Tricot G, Attal M, Merlini G, ment in immunoglobulin light chain amyloidosis based on free light
Powles R, Richardson P, Shimizu K, Tosi P, Morgan G, Rajkumar SV. chain measurement and cardiac biomarkers: impact on survival out-
International uniform response criteria for multiple myeloma. Leukemia comes. J Clin Oncol 30: 45414549, 2012
20: 14671473, 2006 24. Hutchison CA, Plant T, Drayson M, Cockwell P, Kountouri M, Basnayake
12. Rajkumar SV, Dimopoulos MA, Palumbo A, Blade J, Merlini G, Mateos K, Harding S, Bradwell AR, Mead G. Serum free light chain measure-
MV, Kumar S, Hillengass J, Kastritis E, Richardson P, Landgren O, Paiva ment aids the diagnosis of myeloma in patients with severe renal failure.
B, Dispenzieri A, Weiss B, LeLeu X, Zweegman S, Lonial S, Rosinol L, BMC Nephrol 9: 11, 2008
Zamagni E, Jagannath S, Sezer O, Kristinsson SY, Caers J, Usmani SZ, 25. Nakano T, Nagata A, Takahashi H. Ratio of urinary free immunoglobulin
Lahuerta JJ, Johnsen HE, Beksac M, Cavo M, Goldschmidt H, Terpos E, light chain kappa to lambda in the diagnosis of Bence Jones pro-
Kyle RA, Anderson KC, Durie BG, Miguel JF. International Myeloma teinuria. Clin Chem Lab Med 42: 429434, 2004
Working Group updated criteria for the diagnosis of multiple myeloma. 26. Abraham RS, Clark RJ, Bryant SC, Lymp JF, Larson T, Kyle RA, Katzmann
The Lancet Oncology 15: e538e548, 2014 JA: Correlation of serum immunoglobulin free light chain quantication
13. Gay-Bellile C, Bengoufa D, Houze P, Le Carrer D, Benlakehal M, with urinary Bence Jones protein in light chain myeloma. Clin Chem 48:
Bousquet B, Gourmel B, Le Bricon T. Automated multicapillary elec- 655657, 2002
trophoresis for analysis of human serum proteins. Clin Chem 49: 1909 27. Alyanakian MA, Abbas A, Delarue R, Arnulf B, Aucouturier P. Free im-
1915, 2003 munoglobulin light-chain serum levels in the follow-up of patients with
14. Katzmann JA, Kyle RA, Benson J, Larson DR, Snyder MR, Lust JA, monoclonal gammopathies: Correlation with 24-hr urinary light-chain
Rajkumar SV, Dispenzieri A. Screening panels for detection of mono- excretion. Am J Hematol 75: 246248, 2004
clonal gammopathies. Clin Chem 55: 15171522, 2009 28. Bhutani G, Nasr SH, Said SM, Sethi S, Fervenza FC, Morice WG, Kurtin
15. Steensma DP, Kyle RA. A history of the kidney in plasma cell disorders. PJ, Buadi FK, Dingli D, Dispenzieri A, Gertz MA, Lacy MQ, Kapoor P,
Contrib Nephrol 153: 524, 2007 Kumar S, Kyle RA, Rajkumar SV, Leung N. Hematologic characteristics
16. Durie BGM, Harousseau JL, Miguel JS, Blade J, Barlogie B, Anderson K, of proliferative glomerulonephritides with nonorganized monoclonal
Gertz M, Dimopoulos M, Westin J, Sonneveld P, Ludwig H, Gahrton G, immunoglobulin deposits. Mayo Clinic Proc 90: 587596, 2015
Beksac M, Crowley J, Belch A, Boccadaro M, Turesson I, Joshua D, 29. Kuhnemund A, Liebisch P, Bauchmuller K, zur Hausen A, Veelken H,
Vesole D, Kyle R, Alexanian R, Tricot G, Attal M, Merlini G, Powles R, Wasch R, Engelhardt M. Light-chain escape-multiple myeloma-an
Richardson P, Shimizu K, Tosi P, Morgan G, Rajkumar SV, Grp IMW. escape phenomenon from plateau phase: Report of the largest pa-
International uniform response criteria for multiple myeloma (vol. 20, tient series using LC-monitoring. J Cancer Res Clin Oncol 135: 477
pg 1467, 2006). Leukemia 20: 22202220, 2006 484, 2009

4 Onco-Nephrology Curriculum American Society of Nephrology


REVIEW QUESTIONS Answer: e is correct. Prior to the serum free light chain
assay, up to 5% of the multiple myeloma cases were thought
1. A 68-year-old man presents with a 1-year history of sudden- to be nonsecretory. The serum free light chain assay identies
onset nephrotic range proteinuria, easy bruising with 80% of these cases as light chain myeloma.
spontaneous black eyes, and more recent onset of
dyspnea on exertion. Which of the following tests should 3. A 65-year-old man with hypertension, mild anemia, and
be performed for screening? CKD stage IV presents with a mildly depressed serum k to
l free light chain ratio of 0.12 (normal 5 0.261.65). Se-
a. Serum protein electrophoresis
rum and urine immunoxation are negative. What is the
b. Urine protein electrophoresis
most likely possible explanation for his abnormal k to l
c. Serum immunoxation
free light chain ratio?
d. Serum free light chain assay
e. All of the above a. CKD
b. Hypertension
Answer: e is correct. The presentation is concerning for c. Chronic myelogenous leukemia
amyloidosis. Because amyloidosis is usually caused by a low d. Monoclonal gammopathy of undetermined signicance
burden plasma cell dyscrasia, all of the tests should be per- e. None of the above
formed to avoid false negativity.
Answer: d is correct. CKD and renal impairment result in a
2. Which test is most important in the screening of non- mildly elevated k to l ratio. This is because the reduction of
secretory multiple myeloma? glomerular ltration reduces the clearance of k free light chain
a. Urine immunoxation more than l. Hypertension does not affect free light chain
b. Complete blood count clearance. Chronic myelogenous leukemia is a myeloid disease
c. Serum creatinine and does not produce monoclonal protein. With the history of
d. Serum calcium CKD, the low k to l ratio is likely to be due to the presence of a
e. Serum free light chain assay monoclonal gammopathy.

American Society of Nephrology Onco-Nephrology Curriculum 5


AUTHOR QUERIES

AUTHOR PLEASE ANSWER ALL QUERIES

Q1: Please provide mailing address for correspondence.


Chapter 9: Hematopoietic Stem Cell
TransplantRelated Kidney Disease
Joseph R. Angelo, MD,* and Sangeeta Hingorani, MD, MPH
*Pediatric Nephrology and Hypertension, University of Texas Health Science Center at Houston, University of Texas
MD Anderson Cancer Center, Houston, Texas; Division of Nephrology, Seattle Childrens Hospital, Seattle,
Washington; and Department of Pediatrics, Division of Nephrology, University of Washington, Seatlle, Washington

INTRODUCTION decline in kidney function. Other methods of GFR


estimation include 24-hour urine creatinine clear-
Acute and chronic kidney diseases are common ance, inulin clearance, and use of radioactive isotopes
following hematopoietic cell transplantation (HCT) (Tc-DTPA or Cr-EDTA) or iodinated contrast agents
and can lead to long-term effects. Additionally, the (iothalamate or iohexol).
occurrence of kidney disease in the setting of HCT
can negatively affect mortality and morbidity. Eti- Dening AKI
ologies of HCT-associated kidney injury are often Current denitions of AKI are based on increases
multifactorial, including conditioning chemotherapy, in SCr and decreased urine output. Two scoring
radiation, nephrotoxic medications, sepsis, sinu- systems, RIFLE (risk, injury, failure, loss, ESRD) and
soidal obstruction syndrome (SOS), transplantation- Acute Kidney Injury Network (AKIN), have been
associated thrombotic microangiopathy (TA-TMA), developed to standardize stratication of AKI
and graft-versus-host disease (GVHD). Continued severity (5,6). RIFLE criteria include two additional
improvement in survival following HCT highlights categories (loss, ESRD) describing two post-AKI
the importance of monitoring renal function both clinical outcomes. For children, a modied version
before and after transplant and continued follow-up of RIFLE criteria, pRIFLE, has been developed (7).
of patients with CKD. Several studies have shown a correlation between
these scores and clinical outcomes (8). Recently, a
new staging criteria for AKI was created by KDIGO;
however, this newer denition has not been pro-
AKI
spectively studied in the HCT population.
Pretransplant evaluation of renal function
Epidemiology of AKI
Serum creatinine (SCr) is the most widely used The incidence of AKI varies, based on the denition
marker of kidney function in patients undergoing
of AKI, type of HCT, and chemotherapeutic condi-
HCT (1). Measurement of SCr provides estimation
tioning regimen. When AKI is dened as a doubling of
of renal function at the bedside and allows for fol-
SCr during the rst 100 days after stem cell infusion,
lowing trends in renal function. GFR prediction
the prevalence ranges from 15% to 73% (9). Severity
formulas, such as the Modication of Diet in Renal
of AKI also varies. In a study of pediatric and adult
Disease (MDRD) equation for adults and the
allogeneic HCTrecipients, up to a third of all patients
Schwartz formula in children, are available (2,3).
doubled their SCr in the rst 100 days, and 5% re-
However, several shortcomings are inherent in the quired acute dialysis (10). Severity of AKI is associ-
properties of SCr as a functional biomarker of AKI.
ated with increased risk of morbidity and mortality
These include the delay between the onset of kidney
(1113).
injury and an increase in SCr, limiting its utility to
provide the earliest window for intervention (4). In
addition, SCr is affected by factors such as age, Correspondence: Sangeeta Hingorani, Division of Nephrology,
muscle mass, and hydration status, issues particu- Seattle Childrens Hospital, 4800 Sand Point Way NE, Seattle,
Washington 98105.
larly relevant for HCT patients. Even small changes
in SCr in this population can represent signicant Copyright 2016 by the American Society of Nephrology

American Society of Nephrology Onco-Nephrology Curriculum 1


For those receiving high-dose conditioning regimens and Table 2. Urinary indices for AKI classication
allogeneic HCT, the incidence of AKI is as high as 69%. It often AKI classication BUN/Cr ratio FENa* FEurea
occurs before day 28, and risk factors include lung toxi- Prerenal AKI .20 ,1% ,35%
city, hepatic toxicity, SOS, amphotericin exposure, and sepsis Intrinsic AKI Variable .3% .35%
(14,15). Obstructive AKI Variable Variable Variable
For patients receiving reduced-intensity chemotherapy *FENa 5 (urinary Na 3 SCr)/(serum Na 3 urinary Cr).

(RIC) and allogeneic HCT, AKI occurs less frequently, later FEurea 5 (urinary urea 3 SCr)/(serum urea 3 urinary Cr).
after transplant, and less often results in the need for dialysis. A
retrospective cohort study found that 47% of RIC patients
developed AKI compared with 73% in the high-dose treatment SOS and hepatorenal syndrome (HRS) have been identied
group, developing at a median of 2660 days after transplant as an independent risk factor for AKI. HRS results in decreased
in the RIC group. Fewer RIC patients required dialysis, and resistance in the systemic and splanchnic vasculature, leading
mortality was signicantly lower (11,16,17). Compared with to renal hypoperfusion and compensatory increase in renal salt
allogeneic HCT, AKI incidence is lower in autologous HCT, and water reabsorption. It presents as oligo-anuric prerenal
occurring in approximately 21% of these patients (18). AKI with edema and low urinary sodium. Septic shock and
other causes of AKI must be ruled out. Debroitide exhibits
Causes of AKI antithrombotic and brinolytic properties and has been
Common risk factors and causes of AKI after HCT include studied for use in SOS (21).
volume depletion, sepsis, nephrotoxic medication exposure, Common medications related to AKI include vancomycin,
SOS, and GVHD (Table 1). Owing to a propensity for in- aminoglycosides, and amphotericin. Calcineurin inhibitors
creased gastrointestinal (GI) uid losses and poor oral intake, (CNIs) can lead to renal arteriolar vasoconstriction and have
HCT patients are highly susceptible to volume depletion. been associated with development of TA-TMA.
Close tracking of uid intake, urine output, uid losses via
the GI tract and insensible losses, and daily weight measure- Management of AKI
ment are required. Additional measures that can discriminate The management of AKI is mainly supportive and specic to
prerenal AKI from other types include BUN/Cr ratio, frac- the underlying cause. For situations of renal hypoperfusion,
tional excretion of sodium (FENa), and fractional excretion prompt administration of intravenous uids is required to
of urea (FEurea) (Table 2). restore effective circulating volume. However, a critical point is
Sepsis can result in decreased effective circulating volume that uid overload (FO) can itself be an independent predictor
and hypotension and is a major risk factor for AKI. Sepsis- of mortality in critically ill patients (22,23). Stem cell trans-
induced inammation leads to increased capillary permeabil- plant recipients are a population that may be particularly sen-
ity and intravascular uid leak, resulting in total body volume sitive to FO, with one study of critically ill children suggesting
overload while depleting effective circulating volume and end that .10% FO in HCT patients correlates with decreased sur-
organ perfusion (19). vival (%FO 5 [Fluid In Fluid Out]/Intensive Care Unit
GVHD is unique to HCT and likely causes tissue and Admission Weight in kilograms) (24,25). Judicious use and
endothelial damage via T cell and cytokine-mediated injury dose adjustment of antimicrobials should be used to decrease
(20). The GI mucosa is a common site of GVHD, contributing risk of AKI from nephrotoxin exposure. For those not respon-
to inadequate uid intake and increased GI losses. sive to medical interventions, dialysis is used as supportive
therapy for management of AKI-related uid and metabolic
derangements.
Table 1. Risk factors for AKI in HCT
Intravascular volume depletion
Vomiting and diarrhea associated with acute gut GVHD CKD AFTER HCT
Systemic vasodilatation
Sepsis
CKD stage 3 is dened as a GFR of ,60 mL/min per 1.73 m2 for
Renal vasoconstriction
$3 months. This is often the denition of CKD used in studies
Sinusoidal obstruction syndrome
Calcineurin inhibitors of HCT patients. The ve stages of CKD (Table 3) range from
Endothelial injury mild to ESRD. The prevalence of CKD after HCT, using this
Acute GVHD denition, is between 20% and 30% (26). Cohorts including
Calcineurin inhibitors children and adults have shown a CKD prevalence of 19% at
Total body irradiation 1 year and 7% at least 2 years after HCT, with a mean estimated
Thrombotic microangiopathy GFR of 46 mL/min per 1.73 m2 (27,28). Other chronic kidney
Tubular injury disorders following HCT include albuminuria, hypertension,
Medications: amphotericin, vancomycin and renal tubular dysfunction. Regardless of underlying cause,
Conditioning chemotherapy
CKD can progress to ESRD and increases mortality risk after

2 Onco-Nephrology Curriculum American Society of Nephrology


Table 3. Stages of CKD (67) monitoring of levels rather than complete cessation of CNI
Stage Description 2
GFR (mL/min per 1.73 m ) may be more appropriate (36,37). In patients not responsive to
1 Kidney damage with normal GFR $90 these interventions, pharmacologic therapies include rituxi-
2 Mild 6089 mab, debrotide, and eculizumab (33,38,39). Eculizumab, a
3a Moderate 3044 monoclonal immunoglobulin that binds complement factor
3b 4559 5, has been used for treatment of TA-TMA. A retrospective
4 Severe 1529 analysis of 12 patients with post-HCT TMA treated with ecu-
5 ESRD ,15 lizumab reported hematologic response of 50% and overall
survival of 33% (40). In some cases, there may be dysregula-
HCT. For those who progress to ESRD, this mortality risk can tion of the complement system; elevated levels of C5b-9, the
be as high as 90% (29). Monitoring changes in renal function membrane attack complex, have been identied; and eculizu-
and management of any existing chronic renal disease are im- mab has been used in these patients with mixed results (41).
portant to the long-term survival and quality of life for post-
HCT patients.
IDIOPATHIC OR GVHD-RELATED CKD
Clinical entities associated with CKD after HCT
Preexisting CKD before HCT Many HCT survivors will not present with a clear etiology for
Preexisting CKD is not a contraindication to HCT. A study of CKD and are labeled as idiopathic CKD. Some data support a
141 adult patients with leukemia and pretransplant kidney label of GVHD-related CKD, with renal disease resulting from
dysfunction showed that, at 1 year, these patients did not have T cell and cytokine-mediated tissue damage related to the
worse survival than those who initially had normal kidney chronic inammatory state of GVHD (42,43). The presence
function. In addition, for some cancer diagnoses, CKD is of albuminuria in this patient population may be a marker of
related to the underlying disease process and HCT can slow the the renal involvement of GVHD either directly or indirectly as
progression of CKD. described above. Clinically, albuminuria is monitored using
Important points in managing patients with preexisting urine albumin to creatinine ratio (ACR) on a spot urine sam-
CKD include accurate assessment of renal function prior to ple. Microalbuminuria is dened as an ACR of 30300 mg
HCT, appropriate changes to medication dosing, and avoid- albumin/g creatinine, whereas macroalbuminuria is dened
ance of nephrotoxins. by ACR $300 mg/g creatinine. Albuminuria is common after
HCT and can have long-term effects. In a cohort of 142 HCT
Transplantation-associated TMA patients, 94% developed albuminuria within 100 days after
TMA is dened by hemolytic anemia with erythrocyte HCT, 50% developed it at 1 year, and 4% had an ACR dem-
fragmentation, thrombocytopenia, and renal failure. It is onstrating overt proteinuria. Microalbuminuria at day 100
characterized by endothelial damage, leading to thickened was associated with a four times greater risk of CKD, and
glomerular and arteriolar vessels, the presence of fragmented macroalbuminuria was associated with a seven times greater
red blood cells, thrombosis, and endothelial cell swelling (30). risk of nonrelapse mortality (28). In a more recent study, both
Two consensus guidelines outline the clinical criteria for the micro- and macroalbuminuria in the rst 100 days after HCT
diagnosis of TA-TMA. Both require the presence of schisto- were associated with an increased risk of nonrelapse mortality
cytes on peripheral smear and an elevated lactate dehydroge- at 1 year (43). Albuminuria can also provide a readily available
nase. The BMT Clinical Trials Network also includes AKI indicator of other underlying pathologic processes, such as
(doubling of serum creatinine), unexplained CNS dysfunction, TA-TMA (37).
and a negative Coombs test. The International Guidelines from Recent consensus guidelines recommend screening urinal-
the European Group for Blood and Marrow Transplantation yses and ACR as part of the day 180 post-HCTevaluation and
include thrombocytopenia, anemia, and decreased haptoglobin then yearly screening after HCT. If macroalbuminuria is pre-
(3133). sent, more frequent monitoring every 36 months is indicated
In the setting of HCT, the incidence of TMA ranges from 2% (44). Given its utility as a marker of underlying pathology and
to 21% (31). The clinical course of TMA can be rapid with the association between macroalbuminuria and long-term
severe AKI but commonly follows a more indolent course, outcomes, renal biopsy is indicated in patients with persistent
resulting in CKD and, possibly, progression to ESRD (34). macroalbuminuria.
Risk factors for the development of TMA after HCT include
CNI use, total body irradiation, and GVHD (35). Glomerular disorders
The mainstay of TA-TMA management remains reduction Glomerular lesions related to HCT are typically discussed in
in dose or stoppage of CNI and therapeutic plasma exchange. A association with chronic GVHD (cGVHD). HCT-related
response rate of 50%63% has been reported using these two glomerular disease results in albuminuria, ranging from
interventions (33). Given that GVHD itself can be a risk factor mild to nephrotic range proteinuria. Rarely, post-HCT glo-
for the development of TA-TMA, an approach with close merular disease presents as glomerulonephritis (45). In

American Society of Nephrology Onco-Nephrology Curriculum 3


contrast to albuminuria, more severe glomerular diseases are study in pediatric HCT patients reported a 22% prevalance of
less common. Among these, membranous nephropathy (67%) hemorrhagic cystitis, developing at a median of 35 days after
and minimal change disease (33%) are the two most common HCT, which was associated with worse survival (55). Risks for
pathologies (45,46). Both tend to occur fairly late after trans- developing BK-related complications include unrelated do-
plant, 814 months, and often within several months of devel- nor, myeloablative conditioning, and GVHD (56).
opment of GVHD or lowering of immunosuppression for Despite a reported prevalence of 10%30% for BK viremia,
GVHD prophylaxis (27,47). Treatment for HCT-associated ne- less is known about the relationship between BK viremia and
phrotic syndrome is similar to that in other settings, with cor- nephropathy in the HCT population. In a study of 124 adult
ticosteroids being most common, as well as resumption of allogeneic HCT recipients, 65% developed viruria and 17%
GVHD prophylaxis with CNIs (27,48). HCT patients with developed viremia after a median follow-up of 454 days.
macroalbuminuria are likely to benet from antiproteinuric Only 2 of 21 patients had persistent viremia and biopsy-
therapy with angiotensin-converting enzyme inhibitors proven nephropathy; the remaining cases of viremia were
(ACE-Is) or angiotensin receptor blockers (ARBs) (27,49). mild and transient. BK viremia was an independent risk factor
for an increase in post-HCT creatinine (57). In children after
Hypertension HCT, BK viremia has been reported as more predictive of poor
Elevations in BP are a common complication of HCT. In a renal outcomes than viruria, supporting the use of plasma BK
retrospective analysis of a cohort of children and adults fol- polymerase chain reaction (PCR) levels when monitoring for
lowed for a median of 16 years after HCT, the prevalence of the development of nephropathy (58). For those with eleva-
hypertension was 17% (50). Risk factors associated with the de- tions in SCr suspected to be related to BK nephropathy, de-
velopment of hypertension include prior AKI, total body irra- nitive diagnosis requires kidney biopsy.
diation, autologous transplant, TA-TMA, obesity, and diabetes. Current treatment options for hemorrhagic cystitis include
Consensus recommendations are for BP measurement at pain control, continuous bladder irrigation, and urologic
each clinic visit, with a maximum interval of yearly (51). intervention for clearance of clots causing obstruction. Phar-
Thresholds for treating hypertension in the HCT population macologic interventions include cidofovir, leunomide, and
follow those of the general population, as recommended in the uoroquinolones (59). CMX100 is an oral formulation of ci-
Report from the Panel Members of the Eighth Joint National dofovir and may have less kidney toxicity (60). Another novel
Committee. For those .60 years of age, treatment goals are therapy being investigated is the use of exogenous BK-specic
based on a threshold of $150/90 mmHg. For all other adults T cells and manipulation of immunosuppression to maximize
($18 years old), including those with CKD, the threshold for the patients own immune response (61,62).
treatment initiation is $140/90 mmHg (52). In children, the
Fourth Report on the Diagnosis, Treatment, Evaluation, and
Treatment of High Blood Pressure in Children and Adoles- ESRD AND KIDNEY TRANSPLANT AFTER HCT
cents denes hypertension as a systolic or diastolic BP .95th
percentile, based on sex, age, and height (53). There are limited data on the risk of ESRD after HCT. The
Effective treatment of hypertension can decrease cardio- reported prevalence ranges from 0.4% to 4.4% (63). One study
vascular disease risk and slow the progression of CKD. Initial calculated a risk of end-stage kidney disease as being 16 times
interventions are lifestyle modications, including dietary higher than the general population 20 years after HCT (64). For
sodium restriction and regular exercise. Due to antiproteinuric those progressing to ESRD, dialysis and renal transplant remain
and renoprotective properties, ACE-I or ARB therapy should the treatment options. There are several reports of successful
be rst choice for pharmacologic treatment of hypertension. kidney transplantation in both children and adults (65,66).
For patients in which ACE-I/ARB use is contraindicated,
choice of antihypertensive should be individualized.
CONCLUSION
BK nephropathy
BK virus is a double-stranded DNA virus in the polyomavirus HCT patients are a population clearly at risk for the develop-
family with a seroprevalence of 80% reported in healthy blood ment of kidney disease, necessitating close monitoring with a
donors (54). After infection, BK remains dormant in the ur- multidisciplinary approach involving both oncologists and
othelial cells without clinical effects in immunocompetent nephrologists.
individuals. In the immunosuppressed, BK virus has been as-
sociated with nephropathy after both kidney transplant HCT;
however, hemorrhagic cystitis is more common in the HCT TAKE HOME POINTS
population. In post-HCT patients, BK virusassociated hem-
orrhagic cystitis occurs in 10%25% of patients and can lead c Acute and chronic kidney problems are common following HCT and are
to obstructive AKI, long-term urologic dysfunction, need for associated with an increased risk of nonrelapse mortality and a decrease
invasive intervention, and increased mortality. A prospective in overall survival.

4 Onco-Nephrology Curriculum American Society of Nephrology


c Micro- and macroalbuminuria are associated with an increased risk of mor- severe acute graft-versus-host disease through systemic release of in-
tality and CKD at 1 year after HCT. It is unclear if the presence of albuminuria ammatory cytokines. Biol Blood Marrow Transplant 16: 15671575, 2010
is a marker of systemic or local inammation or a marker of GVHD. 17. Kersting S, Drop SV, Theobald M, Verdonck LF. Acute renal failure after
nonmyeloablative stem cell transplantation in adults. Biol Blood Mar-
c Hypertension should be managed with ACE-Is or ARBs.
row Transplant 14: 125131, 2008
c A multidisciplinary approach is needed to insure appropriate manage-
18. Lopes JA, Jorge S, Silva S, de Almeida E, Abreu F, Martins C, do Carmo
ment of renal issues occurring after HCT. JA, Lacerda JF, Parta MM. Acute renal failure following myeloablative
autologous and allogeneic hematopoietic cell transplantation. Bone
Marrow Transplant 38: 707, 2006
REFERENCES 19. Wan L, Bagshaw SM, Langenberg C, Saotome T, May C, Bellomo R.
Pathophysiology of septic acute kidney injury: What do we really know?
Crit Care Med 36[4 Suppl]: S198S203, 2008
1. Tichelli A, Royo A, Gratwohl A. Late pulmonary, cardiovascular, and
20. Parikh CR, Coca SG. Acute renal failure in hematopoietic cell trans-
renal complications after hematopoietic stem cell transplantation and
plantation. Kidney Int 69: 430435, 2006
recommended screening practices. Hematology Am Soc Hematol
21. Keating GM. Debrotide: A review of its use in severe hepatic veno-
Educ Program 2008: 125133, 2008
occlusive disease following haematopoietic stem cell transplantation.
2. Schwartz GJ, Brion LP, Spitzer A. The use of plasma creatinine con-
Clin Drug Investig 34: 895904, 2014
centration for estimating glomerular ltration rate in infants, children,
22. Murphy CV, Schramm GE, Doherty JA, Reichley RM, Gajic O, Afessa B,
and adolescents. Pediatr Clin North Am 34: 571590, 1987
Micek ST, Kollef MH. The importance of uid management in acute
3. Levey AS, Coresh J, Greene T, Stevens LA, Zhang YL, Hendriksen S,
lung injury secondary to septic shock. Chest 136: 102109, 2009
Kusek JW, Van Lente F; Chronic Kidney Disease Epidemiology Col-
23. Boyd JH, Forbes J, Nakada TA, Walley KR, Russell JA. Fluid re-
laboration. Using standardized serum creatinine values in the modi-
suscitation in septic shock: A positive uid balance and elevated central
cation of diet in renal diseases study equation for estimating glomerular
venous pressure are associated witih increased mortality. Crit Care
ltration rate. Ann Intern Med 145: 247254, 2006
Med 39: 259265, 2011
4. Devarajan P. Emerging biomarkers of acute kidney injury. Contrib
24. Michael M, Kuehnle I, Goldstein SL. Fluid overload and acute renal failure
Nephrol 156: 203212, 2007
in pediatric stem cell transplant patients. Pediatr Nephrol 19: 9195, 2004
5. Mehta RL, Kellum JA, Shah SV, Molitoris BA, Ronco C, Warnock DG, Levin A;
25. Goldstein SL, Currier H, Graf C, Cosio CC, Brewer ED, Sachdva R.
Acute Kidney Injury Network. Acute Kidney Injury Network: Report of an
Outcome in children receiving continuous venovenous hemoltration.
initiative to improve outcomes in acute kidney injury. Crit Care 11: R31, 2007
Pediatrics 107: 13091312, 2001
6. Ronco C, Levin A, Warnock DG, Mehta R, Kellum JA, Shah S, Molitoris
BA; AKIN Working Group. Improving outcomews from acute kidney 26. Choi M, Sun CL, Kurian S, Carter A, Francisco L, Forman SJ, Bhatia S.
injury (AKI): Report on an initiative. Int J Arfit Organs 30: 373376, 2007 Incidence and predictors of delayed chronic kidney disease in long-
7. Arkan-Arikan A, Zappitelli M, Loftis LL, Washburn KK, Jefferson LS, term survivors of hematopoietic cell transplantation. Cancer 113: 1580
Goldstein SL. Modied RIFLE criteria in critically ill children with acute 1587, 2008
kidney injury. Kidney Int 71: 10281035, 2007 27. Abboud I, Peraldi MN, Hingorani S. Chronic kidney diseases in long-term
8. Hoste EA, Clermont G, Kersten A, Venkataraman R, Angus DC, De Bacquer survivors after allogeneic hematopoietic stem cell transplantation: Moni-
D, Kellum JA. RIFLE criteria for acute kidney injury are associated with hospital toring and management guidelines. Semin Hematol 49: 7382, 2012
mortality in critically ill patients: A cohort analysis. Crit Care 10: R73, 2006 28. Hingorani SR, Seidel K, Lindner A, Aneja T, Schoch G, McDonald G.
9. Hingorani SR, Guthrie K, Batchelder A, Schoch G, Aboulhosn N, Manchion Albuminuria in hematopoietic cell transplantation patients: Prevalence,
J, McDonald GB. Acute renal failure after myeloablative hematopioetic clinical associations, and impact on survival. Biol Blood Marrow
cell transplant: Incidence and risk factors. Kidney Int 67: 272277, 2005 Transplant 14: 13651372, 2008
10. Gooley TA, Chien JW, Peregam SA, Hingorani S, Sorror ML, Boeckh M, 29. Cohen EP, Piering WF, Kabler-Babbit C, Moulder JE. End-stage renal
Martin PJ, Sandmaier BM, Marr KA, Appelbaum FR, Storb R, McDonald disease (ESRD) after bone marrow transplantation: poor survival com-
GB. Reduced mortality after allogeneic hematopoietic-cell trans- pared to other causes of ESRD. Nephron 79: 408412, 1998
plantation. N Engl J Med 363: 20912101, 2010 30. Batts ED, Lazarus HM. Diagnosis and treatment of transplantation-
11. Parikh CR, Yarlagadda SG, Storer B, Sorror M, Storb R, Sandmaier B. associated thrombotic microangiopathy: Real progress or are we still
Impact of acute kidney injury on long-term mortality after non- waiting? Bone Marrow Transplant 40: 709719, 2007
myeloablative hematopoietic cell transplantation. Biol Blood Marrow 31. Ho VT, Cutler C, Carter S, Martin P, Adams R, Horowitz M, Ferrara J,
Transplant 14: 309315, 2008 Soiffer R, Giralt S. Blood and Marrow Transplant Clinical Trials Network
12. Bao YS, Xie RJ, Wang M, Feng SZ, Han MZ. An evaluation of the RIFLE Toxicity Committee consensus summary: Thrombotic microangiopathy
criteria for acute kidney injury after myeloablative allogeneic haemato- after hematopoietic stem cell transplantation. Biol Blood Marrow
poietic stem cell transplantation. Swiss Med Wkly 141: w13225, 2011 Transplant 11: 571575, 2005
13. Ando M, Mori J, Ohashi K, Akiyama H, Morito T, Tsuchiya K, Nitta K, 32. Ruutu T, Barosi G, Benjamin RJ, Clark RE, George JN, Gratwohl A, Holler E,
Sakamaki H. A comparative assessment of the RIFLE, AKIN and con- Iacobelli M, Kentouche K, Lmmle B, Moake JL, Richardson P, Soci G,
ventional criteria for acute kidney injury after hematopoietic SCT. Bone Zeigler Z, Niederwieser D, Barbui T; European Group for Blood and Marrow
Marrow Transplant 45: 14271434, 2010 Transplantation; European LeukemiaNet. Diagnostic criteria for hematopoi-
14. Kersting S, Koomans HA, Hene RJ, Verdonck LF. Acute renal failure etic stem cell transplant-associated microangiopathy: Results of a consensus
after allogeneic myeloablative stem cell transplantation: Retrospective process by an international working group. Haematologica 92: 100, 2007
analysis of incidence, risk factors and survival. Bone Marrow Transplant 33. Kim SS, Patel M, Yum K, Keyzner A. Hematopoietic stem cell transplant-
39: 359365, 2007 associated thrombotic microangiopathy: Review of pharmacologic
15. Parikh CR, Schrier RW, Storer B, Diaconescu R, Sorror ML, Maris MB, treatment options. Transfusion 55: 452458, 2015
Maloney DG, McSweeney P, Storb R, Sandmaier BM. Comparison of 34. Van Why SK, Friedman AL, Wei LJ, Hong R. Renal insufciency after bone
ARF after myeloablative and nonmyeloablative hematopioetic cell marrow transplantation in children. Bone Marrow Transplant 7: 383388, 1991
transplantation. Am J Kidney Dis 45: 502509, 2005 35. Schriber JR, Herzig GP. Transplantation-associated thrombotic
16. Liu D, Yan C, Xu L, Wang Y, Han W, Zhang X, Liu K, Huang X. Diarrhea thrombocytopenic purpura and hemolytic uremic syndrome. Semin
during the conditioning regimen is correlated with the occurrence of Hematol 34: 126133, 1997

American Society of Nephrology Onco-Nephrology Curriculum 5


36. Mii A, Shimizu A, Kaneko T, Fujita E, Fukui M, Fujino T, Utsumi K, Yamaguchi Transplant Research, and the American Society of Blood and Marrow
H, Tajika K, Tsuchiya S, Iino Y, Katayama Y, Fukuda Y. Renal thrombotic mi- Transplantation. Biol Blood Marrow Transplant 12: 138151, 2006
croangiopathy associated with chronic graft-versus-host disease after allo- 52. James PA, Oparil S, Carter BL, Cushman WC, Dennison-Himmelfarb C,
geneic hematopoietic stem cell transplantation. Pathol Int 61: 518527, 2011 Handler J, Lackland DT, LeFevre ML, MacKenzie TD, Ogedegbe O,
37. Laskin BL, Goebel J, Davies SM, Jodele S. Small vessels, big trouble in the Smith SC Jr, Svetkey LP, Taler SJ, Townsend RR, Wright JT Jr, Narva AS,
kidneys and beyond: Hematopoietic stem cell transplantation-associated Ortiz E. 2014 evidence-based guideline for the management of high
thrombotic microangiopathy. Blood 118: 14521462, 2011 blood pressure in adults: report from the panel members appointed to
38. Au WY, Ma ES, Lee TL, Ha Sy, Fung AT, Lie AK, Kwong YL. Successful the Eighth Joint National Committee. JAMA 311: 507520, 2014
treatment of thrombotic microangiopathy after haematopoietic stem 53. National High Blood Pressure Education Program Working Group on High
cell transplantation with rituximab. Br J Haematol 137: 475478, 2007 Blood Pressure in Children and Adolescents. The fourth report on the
39. Corti P, Uderzo C, Tagliabue A, Della Volpe A, Annaloro C, Tagliaferri E, diagnosis, evaluation, and treatment of high blood pressure in children
Balduzzi A. Debrotide as a promising treatment for thrombotic and adolescents. Pediatrics 114[2 Suppl 4th Report]: 555576, 2004
thrombocytopenic purpura in atient sundergoing bone marrow trans- 54. Egli A, Infanti L, Dumoulin A, Buster A, Samaridis J, Stebler C, Gosert R,
plantation. Bone Marrow Transplant 29: 542543, 2002 Hirsch HH. Prevalence of polyomavirus Bk and JC infection and repli-
40. de Fontbrune FS, Galambrun C, Sirvent A, Huynh A, Faguer S, Nguyen cation in 400 healthy blood donors. J Infect Dis 199: 837846, 2009
S, Bay JO, Neven B, Moussi J, Simon L, Xhaard A, Resche-Riggon M, 55. Cesaro S, Facchin C, Tridello G, Messina C, Calore E, Biasolo MA, Pillon
OMeara A, Fremeaux-Bacchi V, Veyradier A, Soci G, Coppo P, de M, Varotto S, Brugiolo A, Mengoli C, Pal G. A prospective study of
Latour RP. Use of eculizumab in patients with allogeneic stem cell Bi-virus-associated haemorrhagic cystitis in paediatric patients un-
transplant-associated thrombotic microangiopathy: A study from the dergoing allogeneic haematopoietic stem cell transplantation. Bone
SFGM-TC. Transplantation 99: 19531959, 2015 Marrow Transplant 41: 363370, 2008
41. Jodele S, Davies SM, Lane A, Khoury J, Dandoy C, Goebel J, Myers K, 56. Giraud G, Priftakis P, Bogdanovic G, Remberger M, Dubrulle M, Hau A,
Grimley M, Bleesing J, El-Bietar J, Wallace G, Chima RS, Paff Z, Laskin BL. Gutmark R, Mattson J, Svahn BM, Ringden O, Winiarski J, Ljungman P,
Diagnostic and risk criteria for HSCT-associated thrombotic microangiopathy: Dalianis T. BK-viruria and haemorrhagic cystitis are more frequent in
A study in children and young adults. Blood 1214: 645653, 2014 allogeneic haematopoietic stem cell transplant patients receiving full
42. Hingorani S, Guthjrie KA, Schoch G, Weiss NS, McDonald GB. Chronic conditioning and unrelated-HLA-mismatched grafts. Bone Marrow
kidney disease in long-term survivors of hematopoietic cell transplant. Transplant 41: 737742, 2008
Bone Marrow Transplant 39: 223229, 2007 57. ODonnell PH, Swanson K, Josephson MA, Artz AS, Parsad SD,
43. Hingorani S, Gooley T, Pao E, Sandmaier B, McDonald G. Urinary cytokines Ramaprasad C, Pursell K, Rich E, Stock W, van Besien K. BK virus in-
after HCT: Evidence for renal inammation in the pathogenesis of pro- fection is associated with hematuria and renal impairment in recipients
teinuria and kidney disease. Bone Marrow Transplant 49: 403409, 2014 of allogeneic hematopoetic stem cell transplants. Biol Blood Marrow
44. Pulsipher MA, Skinner R, McDonald GB, Hingorani S, Armenian SH, Cooke Transplant 15: 10381048, 2009
KR, Gracia C, Petryk A, Bhatia S, Bunin N, Nieder ML, Dvorak CC, Sung L, 58. Haines HL, Laskin BL, Goebel J, Davies SM, Yin HJ, Lawrence J, Mehta
Sanders JE, Kurtzberg J, Baker KS. National Cancer Institute, National Heart, PA, Bleesing JJ, Filipovich AH, Marsh RA, Jodele S. Blood, and not
Lung and Blood Instsitute/Pediatric Blood and Marrow Transplantation Con- urine, BK viral load predicts renal outcome in children with hemorrhagic
sortium First International consensus conference on late effects after pediatric cystitis following hematopoietic stem cell transplantation. Biol Blood
hematopoietic cell transplantation: The need for pediatric-specic long-term Marrow Transplant 17: 15121519, 2011
followup guidelines. Biol Blood Marrow Transplant 18: 334347, 2012 59. Ramos E, Drachenberg CB, Wali R, Hirsch HH. The decade of poly-
45. Chanswangphuwana C, Townamchai N, Intragumtornchai T, Bunworasate U. omavirus Bi-associated nephropathy: State of affairs. Transplantation
Glomerular diseases associated with chronic graft-versus-host disease after 87: 621630, 2009
allogeneic peripheral blood stem cell transplantation: Case reports. Trans- 60. Dropoulic LK, Cohen JI. Update on new antivirals under development
plant Proc 46: 36163619, 2014 for the treatment of double-stranded DNA virus infections. Clin Phar-
46. Hu SL. The role of graft-versus-host disease in haematopoietic cell macol Ther 88: 610619, 2010
transplantation-associated glomerular disease. Nephrol Dial Trans- 61. Babel N, Volk HD, Reinke P. BK polyomavirus infection and nephropathy:
plant 26: 20252031, 2011 The virus-immune system interplay. Nat Rev Nephrol 7: 399406, 2011
47. Brukamp K, Doyle AM, Bloom RD, Bunin N, Tomaszewski JE, Cizman B. 62. Mani J, Jin N, Schmitt M. Cellular immunotherapy for patients with
Nephrotic syndrome after hematopoietic cell transplantation: Do glo- reactivation of JC and BK polyomaviruses after transplantation. Cyto-
merular lesions represent renal graft-versus-host disease? Clin J Am therapy pii(S14653249(14)00559-3), 2014
Soc Nephrol 1: 685694, 2006 63. Touzot M, Elie C, van Massenhove J, Maillard N, Buzyn A, Fakhouri F.
48. Niscola P, Tendas A, Luo XD, Catalano G, Scaramucci L, Cupelli L, Long-term renal function after allogenic haematopoietic stem cell
Giovannini M, Ferranini M, Bondanini F, Piccioni D, Dentamaro T, Palumbo transplantation in adult patients: A single-centre study. Nephrol Dial
R, Perrotti AP, Liu QF, de Fabritiis P. The management of membranous Transplant 25: 624627, 2010
glomerulopathy in allogeneic stem cells transplantation: Updated litera- 64. Cohen EP, Drobyski WR, Moulder JE. Signicant increase in end-stage
ture. Cardiovasc Hematol Agents Med Chem 11: 6776, 2013 renal disease after hematopoietic stem cell transplantation. Bone
49. Osugi Y, Yamada H, Hosoi G, Noma H, Ikemiya M, Ishii T, Sako M. Treat- Marrow Transplant 39: 571572, 2007
ment with candesartan combined with angiotensin-converting enzyme 65. Hingorani S. Chronic kidney disease in long-term survivors of hema-
inhibitor for immunosuppressive treatment-resistant nephrotic syndrome topoietic cell transplantation: Epidemiology, pathogenesis, and treat-
after allogeneic stem cell transplantation. Int J Hematol 83: 454458, 2006 ment. J Am Soc Nephrol 17: 19952005, 2006
50. Hoffmeister PA, Hingorani SR, Storer BE, Baker KS, Sanders JE. Hy- 66. Bunin N, Guzikowski V, Rand ER, Goldfarb S, Baluarte J, Meyers K,
pertension in long-term survivors of pediatric hematopoietic cell Olthoff KM. Solid organ transplants following hematopoietic stem cell
transplantation. Biol Blood Marrow Transplant 16: 515524, 2010 transplant in children. Pediatr Transplant 14: 10301035, 2010
51. Rizzo JD, Wingard JR, Tichelli A, Lee SJ, Van Lint MT, Burns LJ, Davies 67. Levey AS, Coresh J, Balk E, Kausz AT, Levin A, Steffes MW, Hogg RJ,
SM, Ferrara JL, Soci G. Recommended screening and preventive Perrone RD, Lau J, Eknoyan G; National Kidney Foundation. National
practices for long-term survivors after hematopoietic cell trans- Kidney Foundation practice guidelines for chronic kidney disease:
plantation: Joint recommendations of the European Group for Blood and Evaluation, classicaiton, and stratication. Ann Intern Med 139: 137
Marrow Transplantation, the Center for International blood and Marrow 147, 2003

6 Onco-Nephrology Curriculum American Society of Nephrology


REVIEW QUESTIONS e. Decreased venous return related to positive pressure
ventilation
1. Following hematopoietic stem cell transplant your patient
is having a progressive rise in serum creatinine. A recent Answer: a is correct. Sepsis results in a cytokine and com-
CBC and blood smear showed anemia, thrombocytopenia, plement-stimulated systemic inammatory response causing
and the presence of schistocytes. Considering the possibility increased capillary leak, total body volume overload, and de-
of transplant-associated thrombotic microangiopathy, labs creased effective circulating volume. Sepsis is a common cause
are sent and show an elevated lactate dehydrogenase level, of AKI following HCT. Other common causes include volume
an undetectable haptoglobin level, and the Coombs test is depletion, nephrotoxic medication exposure, sinusoidal ob-
negative. Of the following, the best rst step(s) in the treat- struction syndrome, and GVHD. Volume overload has been
ment of this patient is: associated with increased morbidity and mortality in critically
ill patients and HCT specically.
a. Administration of rituximab 3. As part of annual screening after HCT, your patient is
b. Therapeutic plasma exchange (TPE) noted to have a urine albumin/creatinine ratio of 500 mg/g
c. Red blood cell and platelet transfusion Cr. Your frequency of monitoring for proteinuria should be
d. Decreasing dose of calcineurin inhibitor therapy changed to:
e. Both b and d
a. Monthly
Answer: e is correct. Calcineurin inhibitor therapy, used b. Weekly
for GVHD prophylaxis, has been associated with TA-TMA c. Continue with annual monitoring
in the HCT population. First-line therapy includes lower- d. Every 36 months
ing of CNI dose or stoppage and TPE, with the majority e. You no longer need to check for proteinuria
of patients responding to these two interventions. Other
therapies being studied include rituximab, debrotide, Answer: d is correct. Albuminuria is a common long-term
and eculizumab. consequence of HCTand has been associated with progression to
2. In septic HCT patients, the pathophysiologic mechanism CKD and nonrelapse-associated mortality. Microalbuminuria
most likely to lead to total body volume overload and is dened as an ACR of 30300 mg albumin/g creatinine,
edema is: whereas macroalbuminuria is dened by ACR $ 300 mg/g cre-
atinine. Recent consensus guidelines recommend screening ACR
a. Increased capillary leak related to sepsis-induced in- as part of the day 180 post-HCT evaluation and then yearly
ammatory response screening after HCT. If proteinuria is present, more frequent
b. Heart failure monitoring every 36 months is indicated. Additionally, for
c. Administration of pressors those with persistent macroalbuminuria, renal biopsy is indi-
d. Endothelial damage related to high-dose antibiotics cated to make denitive diagnosis of the underlying etiology.

American Society of Nephrology Onco-Nephrology Curriculum 7


Chapter 10: Radiation Nephropathy
Amaka Edeani, MBBS,* and Eric P. Cohen, MD
*Kidney Diseases Branch, National Institute of Diabetes and Digestive and Kidney Diseases, National Institutes of
Health, Bethesda, Maryland; and Nephrology Division, Department of Medicine, University of Maryland School of
Medicine, and Baltimore Veterans Affairs Medical Center, Baltimore, Maryland

INTRODUCTION Classical radiation nephropathy occurred after


external beam radiation for treatment of solid
The occurrence of renal dysfunction as a consequence cancers such as seminomas (6); the incidence has
of ionizing radiation has been known for more than declined with the advent of more effective chemo-
100 years (1,2). Initial reports termed this condition therapy. In recent years, radiation nephropathy has
radiation nephritis, but that is a misnomer, because occurred due to TBI used as part of chemo-irradiation
it is not an inammatory condition. Renal radiation conditioning just before hematopoietic stem cell
injury may be avoided by the exclusion of an ade- transplantation (HSCT) and also from targeted ra-
quate volume of kidney exposure during radiation dionuclide therapy used for instance in the treatment
therapy, but the kidneys central location can make of neuroendocrine malignancies. TBI may be myelo-
this difcult to impossible when tumors of the abdo- ablative or nonmyeloablative, with myeloablative
men or retroperitoneum are treated, or during total regimens using radiation doses of 1012 Gy to de-
body irradiation (TBI) (3). stroy or suppress the recipients bone marrow. These
doses are given in a single fraction or in nine fractions
over 3 days (4). In addition, TBI for bone marrow
BACKGROUND/CLINICAL SIGNIFICANCE transplantation (BMT) is preceded or accompanied
by cytotoxic chemotherapy, which potentiates the
Radiation nephropathy is renal injury and loss of effects of ionizing radiation (7).
function caused by ionizing radiation; this will occur CKD after HSCT occurs in 10%30% of HSCT
after sufcient irradiation of both kidneys (4). Ioniz- survivors in pediatric and adult populations (8).
ing radiation of sufcient energy disrupts chemical CKD following HSCT can have many causes includ-
bonds and knocks electrons out of atoms. It generates ing medication-induced nephrotoxicity and graft-
oxygen radicals that cause prompt DNA injury within versus-host disease (9); the role of radiation exposure
milliseconds of irradiation. This is the desired action at the time of the HSCT is also well established (4).
of therapeutic irradiation to cause death of cancer CKD has signicant patient impact by predisposing
cells. It is also the cause of injury to irradiated normal to hypertension, by altering medication pharmaco-
tissues such as the kidney. The doses of diagnostic kinetics, and by predisposing to ESRD.
X-ray are orders of magnitude less than those of ther-
apeutic irradiation; whereas there may be a very small Threshold dose
effect of carcinogenesis from a diagnostic X-ray, there Luxton identied 23 Gy as the threshold dose for
is no risk of normal tissue injury. radiation nephropathy (6), from radiation of both
The kidneys are the dose-limiting organs for kidneys when given in 20 fractions over 4 weeks. If
radiation therapy for gastrointestinal cancers, gyne- the total irradiated renal volume is ,30% of both
cologic cancers, lymphomas, and sarcomas of the kidneys, CKD will not occur from irradiation alone
upper abdomen and during TBI (5). Damage to
normal tissues can be reduced by shielding of non-
target tissues, shaping radiation beams that focus the
Correspondence: Amaka Edeani, Kidney Diseases Branch, Na-
high-dose radiation on the cancer and attempting to tional Institute of Diabetes and Digestive and Kidney Diseases,
avoid surrounding normal tissues, and fractionated National Institutes of Health, 10 Center Dr., Rm. 5-5744, Be-
thesda, Maryland 20892.
radiation dosing. Fractionation enables DNA repair
in normal cells between dosing. Copyright 2016 by the American Society of Nephrology

American Society of Nephrology Onco-Nephrology Curriculum 1


(10), although there may be injury to the small, irradiated CLINICAL PRESENTATION
volume of kidneys that leads to hypertension. In the case of
radiation nephropathy after BMT, a 10-Gy TBI single dose of The clinical features of radiation nephropathy will vary
X-rays can cause radiation nephropathy within a year after according to dose and volume of irradiation (17). Presentation
irradiation, as may 14 Gy fractionated over 3 days (11). can be acute and irreversible or subtle, with a gradual pro-
Lower radiation doses may cause kidney injury after many gressive dysfunction over years (5). There is a latent period
years of follow-up. Thus, in survivors of the Hiroshima- that is clinically silent until a stage is reached when there are
Nagasaki atomic bombs, estimated single fraction doses clinical manifestations of disease (11). These were well de-
of ,200 cGy were associated with CKD after many decades scribed by Luxton in his observation of 137 men with semi-
(12). noma who were irradiated with 22503250 cGy over 5 weeks
(3). The incidence of radiation nephropathy in his cohort was
Radiosensitivity about 20%, with four clinical syndromes as identied in Table
Radiation nephropathy occurs as a late phenomenon, usually 1. One may expect similar presentations after sufcient renal
presenting months to years after the radiation exposure. This irradiation from any source, external or internal.
latency is associated with slower cell turnover rates in renal
tissue compared to early-responding (rapidly proliferating) Acute radiation nephritis
tissue such as gastrointestinal epithelium or bone marrow (4). The onset of acute radiation nephritis is only acute relative to the
Normal renal tissue has low mitotic rates, which correlate with other variants. Patients present 612 months following radiation
delayed expression of renal injury after radiation (4). Less than exposure with fatigue, varying degrees of edema, shortness of
half of subjects exposed to threshold or higher radiation doses breath with exertion and headaches with azotemia, malignant
will develop radiation nephropathy. The determinants of in- hypertension, and severe anemia disproportionate to degree of
dividual radiosensitivity are not well known. renal failure (16,18). Malignant hypertension may present with
headaches, encephalopathy, and retinopathy. There may be as-
sociated congestive heart failure with anasarca, pleuropericardial
EXPERIMENTAL MODELS AND PATHOGENESIS effusions, and pulmonary edema. Anemia is normochromic
and normocytic with a nonaplastic marrow (17). This has
Radiation nephropathy has been reproduced in many animal been described as bone marrow transplant nephropathy
models, including in mice, rats, dogs, and nonhuman primates. (4,19) when it occurs after HSCT. Some of these cases may
A recent rat model demonstrates injury at similar doses to those present as hemolytic uremic syndrome or thrombotic throm-
relevant in humans, a similar latency period between time of bocytopenic purpura, i.e., as full-blown thrombotic microan-
irradiation and manifestations of injury, expression of injury as giopathies. Proteinuria is generally not severe, with an average
proteinuria, hypertension, and azotemia, and a similar histo- of 2 g/g urine creatinine. Prognosis has been variably related
pathology (4,13). In this model, suppression of the renin to occurrence of malignant hypertension (6) or severity of
angiotensin system is benecial, and angiotensin II infusion uid retention, with oliguria being a terminal event. Patients
exacerbates injury (14). However, in the irradiated rat, there surviving this acute phase usually are left with progression to
is no evidence of activation of the reninangiotensin system, CKD.
which suggests other countervailing mechanisms relevant in
pathogenesis. Other notable features in this model include a Chronic radiation nephritis
lack for evidence of chronic oxidative stress (15). Multiple There are two variants of chronic nephritis (6): 1) primary
aspects of pathogenesis have been tested for causality in the chronic radiation nephritisinitial presentation up to 2 years
experimental model of TBI-HSCT in which the major lethal or longer following irradiation with proteinuria and other
toxicity is renal failure (8). These include oxidative stress, cell evidence of chronic nephritis; and 2) secondary chronic radi-
proliferation, transforming growth factor-b, glomerular per- ation nephritisseen in patients who survived acute radiation
meability, brosis, reninangiotensin system, and vascular in- nephritis and continued with signs of chronic renal damage.
jury. Of these, the roles of the reninangiotensin system and Signs and symptoms are indistinguishable from renal fail-
vascular injury have not been disproven to date; the others are ure from any other cause, with hypertension, albuminuria,
either absent or do not play a causal role (8). Krochak and Baker anemia, azotemia, and small atrophic kidneys on imaging.
(16) noted a sequence of events in which the initial reaction
to irradiation was an increase in endothelial permeability with
Table 1. Clinical syndromes following renal irradiation (3)
an increase of ultraltrate extruded from the capillaries in the
Type Latent period
glomerular tuft, with an escape of increased protein and other
high-molecular-weight blood components; these events were Acute radiation nephritis (nephropathy) 612 months
transient with normal uid dynamics restored within a few Chronic radiation nephritis $18 months
Malignant hypertension 1218 months
days, but these permeability changes did appear to contribute
Benign hypertension $18 months
to eventual early and late clinical syndromes.

2 Onco-Nephrology Curriculum American Society of Nephrology


Malignant hypertension
In Luxtons series of patients, malignant hypertension devel-
oped either during the acute phase or later, 1218 months
following irradiation (6,20), with some presenting many
years after exposure. Clinical features included hypertensive
encephalopathy, retinopathy, and seizures. Renal size is
variable.

Benign hypertension
Luxton showed that hypertension may occur as a manifestation
of renal radiation injury in the absence of renal failure, with
variable degrees of proteinuria. However, other studies have
shown close correlation between degree of azotemia and
prevalence of hypertension (1). This is also valid for the hy-
Figure 1. Photomicrograph by light microscopy of a renal biopsy
pertensive CKD that occurs after HSCT (4,21).
specimen in case of BMT nephropathy (4). There is mesangiolysis
(*) and extreme widening of the space between endothelium and
Hypertension following unilateral renal irradiation glomerular basement membrane (arrow). The tubular epithelium is
Hypertension alone may be the presenting feature if only a intact, but the tubules are separated by an expanded interstitium.
single kidney is irradiated. This may occur because the periodic acid-Schiff stain; magnication, 2503. Reproduced with
irradiated kidney shrinks and creates renin-dependent hyper- permission from reference 4.
tension in the manner of a Page kidney (22). Such cases have
been successfully treated by unilateral nephrectomy (2325). trial was based on the hypothesis of mitigation: when a subject
has received sufcient irradiation to kidneys, use of an angio-
Histopathology tensin-converting enzyme inhibitor after exposure but before
There are early and late changes following renal irradiation expression of injury may be benecial in mitigating the later
(3,26). Early changes include endothelial microvascular injury injury.
with cell swelling, subendothelial expansion, and capillary Despite treatment, patients with radiation nephropathy may
loop occlusion (Figure 1). There is often mesangiolysis and evolve to ESRD and require chronic dialysis therapy or undergo
variable tubular injury. Ultrastructural examination shows kidney transplant. When this occurs after HSCT, survival on
amorphous material within the subendothelial space, which dialysis is poor (29). Such patients may be eligible for kidney
appears to extend the lamina rara interna (Figure 2). Late transplantation and could avoid the need for immunosuppres-
changes include sclerosis of interlobular and arcuate arteries, sion if the transplanted kidney was from the same donor as gave
with residual parenchymal damage with increased mesangial them their hematopoietic stem cells (30,31).
matrix, glomerular scarring, tubular atrophy, and renal mass
reduction. Fibrosis may be extensive.
RISK OF CANCER FROM IMAGING

TREATMENT High-dose ionizing radiation can cause cancer. The threshold


dose at which there is increased excess risk after a single
Because it is uncommon, there are no controlled trials to guide exposure is 34 mSv (32). For reference, an absorbed dose of
the management of radiation nephropathy. Thus, treatment 1 mGy is equivalent to an effective dose of 1 mSv, the body
of radiation nephropathy is guided by the same principles of radiation dose of a chest X-ray is ,0.1 mSv, and that of a body
treatment of any hypertensive kidney disease, including blood computed tomography (CT) scan is approximately 10 mSv.
pressure control and correction of metabolic acidosis. Standard Recent reports express concern about the possible cancer risks
management of anemia, secondary hyperparathyroidism, of radiologil imaging in patients with kidney disease. Kinsella
and electrolyte disturbances may be useful. Experimental data et al. (33) reported estimated X-ray doses received by dialysis
show that angiotensin converting enzyme inhibitors or angio- patients, and Nguyen (34) reported X-ray exposure in subjects
tensin receptor blockers are preferentially benecial in radiation undergoing workup for kidney transplantation. However,
nephropathy (14). Supportive measures are benecial including rather than create real worry, these reports actually underline
treatment of peripheral and pulmonary edema and treatment of the lack of risk, and neither one reports cancers caused by
anemia with blood transfusions and/or erythropoietin stimulat- radiation. The median cumulative doses were 22 and
ing agents. 29 mSv in each report, respectively. The single fraction equiv-
Prevention may be better than treatment. There are favor- alent doses are about half of those values, well below the
able trends for the benet of captopril to mitigate the late single fraction dose at which there is signicant excess cancer
occurrence of CKD after TBI-based HSCT (28). This clinical risk. Furthermore, because radiation-induced cancer takes

American Society of Nephrology Onco-Nephrology Curriculum 3


Figure 2. Electron micrographs of glomerular capillary loop in a case of radiation nephropathy (27). (A) Wrinkled capillary basement
membrane (BM) with variable widening of lamina rara interna (arrows). Capillary lumen (CL) contains circulating red blood cells (RBCs) and
has intact endothelial cell lining. Endothelial cells (ENs) are occasionally swollen and contain prominent organelles. Original magnication,
6,3003; reduced by 31%. (B) Higher magnication of A reveals variable widening of lamina rara interna and deposits of uffy material (*)
resembling that within the capillary lumen. Newly formed basement membrane material is also evident adjacent to the lining endothelium
(long arrows). Original magnication, 13,5003; reduced by 31%. M, mesangial cells; EP, epithelial cells; US, urinary space. Reproduced with
permission from reference 27.

510 years to develop, for the average dialysis patient, i.e., TAKE HOME POINTS
those starting dialysis at age 65 or more, radiation-induced
excess risk of cancer cannot be a major risk because the ex- c Radiation nephropathy can result from external or internal radiation
exposures.
pected remaining lifetime is 4 years or less for a 65 year old c Radiation nephropathy will not occur after diagnostic X-ray exposures.
starting dialysis (35). c Accidental or belligerent radiation exposures may result in renal radia-
Although cancer after kidney transplant is a genuine tion injury.
concern, one would expect a surfeit of leukemias if pretrans- c Radiation nephropathy is associated with mesangiolysis and glomerular
plant radiation exposure was the culprit, and leukemia after capillary thrombosis on renal biopsy; it can lead to a full blown throm-
botic microangiopathy.
kidney transplant is rare. An unjustied fear of cancer should c Mitigation of radiation nephropathy may be possible with angiotensin
not get in the way of essential radiologic imaging. However, converting enzyme inhibitors.
prevalent kidney transplant patients received cumulative
median doses of 17 mSv in another report, and 12% of that ACKNOWLEDGEMENTS
cohort had cumulative exposures of 100 mSv (36). Thus,
there may be reason for concern in individual patients, per- This work was supported (in part) by the intramural Research Pro-
haps especially in younger people with longer expected gram of the National Institutes of Health, the National Institute of
lifetimes and therefore more possibility of late radiation- Diabetes and Digestive and Kidney Diseases, and Merit Review Awards
induced cancers. Finally, the concern about exposure to IO1 BX002256 from the US Department of Veterans Affairs Bio-
ionizing radiation has affected practice in surveillance for medical Laboratory Research and Development and IO1 CX000569
nephrolithiasis; ultrasound, rather than CT may be preferable Clinical Sciences Research and Development (E.P.C., principal in-
for serial imaging. vestigator, both grants).

DISCLOSURES
FUTURE CONCERNS None.

The apparent excess of CKD in some Hiroshima-Nagasaki


survivors emphasizes that wartime or terrorist radionuclear REFERENCES Q:1
events could cause signicant renal injury in those exposed to
acutely survivable irradiation. Space exploration is another 1. Baerman G, Linser P. Review of localized and general effects of radia-
potential risk for both cardiovascular and renal disease. tion. Munch Med Wschr 7: 996, 1904

4 Onco-Nephrology Curriculum American Society of Nephrology


2. Edsall DL. The attitude of the clinician in regard to exposing patients to 22. Cohen EP. Fibrosis causes progressive kidney failure. Med Hypoth 45:
the x-ray. JAMA 47: 14251429, 1906 459462, 1995
3. Cassady JR. Clinical radiation nephropathy. Int J Radiat Oncol Biol Phys 23. Crummy Jr AB, Hellman S, Stansel Jr HC, Hukill PB. Renal hypertension
31: 12491256, 1995 secondary to unilateral radiation damage relieved by nephrectomy.
4. Cohen EP. Radiation nephropathy after bone marrow transplantation. Radiology 84: 108111, 1965
Kidney Int 58: 903918, 2000 24. Dhaliwal RS, Adelman RD, Turner E, Russo JC, Ruebner B. Radiation
5. Dawson LA, Kavanagh BD, Paulino AC, Das SK, Miften M, Li XA, Pan C, nephritis with hypertension and hyperreninemia: cure by nephrectomy.
Ten Haken RK, Schultheiss TE. Radiation-associated kidney injury. Int J J Pediat 96: 6870, 1980
Radiat Oncol Biol Phys 76: S108S115, 2010 25. Salvi S, Green DM, Brecher ML, Magoos I, Gamboa LN, Fisher JE,
6. Luxton RW. Radiation nephritis: A long term study of 54 patients. Lancet Baliah T, Afshani E. Renal artery stenosis and hypertension after ab-
2: 12211224, 1961 dominal irradiation for Hodgkins disease: Successful treatment with
7. Phillips TL, Wharam MD, Margolis LW. Modication of radiation injury to nephrectomy. Urology 21: 611615, 1983
normal tissues by chemotherapeutic agents. Cancer 35: 16781684, 1975 26. White DC. The histopathologic basis for functional decrement in late
8. Cohen EP, Pais P, Moulder JE. Chronic kidney disease after hemato- radiation injury in diverse organs. Cancer 37: 11261143, 1976
poietic stem cell transplantation. Semin Nephrol 30: 627634, 2010 27. Keane WF, Crosson JT, Staley NA, Anderson WR, Shapiro FL. Radiation-
9. Abboud I, Peraldi M, Hingorani S. Chronic kidney diseases in long-term induced renal disease. Am J Med 60: 127137, 1976
survivors after allogeneic hematopoietic stem cell transplantation: Moni- 28. Cohen EP, Bedi M, Irving AA, Jacobs E, Tomic R, Klein J, Lawton CA,
toring and management guidelines. Semin Hematol 49: 7382, 2012 Moulder JE. Mitigation of late renal and pulmonary injury after hema-
10. Dawson LA, Horgan A, Cohen EP. Kidney and ureter. In: ALERT c Ad- topoietic stem cell transplantation. Int J Radiat Oncol Biol Phys 83:
verse Late Effects of Cancer Treatment, Volume 2: Normal Tissue 292296, 2012
Specific Sites and Systems, edited by Rubin P, Constine LS, Marks LB, 29. Cohen EP, Piering WF, Kabler-Babbitt C, Moulder JE. End-stage-renal-
New York, Springer, 2014 disease after bone marrow transplantation: Poor survival compared to
11. Luxton RW. Radiation nephritis. Q J Med 22: 215242, 1953 other causes of ESRD. Nephron 79: 408412, 1998
12. Sera N, Hida A, Imaizumi M, Nakashima E, Akahoshi M. The association 30. Butcher JA, Hariharan S, Adams MB, Johnson CJ, Roza AM, Cohen EP.
between chronic kidney disease and cardiovascular risk factors in Renal transplantation for end stage renal disease following bone mar-
atomic bomb survivors. Radiat Res 179: 4652, 2013 row transplantation. Clin Transplant 13: 330335, 1999
13. Moulder JE, Fish BL. Late toxicity of total body irradiation with bone 31. Sayegh MH, Fine NA, Smith JL, Rennke HG, Milford EL, Tilney NL.
marrow transplantation in a rat model. Int J Radiat Oncol Biol Phys 16: Immunologic tolerance to renal allografts after bone marrow
15011509, 1989 transplants from the same donor. Ann Intern Med 114: 964965,
14. Cohen EP, Fish BL, Moulder JE. Mitigation of radiation injuries via 1991
suppression of the renin-angiotensin system: Emphasis on radiation 32. Brenner DJ, Doll R, Goodhead DT, Hall EJ, Land CE, Little JB, Lubin JH,
nephropathy. Curr Drug Targets 11: 14231429, 2010 Preston DL, Preston RJ, Puskin JS, Ron E, Sachs RK, Samet JM, Setlow
15. Cohen SR, Cohen EP. Chronic oxidative stress after irradiation: An RB, Zaider M. Cancer risks attributable to low doses of ionizing radia-
unproven hypothesis. Med Hypoth 80: 172175, 2013 tion: Assessing what we really know. Proc Natl Acad Sci U S A 100:
16. Krochak RJ, Baker DG. Radiation nephritis: Clinical manifestations and 1376113766, 2003
pathophysiologic mechanisms. Urology 27: 389393, 1986 33. Kinsella SM, Coyle JP, Long EB, McWilliams SR, Maher MM, Clarkson
17. Baldwin JN, Hagstrom JWC. Acute radiation nephritis. Calif Med 97: MR, Eustace JA. Maintenance hemodialysis patients have high cumu-
359362, 1962 lative radiation exposure. Kidney Int 78: 789793, 2010
18. Cohen EP, Lawton CA, Moulder JE, Becker CG, Ash RC. Clinical course 34. Nguyen KN, Patel AM, Weng FL. Ionizing radiation exposure
of late onset bone marrow transplant nephropathy. Nephron 64: 626 among kidney transplant recipients due to medical imaging during
635, 1993 the pretransplant evaluation. Clin J Am Soc Nephrol 8: 833839,
19. Cohen EP, Robbins MEC. Radiation nephropathy. Semin Nephrol 23: 2013
486499, 2003 35. Collins AJ. USRDS 2013 Annual Data Report, Bethesda, MD, National
20. Fowler JF. Brief summary of radiobiological principles in fractionated Institutes of Health, 2013
radiotherapy. Semin Radiat Oncol 2: 1621, 1992 36. De Mauri A, Brambilla M, Izzo C, Matheoud R, Chiarinotti D, Carriero A,
21. Kersting S, Hen RJ, Koomans HA, Verdonck LF. Chronic kidney dis- Stratta P, De Leo M. Cumulative radiation dose from medical imaging
ease after myelablative allogeneic hematopoietic stem cell trans- in kidney transplant patients. Nephrol Dial Transplant 27: 36453651,
plantation. Biol Blood Marrow Transplant 13: 11691175, 2007 2012

American Society of Nephrology Onco-Nephrology Curriculum 5


REVIEW QUESTIONS f. Endothelial cell swelling
Answer: c is correct. Pathologic features of radiation ne-
1. Which of the following statements is false? phropathy include endothelial cell swelling with subendothe-
a. Radiation nephropathy may result from radionuclide therapy lial expansion with amorphous material and mesangiolysis,
b. Acute radiation nephritis may be associated with throm- and late changes include arteriolar sclerosis and varying de-
botic thrombocytopenic purpura grees of tubular atrophy and interstitial brosis; there is no
c. A single dose of 10 Gy of X-rays can lead to radiation ne- evidence of glomerular leucocytic inltration.
phropathy
d. CKD has been associated solely with myeloablative total 3. Which of the following statements is false regarding hy-
body irradiation regimens pertension associated with radiation nephropathy:
e. Cytotoxic chemotherapy can potentiate the effects of a. Hypertension following unilateral radiation may be treated
ionizing radiation. with nephrectomy of the affected kidney
Answer: d is correct. CKD can complicate both myeloablative b. Acute radiation nephritis is not associated with malignant
and nonmyeloablative regimens. The other statements are correct. hypertension
c. Hypertension may occur in the absence of renal failure
2. The pathologic features of radiation nephropathy include d. ACE inhibitors and/or angiotensin receptor blockers have
the following except: been shown to be benecial in management of hypertension
a. Subendothelial expansion with amorphous material Answer: b is correct. Malignant hypertension in the setting
b. Mesangiolysis of radiation nephropathy may occur as part of the acute radi-
c. Neutrophilic inltration of the mesangium ation nephritis complex, and it may also occur as an initial
d. Arteriolar sclerosis presentation, 1218 months after irradiation. The other state-
e. Varying degrees of tubular atrophy and interstitial brosis ments are correct.

6 Onco-Nephrology Curriculum American Society of Nephrology


Chapter 11. Chemotherapy and Kidney Injury
Ilya G. Glezerman, MD,* and Edgar A. Jaimes, MD*
*Renal Service, Department of Medicine, Memorial Sloan Kettering Cancer Center, New York, New York; and

Department of Medicine, Weill Cornell Medical College, New York, New York

INTRODUCTION cell apoptosis and production of ROS (3). Cisplatin is


excreted and concentrated in the kidneys entering re-
By January 1, 2011, 4,594,732 people in the United nal tubular cells via organic cation transporter 2,
States carried the diagnosis of invasive malignancy. which is kidney specic (3).
On the other hand, with signicant advances in Initial renal toxicity manifests as a decrease in renal
anticancer therapies, the 5-year survival for cancer blood ow and subsequent decline in GFR within
patients has increased from 48.9% in 19751979 to 3 hours of cisplatin administration. These changes are
68.5% in 2006 (1). These statistics show that a sig- probably due to increased vascular resistance sec-
nicant percentage of the population is likely to be ondary to tubulo-glomerular feedback and increased
exposed to chemotherapy and suffer various short- sodium chloride delivery to macula densa. The
term and, in case of survivors, long-term adverse decline in GFR appears to be dose dependent. In
effects of treatment. a group of patients who received four cycles of
Kidneys are vulnerable to the development of drug 100 mg/m2, the 51Cr-EDTAmeasured GFR de-
toxicity due to their role in the metabolism and clined by 11.7%, whereas in patients who received
excretion of toxic agents. The kidneys receive close to three cycles of 200 mg/m2, the mean decline was
25% of cardiac output, and the renal tubules and 35.7%. This effect was noted to be lasting, as GFR
proximal segment in particular have signicant capa- was still 30% below baseline at 2 years (4). Acute
city for uptake of drugs via endocytosis or transporter tubular toxicity of cisplatin causes mitochondrial
proteins. The high rate of delivery and uptake results dysfunction, decreased ATPase activity, impaired
in high intracellular concentration of various sub- solute transport, and altered cation balance. As a re-
stances that then undergo extensive metabolism, sult, sodium and water reabsorption is decreased,
leading to formation of potentially toxic metabolites and salt and water excretion is increased, leading
and reactive oxygen species (ROS) (2). Numerous to polyuria (3). Cisplatin also causes dose-dependent
chemotherapy agents have been associated with renal magnesium wasting.
various renal toxicities including tubulointerstitial Tubulointerstitial injury is a predominant nd-
damage, glomerular disease, electrolyte abnormal- ing on pathologic examination of human kidneys
ities, hypertension, and proteinuria (Table 1). affected by cisplatin toxicity. Both proximal and
distal tubules are affected, and in patients with AKI,
there is usually acute tubular necrosis. Long-term
AGENTS WITH PREDOMONANLTY cisplatin exposure may also cause cyst formation
TUBULAR TOXICITY and interstitial brosis (Figure 1) (3).
Patients with cisplatin toxicity typically present
Platinum compounds with progressive azotemia in the setting of bland
Cisplatin (cis-dichlorodiammineplatinum)platinum urinalysis and minimal proteinuria. Although renal
coordination complex is an effective chemotherapy function improves in most patients, a subgroup of
against a wide spectrum of tumors such as testicular, patients developed permanent renal impairment.
head and neck, ovarian, lung, cervical, and bladder Hypomagnesemia is common and may be present in
cancers. Nephrotoxicity is the dose-limiting toxicity
of cisplatin. Cisplatin induces the production of ROS
and inhibits several antioxidant enzymes, leading to Correspondence: Ilya G. Glezerman, Memorial Sloan Kettering
Cancer Center, 1275 York Ave., New York, New York 10024.
oxidative stress injury. It increases renal expression of
tumor necrosis factor a, leading to increased tubular Copyright 2016 by the American Society of Nephrology

American Society of Nephrology Onco-Nephrology Curriculum 1


Table 1. Nephrotoxicity of common chemotherapy drugs
Agent Common pathologic nding Clinical syndromes
Drugs with tubular toxicity
Cisplatin ATN AKI, hypomagnesemia, renal sodium
Chronic interstitial brosis and cyst formation wasting, CKD
Ifosfamide ATN Fanconi syndrome (partial or complete)
AKI
ESRD
Methotrexate Crystal nephropathy Nonoliguric AKI
Pemetrexed ATN AKI
AIN CKD
Tubular atrophy and interstitial brosis Nephrogenic diabetes insipidus
Ipilimumab AIN AKI
Drugs with glomerular toxicity
Gemcitabine Thrombotic microangiopathy AKI
MAHA
Hypertension
Mitomycin Thrombotic microangiopathy Dose dependent: AKI, MAHA
Hypertension
Bevacizumab Thrombotic microangiopathy Proteinuria
Hypertension
Less common: Nephrotic syndrome
AKI, MAHA
VEGFR mTKI Thrombotic microangiopathy Proteinuria
Sunitinib MCD/cFSGS Hypertension
Sorafenib Less common: Nephrotic syndrome
Axitinib AKI, MAHA
Pazopanib
Drugs causing electrolyte
abnormalities
EGFR antibody Inhibition of TRMP6 in distal Hypomagnesemia
Cetuximab convoluted tubule
Panitumumab
Imatinib Unknown Hypophosphatemia
ATN, acute tubular necrosis; AIN, acute interstitial nephritis; MAHA, microangiopathic hemolytic anemia; VEGFR mTKI, vascular endothelial growth factor mul-
titarget tyrosine kinase inhibitors; MCD/cFSGS, minimal change disease and/or collapsing-like focal segmental glomerulosclerosis; EGFR, epithelial growth factor
receptor; TRMP, transient receptor potential cation channel, subfamily M, member 6.

42%100% of patients depending on total cisplatin dose and Numerous compounds have been studied to prevent cisplatin
length of exposure. Hypomagnesemia and renal magnesium nephrotoxicity but only amifostine is US Food and Drug Ad-
wasting may persist for up to 6 years after initial dose (5). ministration (FDA) approved for protection against cumulative
Renal salt wasting syndrome has been reported in up to 10% nephrotoxicity from cisplatin therapy. Amifostine is protective
of patients, manifesting as hyponatremia and severe ortho- by increasing the binding of ROS to thiol groups. Side effects,
static hypotension in the setting of high urinary sodium cost, and concerns that it also diminishes antitumor effect have
concentration. This syndrome may present 24 months after limited its use in clinical practice (3). A recent study in a murine
initiation of cisplatin therapy (6). Rare cases of thrombotic model showed that magnesium supplementation during cis-
microangiopathy have been reported in patients who were platin therapy may attenuate renal damage; however, further
also receiving bleomycin with cisplatin (4). Syndrome of in- studies in humans are needed to validate these ndings (9).
appropriate antidiuretic hormone secretion (SIADH) has Carboplatin is also a platinum-based agent with a lower po-
been documented in patients receiving vigorous hydration tential for nephrotoxicity compared with cisplatin but can be
(7) but is less common now as cisplatin-associated nausea is nephrotoxic at myeloablative doses of .800 mg/m2 (10). Ox-
treated with new-generation antiemetics, diminishing the aliplatin, another platinum compound, has no nephrotoxic
stimulus for antidiuretic hormone secretion. potential.
Vigorous hydration has been shown to reduce the incidence
of AKI in patients receiving cisplatin. Both mannitol and loop Ifosfamide
diuretics have also been used to ameliorate toxicity; however, Ifosfamide is an alkylating agent used in the treatment of a
randomized studies have not shown a clear benet (8). variety of childhood and adult malignancies. Its use, however, is

2 Onco-Nephrology Curriculum American Society of Nephrology


and secreted by the kidneys. It is a weak organic acid and is
poorly soluble in acidic urine (18).
Although it is administered over a large therapeutic range,
only high-dose methotrexate (HDMTX) therapy of .1 g/m2
has the potential for nephrotoxicity. MTX renal toxicity is pre-
sumed to be due to direct precipitation of the drug, as well to
direct toxic effects on renal tubules. In a large clinical trial of
3,887 patients treated with HDMTX, renal dysfunction occurred
in 1.8% of the subjects and was associated with a 4.4% mortality
in this group (19). Affected patients usually develop nonoliguric
and in more severe cases oliguric AKI shortly after the admin-
istration of HDMTX. Urinalysis is generally bland and without
proteinuria. Because MTX is excreted in the urine, renal im-
pairment affects the clearance of the drug. Prolonged expo-
sure to toxic levels of MTX (.10 mmol/L at 24 hours; .1
mmol/L at 48 hours, and .0.1 mmol/L at 72 hours) may lead
Figure 1. Cisplatin-induced acute tubular injury and necrosis
to life-threatening nonrenal toxicities such as prolonged cy-
(ATI/ATN). Light microscopy of the kidney biopsy specimen re-
topenias, mucositis, neurotoxicity, and hepatic dysfunction.
veals dilated tubules with attened epithelium. There is also apical
blebbing of tubular cells and drop out of tubular cells from the
MTX solubility is 10-fold higher in urine with a pH of 7.5
basement membrane. than in acidic urine, and therefore urinary alkalinization and
aggressive hydration (2.53.5 L/m 2 per 24 hours starting
12 hours prior to chemotherapy administration) are important
associated with a signicant risk for nephrotoxicity. Because it steps to establish brisk diuresis and prevent methotrexate pre-
is commonly used in children, most of the data pertaining to cipitation in the tubules. Probenecid, penicillins, salicylates,
nephrotoxicity of ifosfamide have been obtained in pediatric sulsoxazole, and nonsteroid anti-inammatory drugs may in-
patients. It has been reported that GFR goes to ,90 mL/min crease the risk of nephrotoxicity as they interfere with renal
per 1.73 m2 in 50% of and ,60 mL/min per 1.73 m2 in 11% of tubular secretion of MTX and delay excretion. Leucovorin res-
patients treated with ifosfamide, with an average reduction of cue is used in patients who develop nephrotoxicity and is aimed
GFR of 35.1 mL/min per 1.73 m2 at a median of 6 months after at prevention of nonrenal complications. Leucovorin acts as an
treatment (range, 147 months) (11). In adults, ifosfamide has antidote by bypassing blocked DHFR pathway. In patients who
been shown to reduce mean GFR from 81.5 to 68.5 mL/min have toxic levels of MTX, the leucovorin rescue dose is given
per 1.73 m2 1 year after treatment in patients with prior ex- according to established nomograms (17) with doses of 100
posure to cisplatin (12). 1,000 mg/m2 administered every 6 hours. Leucovorin rescue is
Fanconi syndrome characterized by proximal tubular dys- an effective sole therapy in patients with MTX toxicity (20).
function with variable degrees of glucosuria in the setting of Hemodialysis and hemoperfusion have been used in attempt
normoglycemia, renal phosphate and potassium wasting, to remove MTX from circulation. Although both modalities
proximal tubular acidosis, hypouricemia, and aminoaciduria result in lower MTX plasma levels immediately after treatment,
has been reported in 5% of patients treated with ifosfamide (13). there is signicant rebound effect with levels reaching 90%
Patients who receive cumulative dose ,60 g/m2 are at lower 100% of preprocedure MTX concentrations (19).
risk of renal toxicity, whereas patients receiving .100 g/m2 are Glucarpidase (carboxypeptidase-G2), a recombinant bacte-
at highest. Platinum combination therapy, renal irradiation, rial enzyme that rapidly metabolizes MTX to inactive com-
nephrectomy, and hydronephrosis are additional risk factors pounds, is able to decrease MTX plasma level .98% within
(14). Renal disease may progress even after ifosfamide is dis- 15 minutes after administration and is effective as a single
continued and may lead to ESRD (15). Although the precise dose (21,22). Although a number of studies showed rapid rates
incidence of severe kidney dysfunction after ifosfamide expo- of MTX removal in patients with HDMTX nephrotoxicity, none
sure is unknown, recent review indicates that it appears to be a had a control group, and true clinical impact of glucarpidase is
sporadic complication without clear relationship to cumulative difcult to assess (22). Time to renal recovery in most studies
dose (16). was similar to that of the leucovorin rescue case series (20,22). In
one study, glucarpidase was associated with lower risk of grade
Methotrexate 4 nonrenal toxicity if administered ,96 hours after HDMTX,
Methotrexate (MTX) is an antifolate agent that inhibits dihy- but in the same study, inadequate leucovorin rescue was pre-
drofolate reductase (DHFR), an important step in DNA synthe- dictive of nonrenal toxicities. Glucarpidase only affects extra-
sis. Fifty percent to 70% of the drug is bound to plasma proteins, cellular levels of MTX, which may explain the delay in renal
and 95% is found in the urine 30 hours after administration in recovery after MTX removal from circulation (23). The use of
subjects with normal renal function (17). MTX is both ltered glucarpidase is limited by its high cost (.$100,000/patient),

American Society of Nephrology Onco-Nephrology Curriculum 3


other patients had AKI but did not undergo a kidney biopsy
(30). Most patients improved with prompt discontinuation of
ipilimumab and steroid therapy.

AGENTS WITH PREDOMINANTLY GLOMERULAR


TOXICITY

Gemcitabine
Gemcitabine is a pyrimidine analog used in the treatment of a
variety of solid tumors. Nephrotoxicity of this agent manifests as
thrombotic microangiopathy (TMA). During early clinical
experience, TMA was reported at a low rate of 0.015%; however,
as the drug became more widely used, the incidence was noted to
increase to as high as 2.2%. TMA presents as new-onset renal
Figure 2. Gemcitabine-induced thrombotic microangiopathy. insufciency, various degrees of microangiopathic hemolytic
Light microscopy (H&E stain) of the kidney biopsy shows intra-
anemia (MAHA), and new or worsening hypertension (HTN).
arteriolar microthrombus. (Courtesy of Dr. Surya V. Seshan.)
In a single institution experience of 29 cases of gemcitabine-
induced TMA, de novo renal dysfunction or worsening of pre-
and therefore its use should be considered only after standard existing CKD was noted in all patients (32). Kidney biopsies were
supportive measures are maximized (22). For most patients, performed in four cases and showed thrombi in small blood
supportive care in the form of leucovorin rescue results in vessels, glomerular mesangiolysis, and widening of subendothe-
recovery of renal function, and additional doses of HDMTX lial space with detachment of endothelial cells from the glomer-
may be given without untoward side effects (20). ular basement membrane consistent with TMA (Figure 2). In
this study, the development of TMA was independent of cumu-
Pemetrexed lative dose, which ranged from 4 to 81 g/m2. After discontinu-
Pemetrexed is an antifolate agent that inhibits several enzymes ation of gemcitabine 28% of patients had complete recovery of
involved in DNA synthesis. This drug is not metabolized renal function, and 48% had partial recovery or stable renal
signicantly, and 70%90% of the drug is excreted unchanged function. Although patients in this study did not undergo plas-
in the urine within the rst 24 hours after administration. The mapheresis, some authors advocate this treatment for patients
half-life of pemetrexed is 3.5 hours in patients with normal renal with TMA due to gemcitabine. Literature reviews show no dif-
function but is increased in patients with renal insufciency ference in outcomes between patients treated with plasmaphe-
resulting in higher exposure to the drug (24). Pemetrexed has resis and conservative management with drug withdrawal
not been studied in patients with creatinine clearance (CrCl) (32,33).
, 45 mL/min, but a fatality was reported in a patient with Eculizumab, a monoclonal antibody directed against the com-
CrCl of 19 mL/min who received this drug (25). Mild and re- plement protein C5 approved for treatment of atypical hemolytic
versible renal toxicity has been reported in patients who received uremic syndrome, has been used to treat gemcitabine-induced
high-dose therapy ($600 mg/m2). Recently, several cases of TMA (3436). Of the six patients reported, two had complete
pemetrexed-induced tubular injury were reported (2629) includ- renal response, two had partial improvement in renal function,
ing interstitial nephritis and brosis, as well as diabetes insipidus. and two patients showed no improvement. Given the response
After discontinuation of pemetrexed, the renal function stabilized rates similar to supportive care alone, the use of eculizumab
in these patients, but did not return to pretreatment baseline. should be carefully weighed against its high cost.

Ipilimumab Mitomycin
Ipilimumab is a novel immunotherapy agent that has shown Mitomycin is an antitumor antibiotic isolated from Streptomyces
signicant promise in the treatment of metastatic melanoma. caespitosus used for treatment of gastrointestinal and other solid
Ipilimumab is a fully human monoclonal antibody directed tumors. It has been associated with life-threatening TMA with
against cytotoxic T-lymphocyte antigen-4 (CTLA-4), a key renal failure and MAHA. Mitomycin nephrotoxicity is dose de-
negative regulator of T-cell activation. Because of its immu- pendent, with the risk of TMA being 1.6% with cumulative doses
nomodulatory effects, ipilimumab has been associated with a #49 mg/m2 and as high as 30% at doses exceeding 70 mg/m2 (37).
number of immune-mediated side effects involving skin, liver, Therefore, doses exceeding 40 mg/m2 are not recommended.
gastrointestinal tract, and endocrine system (30). Renal in-
volvement appears less common, but two cases of biopsy Antiangiogenic agents
proven granulomatous acute interstitial nephritis (AIN) and In the last several years, a group of agents called antiangiogenic
one case of lupus nephritis have been reported (30,31). Three therapies have been utilized in the treatment of a variety of solid

4 Onco-Nephrology Curriculum American Society of Nephrology


TMA with intracapillary thrombi, endotheliosis, and obliter-
ated capillary loops (41). In humans, a similar spectrum of
disorders has been associated with VEGF inhibition. Hyper-
tension, proteinuria, and TMA have all been reported after
VEGF inhibition.
The effects of anti-VEGF antibody therapy on blood pressure
were recently reviewed in a meta-analysis of seven randomized
clinical trials that included 1,850 patients treated with bevacizu-
mab. In patients who received a low dose (37.5 mg/kg/dose) of the
drug, the relative risk (RR) of developing HTN was 3.0 (95% CI,
2.24.2; P,0.001). In a high-dose group (1015 mg/kg/dose), the
RR was 7.5 (95% CI, 4.213.4; P,0.001). Grade III HTN (re-
quiring therapy or more intense therapy) was observed in 8.7% of
patients in low-dose and 16.0% in high-dose groups. Proteinuria
was also more common in treated patients. In the low-dose group,
RR for proteinuria was 1.4 (95% CI, 1.11.7; P50.003), and in the
Figure 3. Vascular endothelial growth factor angiogenic
pathway inhibition. VEGF binds its receptor expressed on the
high-dose group, RR was 2.2 (95% CI, 1.62.9; P,0.001). Grade
surface of endothelial cells and podocytes triggering intracellular III (.3.5 g/24 h) proteinuria was noted in 1.8% of patients in the
and extracellular processes resulting in vascular proliferation. Sev- high-dose group vs. only 0.1% of controls (42).
eral drugs classes have been employed to inhibit the activation of TMA is the predominant glomerular lesion associated with
VEGFR and prevent angiogenesis, which is seminal for tumor anti-VEGF antibody therapy. It has been reported after in-
growth. VEGF, vascular endothelial growth factor; VEGFR, vascular travenous (41,4345) and ntraocular administration (46).
endothelial growth factor receptor; TK, intracellular VEGFR tyrosine However, concurrent mesangial IgA deposits, cryoglobuline-
kinases; mTKI, multitarget tyrosine kinase inhibitors. mic glomerulonephritis (47), and immune complexmediated
focal proliferative glomerulonephritis (48) have also been re-
ported. In patients with kidney biopsy ndings of TMA, the
tumors. Angiogenesis is seminal for tumor growth and de- clinical course varied from subnephrotic range proteinuria to
velopment of metastases, making it an attractive target for more fulminant disease with worsening renal function, hyper-
therapeutic intervention. Vascular endothelial growth factor tension, and microangiopathic anemia (41,4345).
(VEGF) is a proangiogenic factor that binds to a family of VEGF Hypertension is another major side effect of mTKI therapy.
receptors (VEGFRs), with tyrosine kinase activity (TKR). The In a meta-analysis of 13 clinical trials that included 4,999 patients
receptor binding triggers intracytoplasmic signaling pathways, with renal cell carcinoma (RCC) and other malignancies treated
leading to proliferation of endothelial cells and pericytes, with sunitinib, the incidence of all-grade hypertension was 21.6%
recruitment of endothelial cell precursors, and growth of and high grade was 6.8%. However, the RR was only statically
capillaries (38). In the kidneys, VEGF is expressed in podo- signicant for patients with high-grade hypertension at 22.72
cytes, signals glomerular endothelial cells, and regulates sur- (95% CI, 4.48115.3). Furthermore, when patients were ana-
vival of podocytes via autocrine mechanisms. VEGF maintains lyzed by the type of malignancy, only those with RCC had a
podocyte cytosolic calcium concentration and selective barrier statistically signicant RR of developing both all-grade and
to macromolecules (39). In addition, VEGF inuences BP by high-grade HTN. More pronounced effects of mTKI in RCC
up-regulating the synthesis of nitric oxide in the vascular en- may be due to higher VEGF levels in patients with RCC, resulting
dothelium and increasing the production of prostacycline re- in a more evident anti-VEGF effect. In addition, the majority of
sulting in vasodilatation (40). patients with RCC also undergo nephrectomies, resulting in re-
Several classes of antiangiogenic therapies targeting VEGF duction in renal function and decreased excretion of sunitinib,
pathway are now available (Figure 3). Bevacizumab is a block- leading to prolonged exposure to the drug (49).
ing humanized monoclonal antibody directed against VEGF. In phase 2 trials of axitinib proteinuria, all-grade proteinuria
Another class is represented by a group of drugs known as was reported in 18%36% of subjects and grade $3 was 0%5%
small molecule multitarget tyrosine kinase inhibitors (50). Proteinuria and nephrotic syndrome due to sunitinib or
(mTKIs). These agents inhibit VEGFR and a number of other sorafenib have been described in a number of case reports and
TKRs and include sunitinib, sorafenib, axitinib, and other one case series (5155). In these publications, proteinuria of up
drugs. Ramucirumab is a recombinant human monoclonal to 20 g/24 h has been reported, usually in association with new or
antibody directed against VEGFR. worsening hypertension. These side effects generally resolved
The renal effects of VEGF inhibition have been studied in after discontinuation of mTKI. Two patients had a kidney biopsy
murine models. When the VEGF gene is deleted only from that showed features of TMA in one patient and TMA and po-
podocytes in mice, they become hypertensive and proteinuric. docyte effacement in another. MAHA was not present in either
Pathologic ndings in the kidneys revealed typical features of case (53,55).

American Society of Nephrology Onco-Nephrology Curriculum 5


Several more fulminant cases of TMA with worsening renal However, if complications such as nephrotic syndrome, HTN
function, severe hypertension and MAHAwith low haptoglobin, with end-organ damage, renal insufciency, or evidence of
high lactate dehydrogenase levels, and schistocytosis have also MAHA develop, discontinuation of antiangiogenic therapy
been reported (43,5658). However, in a cohort study of 29 should be considered promptly.
patients treated with mTKIs who developed proteinuria and In addition to HTN, proteinuria, and TMA, both mTKI and
HTN and underwent a biopsy, minimal change disease and/or anti-VEGF antibody agents have been reported to cause acute AIN
collapsing-like focal segmental glomerulosclerosis (MCD/cFSGS) (54,6669). Although some cases have been conrmed by renal
was found in 20 cases (45). However, .55.5% of these pa- biopsy, in others the diagnosis was made on clinical grounds
tients had a history of nephrectomy, and hyperltration injury because biopsy was precluded by thrombocytopenia or the pres-
as a cause of MCD/cFSGS could not be completely ruled out. ence of a solitary kidney. These patients had eosinophilia, eo-
Because the putative mechanism of hypertension in patients sinophiluria, and kidney dysfunction, and renal function either
treated with anti-VEGF therapies is intricately related to the improved or stabilized after discontinuation of antiangiogenic
antitumor action of these drugs, it has been proposed that the therapy. In two cases, mTKI was administered intermittently
development of HTN could be used as biomarker of response (4 weeks on and 2 weeks off), and the patients exhibited saw
(59). Two small retrospective studies showed that the develop- tooth uctuations in eosinophilia and SCr levels, with both pa-
ment of hypertension was associated with improved oncologic rameters improving just before the initiation of the next cycle (54).
outcomes in patients with RCC treated with axitinib and
sunitinib (60,61). In a retrospective analysis of .500 patients
treated with sunitinib for RCC, the overall survival (OS) and AGENTS ASSOCIATED WITH ELECTROLYTE
progression-free survival (PFS) was more than four-fold higher ABNORMALITIES
in the group of patients who developed sunitinib-induced hy-
pertension dened as a maximum systolic blood pressure of Hypomagnesemia as a common complication of cisplatin
$140 mmHg. However, hypertensive patients had more renal therapy and Fanconi syndrome due to ifosfamide treatment
adverse events (5% versus 3%, P 5 0.013) (62). OS and PFS were have been addressed by this review already. However, a number
also improved in patients with advanced nonsmall-cell lung of targeted biological agents have been associated with electrolyte
cancer treated with bevacizumab who developed treatment- imbalance.
related hypertension. In this study, hypertension was dened
as BP .150/100 mmHg or a $20-mmHg rise in diastolic blood Cetuximab
pressure (DBP) (63). Cetuximab is a chimeric monoclonal antibody directed against
Nephrologists should be aware of these data as recommen- epithelial growth factor receptor (EGFR). The EGFR is overex-
dations to discontinue anti-VEGF therapy due to the develop- pressed in several tumors of epithelial origin, and cetuximab is
ment of hypertension and proteinuria should be weighed against often used in combination with chemotherapy for their treat-
the possible enhanced antitumor effects in this setting. An expert ment. Although in initial clinical trials hypomagnesemia was not
panel from the National Institute of Cancer has issued guidelines reported (70), numerous published reports have established a
in management of anti-VEGF therapyinduced hypertension. It link between low serum Mg21 and use of cetuximab.
recommends careful assessment of the patients prior to the ini- Active Mg21 transport in the kidney occurs predominantly in
tiation of therapy to identify those with cardiovascular risk fac- the distal convoluted tubule (DCT) and where EGFR is also ex-
tors, addressing preexisting hypertension prior to initiation of pressed. TRPM 6 (transient receptor potential cation channel,
anti-VEGF therapy, and frequent monitoring of BP particularly subfamily M, member 6) has been demonstrated to play a role in
during the rst cycle. The patients should be treated if they de- this process. The epithelial growth factor (EGF) markedly in-
velop BP .140/90 mmHg or DBP $20 mmHg higher than creases the activity of TRMP 6, leading to the hypothesis that
baseline. The panel did not make any specic recommendations EGFR activation is necessary for reabsorption of Mg21 and that
about antihypertensive regimen due to lack of data and stated blockade of EGFR leads to renal Mg21 wasting (71) by blocking
that treatment should be individualized to t the patients co- the activity of TRMP6. In one of the earlier reports, 34 patients
morbid conditions and to minimize drug interaction (64). Other on cetuximab had their Mg21 level measured at least once. Of
considerations include concurrent development of proteinuria these patients, 23% had grade 3 (,0.90.7 mg/dL) and 6% had
as a complication of anti-VEGF therapy. In this setting, it may be grade 4 (,0.7 mg/dL) hypomagnesemia (72). In another report,
appropriate to use angiotensin converting enzyme inhibitors the incidence of grade 3/4 hypomagnesemia was 27% (73). The
(ACE-Is) or angiotensin receptor blockers (ARBs) for their anti- severity of hypomagnesemia appears to correlate with duration
proteinuric effect. Additionally, use of ACE-Is or ARBs in com- of exposure and is difcult to manage. Daily infusions of up to
bination with anti-VEGF therapy may have a synergistic effect 610 g of MgSO4 were required to correct the decit in one
on OS in patients with RCC (65). Although long-term effects of cohort (73). Hypomagnesemia resolved in all cases after 4 weeks
anti-VEGF therapyinduced HTN and proteinuria are un- of discontinuation of cetuximab.
known, it is probably prudent to continue the anticancer therapy Patients who develop clinically signicant hypomagnesemia
if HTN and proteinuria are controlled with medical therapy. are also hypocalcemic due to parathyroid hormone resistance,

6 Onco-Nephrology Curriculum American Society of Nephrology


which is often seen in the presence of hypomagnesemia and therapies have been associated with kidney disease due to
resolves after Mg21 levels are normalized (72,73). interference with signaling pathways in nonmalignant cells.
In more recent randomized trials, the incidence of grade 3/4
hypomagnesemia ranged between 1.8%5.8% in the
cetuximab arm and 0%0.4% in chemotherapy or best sup-
portive care (BSC) arms. However, in these studies, the Mg21 TAKE HOME POINTS
levels were not routinely measured, which likely explain the
lower incidence of hypomagnesemia (74). c A high percentage of the US population undergoes chemotherapy
treatments and is at risk for renal complications because kidneys are a
major route of elimination of these drugs.
Panitumumab c Cisplatin and ifosfamide are major tubular toxins leading to both AKI
Panitumumab is a fully human antibody directed at EGFR and CKD.
and is used in treatment of metastatic colorectal cancer. In c Glomerular toxicity of chemotherapy most commonly manifests as
randomized trials, it has also been shown to cause low serum TMA, with gemcitabine and mitomycin as major offenders.
c VEGF antagonists such as anti-VEGF antibody and VEGFR TKI are as-
Mg21 levels, with an incidence of grade 3/4 hypomagnesemia
sociated with development of hypertension and proteinuria and in se-
ranging between 3%5% in the panitumumab arm and 0% vere cases TMA. Most patients are managed with antihypertensive
,1% in chemotherapy or BSC arms (75,76). drugs, with discontinuation of therapy only in patients who develop
nephrotic syndrome, malignant hypertension, or TMA.
Imatinib
Imatinib is a small molecule mTKI with specicity for BCR-
Abl, C-kit, and platelet-derived growth factor receptor
(PDGFR) and activity against tumors characterized by dysre-
REFERENCES
gulation of function of these enzymes. Use of imatinib has
been shown to cause hypophosphatemia. In the initial report, 1. National Cancer Institute. Surveillance, Epidemiology and End Results
hypophosphatemia developed in 25 (51%) of 49 patients who Program. Cancer Statistics. Available at: http://seer.cancer.gov/csr/
had at least one measurement of serum phosphorus. Patients 1975_2011/sections.html. Accessed March 9, 2015
with both low and normal serum phosphate levels were found 2. Perazella MA. Renal vulnerability to drug toxicity. Clin J Am Soc
Nephrol 4: 12751283, 2009
to have high urine fractional excretion of phosphate com-
3. Yao X, Panichpisal K, Kurtzman N, Nugent K. Cisplatin nephrotoxicity: A
pared with controls, but only hypophosphatemic patients had review. Am J Med Sci 334: 115124, 2007
elevated parathyroid hormone (PTH) levels (77). In another 4. Meyer KB, Madias NE. Cisplatin nephrotoxicity. Miner Electrolyte
study, 14 (39%) of 36 patients treated with imatinib developed Metab 20: 201213, 1994
hypophosphatemia and low PTH levels (78). Additionally, se- 5. Lajer H, Daugaard G. Cisplatin and hypomagnesemia. Cancer Treat
Rev 25: 4758 1999
rum phosphate levels were measured routinely in two clinical
6. Hutchison FN, Perez EA, Gandara DR, Lawrence HJ, Kaysen GA. Renal
trials of 403 patients with chronic myeloid leukemia receiving salt wasting in patients treated with cisplatin. Ann Intern Med 108: 21
imatinib. Hypophosphatemia was observed in 50% of the pa- 25, 1988
tients, but hypophosphatemia as an adverse event was only 7. Littlewood TJ, Smith AP. Syndrome of inappropriate antidiuretic hor-
reported in 3% of the patients (79). The exact mechanism mone secretion due to treatment of lung cancer with cisplatin. Thorax
39: 636637, 1984
by which imatinib causes hypophosphatemia is unknown,
8. Launay-Vacher V, Rey JB, Isnard-Bagnis C, Deray G, Daouphars M. Pre-
but it has been proposed that it may inhibit bone resorption vention of cisplatin nephrotoxicity: State of the art and recommendations
via inhibition of PDGFR and lead to decreased calcium and from the European Society of Clinical Pharmacy Special Interest Group on
phosphate efux from the bone. Lower calcium egress from Cancer Care. Cancer Chemother Pharmacol. 61: 903909, 2008
bone has been postulated to cause mild secondary hyperpara- 9. Solanki MH, Chatterjee PK, Gupta M, Xue X, Plagov A, Metz MH,
Mintz R, Singhal PC, Metz C. Magnesium protects against cisplatin-
thyroidism, which in turn leads to increased renal phosphate
induced acute kidney injury by regulating platinum accumulation. Am J
losses (77). Physiol Renal Physiol 307: F369F384, 2014
10. Isnard-Bagnis C, Launay-Vacher V, Karie S, Deray G. Anticancer drugs.
In: Clinical Nephrotoxins Renal Injury from Drug and Chemicals, edited
CONCLUSIONS by De Broe M, Porter G, Bennett W, Deray G, 3rd Ed., New York,
Springer Scientic, 511535 2008
11. Skinner R, Cotterill SJ, Stevens MC. Risk factors for nephrotoxicity after
Despite advances in diagnosis, treatment, and prevention of ifosfamide treatment in children: A UKCCSG Late Effects Group study.
chemotherapy-induced kidney injury, signicant challenges United Kingdom Childrens Cancer Study Group. Br J Cancer 82: 1636
still remain. In many cases, the only therapeutic intervention 1645, 2000
available is the discontinuation of the offending agent. Future 12. Farry JK, Flombaum CD, Latcha S. Long term renal toxicity of ifosfamide
in adult patients: 5 year data. Eur J Cancer 48: 13261331, 2012
research should be directed toward development of antidote
13. Suarez A, McDowell H, Niaudet P, Comoy E, Flamant F. Long-term
agents that protect normal cells and allow continuation of follow-up of ifosfamide renal toxicity in children treated for malignant
chemotherapy without compromising antitumor effects. In mesenchymal tumors: An International Society of Pediatric Oncology
addition to traditional cytotoxic agents, new targeted biological report. J Clin Oncol 9: 21772182, 1991

American Society of Nephrology Onco-Nephrology Curriculum 7


14. Jones DP, Spunt SL, Green D, Springate JE. Renal late effects in pa- 36. Tsai HM, Kuo E. Eculizumab therapy leads to rapid resolution of
tients treated for cancer in childhood: A report from the Childrens thrombocytopenia in atypical hemolytic uremic syndrome. Adv He-
Oncology Group. Pediatr Blood Cancer 51: 724731, 2008 matol 2014: 295323, 2014
15. Berns JS, Haghighat A, Staddon A, Cohen RM, Schmidt R, Fisher S, 37. Valavaara R, Nordman E. Renal complications of mitomycin C therapy
Rudnick MR, Tomaszewski JE. Severe, irreversible renal failure after with special reference to the total dose. Cancer 55: 4750, 1985
ifosfamide treatment. A clinicopathologic report of two patients. 38. Mena AC, Pulido EG, Guillen-Ponce C. Understanding the molecular-
Cancer 76:497500, 1995 based mechanism of action of the tyrosine kinase inhibitor: Sunitinib.
16. Akilesh S, Juaire N, Dufeld JS, Smith KD. Chronic Ifosfamide toxicity: Anticancer Drugs 21(Suppl 1): S3S11, 2010
Kidney pathology and pathophysiology. Am J Kidney Dis 63:843850, 39. Breen EC. VEGF in biological control. J Cell Biochem 102: 13581367,
2014 2007
17. Bleyer WA. The clinical pharmacology of methotrexate: new applica- 40. Yogi A, OConnor SE, Callera GE, Tostes RC, Touyz RM. Receptor and
tions of an old drug. Cancer 41: 3651, 1978 nonreceptor tyrosine kinases in vascular biology of hypertension. Curr
18. Smith SW, Nelson LS. Case les of the New York City Poison Control Opin Nephrol Hypertens 19: 169176, 2010
Center: Antidotal strategies for the management of methotrexate 41. Eremina V, Jefferson JA, Kowalewska J, Hochster H, Haas M, Weisstuch
toxicity. J Med Toxicol 4: 132140, 2008 J, Richardson C, Kopp JB, Kabir MG, Backx PH, Gerber HP, Ferrara N,
19. Widemann BC, Adamson PC. Understanding and managing metho- Barisoni L, Alpers CE, Quaggin SE. VEGF inhibition and renal throm-
trexate nephrotoxicity. Oncologist 11: 694703, 2006 botic microangiopathy. N Engl J Med 358:11291136, 2008
20. Flombaum CD, Meyers PA. High-dose leucovorin as sole therapy for 42. Zhu X, Wu S, Dahut WL, Parikh CR. Risks of proteinuria and hyperten-
methotrexate toxicity. J Clin Oncol 17: 15891594, 1999 sion with bevacizumab, an antibody against vascular endothelial
21. Widemann BC, Balis FM, Kim A, Boron M, Jayaprakash N, Shalabi A, growth factor: Systematic review and meta-analysis. Am J Kidney Dis
OBrien M, Eby M, Cole DE, Murphy RF, Fox E, Ivy P, Adamson PC. 49: 186193, 2007
Glucarpidase, leucovorin, and thymidine for high-dose methotrexate- 43. Frangie C, Lefaucheur C, Medioni J, Jacquot C, Hill GS, Nochy D.
induced renal dysfunction: Clinical and pharmacologic factors affecting Renal thrombotic microangiopathy caused by anti-VEGF-antibody
outcome. J Clin Oncol 28: 39793986, 2010 treatment for metastatic renal-cell carcinoma. Lancet Oncol 8: 177
22. Cavone JL, Yang D, Wang A. Glucarpidase intervention for delayed 178, 2007
methotrexate clearance. Ann Pharmacother 48: 897907, 2014 44. Roncone D, Satoskar A, Nadasdy T, Monk JP, Rovin BH. Proteinuria in a
23. Meyers PA, Flombaum C. High-dose methotrexate-induced renal patient receiving anti-VEGF therapy for metastatic renal cell carcinoma.
dysfunction: Is glucarpidase necessary for rescue? J Clin Oncol 29: Nat Clin Pract Nephrol 3: 287293, 2007
e180, 2011 45. Izzedine H, Escudier B, Lhomme C, Pautier P, Rouvier P, Gueutin V,
24. Villela LR, Stanford BL, Shah SR. Pemetrexed, a novel antifolate ther- Baumelou A, Derosa L, Bahleda R, Hollebecgue A, Sahali D, Soria JC.
apeutic alternative for cancer chemotherapy. Pharmacotherapy 26: Kidney diseases associated with anti-vascular endothelial growth factor
641654, 2006 (VEGF): An 8-year observational study at a single center. Medicine 93:
25. Mita AC, Sweeney CJ, Baker SD, Goetz A, Hammond LA, Patnaik A, 333339, 2014
Tolcher AW, Villalona-Calero M, Sandler A, Chaudhuri T, Molpus K, Latz 46. Pelle G, Shweke N, Van Huyen JP, Tricot L, Hessaine S, Fremeaux-
JE, Simms L, Chaudhary AK, Johnson RD, Rowinsky EK, Takimoto CH. Bacchi V, Hiesse C, Delahousse M. Systemic and kidney toxicity of
Phase I and pharmacokinetic study of pemetrexed administered every intraocular administration of vascular endothelial growth factor inhibi-
3 weeks to advanced cancer patients with normal and impaired renal tors. Am J Kidney Dis 57: 756759, 2011
function. J Clin Oncol 24: 552562, 2006 47. Johnson DH, Fehrenbacher L, Novotny WF, Herbst RS, Nemunaitis JJ,
26. Glezerman IG, Pietanza MC, Miller V, Seshan SV. Kidney tubular toxicity Jablons DM, Langer CJ, DeVore RF 3rd, Gaudreault J, Damico LA,
of maintenance pemetrexed therapy. Am J Kidney Dis 58: 817820, 2011 Holmgren E, Kabbinavar F. Randomized phase II trial comparing
27. Michels J, Spano JP, Brocheriou I, Deray G, Khayat D, Izzdine H. Acute bevacizumab plus carboplatin and paclitaxel with carboplatin and
tubular necrosis and interstitial nephritits during pemetrexed Ttherapy. paclitaxel alone in previously untreated locally advanced or metastatic
Case Rep Oncol 2: 5356, 2009 non-small-cell lung cancer. J Clin Oncol 22: 21842191, 2004
28. Stavroulopoulos A, Nakopoulou L, Xydakis AM, Aresti V, Nikolakopoulou 48. George BA, Zhou XJ, Toto R. Nephrotic syndrome after bevacizumab:
A, Klouvas G. Interstitial nephritis and nephrogenic diabetes insipidus in a Case report and literature review. Am J Kidney Dis. 49: e23e29, 2007
patient treated with pemetrexed. Ren Fail 32: 10001004, 2010 49. Zhu X, Stergiopoulos K, Wu S. Risk of hypertension and renal dys-
29. Vootukuru V, Liew YP, Nally JV Jr. Pemetrexed-induced acute renal function with an angiogenesis inhibitor sunitinib: Systematic review and
failure, nephrogenic diabetes insipidus, and renal tubular acidosis in a meta-analysis. Acta Oncol 48: 917, 2009
patient with non-small cell lung cancer. Med Oncol 23: 419422, 2006 50. Izzedine H, Massard C, Spano JP, Goldwasser F, Khayat D, Soria JC.
30. Izzedine H, Gueutin V, Gharbi C, Mateus C, Robert C, Routier E, Thomas VEGF signalling inhibition-induced proteinuria: Mechanisms, signi-
M, Baumelou A, Rouvier P. Kidney injuries related to ipilimumab. In- cance and management. Eur J Cancer 46: 439448, 2010
vestig New Drugs 32: 769773, 2014 51. Patel TV, Morgan JA, Demetri GD, George S, Maki RG, Quigley M,
31. Fadel F, El Karoui K, Knebelmann B. Anti-CTLA4 antibody-induced Humphreys BD. A preeclampsia-like syndrome characterized by re-
lupus nephritis. N Engl J Med 361: 211212, 2009 versible hypertension and proteinuria induced by the multitargeted
32. Glezerman I, Kris MG, Miller V, Seshan S, Flombaum CD. Gemcitabine kinase inhibitors sunitinib and sorafenib. J Natl Cancer Inst 100: 282
nephrotoxicity and hemolytic uremic syndrome: Report of 29 cases 284, 2008
from a single institution. Clin Nephrol 71: 130139, 2009 52. Obhrai JS, Patel TV, Humphreys BD. The case/progressive hyperten-
33. Gore EM, Jones BS, Marques MB. Is therapeutic plasma exchange in- sion and proteinuria on anti-angiogenic therapy. Kidney Int 74: 685
dicated for patients with gemcitabine-induced hemolytic uremic syn- 686, 2008
drome? J Clin Apheresis 24: 209214, 2009 53. Overkleeft EN, Goldschmeding R, van Reekum F, Voest EE, Verheul
34. Starck M, Wendtner CM. Use of eculizumab in refractory gemcitabine- HM. Nephrotic syndrome caused by the angiogenesis inhibitor
induced thrombotic microangiopathy. Br J Haematol 164: 894896, sorafenib. Ann Oncol 21: 184185, 2010
2014 54. Jhaveri KD, Flombaum CD, Kroog G, Glezerman IG. Nephrotoxicities
35. Al Ustwani O, Lohr J, Dy G, Levea C, Connolly G, Arora P, Iyer R. Eculi- associated with the use of tyrosine kinase inhibitors: A single-center
zumab therapy for gemcitabine induced hemolytic uremic syndrome: experience and review of the literature. Nephron Clin Pract 117: c312
Case series and concise review. J Gastrointestinal Oncol 5: E30E33, 2014 c319, 2011

8 Onco-Nephrology Curriculum American Society of Nephrology


55. Bollee G, Patey N, Cazajous G, Robert C, Goujon JM, Fakhouri F, 67. Izzedine H, Brocheriou I, Rixe O, Deray G. Interstitial nephritis in a pa-
Bruneval P, Noel LH, Knebelmann B. Thrombotic microangiopathy tient taking sorafenib. Nephrol Dial Transplant 22 :2411, 2007
secondary to VEGF pathway inhibition by sunitinib. Nephrol Dial 68. Khurana A. Allergic interstitial nephritis possibly related to sunitinib
Transplant 24: 682685, 2009 use. Am J Geriatr Pharmacother 5: 341344, 2007
56. Kapiteijn E, Brand A, Kroep J, Gelderblom H. Sunitinib induced hy- 69. Barakat RK, Singh N, Lal R, Verani RR, Finkel KW, Foringer JR. Interstitial
pertension, thrombotic microangiopathy and reversible posterior leu- nephritis secondary to bevacizumab treatment in metastatic leiomyo-
kencephalopathy syndrome. Ann Oncol 18: 17451747, 2007 sarcoma. Ann Pharmacother 41: 707710, 2007
57. Choi MK, Hong JY, Jang JH, Lim HY. TTP-HUS associated with sunitinib. 70. Cunningham D, Humblet Y, Siena S, Khayat D, Bleiberg H, Santoro A, bets
Cancer Res Treat 40: 211213, 2008 D, Mueser M, Harstrick A, Verslype C, Chau I, van Cutsen E. Cetuximab
58. Levey SA, Bajwa RS, Picken MM, Clark JI, Baron K, Leehey DJ. monotherapy and cetuximab plus irinotecan in irinotecan-refractory
Thrombotic microangiopathy associated with sunutinib, a VEGF in- metastatic colorectal cancer. N Engl J Med 351: 337345, 2004
hibitor, in a patient with factor V Leiden mutation. Nephrol Dialysis 71. Izzedine H, Bahleda R, Khayat D, Massard C, Magne N, Spano JP, Soria
Transplant Plus 3: 154156, 2008 JC. Electrolyte disorders related to EGFR-targeting drugs. Crit Rev
59. van Heeckeren WJ, Ortiz J, Cooney MM, Remick SC. Hypertension, Oncol Hematol 73: 213219, 2010
proteinuria, and antagonism of vascular endothelial growth factor sig- 72. Schrag D, Chung KY, Flombaum C, Saltz L. Cetuximab therapy and
naling: Clinical toxicity, therapeutic target, or novel biomarker? J Clin symptomatic hypomagnesemia. J Natl Cancer Inst 97: 12211224, 2005
Oncol 25: 29932995, 2007 73. Fakih MG, Wilding G, Lombardo J. Cetuximab-induced hypomagne-
60. Rixe O, Billemont B, Izzedine H. Hypertension as a predictive factor of semia in patients with colorectal cancer. Clin Colorectal Cancer 6: 152
Sunitinib activity. Ann Oncol 18: 1117, 2007
156, 2006
61. Rixe O, Dutcher JP, Motzer RJ, Wilding G, Stadler WM, Kim S, Tarazi J,
74. Fakih M. Management of anti-EGFR-targeting monoclonal antibody-
Motzer RJ. Association between diastolic blood pressure (DBP) $ 90 mm
induced hypomagnesemia. Oncology (Williston Park) 22: 7476, 2008
Hg and efcacy in patients (pts) with metastatic renal cell carcinoma
75. Peeters M, Price TJ, Cervantes A, Sobrero AF, Ducreux M, Hotko Y,
(MRCC) receiving axitinib (AG-013736; AG). Ann Oncol 19(Suppl 8):
Andre T, Chan E, Lordick F, Punt CJ, Strickland AH, Wilson G, Ciuleanu
viii189, 2008
TE, Roman L, Van Cutsen E, Tzekova V, Collins S, Oliner KS, Rong A,
62. Rini BI, Cohen DP, Lu DR, Chen I, Hariharan S, Gore ME, Figlin RA,
Gansert J. Randomized phase III study of panitumumab with uoro-
Baum MS, Motzer RJ. Hypertension as a biomarker of efcacy in pa-
uracil, leucovorin, and irinotecan (FOLFIRI) compared with FOLFIRI
tients with metastatic renal cell carcinoma treated with sunitinib. J Natl
alone as second-line treatment in patients with metastatic colorectal
Cancer Inst 103: 763773, 2011
cancer. J Clin Oncol 28: 47064713, 2010
63. Dahlberg SE, Sandler AB, Brahmer JR, Schiller JH, Johnson DH. Clinical
76. Van Cutsem E, Peeters M, Siena S, Humblet Y, Hendlisz A, Neyns B,
course of advanced non-small-cell lung cancer patients experiencing
hypertension during treatment with bevacizumab in combination with Canon JL, Van Laethem JL, Maurel J, Richardson G, Wolf M, Amado
carboplatin and paclitaxel on ECOG 4599. J Clin Oncol 28: 949954, RG. Open-label phase III trial of panitumumab plus best supportive
2010 care compared with best supportive care alone in patients with
64. Maitland ML, Bakris GL, Black HR, Chen HX, Durand JB, Elliott WJ, Ivy chemotherapy-refractory metastatic colorectal cancer. J Clin Oncol
SP, Leier CV, Lindenfeld J, Liu G, Remick SC, Steingart R, Tang WH. 25: 16581664, 2007
Initial assessment, surveillance, and management of blood pressure in 77. Berman E, Nicolaides M, Maki RG, Fleisher M, Chanel S, Scheu K,
patients receiving vascular endothelial growth factor signaling pathway Wilson BA, Heller G, Sauter NP. Altered bone and mineral metabolism
inhibitors. J Natl Cancer Inst 102: 596604, 2010 in patients receiving imatinib mesylate. N Engl J Med 354:20062013,
65. McKay RRRGE, Lin X, Simantov R, Choueiri TK. Impact of angiotensin 2006
system inhibitors on outcomes in patients with metastatic renal cell 78. Osorio S, Noblejas AG, Duran A, Steegmann JL. Imatinib mesylate
carcinoma: Results from a pooled clinical trials database. J Clin Oncol induces hypophosphatemia in patients with chronic myeloid leukemia
32(4 Suppl): 437, 2014 in late chronic phase, and this effect is associated with response. Am J
66. Winn SK, Ellis S, Savage P, Sampson S, Marsh JE. Biopsy-proven Hematol 82:394395, 2007
acute interstitial nephritis associated with the tyrosine kinase inhibitor 79. Owen S, Hateld A, Letvak L. Imatinib and altered bone and mineral
sunitinib: a class effect? Nephrol Dial Transplant 24: 673675, 2009 metabolism. N Engl J Med 355: 627, 2006

American Society of Nephrology Onco-Nephrology Curriculum 9


REVIEW QUESTIONS cause nephrotic syndrome, and thrombotic microangiopathy
and Fanconi syndrome are rare.
1. The best treatment options for gemcitabine induced throm-
botic microangiopathy are: 3. A 55-year-old man with a history of metastatic renal cell
a. Plasmapheresis carcinoma was begun on treatment with sunitinib (VEGFR
b. Administration of eculizumab TKI). Two months after starting the treatment, he was
c. Discontinuation of gemcitabine and best supportive care noted to have a BP of 154/90 mmHg, and his random
d. All of the above urinary protein to creatinine ratio was 2.3. The patient was
asymptomatic. His renal function remained normal, and
Answer: c is correct. Whereas both plasmapheresis and there was no evidence of hemolysis on his blood work. The
eculizumab have been used in the treatment of gemcitabine- next step is:
induced thrombotic microangiopathy, there is little evidence
that outcomes of these treatments are superior to supportive a. Discontinue sunitinib and offer best supportive care
care alone. b. Begin antihypertensive therapy aimed at reducing his
blood pressure to ,140/90 mmHg and continue to mon-
2. Which presentation is most consistent with cisplatin neph- itor urinary protein to creatinine ratio closely
c. Reduce sunitinib dose
rotoxicity? d. Switch the patient from sunitinib to sorafenib
a. Elevated serum creatinine, minimal proteinuria, hypo-
magnesemia Answer: b is correct. There are no evidence-based rec-
b. Hypertension, elevated serum creatinine, low platelet
ommendations on management of proteinuria and hyper-
count
c. Nephrotic range proteinuria, hypertension, and edema tension induced by VEGF inhibitors. In practice, these
d. Elevated serum creatinine, hypophosphatemia, glucosuria agents are generally continued unless patients develop ne-
phrotic syndrome, malignant hypertension, or thrombotic
Answer: a is correct. Cisplatin toxicity involves damage to microangiopathy. Reduction of the sunitinib dose may be
the tubulo-interstitial compartment and manifests as AKI with attempted as a next step if hypertension is difcult to con-
relatively normal urinalysis. Hypomagnesemia is a common trol (answer c). Sorafenib is likely to have a similar side
manifestation of cisplatin tubular toxicity. Cisplatin does not effect prole (answer d).

10 Onco-Nephrology Curriculum American Society of Nephrology


Chapter 12: Pharmacokinetics of Chemotherapeutic
Agents in Kidney Disease
Sheron Latcha, MD, FASN
Department of Medicine, Memorial Sloan Kettering Cancer Center, New York, New York

INTRODUCTION the isotope dilution mass spectroscopy (IDMS)


creatinine assay, the variability in creatinine assays
The liver and kidneys serve as the major pathways for likely affected the PK and PD data from past studies,
drug metabolism and elimination, with much smaller and therefore, the resulting dosing recommendations
contributions from the fecal and reticuloendothelial being used currently (7). What remains largely un-
systems. As shown in Figure 1, some unique aspects of addressed in all PK and PD studies to date is that
renal physiology that make the kidneys particularly CKD can signicantly alter nonrenal clearance and
susceptible to drug exposure and injury include 1) a modify bioavailability of drugs predominantly me-
high blood ow rate and therefore high drug delivery tabolized by the liver and intestine. It has been shown
rate to the kidney (blood ow to the kidney approx- that CKD suppresses various liver metabolic enzymes
imates 25% of cardiac output); 2) the medullas con- (CPT2C9, CYP2C19, and CYP3A4), and these effects
siderable concentrating ability, which enhances local are clinically signicant (8). GFR is the metric used to
drug tissue concentration; 3) the presence of organic guide dose adjustment, and as shown in Table 1 a
anion transporters within the tubules, which allow number of equations are available to calculate GFR.
nephrotoxic medications and their toxic metabolites In cancer patients, compared with Tc99mDPTA
to become concentrated within the tubules; and 4) clearance, the Martin and Wright formulae seem
the presence of renal enzymes, which can form toxic to have the best concordance, followed by the
metabolites and reactive oxygen species (CYP450 and Chronic Kidney Disease Epidemiology Collaboration
avin-containing monooxygenase) (1,2). (CKD-EPI) equation (60.2%, 56.5%, and 56.3%, re-
Cancer drugs have been demonstrated to cause spectively). However, there were similar levels con-
nephrotoxicity via direct tubular injury, tubular ob- cordance in dosage selection between the assorted
struction, injury to the tubulointerstitium, and glo- formulae and Tc99mDPTA clearance when selecting
merular damage. Certainly, the prevalence of renal carboplatin dose (9), so the variations in concor-
insufciency in cancer patients, and the kidneys role in dance may not appreciably change nal dose recom-
drug metabolism, has implications for the choice and mendations.
dosing of chemotherapeutic agents. In this section,
recommendations for dose modications for some
of the more frequently used chemotherapeutic agents, CARBOPLATIN
which require adjustments in patients with various
levels of CKD, will be discussed. For a more complete Notwithstanding all of these limitations, carbo-
discussion of all chemotherapeutic agents that require platin is one of the few chemotherapeutic agents with
renal dosing, the reader is referred elsewhere (36). good prospective PK and PD data in CKD patients
Published guidelines for dose modication of (1012). It is the third most commonly prescribed
chemotherapy for cancer patients with CKD are cytotoxic agent (3). About 70% of the administered
largely based on limited pharmacokinetic (PK) and dose is eliminated by the kidneys, and the drug has
pharmacodynamic (PD) data and often on studies of
poor quality (physician-initiated postmarketing stud-
ies, small sample sizes). Historically, the Cockcroft Correspondence: Department of Medicine, Memorial Sloan
and Gault (CG) formula was most often used to Kettering Cancer Center, 1275 York Ave., Suite 1204b, New York,
New York 10065.
estimate GFR, and this equation has been shown to
overestimate GFR. Additionally, before the advent of Copyright 2016 by the American Society of Nephrology

American Society of Nephrology Onco-Nephrology Curriculum 1


The drug can be administered to hemodialysis (HD)
patients to achieve an AUC of 46 such that carboplatin
dose (mg)=Target AUCx25. The drug is not avidly protein
bound shortly after administration, and approximately 70%
is cleared by HD if performed within several hours after in-
fusion (15). Importantly, a majority of carboplatin becomes
protein bound after 24 hours (12), so carboplatin administra-
tion should be coordinated to perform dialysis within 24 hours
of dosing (16).

CISPLATIN

Although they are members of the same chemical family, cis-


platin is considered to be superior therapy for specic tumor
types compared with carboplatin. Unfortunately, nephrotoxicity
is the dose-limiting side effect of this very effective chemother-
apeutic agent (1719). In the earliest reports of renal toxicity,
incidence rates of 28%36% were reported in patients
Figure 1. Susceptibility of the kidneys to drug exposure and
receiving a single dose of 50 mg/m2 (Bristol Meyer Packaging),
delivery. but the severity of renal toxicity decreased following institution
of vigorous hydration protocols with normal saline (NS). The
chloride in NS decreases the formation of toxic reactive plati-
only rarely been associated with AKI at high doses (1,600 num compounds (20). Initial declines in a GFR range from 12%
2,400 mg/m2) following bone marrow transplant (BMT) (13,14). to 19% (2123), but some patients have shown improvement in
At the usual doses range from 400 to 600 mg/m2, the drug is renal function over time, implying that renal recovery is possible
much less nephrotoxic. Neuropathy and myelosuppression in some cases (23,24). However, there is usually persistent sub-
are its main toxicities. Calverts formula is used to calculate clinical renal injury following cisplatin exposure (20).
the area under the curve (AUC): carboplatin dose (mg) 5 A detailed understanding of the mechanisms by which
target AUC 3 (GFR 1 25). cisplatin induces renal toxicity remains unclear. The kidney

Table 1. eGFR formulas in cancer patients

2 Onco-Nephrology Curriculum American Society of Nephrology


selectively accumulates cisplatin and its analogues to a greater follow in the range of 314 hours after drug administration.
extent than other organs(18). On microscopic examination, Subsequent dose adjustments were based on signs of neurotox-
cisplatin produces a tubulointerstitial lesion, with acute icity or myelosuppression (33). For HD patients with urine
tubular necrosis being the predominant lesion, whereas the output, hydration and Mesna are needed to prevent hemor-
glomeruli are spared (20,25). Cisplatin-induced tubular dam- rhagic cystitis, and a hydration protocol is outlined elsewhere
age affects the PK of subsequent cisplatin doses and causes a (33,34).
decrease in the percentage of cisplatin excreted and an in-
crease in the AUC following sequential doses of the drug
(26,27). CYCLOPHOSPHAMIDE
The drug is 90% protein bound within 2 hours after infusion,
and 30% is excreted in the urine within 24 hours. Dosing There are no clear guidelines for dose adjustment of cyclo-
guidelines for patients with CKD are empiric. Kintzel recom- phosphamide in the setting of renal insufciency. Although
mends for a creatinine clearance (CrCl) of 4660 mL/min, give cyclophosphamide is largely cleared by hepatic metabolism, up
75% of the usual dose; for a CrCl of 3145 mL/min, give 50% of to 60% of the total dose is eliminated by the kidney as the parent
the usual dose, and for a CrCl ,30 mL/min, the drug is not drug or metabolites (35), and renal insufciency is associated
recommended (4). Arnoff recommends for CrCl of 1050 in changes in the PK prole of the parent drug and its metab-
mL/min, give 75% of the usual dose, and for CrCl ,10 mL/min, olites. Aronoff recommends giving 75% of the usual dose for
give 50% of the usual dose. The manufacturer recommends GFR ,10 mL/min. Because the AUC of cyclophosphamide is
withholding repeat administration of the drug until the serum increased in HD patients, it is recommended that the dose be
creatinine concentration is ,1.5 mg/dL. Cisplatin has been suc- reduced by 25% in this group (36,37). The drug should be
cessfully administered to HD patients with similar tolerance as in given after HD because the drug and its metabolites are di-
patients with normal renal function. Because the drug is highly alyzable.
and irreversibly protein bound and because free cisplatin is well
dialyzed, drug that is dialyzed off cannot be replaced by bound
drug. As such, cisplatin must be given on a nondialysis day. For METHOTREXATE (MTX)
HD patients, it is recommended that the initial dose be reduced
by 50%, or 2550 mg/m2 every 36 weeks (16). Renal excretion of this antifolate agent is 60% and 94% when
the drug is administered over 6 and 24 hours, respectively.
Nephrotoxicity has been observed at doses exceeding 1 g/m2
IFOSFAMIDE (38) and results from intratubular precipitation of MTX and
its metabolites in the distal tubules, which causes an obstruc-
Although hemorrhagic cystitis is the predominant toxicity of this tive tubulopathy and decreased glomerular ltration (4,39).
alkylating agent, renal toxicity can be dose limiting. A number of Pretreatment with cisplatin has been reported to increase
histopathologic changes in the kidney have been observed with MTX toxicity, possibly by decreasing renal clearance (40).
ifosfamide and include segmental glomerular sclerosis, tubu- AKI prolongs extrarenal toxicity (myelosuppression and gas-
lointerstitial nephritis, tubular atrophy, and interstitial brosis trointestinal toxicity) and is observed at plasma concentra-
(28). Chloroacetaldehyde, a metabolite of ifosfamide, is toxic to tions 520 mmol/L at 24 hours, 0.52 mmol/L at 48 hours,
renal epithelial cells and may contribute to nephrotoxicity of the and 0.050.1 mmol/L at 72 hours following drug administra-
parent drug (29,30). Clinical manifestations of renal tubular tion. When serum drug levels reach toxic levels, intravenous
toxicity include Fanconi syndrome, proximal and distal renal leucovorin is generally used to circumvent MTXs inhibition of
tubular acidosis, hypophosphatemia, hypokalemia, and nephro- dihydrofolate reductase to rescue normal cells from MTX
genic diabetes insipidus. (38).
Up to 87% of ifosfamide and its metabolites are recovered in MTX is poorly soluble in acid urine, and decreased ow
the urine, and up to 41% of the dose is recovered in the urine as rates increase its concentration in the renal tubules. Prevention
alkylating activity (31). Central nervous system (CNS) toxicity remains the best treatment for MTX toxicity and includes
may be linked to the accumulation of the metabolite chloroace- measures to alkalinize and maximally dilute the urine with
taldehyde, and the risk of CNS toxicity is greater in patients with bicarbonate containing intravenous uids as needed to
abnormal renal function (32). Dosing guidelines are empiric and achieve a urine pH . 7.0 and a urine output of $150 mL/h.
vary widely. Kintzel recommends for CrCl of 4660 mL/min, Because the drug is not lipophilic, it accumulates at high con-
give 80% of dose; for CrCl of 3145 mL/min, give 75% of dose; centrations in ascites and pleural uid, which can prolong
and for CrCl of ,30 mL/min, give 70% of dose. Aronoff rec- drug elimination and toxicity, especially in the setting of
ommends giving 75% of the usual dose for a GFR ,10 mL/min AKI. Consequently, it is recommended that uid collections
(4,5). The drug has been used without signicant myelosuppres- be drained prior to high-dose MTX.
sion or neurotoxicity in anuric and oliguric patients on HD at When AKI occurs, it is reversible in 70%100% of cases with
starting doses of 1.5 g/m2 at 48- to 72-hour intervals, with HD to conservative medical management (38), and the drug can be

American Society of Nephrology Onco-Nephrology Curriculum 3


readministered following recovery of renal function. In cases CKD patients (51,52). Rare cases of AKI due to acute and
where dialysis requiring AKI occurs and is associated with chronic interstitial nephritis (AIN) have been reported with
prolonged myelosuppression and/or gastrointestinal (GI) ul- lenalidomide (5355). For MDS, the manufacturer recom-
ceration, carboxypeptidase-G(2) can be obtained on a com- mends 5 mg daily for CrCl of ,59 mL/min, and increasing
passionate, albeit expensive basis, for management of severe the dosing interval to every 48 hours for CrCl of ,30 mL/min.
MTX intoxication (4143). Carboxypeptidase-G(2) rapidly For patients on HD, a dose of 5 mg should be given three
hydrolyzes MTX into its inactive metabolites and decreases times a week following each hemodialysis. For MM, the man-
median MTX concentrations by 98.7%. ufacturer recommends a 10-mg daily dose for CrCl of .3059
Kintzel recommends for CrCl of 4660 mL/min, give 65% mL/min, a 15-mg dose every 48 hours for CrCl of ,30
of dose; for CrCl of 3145 mL/min, give 50% of dose, and for mL/min, and 5 mg daily for HD patients to be given after
CrCl ,30 mL/min, do not administer. Aronoff recommends a HD on dialysis days (56).
dose reduction of 50% with a CrCl of .1050 mL/min and
avoiding the drug for CrCl of ,10 mL/min (4,5). Although the
drug is removed by high-ux HD and charcoal hemoperfu- CYTARABINE
sion, because it is 50% protein bound, there is postdialysis
rebound in MTX concentrations of 90%100% of the prepro- Cytarabine is an antimetabolite that is extensively converted to
cedure levels. Therefore, patients may require daily or contin- uridine-arabinoside (Ara-U), and 10%30% of the parent
uous renal replacement therapy to avoid rebound toxicity. drug and 85% of the inactive metabolite are eliminated by
the kidneys (57). High-dose cytarabine (HDAC) can produce
signicant and sometimes irreversible neurotoxicity. Renal
PEMETREXED dysfunction is an independent risk factor for cerebellar (dys-
arthria, nystagmus, gait ataxia, dysdiadochokinesia) and non-
Pemetrexed is a derivative of MTX, and up to 90% of the drug cerebellar (somnolence, seizures) neurotoxicity. The inactive
is excreted unchanged in the urine. The drug has been associated metabolite Ara-U inhibits cytidine deaminase activity, and in
with acute tubular necrosis and interstitial brosis (44,45). the setting of renal dysfunction, further delays cytarabine
The manufacturer recommends avoiding the drug for a CrCl clearance and increases serum and cerebral spinal uid levels
of ,45 mL/min and avoiding nonsteroidal anti-inammatory of the parent drug (58). For patients with CKD receiving
medications in the days before and after pemetrexed dosing in HDAC, Kintzel recommends for CrCl of 4660 mL/min,
patients with a CrCl of 4579 mL/min (46). When nephrotox- give 60% of the dose; for CrCl of 3145 mL/min, give 50%
icity occurs, it is probably best to avoid re-exposure. of the dose; and for CrCl of ,30 mL/min, do not administer
(4). Smith developed a dosing algorithm for HDAC based on
daily serum creatinine measurements while the patients were
MELPHALAN on therapy. If the patients baseline serum creatinine concen-
tration was between 1.5 and 1.9 mg/day, or if baseline serum
Melphalan is effective therapy for multiple myeloma (MM) and creatinine level increased by 0.51.2 mg/dL during treatment,
amyloidosis. Urinary excretion of melphalan ranges from 10% then the dose of Ara-C was reduced to 1 g/m2 per dose. If the
to 34%, and the AUC for melphalan is inversely correlated with serum creatinine level was .2 or the change was .1.2 mg/dL,
the GFR (47). Bone marrow suppression, the limiting side then the dose of Ara-C was reduced to 0.1 g/m2/day (standard
effect of melphalan, increases when the drug is given in dose) as a continuous infusion. (59). Cytarabine and Ara-U
CKD patients (48). Kintzel recommends for CrCl of 4660 are both cleared by HD. There are a few case reports of patients
mL/min, give 85% of dose; for CrCl of 3145 mL/min, give on HD who have tolerated treatment with standard doses cy-
75% of dose; and for CrCl of ,30 mL/min, give 70% of dose. tarabine (continuous infusion, 100 mg/m2 per day) when HD
Aronoff recommends giving 75% of the dose for a CrCl of was performed on day 1 of cytarabine infusion and then every
.1050 mL/min and 50% for those with a CrCl of ,10 other day. In one case, HD was performed consecutively on
mL/min (4,5). Patients on dialysis have been safely and suc- days 1 and 2 as well (60,61). HDAC has been safely used in a
cessfully treated with melphalan prior to stem cell transplant patient with lymphoma on hemodialysis. The patient received
at doses between 60 and 140 mg/m2 (49,50). two doses of HDAC 1 g/m2, 24 hours apart, was dialyzed 6
hours after each dose, and then resumed his usual dialysis
schedule (62).
LENALIDOMIDE

Lenalidomide is a thalidomide analogue used to treat mul- CAPECITABINE


tiple myeloma (MM) and myelodysplastic syndrome (MDS).
Eighty-two percent of the drug is excreted unchanged in the Capecitabine is a pyrimidine analogue that is preferentially
urine, and the AUC and risk for drug toxicity are increased in converted to 5-uorouracil (5-FU) within tumor cells.

4 Onco-Nephrology Curriculum American Society of Nephrology


Although neither capecitabine nor 5-FU is renally cleared, in Tyrosine kinase inhibitors
patients with CKD, there is retention of active metabolites Erlotinib
and a resultant increase in systemic toxicity (63). The manu- The PK of erlotinib were never studied in patients with renal
facturer recommends a dose reduction of 75% from the start- insufciency. There are case reports of patients with CrCl
ing dose of 1,250 mg/m2 for patients with a CrCl between 30 between 25 and 41 mL/min who tolerated the usual dose of 150
and 50 mL/min, and for patients with a CrCl of ,30 mL/min, mg/day without signicant toxicity (74). There are no data on
it is recommended that capecitabine be discontinued (64). its use in hemodialysis patients.
With careful monitoring and dose reduction, the drug has
been effectively and safely used in patients with advanced Imatinib
CKD (GFR , 30 mL/min) and patients on hemodialysis Although there is no signicant renal excretion, the manu-
(65). facturer recommends that patients with a CrCl of 2030
mL/min receive 50% of the starting dose, with dose escalation
as tolerated but not to exceed 400 mg/day. For those with
BLEOMYCIN CrCl of 4059 mL/min, doses .600 mg are not recom-
mended. Imatinib exposure can increase up to two-fold in
Bleomycin is not nephrotoxic, but urinary excretion accounts patients even with mild renal impairment (CrCl , 60 mL/min)
for close to 70% of the intravenous dose of bleomycin (66,67), and that there is a signicant correlation between de-
and patients with renal dysfunction appeared to be at higher creased renal function and the incidence of serious adverse
risk for bleomycin pulmonary toxicity, the major dose-limiting events (75). PK data on one patient with ESRD on dialysis
toxicity of this drug (48,68). Kintzel recommends for CrCl of that received 400 mg/day indicates that imatinib and its me-
4660 mL/min, give 70% of dose; for CrCl of 3145 mL/min, tabolite are unchanged in patients with ESRD on hemodialysis
give 60% of dose; and for CrCl of ,30 mL/min, avoid the drug. (74).
Aronoff recommends giving 75% of the dose for CrCl of .10
50 mL/min and 50% for those with a CrCl of ,10 mL/min Lenvatinib
(4,5). Because pulmonary toxicity can be cumulative in pa- The package insert recommends a dose reduction to 14 mg
tients with CKD, if bleomycin is administered to patients daily for patients with CrCl is ,30 mL/min. There are no data
with CKD, repeat pulmonary function tests prior to each on the use of lenvatinib in patients on hemodialysis.
drug administration may be prudent.
Sunitinib
Sunitinib use was studied in two ESRD patients. The PK pa-
rameters of sunitinib and its major metabolite were similar in
MOLECULARLY TARGETED AGENTS
patients on HD and those with normal renal function. Further-
more, sunitinib is nondialyzable. Doses of 50 mg/day for 4 weeks
Vascular endothelial growth factor (VEGF) pathway inhibi-
every 6 weeks were well tolerated (76).
tors (bevacizumab), tyrosine kinase inhibitors (sorafenib,
nilotinib, and dasatinib), and epithelial growth factor receptor
(EGFR) pathway inhibitors (erlotinib) have been described to Sorafenib
Although the manufacturer does not recommend any dose
cause nephrotoxicity. Observed renal toxicities among these
adjustment for patients with any level of renal insufciency
relatively new agents include acute tubular necrosis, pro-
(77), based on dose-limiting toxicity in a phase 1 study, a
teinuria, hypertension, thrombotic microangiopathy, acute
starting dose of 200 mg twice a day for CrCl of 2039 mL/min
interstitial nephritis, tumor lysis syndrome, and glomerulo-
and 200 mg once daily for patients on hemodialysis was recom-
nephritis (6971).
mended. No recommendations could be made for those with a
CrCl ,20 mL/min and not on dialysis (78).
Bevacizumab
As per the manufacturers guidelines, no studies were done to in-
Vandetanib
vestigate the PK of bevacizumab in patients with CKD. For patients The manufacturer recommends that the starting dose should
with a CrCl between 20 and 39 mL/min, the manufacturer be reduced to 200 mg in patients with a CrCl of ,50 mL/min (79).
recommends a 50% decrease in the usual starting dose with in- There are no data on use of this drug in hemodialysis patients.
creases in subsequent doses as tolerated, but no .400 mg. For
patients with a CrCl of 4059 mL/min, doses .600 mg are not
recommended (72). There is only one report on the PK of BISPHOSPHONATES
bevacizumab in a dialysis-dependent patient with metastatic renal
cancer who received 5 mg/kg every 2 weeks. The drug was Bisphosphonates are frequently administered to cancer pa-
not dialyzable, and its pharmacokinetic parameters were similar tients for management of hypercalcemia of malignancy and
to the reference values of patients with normal renal function. The osteolytic bone lesions. Zoledronic acid has been associated
drug can be administered any time before or after hemodialysis (73). with acute tubular necrosis, especially after repeated dosing.

American Society of Nephrology Onco-Nephrology Curriculum 5


Table 2. Recommended dose adjustments for selected chemotherapies for patients with CKD and ESRD on HD

Although pamidronate is more notoriously associated with commonly prescribed chemotherapeutic agents for CKD pa-
collapsing glomerulopathy, there are case reports of acute tients requiring chemotherapy is quite poor. This chapter was an
tubular necrosis with this agent as well (80). Dehydration, effort to summarize the data that are available and to provide the
concomitant use of nephrotoxic medication, and overly fre- treating physician with some guidance when treating patients
quent dosing of the bisphosphonates all increase susceptibility with cancer and CKD.
to deterioration of renal function following bisphosphonate
exposure. The American Society of Clinical Oncology (ASCO)
recommends that his zoledronate be avoided in patients with TAKE HOME POINTS
CrCl of ,30 mL/min and that the initial dose of 4 mg be
reduced to 3.5 mg for CrCl of 5060 mL/min; 3.3 mg for c The liver and the kidneys serve the major pathways of drug metabolism
CrCl of 4049 mL/min; and 3 mg for CrCl of 3039 mL/ and elimination with much smaller contributions from the fecal and re-
min. For pamidronate, the usual dose of 90 mg over 23 hours ticuloendothelial systems.
and ASCO recommends giving 90 mg over 46 hours for CrCl c Published guidelines for dose modication of chemotherapy for cancer
of ,59 mL/min. Neither agent should be used more fre- patients, with the exception of carboplatin, are largely based on limited
quently than every 34 weeks, the serum creatinine should pharmacokinetic and pharmacodynamic data.
be checked prior to each administration, and the medication c In several cases, the parent drug and its metabolites are responsible for
should be held if the creatinine increases .0.5 mg/dL with systemic toxicity, and the presence of renal insufciency can potentiate
normal renal function or .1 mg/dL if there is abnormal renal toxicity of the parent drug and its metabolites.
function at baseline (81).
Table 2 includes commonly used chemotherapeutic drugs
and their dose adjustment in the setting of CKD.
REFERENCES

1. Perazella MA. Renal vulnerability to drug toxicity. Clin J Am Soc


CONCLUSIONS
Nephrol 4: 12751283, 2009
2. Aleksa K, Matsell D, Krausz K, Gelboin H, Ito S, Koren G. Cytochrome
In summary, although the kidneys are a major pathway for drug P450 3A and 2B6 in the developing kidney: implications for ifosfamide
metabolism, unfortunately, the quality of PK and PD data on nephrotoxicity. Pediatr Nephrol 20: 872885, 2005

6 Onco-Nephrology Curriculum American Society of Nephrology


3. Launay-Vacher V, Oudard S, Janus N, Gligorov J, Pourrat X, Rixe O, 22. Hamilton CR, Bliss JM, Horwich A. The late effects of cis-platinum on
Morere JF, Beuzeboc P, Deray G; Renal Insufciency and Cancer renal function. Eur J Cancer Clin Oncol 25: 185189, 1989
Medications (IRMA) Study Group. Prevalence of Renal Insufciency in 23. Hansen SW, Groth S, Daugaard G, Rossing N, Rrth M. Long-term ef-
cancer patients and implications for anticancer drug management: the fects on renal function and blood pressure of treatment with cisplatin,
renal insufciency and anticancer medications (IRMA) study. Cancer vinblastine, and bleomycin in patients with germ cell cancer. J Clin
110: 13761384, 2007 Oncol 6: 17281731, 1988
4. Kintzel PE, Dorr RT. Anticancer drug renal toxicity and elimination: 24. Fjeldborg P, Srensen J, Helkjaer PE. The long-term effect of cisplatin
dosing guidelines for altered renal function. Cancer Treat Rev 21: 33 on renal function. Cancer 58: 22142217, 1986
64, 1995 25. Madias NE, Harrington JT. Platinum nephrotoxicity. Am J Med 65: 307
5. Aronoff GR. Drug Prescribing in Renal Failure, Philadelphia, American 314, 1978
College of Physicians, 2007 26. Erlichman C, Soldin SJ, Thiessen JJ, Sturgeon JF, Fine S. Disposition of
6. Lichtman SM, Wildiers H, Launay-Vacher V, Steer C, Chatelut E, Aapro total and free cisplatin on two consecutive treatment cycles in patients
M. International Society of Geriatric Oncology (SIOG) recommenda- with ovarian cancer. Cancer Chemother Pharmacol 19: 7579, 1987
tions for the adjustment of dosing in elderly cancer patients with renal 27. Reece PA, Stafford I, Russell J, Gill PG. Reduced ability to clear ultra-
insufciency. Eur J Cancer 43: 1434, 2007 lterable platinum with repeated courses of cisplatin. J Clin Oncol 4:
7. Matzke GR, Aronoff GR, Atkinson AJ Jr, Bennett WM, Decker BS, 13921398, 1986
Eckardt KU, Golper T, Grabe DW, Kasiske B, Keller F, Kielstein JT, 28. Morland BJ, Mann JR, Milford DV, Raafat F, Stevens MC. Ifosfamide
Mehta R, Mueller BA, Pasko DA, Schaefer F, Sica DA, Inker LA, Umans nephrotoxicity in children: Histopathological features in two cases.
JG, Murray P. Drug dosing consideration in patients with acute and Med Pediatr Oncol 27: 5761, 1996
chronic kidney disease: A clinical update from Kidney Disease: Im- 29. Zaki EL, Springate JE, Taub M. Comparative toxicity of ifosfamide
proving Global Outcomes (KDIGO). Kidney Int 80: 11221137, 2011 metabolites and protective effect of mesna and amifostine in cultured
8. Dreisbach AW, Lertora JJ. The effect of chronic renal failure on drug renal tubule cells.Toxicol In Vitro 17: 397402, 2003
metabolism and transport. Expert Opin Drug Metab Toxicol 4: 1065 30. Furlanut M, Franceschi L. Pharmacology of ifosfamide. Oncology 65
1074, 2008 [Suppl 2]: 26, 2003
9. Dooley MJ, Poole SG, Rischin D. Dosing of cytotoxic chemotherapy: 31. Allen LM, Creaven PJ. Pharmacokinetics of ifosfamide. Clin Pharmacol
impact of renal function estimates on dose. Ann Oncol 24: 27462752, Ther 17: 492498, 1975
2013 32. Meanwell CA, Blake AE, Kelly KA, Honigsberger L, Blackledge G.
10. Chatelut E, Canal P, Brunner V, Chevreau C, Pujol A, Boneu A, Roch H, Prediction of ifosfamide/mesna associated encephalopathy. Eur J
Houin G, Bugat R. Prediction of carboplatin clearance from standard Cancer Clin Oncol 22: 815819, 1986
morphological and biological patient characteristics. J Natl Cancer Inst 33. Latcha S, Maki RG, Schwartz GK, Flombaum CD. Ifosfamide may be
87: 573580, 1995 safely used in patients with end stage renal disease on hemodialysis.
11. Calvert AH, Newell DR, Gumbrell LA, OReilly S, Burnell M, Boxall FE, Sarcoma 2009: 575629, 2009
Siddik ZH, Judson IR, Gore ME, Wiltshaw E: Carboplatin dosage: Pro- 34. Carlson L, Goren MP, Bush DA, Griener JC, Quigley R, Tkaczewski I,
spective evaluation of a simple formula based on renal function. J Clin Kamen BA, Weitman SD. Toxicity, pharmacokinetics, and in vitro he-
Oncol 7: 17481756, 1989 modialysis clearance of ifosfamide and metabolites in an anephric
12. Egorin MJ, Van Echo DA, Tipping SJ, Olman EA, Whitacre MY, pediatric patient with Wilms tumor. Cancer Chemother Pharmacol 41:
Thompson BW, Aisner J. Pharmacokinetics and dosage reduction of 140146, 1998
cis-diammine(1,1-cyclobutanedicarboxylato)platinum in patients with 35. de Jonge ME, Huitema AD, Rodenhuis S, Beijnen JH. Clinical phar-
impaired renal function. Cancer Res 44: 54325438, 1984 macokinetics of cyclophosphamide. Clin Pharmacokinet 44: 1135
13. Shea TC, Storniolo AM, Mason JR, Newton B, Mullen M, Taetle R, 1164, 2005
Green MR. A dose-escalation study of carboplatin/cyclophosphamide/ 36. Haubitz M, Bohnenstengel F, Brunkhorst R, Schwab M, Hofmann U,
etoposide along with autologous bone marrow or peripheral blood Busse D. Cyclophosphamide pharmacokinetics and dose requirements
stem cell rescue. Semin Oncol 19[Suppl 2]: 139144, 1992 in patients with renal insufciency. Kidney Int 61: 14951501, 2002
14. Beyer J, Rick O, Weinknecht S, Kingreen D, Lenz K, Siegert W. Neph- 37. Juma FD, Rogers HJ, Trounce JR. Effect of renal insufciency on the
rotoxicity after high-dose carboplatin, etoposide and ifosfamide in pharmacokinetics of cyclophosphamide and some of its metabolites.
germ-cell tumors: Incidence and implications for hematologic recovery Eur J Clin Pharmacol 19: 443451, 1981
and clinical outcome. Bone Marrow Transplant 20: 813819, 1997 38. Flombaum CD, Meyers PA. High-dose leucovorin as sole therapy for
15. Guddati AK, Joy PS, Marak CP. Dose adjustment of carboplatin in pa- methotrexate toxicity. J Clin Oncol 17: 15891594, 1999
tients on hemodialysis. Med Oncol 31: 848, 2014 39. Howell SB, Carmody J. Changes in glomerular ltration rate associated
16. Janus N, Thariat J, Boulanger H, Deray G, Launay-Vacher V. Proposal with high-dose methotrexate therapy in adults. Cancer Treat Rep 61:
for dosage adjustment and timing of chemotherapy in hemodialyzed 13891391, 1977
patients. Ann Oncol 21: 13951403, 2010 40. Crom WR, Pratt CB, Green AA, Champion JE, Crom DB, Stewart CF,
17. Sastry J, Kellie SJ. Severe neurotoxicity, ototoxicity and nephrotoxicity Evans WE. The effect of prior cisplatin therapy on the pharmacokinetics
following high-dose cisplatin and amifostine. Pediatr Hematol Oncol of high-dose methotrexate. J Clin Oncol 2: 655661, 1984
22: 441445, 2005 41. Wall SM, Johansen MJ, Molony DA, DuBose TD Jr, Jaffe N, Madden T.
18. Arany I, Sarstein RL. Cisplatin nephrotoxicity. Semin Nephrol 23: 460 Effective clearance of methotrexate using high-ux hemodialysis
464, 2003 membranes. Am J Kidney Dis 28: 846854, 1996
19. Boulikas T, Vougiouka M:. Cisplatin and platinum drugs at the molec- 42. Relling MV, Stapleton FB, Ochs J, Jones DP, Meyer W, Wainer IW,
ular level. (Review). Oncol Rep 10: 16631682, 2003 Crom WR, McKay CP, Evans WE. Removal of methotrexate, leucovorin,
20. Dentino M, Luft FC, Yum MN, Williams SD, Einhorn LH. Long term ef- and their metabolites by combined hemodialysis and hemoperfusion.
fect of cis-diamminedichloride platinum (CDDP) on renal function and Cancer 62: 884888, 1988
structure in man. Cancer 41: 12741281, 1978 43. Widemann BC, Adamson PC. Understanding and managing metho-
21. Meijer S, Mulder NH, Sleijfer DT, Donker AJ, Sluiter WJ, de Jong PE, trexate nephrotoxicity. Oncologist 11: 694703, 2006
Schraffordt Koops H, van der Hem GK. Inuence of combination che- 44. Chauvet S, Courbebaisse M, Ronco P, Plaisier E. Pemetrexed-induced
motherapy with cis-diamminedichloroplatinum on renal function: acute kidney injury leading to chronic kidney disease. Clin Nephrol 82:
Long-term effects. Oncology 40: 170173, 1983 402406, 2014

American Society of Nephrology Onco-Nephrology Curriculum 7


45. Glezerman IG, Pietanza MC, Miller V, Seshan SV. Kidney tubular toxicity impairment on the pharmacokinetics and tolerability of capecitabine
of maintenance pemetrexed therapy. Am J Kidney Dis 58: 817820, (Xeloda) in cancer patients. Cancer Chemother Pharmacol 49: 225
2011 234, 2002
46. Eli Lilly Canada Inc: Product Monograph Altima Pemetrexed Disodium 64. Hoffman-LaRoche Limited: Product Monograph Xeloda. Available
for Injection, 2013. Available at: http://www.lilly.ca/en/pdf/product- at: http://www.rochecanada.com/fmles/re7234008/Research/
monograph/14_alimta-pm-10may2013.pdf. Accessed January 28, 2016 ClinicalTrialsForms/Products/ConsumerInformation/Monographsand
47. Alberts DS, Chang SY, Chen HS, Moon TE, Evans TL, Furner RL, PublicAdvisories/Xeloda/Xeloda_PM_E.pdf. Accessed January 28, 2016
Himmelstein K, Gross JF. Kinetics of intravenous melphalan. Clin 65. Jhaveri KD, Flombaum C, Shah M, Latcha S. A retrospective observa-
Pharmacol Ther 26: 7380, 1979 tional study on the use of capecitabine in patients with severe renal
48. Cornwell GG 3rd, Pajak TF, McIntyre OR, Kochwa S, Dosik H. Inuence impairment (GFR ,30 mL/min) and end stage renal disease on he-
of renal failure on myelosuppressive effects of melphalan: Cancer and modialysis. J Oncol Pharm Pract 18: 140147, 2012
Leukemia Group B experience. Cancer Treat Rep 66: 475481, 1982 66. Alberts DS, Chen HS, Mayersohn M, Perrier D, Moon TE, Gross JF.
49. Badros A, Barlogie B, Siegel E, Roberts J, Langmaid C, Zangari M, Bleomycin pharmacokinetics in man. II. Intracavitary administration.
Desikan R, Shaver MJ, Fassas A, McConnell S, Muwalla F, Barri Y, Cancer Chemother Pharmacol 2: 127132, 1979
Anaissie E, Munshi N, Tricot G. Results of autologous stem cell trans- 67. Crooke ST, Comis RL, Einhorn LH, Strong JE, Broughton A, Prestayko
plant in multiple myeloma patients with renal failure. Br J Haematol AW. Effects of variations in renal function on the clinical pharmacology
114: 822829, 2001 of bleomycin administered as an iv bolus. Cancer Treat Rep 61: 1631
50. Hamaki T, Katori H, Kami M, Yamato T, Yamakado H, Itoh T, Kusumi E, 1636, 1977
Igarashi M, Ueyama J, Kanda Y, Miyakoshi S, Mineishi S, Morinaga S, 68. Dalgleish AG, Woods RL, Levi JA. Bleomycin pulmonary toxicity: Its
Mukai M, Hayashi M, Takaue Y, Hara S, Mutou Y. Successful allogeneic relationship to renal dysfunction. Med Pediatr Oncol 12: 313317, 1984
blood stem cell transplantation for aplastic anemia in a patient with 69. Gafter-Gvili A, Ram R, Gafter U, Shpilberg O, Raanani P. Renal failure
renal insufciency requiring dialysis. Bone Marrow Transplant 30: 195 associated with tyrosine kinase inhibitors: Case report and review of the
198, 2002 literature. Leuk Res 34: 123127, 2010
51. Chen N, Lau H, Kong L, Kumar G, Zeldis JB, Knight R, Laskin OL. 70. Kurita N, Mise N, Fujii A, Ikeda S, Sugimoto T. Crescentic glomerulo-
Pharmacokinetics of lenalidomide in subjects with various degrees of nephritis in a patient with advanced lung cancer during erlotinib ther-
renal impairment and in subjects on hemodialysis. J Clin Pharmacol 47: apy. NDT Plus 2: 512513, 2009
14661475, 2007 71. Usui J, Glezerman IG, Salvatore SP, Chandran CB, Flombaum CD,
52. Niesvizky R, Naib T, Christos PJ, Jayabalan D, Furst JR, Jalbrzikowski J, Seshan SV. Clinicopathological spectrum of kidney diseases in cancer
Zafar F, Mark T, Lent R, Pearse RN, Ely S, Leonard JP, Mazumdar M, patients treated with vascular endothelial growth factor inhibitors: A re-
Chen-Kiang S, Coleman M. Lenalidomide-induced myelosuppression port of 5 cases and review of literature. Hum Pathol 45: 19181927, 2014
is associated with renal dysfunction: Adverse events evaluation of 72. Hoffman-LaRoche Limited: Product Monograph Avastin. Available
treatment-nave patients undergoing front-line lenalidomide and at: http://rochecanada.com/fmles/re7234008/Research/Clinical
dexamethasone therapy. Br J Haematol 138: 640643, 2007 TrialsForms/Products/ConsumerInformation/MonographsandPublic
53. Shaaban H, Layne T, Guron G. A case of DRESS (drug reaction with Advisories/Avastin/Avastin_PM_E.pdf. Accessed January 28, 2016
eosinophilia and systemic symptoms) with acute interstitial nephritis 73. Garnier-Viougeat N, Rixe O, Paintaud G, Ternant D, Degenne D,
secondary to lenalidomide. J Oncol Pharm Pract 20: 302304, 2013 Mouawad R, Deray G, Izzedine H. Pharmacokinetics of bevacizumab in
54. Batts ED, Sanchorawala V, Hegerfeldt Y, Lazarus HM. Azotemia asso- haemodialysis. Nephrol Dial Transplant 22: 975, 2007
ciated with use of lenalidomide in plasma cell dyscrasias. Leuk Lym- 74. Pappas P, Karavasilis V, Briasoulis E, Pavlidis N, Marselos M. Pharma-
phoma 49: 11081115, 2008 cokinetics of imatinib mesylate in end stage renal disease. A case study.
55. Lipson EJ, Huff CA, Holanda DG, McDevitt MA, Fine DM. Lenalidomide- Cancer Chemother Pharmacol 56: 358360, 2005
induced acute interstitial nephritis. Oncologist 15: 961964, 2010 75. Novartis Pharmaceuticals Corporation: Highlights of Prescribing In-
56. Available at: http://www.celgenecanada.net/pdfs/Revlimid%20 formation Gleevac, 2015. Available at: https://www.pharma.us.
Product_Monograph_-_English_Version.pdf novartis.com/product/pi/pdf/gleevec_tabs.pdf. Accessed January 28,
57. Powis G. Effect of human renal and hepatic disease on the pharmaco- 2016
kinetics of anticancer drugs. Cancer Treat Rev 9: 85124, 1982 76. Izzedine H, Etienne-Grimaldi MC, Rene N, Vignot S, Milano G. Phar-
58. Damon LE, Mass R, Linker CA. The association between high-dose macokinetics of sunitinib in hemodialysis. Ann Oncol 20: 190192,
cytarabine neurotoxicity and renal insufciency. J Clin Oncol 7: 1563 2009
1568, 1989 77. Baryer Inc: Product Monograph Nexavar, 2014. Available at: http://
59. Smith GA, Damon LE, Rugo HS, Ries CA, Linker CA. High-dose cytar- www.bayer.ca/omr/online/nexavar-pm-en-17dec2014-178086.pdf.
abine dose modication reduces the incidence of neurotoxicity in pa- Accessed January 28, 2016
tients with renal insufciency. J Clin Oncol 15: 833839, 1997 78. Miller AA, Murry DJ, Owzar K, Hollis DR, Kennedy EB, Abou-Alfa G,
60. Pschl JM, Klaus G, Querfeld U, Ludwig R, Mehls O. Chemotherapy Desai A, Hwang J, Villalona-Calero MA, Dees EC, Lewis LD, Fakih MG,
with cytosine arabinoside in a child with Burkitts lymphoma on main- Edelman MJ, Millard F, Frank RC, Hohl RJ, Ratain MJ. Phase I and
tenance hemodialysis and hemoltration. Ann Hematol 67: 3739, pharmacokinetic study of sorafenib in patients with hepatic or renal
1993 dysfunction: CALGB 60301. J Clin Oncol 27: 18001805, 2009
61. Tsuchiya Y, Ubara Y, Suwabe T, Hoshino J, Sumida K, Hiramatsu R, 79. AstraZeneca Pharmaceuticals Corporation: Highlights of Prescribing
Hasegawa E, Yamanouchi M, Hayami N, Marui Y, Sawa N, Takemoto F, Information Caprelsa. Available at: http://www1.astrazeneca-us.com/
Takaichi K. Successful treatment of acute promyelocytic leukemia in a pi/vandetanib.pdf. Accessed January 28, 2016
patient on hemodialysis. Clin Exp Nephrol 15: 434437, 2011 80. Perazella MA, Markowitz GS. Bisphosphonate nephrotoxicity. Kidney
62. Radeski D, Cull GM, Cain M, Hackett LP, Ilett KF. Effective clearance of Int 74: 13851393, 2008
Ara-U the major metabolite of cytosine arabinoside (Ara-C) by hemo- 81. Kyle RA, Yee GC, Somereld MR, Flynn PJ, Halabi S, Jagannath S,
dialysis in a patient with lymphoma and end-stage renal failure. Cancer Orlowski RZ, Roodman DG, Twilde P, Anderson K; American Society of
Chemother Pharmacol 67: 765768, 2011 Clinical Oncology. American Society of Clinical Oncology 2007 clinical
63. Poole C, Gardiner J, Twelves C, Johnston P, Harper P, Cassidy J, practice guideline update on the role of bisphosphonates in multiple
Monkhouse J, Banken L, Weidekamm E, Reigner B. Effect of renal myeloma. J Clin Oncol 25: 24642472, 2007

8 Onco-Nephrology Curriculum American Society of Nephrology


REVIEW QUESTIONS a. 4 mg
b. 3.5 mg
1. What is the best option for management of a patient on c. 3 mg
cisplatin who has lost approximately 50% of the GFR fol- d. Do not administer
lowing the rst three cycles?
Answer: c is correct. The American Society of Clinical On-
a. Continue cisplatin at the present dose
cology recommends that zoledronate should be avoided in
b. Continue cisplatin at a reduced dose
patients with CrCl of ,30 mL/min and that the initial dose of
c. Consider alternative therapies
4 mg be reduced to 3.5 mg for CrCl of 5060 mL/min; 3.3 mg for
Answer: c is correct. Patients can have episodes of AKI CrCl of 4049 mL/min; and 3 mg for CrCl of 3039 mL/min.
following each dose of cis-platinum. With each subsequent
3. What is the appropriate timing of cisplatin administration
episode of AKI, the baseline serum creatinine may not re-
in an ESRD patient receiving hemodialysis?
turn to its pretreatment value. Up to 30% of patients may
have residual renal insufciency due to cis-platinum neph- a. 2 hours prior to hemodialysis
rotoxicity. In this patient who has lost about 50% of his GFR b. 6 hours prior to hemodialysis
following his initial treatment, cisplatin is not re- c. 12 hours prior to hemodialysis
commended, and it may be best to consider alternative d. On the off-dialysis day
therapies.
Answer: d is correct. Because the drug is highly and irrevers-
2. What is the dose of zoledronic acid recommended by ibly protein bound and because free cisplatin is well dialyzed,
American Society of Clinical Oncology for the adminis- drug that is removed by dialysis cannot be replaced by bound
tration to patients with CrCl of 3039 mL/min? drug. As such, cisplatin must be given on a nondialysis day.

American Society of Nephrology Onco-Nephrology Curriculum 9


Chapter 13: CKD as a Complication of Cancer
Laura Cosmai, MD,* Camillo Porta, MD, and Maurizio Gallieni, MD, FASN
*Nephrology and Dialysis, Istituti Ospitalieri Cremona, Cremona, Italy; Medical Oncology, I.R.C.C.S., San Matteo
University Hospital Foundation, Pavia, Italy; and Nephrology and Dialysis, San Carlo Borromeo Hospital, University of
Milano, Milano, Italy

CKD AND CANCER: A CIRCULAR The authors also observed an increased risk of
RELATIONSHIP urothelial cancer at an eGFR ,30 mL/min per
1.73 m2 but no signicant associations between
CKD is recognized as a disease that may complicate eGFR and other cancers. Finally, CKD conferred an
cancer and its therapy. This is in part related to the fact increased cancer-specic mortality in patients with
that preexisting CKD is highly prevalent in oncologic kidney and urinary tract cancer (6). In ESRD patients
patients. Indeed, as observed in the Renal Insuf- on dialysis, the observed increased risk for renal paren-
ciency and Anticancer Medications (IRMA)-1 and -2 chymal cancer is related to the development of ac-
studies (1,2), more than half of patients with an active quired renal cystic disease, which increases with time
malignancy present with an eGFR of ,90 mL/min on dialysis (7).
per 1.73 m2. Furthermore, the prevalence of more
severe CKD (i.e., stages 35), not requiring dialysis, CKD and antineoplastic drugs
was 12.0% and 11.8%, respectively (1,2). Similar re- Acute and chronic kidney injury associated with
sults have been reported in other series from different antineoplastic drug exposure is well described for the
countries, thereby conrming that CKD is a relatively classic cytotoxic agents that are used. In addition,
common occurrence in cancer patients, irrespective there is a large body of literature that describes dosing
of the type of malignancy. As a whole, causes poten- of these drugs in patients with underlying renal
tially able to have a negative impact on kidney func- dysfunction and those on dialysis; however, little is
tion are summarized in Table 1. known about the appropriate use of the new targeted
Interestingly, the relationship between the kidney agents in this population. This creates a complicated
and cancer appears to be bidirectional (3). For exam- issue for oncologists and nephrologists who care for
ple, preexisting CKD may impact the bioavailability these patients and must provide both safe and effective
and/or safety prole of an anticancer drug, poten- anticancer therapy. After decades of use of common
tially leading to different and sometimes suboptimal cytotoxic drugs, clinicians versed in cancer care and its
treatment choices. On the other hand, it is also pos- complications are well aware of the main toxicities of
sible that the renal effects of a novel anticancer drug these agents. The new, molecularly targeted, antican-
may lead to progressive kidney injury or to worsening cer drugs that are entering clinical practice have a
of preexisting CKD (3). In addition to the observed wide array of previously unrecognized and ill-dened
increase in CKD prevalence in cancer patients, both adverse effects (8). Ultimately, these toxicities must
CKD and ESRD are risk factors for a number of ma- be readily recognized and managed by those provid-
lignancies (4). However, not all solid tumors appear ing care for patients exposed to these drugs. This
to be equally represented in this population.
A retrospective cohort study of 1,190,538 adults Correspondence: Laura Cosmai, Division of Nephrology and
assessed the association between eGFR level and the Dialysis, Istituti Ospitalieri di Cremona, Largo Priori, 1, 26100,
Cremona, Italy. Email: lacos@iol.it
risk of incident cancer (5). During 6,000,420 person-
years of follow-up, 76,809 incident cancers were All authors are members of the Joint Italian Association of Medical
Oncology (AIOM)/Italian Society of Nephrology (SIN) Working
identied in 72,875 subjects. After adjustment for
Group on Onco-Nephrology.
time-updated confounders, lower eGFR was associ-
ated with an increased risk of renal cancer, with an L.C. and C.P. contributed equally to the preparation of this
chapter.
adjusted hazard ratio (HR) of 2.28 (95% CI, 1.78
2.92) for an eGFR ,30 mL/min per 1.73 m2 (5). Copyright 2016 by the American Society of Nephrology

American Society of Nephrology Onco-Nephrology Curriculum 1


Table 1. Causes of kidney disease in cancer patients
Cause Mechanism(s)
Antineoplastic drugs
Traditional chemotherapeutic agents c Direct nephrotoxicity (e.g., cisplatin)
Novel targeted therapies c Hypertension and/or proteinuria (e.g., VEGF[Rs]-targeted agents)
c TMA (e.g., VEGF-targeted agents)
c Interstitial nephritis and other glomerulonephritis
c Autoimmune nephropathies (e.g., anti-CTLA4 and anti-PD1/PDL1
antibodies)
c Indirect toxicities (e.g., nausea/vomiting, diarrhea, dysgeusia)
leading to dehydration/volume depletion
Other drugs used in cancer patients c Direct nephrotoxicity (e.g., NSAIDs, bisphosphonates)
Anti-pain drugs
Bisphosphonates
Radiation therapy c Still ill dened
Contrast medium c Direct nephrotoxicity
Paraneoplastic renal syndromes c Autoimmune mechanism?
Nephrectomy c Loss of nephrons
For cancer c AKI
For other causes
Obstruction/compression c Mechanical injury
Tumor Inltration c Kidney inltration
Comorbid risk factors c Hypertension
c Preexisting CKD
c Diabetes mellitus
c AKI
c Previous use of nephrotoxic cancer therapies
VEGF(Rs), vascular endothelial growth factor (receptors); TMA, thrombotic microangiopathies; CTLA4, cytotoxic T-lymphocyte antigen 4; PD1, programmed cell
death 1; PDL1, programmed cell death ligand 1; NSAIDs, nonsteroidal anti-inammatory drugs.

includes understanding risk factors for targeted drug-induced Although the relationship between kidney function and
kidney injury, appropriate drug dosing (if known) for the patient cytotoxic agents will be covered in other chapters of the
with AKI, CKD, and those on dialysis, the clinical manifestations curriculum, the pharmacokinetic properties and specic renal
of drug nephrotoxicity, and the optimal management of neph- toxicities of novel anticancer agents are summarized in Table 2.
rotoxic complications (8). The pharmacokinetics of these drugs are similar as most are
Frequently, oncologists ask their nephrology colleagues to 90%98% bound to plasma proteins, and their excretion oc-
assess the degree of kidney function impairment to provide curs predominantly via the feces (or the reticulo-endothelial
insight intodosageadjustmentofanticancer therapy.Toaccomplish system), whereas urinary excretion is quite variable from one
this, a thorough knowledge of the specic metabolism of anticancer drug to the other (8).
agents and of their pharmacokinetic and pharmacodynamic As stated in the drugs Summary of Product Characteristics
properties is mandatory. The thought process includes deciding (SmPC), the pharmacokinetic properties of the majority of these
if the drug should be administered, when it is appropriate to drugs are not inuenced by kidney function (3). A population
dose the drugs, and to what extent dosage adjustment should be pharmacokinetic model, which includes data from subjects with
used in the setting of underlying kidney disease (3). This approach baseline creatinine clearance ranging from 30 to 150 mL/min,
must be carefully done, as unnecessary treatment interruptions and indicated that it is unlikely that renal insufciency has a clinically
drug dose reductions may be associated with suboptimal cancer relevant effect on the pharmacokinetics of targeted therapies (9).
therapy and hamper the clinical benets of cancer therapy. Thus, no dosage adjustment is recommended in patients
Optimal management of underlying CKD and its complica- with a creatinine clearance .30 mL/min. To date, only patients
tions, which may be signicantly ameliorated in many cases, as with adequate kidney function (serum creatinine #1.5 times the
well as prevention of further kidney damage from other upper limit of normal) have been included in registrative
exogenous nephrotoxins (e.g., contrast medium, nonsteroidal randomized controlled trials. In patients with a creatinine
anti-inammatory drugs, bisphosphonates) in cancer patients clearance ,30 mL/min, a patient population that is poorly
with preexisting CKD, is also key to minimizing drug-related studied, caution is recommended (10). Interestingly, this con-
complications. Unfortunately, patients with CDK and those on servative recommendation is not based on data, as drug expo-
dialysis are often undertreated for their neoplastic disease due sure in patients with severe renal impairment was similar to
to the fear of drug-induced adverse effects. that observed in patients with normal kidney function (10).

2 Onco-Nephrology Curriculum American Society of Nephrology


Table 2. Renal toxicities of anticancer targeted agents (modied from reference 3)
Patients with renal Dose reduction required?
function impairment Renal Most-frequent Patients
Drug Patients with mild Patients with
included in pivotal excretion renal AEs receiving
trial to moderate CKD severe CKD
dialysis
VEGF/VEGFR-tageting
agents
Bevacizumab No No Hypertension, proteinuria No No (no data) No
Aibercept No No Hypertension, proteinuria No No (no data) No
Sunitinib No 16% Hypertension, proteinuria No No (no data) No
Pazopanib No ,4% Hypertension, proteinuria No No (no data) No
Axitinib No 23% Hypertension, (proteinuria) No No (no data) No
Other multikinase
inhibitors
Sorafenib No 19% Hypertension, proteinuria, No No (no data) No
hypophosphatemia
Regorafenib No 19% Hypertension, proteinuria, No No (no data) No data
electrolyte disorders
Vandetanib No 25% Hypertension, proteinuria, AKI No Yes (few data) No
Imatinib No 13% More renoprotective effects No No (no data) No
mTOR inhibitors
Everolimus No 2% Proteinuria, AKI, electrolyte No No (no data); No
disorders suspend if AKI
Temsirolimus No 4.6% As for everolimus, but less No No (no data); No
frequent suspend if AKI
EGFR inhibitors
Getinib No ,4% Electrolyte disorders No No (no data) No
Erlotinib No ,9% Electrolyte disorders No No (no data) No
Afatinib No ,5% Electrolyte disorders No No (no data) No (no data)
Cetuximab No No Hypomagnesemia, other No No (no data) No
electrolyte disorders
Panitumumab No No Hypomagnesemia, other No No (no data) No (no data)
electrolyte disorders
B-Raf inhibitors and
MEK inhibitors
Vemurafenib No 1% AKI (tubular necrosis?) No No (no data) Possible
(risk of
arrhythmia)
Dabrafenib No 23% Hypophosphatemia, No No (no data) No (no data)
(granulomatous nephritis?)
Trametinib No ,20% Hypertension, hyponatremia No No (no data) No (no data)
(with dabrafenib)
ERBB2-targeting agents
Trastuzumab No No Hypertension, AKI No No (no data) No
(with cisplatin)
Pertuzumab No No No issues No No (no data) No (no data)
Lapatinib No 2 No issues No No (no data) No
Trastuzumab emtansine No ,5% Hypokalemia No No (no data) No (no data)
Antibodies against CTLA4
Ipilimumab No No Autoimmune nephritis, (drug No No (no data) No (no data)
reaction with eosinophilia
and systemic symptom
syndrome?)
Other agents
Crizotinib No No Reduction of eGFR Possible, with Possible, with No (no data)
(tubular necrosis?), caution caution (no data)
renal cysts
Catumaxomab No No No issues No No (no data) No (no data)
AE, adverse event.

American Society of Nephrology Onco-Nephrology Curriculum 3


CKD postnephrectomy for kidney cancer associated with a 21% reduction in the absolute risk of devel-
Surgical resection remains the gold standard treatment for local- oping moderate CKD (eGFR , 60 mL/min per 1.73 m2) over a
ized renal cell carcinoma (RCC) and is also commonly performed median follow-up of 6.7 years, whereas the difference in the
for synchronous metastatic disease. The type of surgical resection incidence of severe CKD (eGFR ,30 mL/min per 1.73 m2)
utilized to treat RCC for the last several decades has been radical between the two groups was 3.7% (19).
nephrectomy, although, more recently, increasing emphasis has These results have generated various hypotheses about the
been placed on the concept of nephron-sparing procedures (11). effect of medical- versus surgical-associated CKD on patient
Although radical nephrectomy and nephron-sparing surgery survival. It is speculated that medical diseaserelated CKD has
do not appear to differ in terms of oncologic outcome, the two worse outcomes than CKD due to surgery (nephron loss),
different strategies differ in terms of incidence of postoperative likely related to other comorbidities and the primary renal
CKD and of cardiovascular complications (12). Less invasive sur- disease (i.e., diabetes mellitus). Indeed, a recent report sugges-
gical procedures for RCC are associated with improved outcomes ted that patients with medical risk factors for CKD are at in-
with less postoperative AKI and CKD and less cardiovascular com- creased risk of progressive renal impairment, irrespective of
plications (12). This is not trivial considering that 22% of patients the use of partial nephrectomy (20).
with renal tumors had a pre-nephrectomy stage 3 or greater CKD The duration of renal ischemia during partial nephrectomy
(eGFR , 60 mL/min per 1.73 m2) (13). In patients 70 years of age may also play a key role in the development of postoperative CKD.
and older, the percentage approaches 40% (13). Overall, among In a recent collaborative review (21), a strong association was
662 patients scheduled for partial or radical nephrectomy, the noted between the quality and quantity of renal tissue that is pre-
prevalence of stage 3 or greater CKD was 26% (14). Furthermore, served after surgery and long-term kidney function, and also the
patients that had postoperative AKI following radical nephrectomy duration of ischemia proved to be an important modiable pre-
had a 4.24-fold higher risk of developing new-onset CKD (15). dictor of postoperative kidney function. Prolonged warm ische-
Progression of underlying CKD was also noted in patients mia time (WIT) proved to be signicantly associated with adverse
undergoing a radical nephrectomy procedure for an RCC (15). postoperative kidney function (21). Available data suggest a renal
Accordingly, both the American Urological Association (16) benet of keeping WIT ,25 minutes (21). Conversely, cold is-
and the European Association of Urology (17) endorsed partial chemia appears to safely allow longer durations of ischemia (21).
nephrectomy as the novel standard of care for organ-conned Finally, patients with CKD (irrespective of its cause) within
tumors #4 cm (T1a) and suggested that it be considered as a the setting of a metastatic cancer usually tolerate cancer-targeted
viable option for patients with tumors .4 but #7 cm (T1b). agents poorly, experiencing higher-grade adverse events com-
Evidence for a relationship between the extent of kidney tissue pared with patients with normal kidney function (3).
removal and the risk of CKD comes from single-center retro-
spective studies, population-based studies, and a single random- Tumor-induced CKD
ized, controlled, phase 3 study (12). Among the population-based End-stage cancer is often associated with malignant ureteral
studies available between 1990 and 2011, a meta-analysis of 36 obstruction (MUO), leading to obstructive nephropathy and
studies, of which only one was prospective, examined 31,729 CKD. Direct ureteral inltration by tumor or extrinsic ureter
patients treated with radical nephrectomy and 9,281 patients compression by bulky tumor masses frequently cause these
managed conservatively (18). The results demonstrated that par- sequelae (22). In general, cervical, bladder, and prostate cancers
tial nephrectomy was associated with a 19% reduction in the risk are the most common culprits (22). However, urinary obstruc-
of all-cause mortality (HR, 0.81; P ,0.0001), a 29% reduction in tion can also occur from retroperitoneal brosis due to surgery,
cancer-specic mortality (HR, 0.71; P ,0.001), and a 61% re- chemotherapy, and/or radiotherapy.
duction in the risk of severe CKD (HR, 0.39; P ,0.0001), sup- In the setting of malignant obstruction, percutaneous ne-
porting the ndings observed in a number of smaller studies (12). phrostomy tube placement or retrograde ureteral double-J stent
In contrast to these ndings, however, the results of a highly placement should be urgently performed, recognizing the asso-
controversial randomized controlled trial, conducted by the ciated procedural complications (22). Recently, a prognostic
European Organization for the Research and Treatment of Cancer model for survival after palliative urinary diversion for malignant
(EORTC) (19), complicate the surgical approach to renal cell urinary obstruction has been developed (23). Two risk factors (at
cancer. In this trial, a more favorable outcome was observed in least four events related to malignancy and an Eastern Cooperative
patients treated with radical nephrectomy compared with those Oncology Group [ECOG] index $2) were used to stratify patients
treated conservatively. During a median follow-up of .9 years, into three groups by survival type: favorable (no factors), inter-
death occurred in 25% of patients treated by partial nephrectomy mediate (one factor), and unfavorable (two factors). The median
and in 18.3% of those undergoing radical nephrectomy. Cardio- survival at 1, 6, and 12 months was 94.4%, 57.3%, and 44.9% in
vascular diseases were noted to be the most common cause of the favorable group, 78.0%, 36.3%, and 15.5% in the intermediate
death. The intention-to-treat analysis showed a 10-year overall group, and 46.4%, 14.3%, and 7.1% in the unfavorable group,
survival rate of 81.1% in the radical nephrectomy group com- respectively (23). This simple model could help to guide clinical
pared with 75.5% in the partial nephrectomy group (HR, 1.5; decisions when choosing which patients are reasonable candidates
95% CI, 1.032.16). Interestingly, partial nephrectomy was for urinary diversion in a palliative setting.

4 Onco-Nephrology Curriculum American Society of Nephrology


CONCLUSION 8. Porta C, Paglino C, Imarisio I, Bonomi L. Uncovering Pandoras vase:
The growing problem of new toxicities from novel anticancer agents.
The case of sorafenib and sunitinib. Clin Exp Med 7: 127134, 2007
CKD is highly prevalent in oncologic patients and appears to
9. Thomas SM, Grandis JR. Pharmacokinetic and pharmacodynamic
be a risk factor for the development of cancer. Furthermore, properties of EGFR inhibitors under clinical investigation. Cancer Treat
the use of antineoplastic drugs in patients with underlying Rev 30: 255268, 2004
CKD raises several specic issues: 1) the direct nephrotoxicity 10. Josephs D, Hutson TE, Cowey CL, Pickering LM, Larkin JM, Gore ME,
of several anticancer agents (especially novel molecularly Van Hemelrijck M, McDermott DF, Powles T, Chowdhury P, Karapetis
C, Harper PG, Choueiri TK, Chowdhury S. Efcacy and toxicity of su-
targeted agents); 2) the need to adjust antineoplastic doses
nitinib in patients with metastatic renal cell carcinoma with severe renal
due to concomitant CKD; 3) the lack of prospective drug impairment or on haemodialysis. BJU Int 108: 12791283, 2011
dosing data in patients with advanced CKD or those on 11. Manikandan R, Srinivasan V, Ran A. Which is the real gold standard for
dialysis; and 4) the nihilistic approach to the treatment of this small-volume renal tumors? Radical nephrectomy versus nephron-
population of patients with CKD, leading to the frequent sparing surgery. J Endourol 18: 3944, 2004
12. Li L, Lau WL, Rhee CM, Harley K, Kovesdy CP, Sim JJ, Jacobsen S,
undertreatment (or even absence of treatment) of these
Chang A, Landman J, Kalantar-Zadeh K. Risk of chronic kidney disease
patients, ultimately denying them potentially life-prolonging after cancer nephrectomy. Nat Rev Nephrol 10: 135145, 2014
options (3). Indeed, in our opinion, CKD should not be 13. Canter D, Kutikov A, Sirohi M, Street R, Viterbo R, Chen DY, Greenberg
regarded, in and of itself, as a reason to reduce or hold tar- RE, Uzzo RG. Prevalence of baseline chronic kidney disease in patients
geted therapies in the absence of other comorbidities or presenting with solid renal tumors. Urology 77: 781785, 2011
14. Huang WC, Levey AS, Serio AM, Snyder M, Vickers AJ, Raj GV, Scardino
medical indications (3).
PT, Russo P. Chronic kidney disease after nephrectomy in patients with
renal cortical tumours: a retrospective cohort study. Lancet Oncol 7:
735740, 2006
TAKE HOME POINTS 15. Cho A, Lee JE, Kwon GY, Huh W, Lee HM, Kim YG, Kim DJ, Oh HY,
Choi HY. Post-operative acute kidney injury in patients with renal cell
c The relationship between CKD and cancer should be regarded as carcinoma is a potent risk factor for new-onset chronic kidney disease
bidirectional. after radical nephrectomy. Nephrol Dial Transplant 26: 34963501,
c The use of targeted agents in cancer patients with CKD is ill dened. 2011
c CKD and ESRD requiring dialysis, per se, should not be regarded as 16. Campbell SC, Novick AC, Belldegrun A, Blute ML, Chow GK, Derweesh
reasons not to administer anticancer treatments. IH, Faraday MM, Kaouk JH, Leveillee RJ, Matin SF, Russo P, Uzzo RG;
c Nephrectomy for kidney cancer is another common cause of CKD, Practice Guidelines Committee of the American Urological Associa-
but the use of nephron-sparing surgical techniques have been de- tion. Guideline for management of the clinical T1 renal mass. J Urol
veloped to limit this issue. 182: 12711279, 2009
c Medical diseaserelated CKD has worse outcomes than CKD due to 17. Ljungberg B, Bensalah K, Caneld S, Dabestani S, Hofmann F, Hora M,
surgery. Kuczyk MA, Lam T, Marconi L, Merseburger AS, Mulders P, Powles T,
c The tumor may cause CKD by causing urinary obstruction with ob-
Staehler M, Volpe A, Bex A. EAU guidelines on renal cell carcinoma:
structive nephropathy.
2014 update. Eur Urol 67: 913924, 2015
18. Kim SP, Thompson RH, Boorjian SA, Weight CJ, Han LC, Murad MH,
Shippee ND, Erwin PJ, Costello BA, Chow GK, Leibovich BC. Com-
REFERENCES parative effectiveness for survival and renal function of partial and
radical nephrectomy for localized renal tumors: A systematic review
and meta-analysis. J Urol 188: 5157, 2012
1. Launay-Vacher V, Oudard S, Janus N, Gligorov J, Pourrat X, Rixe O,
19. Van Poppel H, Da Pozzo L, Albrecht W, Matveev V, Bono A, Borkowski
Morere JF, Beuzeboc P, Deray G; Renal Insufciency and Cancer
A, Colombel M, Klotz L, Skinner E, Keane T, Marreaud S, Collette S,
Medications (IRMA) Study Group. Prevalence of renal insufciency in
Sylvester R. A prospective, randomised EORTC intergroup phase 3
cancer patients and implications for anticancer drug management: The
study comparing the oncologic outcome of elective nephron-sparing
renal insufciency and anticancer medications (IRMA) study. Cancer
surgery and radical nephrectomy for low-stage renal cell carcinoma.
110: 13761384, 2007 Eur Urol 59: 543552, 2011
2. Launay-Vacher V. Epidemiology of chronic kidney disease in cancer 20. Satasivam P, Reeves F, Rao K, Ivey Z, Basto M, Yip M, Roth H, Grummet
patients: Lessons from the IRMA study group. Semin Nephrol 30: 548 J, Goad J, Moon D, Murphy D, Appu S, Lawrentschuk N, Bolton D,
556, 2010 Kearsley J, Costello A, Frydenberg M. Patients with medical risk factors
3. Porta C, Cosmai L, Gallieni M, Pedrazzoli P, Malberti F. Renal effects of for chronic kidney disease are at increased risk of renal impairment
targeted anticancer therapies. Nat Rev Nephrol 11: 354370, 2015 despite the use of nephron-sparing surgery. BJU Int 116: 590595,
4. Stengel B. Chronic kidney disease and cancer: A troubling connection. 2015
J Nephrol 23: 253262, 2010 21. Volpe A, Blute ML, Ficarra V, Gill IS, Kutikov A, Porpiglia F, Rogers C,
5. Lowrance WT, Ordoez J, Udaltsova N, Russo P, Go AS. CKD and the Touijer KA, Van Poppel H, Thompson RH. Renal ischemia and function
risk of incident cancer. J Am Soc Nephrol 25: 23272334, 2014 after partial nephrectomy: A collaborative review of the literature. Eur
6. Weng PH, Hung KY, Huang HL, Chen JH, Sung PK, Huang KC. Cancer- Urol 68: 6174, 2015
specic mortality in chronic kidney disease: Longitudinal follow-up of a 22. Russo P: Ureteral obstruction and stents: Still a difcult problem for
large cohort. Clin J Am Soc Nephrol 6: 11211128, 2011 patients and urologists alike. J Urol 174: 2088, 2005
7. Stewart JH, Buccianti G, Agodoa L, Gellert R, McCredie MR, Lowenfels 23. Cordeiro MD, Coelho RF, Chade DC, Pessoa RR, Chaib MS, Colombo-
AB, Disney AP, Wolfe RA, Boyle P, Maisonneuve P. Cancers of the Jnior JR, Pontes-Jnior J, Guglielmetti GB, Srougi M. A prognostic
kidney and urinary tract in patients on dialysis for end-stage renal dis- model for survival after palliative urinary diversion for malignant
ease: Analysis of data from the United States, Europe, and Australia and ureteric obstruction: A prospective study of 208 patients. BJU Int 117:
New Zealand. J Am Soc Nephrol 14: 197207, 2003 266271, 2016

American Society of Nephrology Onco-Nephrology Curriculum 5


REVIEW QUESTIONS plasma proteins, and their excretion occurs predominantly via
the feces (or the reticulo-endothelial system), whereas urinary
1. Which types of cancer are more commonly observed in excretion is quite variable among the drugs. A population phar-
CKD patients? macokinetic model indicated that it is unlikely that renal insuf-
a. Gastrointestinal cancers ciency has a clinically relevant effect on the pharmacokinetics
b. Lung cancers of targeted therapies. Thus, no dosage adjustment is recom-
c. Breast cancers mended in patients with a creatinine clearance .30 mL/min.
d. Renal and urothelial cancers
3. Which of the following contributes to the development of
Answer: d is correct. A large retrospective cohort study as- CKD in nephrectomized patients?
sessed the association between eGFR level and the risk of inci-
a. Nephrectomy itself (loss of nephrons)
dent cancer. Lower eGFR was associated with an increased risk of
b. Concomitant comorbidities
renal cancer with an adjusted HR of 2.28 (95% CI, 1.782.92) for
c. Ischemia time
an eGFR ,30 mL/min per 1.73 m2. An increased risk of uro-
d. All the above
thelial cancer at an eGFR ,30 mL/min per 1.73 m2 was also
evidenced. Finally, CKD conferred an increased cancer-specic Answer: d is correct. Radical nephrectomy and nephron-
mortality in patients with kidney and urinary tract cancer. sparing surgery greatly differ in terms of incidence of post-
operative CKD, due to the different amount of nephron loss.
2. Should the dose of a targeted anticancer agent be reduced Less invasive surgical procedures for renal cell carcinoma are
in a patient with mild to moderate CKD? associated with less postoperative AKI and CKD. Despite
a. Yes, almost always this, a recent report suggested that patients with medical risk
b. No, almost never factors for CKD are at increased risk of progressive renal im-
c. Yes, depending on the drugs pharmacokinetic properties pairment, irrespective of the use of partial nephrectomy, thus
d. The targeted anticancer agent should not be used in this highlighting the key role played by comorbidities in the devel-
setting opment of post-nephrectomy CKD. The duration of ischemia
is another important predictor of postoperative kidney func-
Answer: b is correct. The pharmacokinetics of all targeted tion. Prolonged warm ischemia time is signicantly associated
anticancer agents are similar. Most are 90%98% bound to with adverse postoperative kidney function.

6 Onco-Nephrology Curriculum American Society of Nephrology


Chapter 14: Hereditary Renal Cancer Syndromes
Katherine L. Nathanson, MD
Department of Medicine, Division of Translational Medicine and Human Genetics, and Cancer Control Program,
Abramson Cancer Center, Perelman School of Medicine at the University of Pennsylvania, Philadelphia, Pennsylvania

INTRODUCTION a wide spectrum of hemangioblastomas of the brain,


spine, and retina, pancreatic cysts and neuroendo-
Inherited forms of renal cancer are estimated to ac- crine tumors, renal cysts and clear cell renal tumors,
count for 2%5% of all kidney cancer (1). Currently, endolympatic sac tumors, and pheochromocyto-
10 inherited cancer susceptibility syndromes are de- mas. vHL disease is found across all ethnic groups,
nitively associated with an increased risk of renal with approximately one-quarter of the incidence due
cancer (Table 1) and are described in more detail be- to de novo mutations; genetic testing for mutations
low. Patients with these inherited syndromes develop in VHL detects nearly 100% of individuals with vHL
kidney cancer at an earlier age; furthermore, the le- disease (6). Disease usually presents in the late teens
sions can be multifocal, bilateral, and heterogeneous. to early 20s, although an occasional individual may
Several, including Birt-Hogg-Dub syndrome be diagnosed in their mid-40s. The presentation of
(BHD), familial clear cell renal cancer due to chromo- renal disease is quite variable even within family
some 3 translocation, hereditary papillary renal cancer, members, with some patients never developing renal
hereditary leiomyomatosis and renal cell cancer cancer, others having a few renal cysts, and others
(HLRCC), and von Hippel-Lindau disease (vHL) with who have bilateral renal cancers and hundreds
have renal cancer as a primary feature, whereas in an- of lesions within each kidney. In part, this variability
other inherited cancer susceptibility syndrome, such as is due to a strong genotypephenotype correlation
BAP1 mutationassociated disease, Lynch syndrome, that is seen with a mutational type predictive of dis-
phosphatase and tensin homologue (PTEN) hamar- ease (7). Patients with type 1 (truncating) mutations
toma syndrome, hereditary pheochromocytoma and have a decreased incidence of pheochromocytoma
paraganglioma (due to SDHx mutations), and tuber- compared with those with type 2 (missense) muta-
ous sclerosis complex, it is a secondary feature. Re- tions (8). Frameshift and nonsense mutations in VHL
cently, mutations in CDKN2B and PBRM1 also have are associated with a high penetrance of clear cell re-
been reported to predispose to clear cell renal cancer in nal cancer, with a risk at age 50 of 70% (8). Full and
single case series (2,3) and, as such, need further vali- partial gene deletions of VHL confer a lower risk at
dation. Many of the genes identied through the studies age 50 of 40%. Families with type 2 mutations have
of familial renal cancer have proven to play a critical role either a low (type 2A) or high risk of clear cell renal cell
in renal cancer development through somatic muta- carcinoma (ccRCC) (type 2B); type 2C families de-
tion, with vHL disease being the exemplar of this par- velop pheochromocytoma only. Type 2A disease is
adigm. The description of families with inherited cancer associated with the Black Forest founder mutation
susceptibility syndromes associated with an increased (Tyr98His) originating from southwestern Germany,
risk of renal cancer has and will lead to the discovery commonly found in the Pennsylvania Dutch pop-
of mutated genes important in the pathogenesis of ulation (9). Despite the variability in phenotype,
renal cancer. Below, the features of the inherited can- screening recommendations for vHL patients are
cer susceptibility syndromes associated with an in- standardized, and in adult include annual CNS (brain,
creased risk of renal cancer, with a focus on the renal spine) and abdominal/pelvic magnetic resonance
manifestations and pathologic features, are reviewed.

Correspondence: Katherine L. Nathanson, Perelman School of


VON-HIPPEL LINDAU DISEASE Medicine at the University of Pennsylvania, 351 BRB 2/3, 421
Curie Blvd., Philadelphia, Pennsylvania 19104. Email: knathans@
exchange.upenn.edu
Patients with this highly penetrant autosomal dom-
inant cancer susceptibility syndrome can present with Copyright 2016 by the American Society of Nephrology

American Society of Nephrology Onco-Nephrology Curriculum 1


Table 1. Inherited cancer susceptibility syndromes associated with an increased risk of renal cancer
Predominant renal
Syndrome Gene Protein Other Cancers Non-Neoplastic Findings
Cancer Type
BAP1 mutation associated BAP1 BRCA associated Clear cell Melanoma Epithelioid atypical
cancer susceptibility protein Uveal melanoma Spitz tumors
Mesothelioma
Birt-Hogg-Dub syndrome FLCN folliculin Oncocytic, Fibrofolliculomas
chromophobe, Lung cysts,
hybrid pneumothorax
Familial clear cell renal Translocation Clear cell
cancer due to chr 3
chromosome 3
translocation
Hereditary FH fumarate hydratase Papillary type 2 Cutaneous
Leiomyomatosis and leiomyomas
Renal Cell Cancer Uterine
leiomyomas
Hereditary Papillary MET c-MET Papillary type 1
Renal Cancer
Hereditary SDHB succinate Clear cell Paraganglioma
Pheochromocytoma SDHC dehydrogenase (distinct phenotype) Pheochromocytoma
and Paraganglioma SDHD subunits B, C, D Gastrointestinal
Stromal Tumor
Lynch Syndrome MLH1 Mismatch repair Urothelial cancer Colorectal cancer
MSH2 proteins (upper tract) Endometrial (uterine)
cancer
MSH6 Ovarian cancer
PMS2
PTEN Hamartoma PTEN PTEN Clear cell Breast cancer Mucocutaneous papules,
Syndrome Thyroid cancer hamartomas, lipomas,
(Cowden syndrome) macrocephaly
Tuberous Sclerosis TSC1 hamartin Angiomyolipoma Angiomyolipomas Facial angiobroma
Complex TSC2 Tuberin Epitheliod Subependymal giant Hypomelanotic macule
angiomyolipoma cell astrocytomas Connective tissue nevus
Forehead plaque
Ungal and peri-ungal
bromas
Von Hippel Lindau VHL pVHL Clear cell CNS - hemangioblastoma Pancreatic, renal cysts
disease Clear cell papillary (brain, spine, retina)
Adrenal - pheochromocytoma
Inner ear - endolymphatic sac
tumors
Pancreas - neuroendocrine
tumors

(MR) imaging, ophthalmologic evaluation, plasma metaneph- chromosome 3p in these tumors and differing ndings in re-
rines, and consultation with vHL expert. The implementation gards to cytokeratin 7 staining (12). Based on multiple natural
of screening guidelines has led to vast improvements in history studies done at the National Cancer Institute, the standard
survival (10). of care for timing of resection of renal cancer in patients with vHL
Classically, vHL disease and mutations in VHL have been as- disease is when there is a solid component of 3 cm. In the initial
sociated with clear cell renal cancers. However, clear cell papillary series, with a follow-up of .5 years, Walther et al. reported no
renal cell carcinoma (CCPRCC), a relatively recently de- evidence of metastatic disease progression and no need for renal
scribed entity with prominent papillary architecture, exclu- transplantation or dialysis among 52 patients with tumors ,3 cm
sive clear cell morphology, and a partially cystic appearance, at diagnosis. In contrast, distant metastases developed in 11 of 44
has been reported in patients with vHL disease (11). The exact patients (25%) with lesions .3 cm in size, including 3 of 27
characteristics of CCPRCC in this context, and relationship to patients (11%) with lesions between 3 and 6 cm (13). Similar
sporadic disease are somewhat controversial, due to loss of results were obtained in a follow-up study 5 years later (14).

2 Onco-Nephrology Curriculum American Society of Nephrology


HEREDITARY PAPILLARY RENAL CELL CARCINOMA BIRT-HOGG-DUB DISEASE
(TYPE 1 PAPILLARY)
BHD disease is an autosomal dominant cancer susceptibility
Hereditary papillary renal cell carcinoma (HPRCC) is an syndrome characterized by the development of brofolliculomas
autosomal dominant syndrome characterized by multifo- (dysplastic hair follicules), lung cysts and spontaneous pneu-
cal, bilateral type 1 papillary renal cell carcinomas, with mothorax, and renal cancer, and is due to mutations in folliculin
hundreds of tumors observed due to mutations in MET (FLCN) (21). The dermatologic features and lung disease are the
(15). The tumors are indistinguishable pathologically most common presenting features; BHD is underdiagnosed due
from sporadic type 1 papillary renal cancer. Mutations of to its variable, often mild, presentation. Awide spectrum of renal
the MET gene on 7q31 have been causally associated with cancers (papillary RCC, ccRCC, mixed, and oncocytomas) has
HPRCC. Families with inherited mutations in MET lead- been observed in patients with BHD, even within the same kid-
ing to multifocal papillary renal cancer (type 1) are quite ney (22). The renal parenchyma surrounding the renal tumor
rare, more so than other described inherited renal cancer can often contain multifocal oncocytosis. The most common
syndromes. type of tumor is an unusual hybrid oncocytic tumor (mixed
oncocytoma and chromophobe). As a hybrid oncocytic tumor
is characteristic of BHD, any patient presenting with one should
HEREDITARY LEIOMYOMATOSIS AND RENAL CELL be evaluated for BHD. Renal cancer is observed in approximately
CANCER (TYPE 2 PAPILLARY) 30% of patients with a highly variable age of diagnosis ranging
from 20 to 83 years, with an average age of 46 years (23). Given
Hereditary leiomyomatosis and renal cell cancer (HLRCC), the low malignant potential of these tumors, it is generally rec-
otherwise known as Reeds syndrome, is an autosomal cancer ommended that screening with abdominal MRI take place every
susceptibility syndrome characterized by the development of 2 years and that the tumors can be observed until they are 3 cm in
cutaneous and uterine leiomyomas and renal cancer, due to diameter prior to resection (24).
mutations in fumarate hydratase (FH) (16). The lifetime risk
of renal cancer is currently estimated to be 15% (17). The
pattern of renal cancer in HLRCC differs from other inherited OTHER INHERITED SYNDROMES WITH AN
renal cancer susceptibility syndromes in that the tumors tend INCREASED RISK OF RENAL CANCER
to be solitary and unilateral and have a more aggressive course
of disease. Independent of underlying architecture, which is BAP1 mutations and familial renal cancer
most commonly described as a subtype of type 2 papillary Mutations in BAP1 (BRCA-associated protein 1) were initially
renal cancer, cells in the renal cancers associated with HLRCC identied through somatic sequencing of renal tumors. One of
have a characteristic pathologic appearance with large nuclei, the tumors in the initial study was found to carry a germ-line
with inclusion-like orangiophilic or eosinophilic nucleoi sur- BAP1 mutation, and subsequent studies suggested that BAP1
rounded by a clear halo, which can be recognized by knowl- mutations predispose to familial clear cell renal cancer, along
edgeable pathologists (18). with uveal and cutaneous melanoma and mesothelioma
Recently, several studies have focused on using immuno- (25,26).
histochemistry with S-(2-succinyl) cysteine (2SC) as a sur-
rogate for FH deciency, as an adjunct to pathologic features Chromosome 3 translocations associated with clear cell
to accurately diagnose HLRCC associated renal cancer. An renal cancer
initial study from Bardella et al. suggested that positive stain- The rst genetic changes identied as associated with inherited
ing with 2SC was sensitive and specic to detect renal cancers risk of clear cell renal cancer were balanced translocations
associated with FH mutations (19). An independent follow- involving chromosome 3; since then, multiple families have
up study conrmed that 2SC demonstrated diffuse and strong been reported with multifocal bilateral disease (27). The
cytoplasmic staining in the conrmed HLRCC tumors com- mechanism behind the increased risk of multifocal clear cell
pared with other tumor types (20). Thus, in addition to renal cancer is thought to be loss of the rearranged chromo-
family and personal history, histology and immunohisto- some during mitosis, which requires a quadrivalent (four
chemistry can be used to assist in the diagnosis of HLRCC chromosomes coming together), leading to greater errors dur-
when the initial presenting manifestation is renal cancer. ing chromosomal segregation. As multiple genes involved in
The mean age of renal cancer diagnosis is 40 years, but the pathogenesis of clear cell renal cancer are located on chro-
metastatic renal cancer can present in the teens. Given the mosome 3p, including VHL, PBRM1, BAP1, and SETD2 (28),
potential early age of renal cancer diagnosis, genetic testing it is not surprising that a mechanism of increased loss of one
is recommended at age 810 for familial FH mutations with allele leads to an increased risk of clear cell renal cancer. His-
annual MRI; however, the risk of renal cancers is relatively tologically, the clear cell renal cancers are indistinguishable
low before age 20, so the drawbacks of screening should be from those associated with VHL mutations, although the age
considered (17). of onset tends to be later than in vHL disease.

American Society of Nephrology Onco-Nephrology Curriculum 3


Lynch syndrome immunohistochemistry (absence of staining) also can be used
Although Lynch syndrome (also known hereditary non- to assist in making the diagnosis.
polyposis colorectal cancer), due to mutations in the mismatch
repair genes (MMR) MLH1, MSH2, MSH6, and PMS2, is most Tuberous sclerosis complex
commonly associated with an increased risk of colorectal, en- Tuberous sclerosis complex (TSC) is an autosomal dominant
dometrial, and ovarian cancers, upper tract urothelial cancers genetic disorder characterized by the formation of hamarto-
are a well-recognized feature (29). The estimated risk ranges mas in multiple organs, including brain, kidney, skin, and lung.
from 2.6% to 11% (30,31). The risk appears to be highest in The formation of hamartomas leads to neurologic disorders,
patients with MSH2 and MLH1 mutations and presents at including epilepsy, mental retardation, and autism, as well
earlier ages than sporadic disease (32). A recent review for as dermatologic manifestations such as facial angiobromas,
urologists suggests that evaluation for Lynch syndrome should renal angiomyolipomas, and pulmonary lymphangiomyoma-
be done when a patient with upper tract urothelial cancer tosis (38). Inactivating mutations in TSC1 encoding hamartin,
presents before age 60 or the family meets Amsterdam I or II or TSC2 encoding tuberin are responsible for the phenotype.
criteria, which includes colorectal, small bowel, ureter, en- Patients with TSC2 mutations are more severely affected with
dometrial, and ovarian cancers (29). Screening for Lynch greater renal involvement among other features. Fifty per-
syndrome can be done using immunohistochemistry for cent to 80% of patients with TSC develop renal lesions in-
the MMR proteins, in which loss of staining suggests the cluding angiomyolipomas (AMLs), cysts, and oncocytomas;
diagnosis. renal cell cancer is estimated to occur in ,5% (with precise
estimates varying across studies) (39). A recent review of
PTEN hamartoma tumor syndrome TSC-associated renal cancer demonstrated young age at di-
Mutations in PTEN are associated with a pleomorphic syndrome agnosis, multifocal disease, an indolent clinical course, and
(PHTS, also known as Cowden disease and Bannayan-Riley- three morphologies: renal angiomyoadenomatous tumor,
Ruvalcaba), which has a variety of disease manifestations rang- chromophobe renal cancer, and a granular eosinophilic-
ing from cancer susceptibility to intellectual disability. Patients macrocystic morphology (40). In patients with TSC2 muta-
are at increased risk of benign and malignant tumors of the tions and multiple renal tumors, it has been shown that they
thyroid, breast, and endometrium; it has been suggested that are due to a shower of second hits with different secondary
renal cancer is an underappreciated component of the cancer TSC2 mutations in each tumor (41). Everolimus is US Food
spectrum (33). Recent estimates suggesting that 3%5% of of and Drug Administration (FDA) approved for treating AMLs
patients with PHTS have renal cancer, with a standardized in the setting of TSC and also should be considered for TSC-
incidence ratio of 31.7 (95% CI, 15.458.4); however, these associated renal cancer.
estimates are based on small numbers and may be due to as-
certainment basis (34). When centrally reviewed, the pathology
of the renal cancers was either papillary or chromophobe (34). CONTROVERSIES IN GENETIC TESTING FOR
Screening for renal cancers is part of the standard surveillance INHERITED SUSCEPTIBILITY TO RENAL CANCER
recommendations for patients with PHTS and is recom-
mended biennially starting at age 40. Two questions recently have arisen in relationship to genetic
testing for inherited susceptibility to renal cancer. Marston
SDHx-associated paraganglioma/pheochromocytoma Linehan and colleagues from the National Cancer Institute have
Mutations in the ve proteins (SDHA, SDHB, SDHC, SDHD, suggested that all patients with renal cancer diagnosed under the
and associated cofactor SDHAF2) that comprise the succinate age of 45 should have consideration of genetic counseling/germ-
dehydrogenase complex, which participates in both the Krebs line mutation testing, even in the absence of a personal or family
cycle, converting fumarate to succinate, and as mitochondrial medical history suggestive of an inherited syndrome (4). The
respiratory chain complex II, have been associated with an rates of renal cancer in patients under age 50 is steadily increas-
increased risk of pheochromocytomas, paragangliomas, gas- ing and has doubled since 1995, going from 3 per 100,000 to 6
trointestinal stromal tumors, and renal cancer (35). Renal can- per 100,000 (5), presumably due to the increasing number of
cer has been most commonly observed in association with incidental renal cancers detected on imaging studies. Although a
patients carrying SDHB mutations, which has the highest cutoff age of 45 for detecting cases of inherited renal cancer may
risk of malignant disease, and these renal tumors have been be quite sensitive, with the background rate increasing so dra-
reported to be particularly aggressive (36). SDHB-associated matically, it would require testing of many individuals to identify
renal cancer displays a characteristic histopathology with solid only a few with inherited disease. Population-based studies of
or focally cystic growth, uniform cytology with eosinophilic mutation testing in early-onset renal cancer are required to an-
occulent cytoplasm, intracytoplasmic vacuolations and in- swer the question of utility of general genetic counseling and
clusions, and round to oval low-grade nuclei (37). Renal can- testing in this setting. An earlier age cutoff, such as at 30, may
cer can be the sentinel diagnosis in the family, so pathologists emerge as a more feasible alternative. As with many other genetic
should be aware of and alert to this potential diagnosis. SDHB diseases, multiplex gene panels using massively parallel

4 Onco-Nephrology Curriculum American Society of Nephrology


sequencing have emerged as an alternative for genetic testing 8. Ong KR, Woodward ER, Killick P, Lim C, Macdonald F, Maher ER. Geno-
for renal cancer susceptibility syndromes (e.g., RenalNext from type-phenotype correlations in von Hippel-Lindau disease. Hum Mutat 28:
143149, 2007
Ambry Genetics). Given the usefulness of renal pathology and
9. Brauch H, Kishida T, Glavac D, Chen F, Pausch F, Her H, Latif F,
extra-kidney manifestations to guide genetic testing, which is rel- Lerman MI, Zbar B, Neumann HP. Von Hippel-Lindau (VHL) disease with
atively unusual for other cancer types, the usefulness of panels pheochromocytoma in the Black Forest region of Germany: Evidence
that include genes in which mutations predispose to vastly differ- for a founder effect. Hum Genet 95: 551556, 1995
ent diseases (e.g., vHL and HLRCC) is not immediately apparent. 10. Schmid S, Gillessen S, Binet I, Brndle M, Engeler D, Greiner J, Hader C,
However, many institutions do not always differentiate renal can- Heinimann K, Kloos P, Krek W, Krull I, Stoeckli SJ, Sulz MC, van Leyen K,
Weber J, Rothermundt C, Hundsberger T. Management of von hippel-
cer pathologies to the degree needed for prioritization of genetic
lindau disease: an interdisciplinary review. Oncol Res Treat 37: 761
testing studies, and even at experienced centers, discrimination 771, 2014
of renal cancer pathologies can be complex on occasion. Thus, 11. Tickoo SK, dePeralta-Venturina MN, Harik LR, Worcester HD, Salama
there has been uptake of massively parallel sequencing panels ME, Young AN, Moch H, Amin MB. Spectrum of epithelial neoplasms in
for renal cancerassociated syndromes, but not to the same ex- end-stage renal disease: An experience from 66 tumor-bearing kidneys
tent as for other cancer types (e.g., breast and ovarian cancers, with emphasis on histologic patterns distinct from those in sporadic
adult renal neoplasia. Am J Surg Pathol 30: 141153, 2006
pheochromocytoma).
12. Rao P, Monzon F, Jonasch E, Matin SF, Tamboli P. Clear cell papillary
renal cell carcinoma in patients with von Hippel-Lindau syndrome:
TAKE HOME POINTS Clinicopathological features and comparative genomic analysis of 3
cases. Hum Pathol 45: 19661972, 2014
13. Walther MM, Choyke PL, Glenn G, Lyne JC, Rayford W, Venzon D,
c Many different types of hereditary renal cancer exist, and in general,
Linehan WM. Renal cancer in families with hereditary renal cancer:
each is associated with a histologic subtype.
Prospective analysis of a tumor size threshold for renal parenchymal
c Renal cancer can either be a major or a minor feature of a cancer suscep- sparing surgery. J Urol 161: 14751479, 1999
tibility syndrome, but early age of onset, unusual or pathognomic pa- 14. Duffey BG, Choyke PL, Glenn G, Grubb RL, Venzon D, Linehan WM,
thology, and multiplicity of tumors all should be red ags, which prompt Walther MM. The relationship between renal tumor size and metasta-
questions about family history and consideration of inherited disease. ses in patients with von Hippel-Lindau disease. J Urol 172: 6365, 2004
15. Schmidt L, Duh FM, Chen F, Kishida T, Glenn G, Choyke P, Scherer SW,
c Standard of care surveillance recommendations are available for essen-
Zhuang Z, Lubensky I, Dean M, Allikmets R, Chidambaram A,
tially all renal cancer susceptibility syndromes and should be followed.
Bergerheim UR, Feltis JT, Casadevall C, Zamarron A, Bernues M,
Richard S, Lips CJ, Walther MM, Tsui LC, Geil L, Orcutt ML, Stackhouse
REFERENCES T, Lipan J, Slife L, Brauch H, Decker J, Niehans G, Hughson MD, Moch
H, Storkel S, Lerman MI, Linehan WM, Zbar B. Germline and somatic
mutations in the tyrosine kinase domain of the MET proto-oncogene in
1. Cho E, Adami HO, Lindblad P. Epidemiology of renal cell cancer. He-
papillary renal carcinomas. Nat Genet 16: 6873, 1997
matol Oncol Clin North Am 25: 651665, 2011
16. Tomlinson IP, Alam NA, Rowan AJ, Barclay E, Jaeger EE, Kelsell D,
2. Jafri M, Wake NC, Ascher DB, Pires DE, Gentle D, Morris MR,
Leigh I, Gorman P, Lamlum H, Rahman S, Roylance RR, Olpin S, Bevan
Rattenberry E, Simpson MA, Trembath RC, Weber A, Woodward ER,
S, Barker K, Hearle N, Houlston RS, Kiuru M, Lehtonen R, Karhu A, Vilkki
Donaldson A, Blundell TL, Latif F, Maher ER. Germline Mutations in the
S, Laiho P, Eklund C, Vierimaa O, Aittomki K, Hietala M, Sistonen P,
CDKN2B tumor suppressor gene predispose to renal cell carcinoma.
Paetau A, Salovaara R, Herva R, Launonen V, Aaltonen LA; Multiple
Cancer Discov 5: 723729, 2015
Leiomyoma Consortium. Germline mutations in FH predispose to
3. Benusiglio PR, Couv S, Gilbert-Dussardier B, Deveaux S, Le Jeune H,
dominantly inherited uterine broids, skin leiomyomata and papillary
Da Costa M, Fromont G, Memeteau F, Yacoub M, Coupier I, Leroux D,
renal cell cancer. Nat Genet 30: 406410, 2002
Mjean A, Escudier B, Giraud S, Gimenez-Roqueplo AP, Blondel C, 17. Menko FH, Maher ER, Schmidt LS, Middelton LA, Aittomki K,
Frouin E, Teh BT, Ferlicot S, Bressac-de Paillerets B, Richard S, Gad S. A Tomlinson I, Richard S, Linehan WM. Hereditary leiomyomatosis and
germline mutation in PBRM1 predisposes to renal cell carcinoma. J renal cell cancer (HLRCC): Renal cancer risk, surveillance and treatment.
Med Genet 52: 426430, 2015 Fam Cancer 13: 637644, 2014
4. Shuch B, Vourganti S, Ricketts CJ, Middleton L, Peterson J, Merino MJ, 18. Merino MJ, Torres-Cabala C, Pinto P, Linehan WM. The morphologic
Metwalli AR, Srinivasan R, Linehan WM. Dening early-onset kidney spectrum of kidney tumors in hereditary leiomyomatosis and renal cell
cancer: implications for germline and somatic mutation testing and carcinoma (HLRCC) syndrome. Am J Surg Pathol 31: 15781585, 2007
clinical management. J Clin Oncol 32: 431437, 2014 19. Bardella C, El-Bahrawy M, Frizzell N, Adam J, Ternette N, Hatipoglu E,
5. Howlader N, Noone AM, Krapcho M, Garshell J, Miller D, Altekruse SF, Howarth K, OFlaherty L, Roberts I, Turner G, Taylor J, Giaslakiotis K,
Kosary CL, Yu M, Ruhl J, Tatalovich Z, Mariotto A, Lewis DR, Chen HS, Macaulay VM, Harris AL, Chandra A, Lehtonen HJ, Launonen V,
Feuer EJ, Cronin KA. SEER Cancer Statistics Review, 1975-2012, Be- Aaltonen LA, Pugh CW, Mihai R, Trudgian D, Kessler B, Baynes JW,
thesda, MD, National Cancer Institute, 2015 Ratcliffe PJ, Tomlinson IP, Pollard PJ. Aberrant succination of proteins
6. Crossey PA, Foster K, Richards FM, Phipps ME, Latif F, Tory K, Jones in fumarate hydratase-decient mice and HLRCC patients is a robust
MH, Bentley E, Kumar R, Lerman MI. Molecular genetic investigations of biomarker of mutation status. J Pathol 225: 411, 2011
the mechanism of tumourigenesis in von Hippel-Lindau disease: Analysis 20. Chen YB, Brannon AR, Toubaji A, Dudas ME, Won HH, Al-Ahmadie HA,
of allele loss in VHL tumours. Hum Genet 93: 5358, 1994 Fine SW, Gopalan A, Frizzell N, Voss MH, Russo P, Berger MF, Tickoo
7. Maher ER, Webster AR, Richards FM, Green JS, Crossey PA, Payne SJ, SK, Reuter VE. Hereditary leiomyomatosis and renal cell carcinoma
Moore AT. Phenotypic expression in von Hippel-Lindau disease: cor- syndrome-associated renal cancer: Recognition of the syndrome by
relations with germline VHL gene mutations. J Med Genet 33: 328332, pathologic features and the utility of detecting aberrant succination by
1996 immunohistochemistry. Am J Surg Pathol 38: 627637, 2014

American Society of Nephrology Onco-Nephrology Curriculum 5


21. Nickerson ML, Warren MB, Toro JR, Matrosova V, Glenn G, Turner ML, 31. Win AK, Lindor NM, Winship I, Tucker KM, Buchanan DD, Young JP,
Duray P, Merino M, Choyke P, Pavlovich CP, Sharma N, Walther M, Rosty C, Leggett B, Giles GG, Goldblatt J, Macrae FA, Parry S, Kalady
Munroe D, Hill R, Maher E, Greenberg C, Lerman MI, Linehan WM, Zbar MF, Baron JA, Ahnen DJ, Marchand LL, Gallinger S, Haile RW,
B, Schmidt LS. Mutations in a novel gene lead to kidney tumors, lung Newcomb PA, Hopper JL, Jenkins MA. Risks of colorectal and other
wall defects, and benign tumors of the hair follicle in patients with the cancers after endometrial cancer for women with Lynch syndrome. J
Birt-Hogg-Dub syndrome. Cancer Cell 2: 157164, 2002 Natl Cancer Inst 105: 274279, 2013
22. Pavlovich CP, Grubb RL 3rd, Hurley K, Glenn GM, Toro J, Schmidt LS, 32. Barrow PJ, Ingham S, OHara C, Green K, McIntyre I, Lalloo F, Hill J,
Torres-Cabala C, Merino MJ, Zbar B, Choyke P, Walther MM, Linehan Evans DG. The spectrum of urological malignancy in Lynch syndrome.
WM. Evaluation and management of renal tumors in the Birt-Hogg- Fam Cancer 12: 5763, 2013
Dub syndrome. J Urol 173: 14821486, 2005 33. Shuch B, Ricketts CJ, Vocke CD, Komiya T, Middelton LA, Kauffman EC,
23. Benusiglio PR, Giraud S, Deveaux S, Mjean A, Correas JM, Joly D, Merino MJ, Metwalli AR, Dennis P, Linehan WM. Germline PTEN mu-
Timsit MO, Ferlicot S, Verkarre V, Abadie C, Chauveau D, Leroux D, tation Cowden syndrome: An underappreciated form of hereditary
Avril MF, Cordier JF, Richard S; French National Cancer Institute In- kidney cancer. J Urol 190: 19901998, 2013
herited Predisposition to Kidney Cancer Network. Renal cell tumour 34. Mester JL, Zhou M, Prescott N, Eng C. Papillary renal cell carcinoma is
characteristics in patients with the Birt-Hogg-Dub cancer suscepti- associated with PTEN hamartoma tumor syndrome. Urology 79: 1187.
bility syndrome: A retrospective, multicentre study. Orphanet J Rare e11187.e7, 2012
Dis 9: 163, 2014 35. Evenepoel L, Papathomas TG, Krol N, Korpershoek E, de Krijger RR,
24. Houweling AC, Gijezen LM, Jonker MA, van Doorn MB, Oldenburg RA, Persu A, Dinjens WN. Toward an improved denition of the genetic and
van Spaendonck-Zwarts KY, Leter EM, van Os TA, van Grieken NC, tumor spectrum associated with SDH germ-line mutations. Genet Med
Jaspars EH, de Jong MM, Bongers EM, Johannesma PC, Postmus PE, 17: 610620, 2015
van Moorselaar RJ, van Waesberghe JH, Starink TM, van Steensel MA, 36. Ricketts CJ, Shuch B, Vocke CD, Metwalli AR, Bratslavsky G,
Gille JJ, Menko FH. Renal cancer and pneumothorax risk in Birt-Hogg- Middelton L, Yang Y, Wei MH, Pautler SE, Peterson J, Stolle CA,
Dub syndrome; an analysis of 115 FLCN mutation carriers from 35 Zbar B, Merino MJ, Schmidt LS, Pinto PA, Srinivasan R, Pacak K,
BHD families. Br J Cancer 105: 19121919, 2011 Linehan WM. Succinate dehydrogenase kidney cancer: An ag-
25. Farley MN, Schmidt LS, Mester JL, Pea-Llopis S, Pavia-Jimenez A, gressive example of the Warburg effect in cancer. J Urol 188:
Christie A, Vocke CD, Ricketts CJ, Peterson J, Middelton L, Kinch L, 20632071, 2012
Grishin N, Merino MJ, Metwalli AR, Xing C, Xie XJ, Dahia PL, Eng C, 
37. Gill AJ, Hes O, Papathomas T, Sedivcov M, Tan PH, Agaimy A,
Linehan WM, Brugarolas J. A novel germline mutation in BAP1 pre- Andresen PA, Kedziora A, Clarkson A, Toon CW, Sioson L, Watson N,
disposes to familial clear-cell renal cell carcinoma. Mol Cancer Res 11: Chou A, Paik J, Clifton-Bligh RJ, Robinson BG, Benn DE, Hills K,
10611071, 2013 Maclean F, Niemeijer ND, Vlatkovic L, Hartmann A, Corssmit EP, van
26. Popova T, Hebert L, Jacquemin V, Gad S, Caux-Moncoutier V, Dubois- Leenders GJ, Przybycin C, McKenney JK, Magi-Galluzzi C, Yilmaz A, Yu
dEnghien C, Richaudeau B, Renaudin X, Sellers J, Nicolas A, Sastre- D, Nicoll KD, Yong JL, Sibony M, Yakirevich E, Fleming S, Chow CW,
Garau X, Desjardins L, Gyapay G, Raynal V, Sinilnikova OM, Andrieu N, Miettinen M, Michal M, Trpkov K. Succinate dehydrogenase (SDH)-
Mani E, de Pauw A, Gesta P, Bonadona V, Maugard CM, Penet C, Avril decient renal carcinoma: A morphologically distinct entity: a clinico-
MF, Barillot E, Cabaret O, Delattre O, Richard S, Caron O, Benfodda M, pathologic series of 36 tumors from 27 patients. Am J Surg Pathol 38:
Hu HH, Sour N, Bressac-de Paillerets B, Stoppa-Lyonnet D, Stern MH. 15881602, 2014
Germline BAP1 mutations predispose to renal cell carcinomas. Am J 38. Crino PB, Nathanson KL, Henske EP. The tuberous sclerosis complex. N
Hum Genet 92: 974980, 2013 Engl J Med 355: 13451356, 2006
27. Woodward ER, Skytte AB, Cruger DG, Maher ER. Population-based 39. Dixon BP, Hulbert JC, Bissler JJ. Tuberous sclerosis complex renal
survey of cancer risks in chromosome 3 translocation carriers. Genes disease. Nephron, Exp Nephrol 118: e15e20, 2011
Chromosomes Cancer 49: 5258, 2010 40. Guo J, Tretiakova MS, Troxell ML, Osunkoya AO, Fadare O, Sangoi AR,
28. Cancer Genome Atlas Research Network. Comprehensive molecular Shen SS, Lopez-Beltran A, Mehra R, Heider A, Higgins JP, Harik LR,
characterization of clear cell renal cell carcinoma. Nature 499: 4349, Leroy X, Gill AJ, Trpkov K, Campbell SC, Przybycin C, Magi-Galluzzi C,
2013 McKenney JK. Tuberous sclerosis-associated renal cell carcinoma: A
29. Mork M, Hubosky SG, Rouprt M, Margulis V, Raman J, Lotan Y, OBrien clinicopathologic study of 57 separate carcinomas in 18 patients. Am J
T, You N, Shariat SF, Matin SF. Lynch syndrome: A primer for urologists Surg Pathol 38: 14571467, 2014
and panel recommendations. J Urol 194: 2129, 2015 41. Tyburczy ME, Jozwiak S, Malinowska IA, Chekaluk Y, Pugh TJ, Wu CL,
30. Sijmons RH, Kiemeney LA, Witjes JA, Vasen HF. Urinary tract cancer Nussbaum RL, Seepo S, Dzik T, Kotulska K, Kwiatkowski DJ. A shower of
and hereditary nonpolyposis colorectal cancer: Risks and screening second hit events as the cause of multifocal renal cell carcinoma in
options. J Urol 160: 466470, 1998 tuberous sclerosis complex. Hum Mol Genet 24: 18361842, 2015

6 Onco-Nephrology Curriculum American Society of Nephrology


REVIEW QUESTIONS have loss of staining on SDHB immunohistochemistry. Col-
lection of additional history also could be useful, in that
1. What is the exception to the 3-cm rule of tumor removal further history of pheochromocytomas and paragangliomas in
in hereditary renal cancer syndromes? the family supports the diagnosis of an SDHx-related tumor.
a. von Hippel Lindau disease However, it is important to note that genetic testing is recom-
b. Birt-Hogg-Dub syndrome mended for all patients with pheochromocytoma, so the family
c. Hereditary leiomyomatosis and renal cell cancer should be referred to a cancer geneticist independent of family
history.
Answer: c is correct. Hereditary leiomyomatosis and renal
cell cancer (HLRCC or Reeds syndrome) is associated with a 3. One of your patients has a hybrid chromophobe/oncocytic
very aggressive form of renal cancer, and tumors should be renal tumor, but is aged 65 and has no family history of
removed as soon as they are detected. renal cancer or other cancers. You vaguely remember
reading this chapter, and remember that tumor type is a red
2. A patient with renal cancer tells you that a sister had a ag,and deserves further evaluation, but cant remember
pheochromocytoma. What further evaluation might that for what?
prompt? a. BAP-1associated renal cancer
a. Review of pathology of the renal cancer and potentially b. Birt-Hogg-Dub syndrome
additional immunohistochemistry c. Lynch syndrome
b. Further family history collection and potentially genetics
evaluation Answer: b is correct. The three major manifestations of
c. Nothing, probably unrelated Birt-Hogg-Dub (BHD) are brofolliculomas (facial), lung
d. a and b cysts and pneumothorax, and renal cancer. A hybrid chromo-
phobe/oncocytic renal tumor is considered characteristic of
Answer: d is correct. Renal cancer is associated with he- BHD, and the nding on pathology should prompt further
reditary pheochromocytoma and paraganglioma syn- evaluation. All three manifestations of BHD are incompletely
dromes caused by SDHx mutations. These renal cancers penetrant, particularly renal cancer. Thus, the lack of family
have a characteristic pathology, which is well described. history of renal cancer and older age of diagnosis should not
The renal cancers associated with SDHB mutations also preclude evaluation for BHD.

American Society of Nephrology Onco-Nephrology Curriculum 7


Chapter 15: Workup and Management of Small
Renal Masses
Susie L. Hu, MD,* and Anthony Chang, MD
*Division of Kidney Disease and Hypertension, Warren Alpert Medical School of Brown University, Rhode Island
Hospital, Providence, Rhode Island; and Department of Pathology, The University of Chicago Medicine,
Chicago, Illinois

INTRODUCTION death) (9). Consideration of preoperative kidney


function, comorbidities (10), nephron-sparing sur-
Nephrologists are frequently asked by urology and gical methods, and tumor size (11) should be made
oncology colleagues to participate in the manage- when determining the management plan for pa-
ment of patients diagnosed with a renal mass. This is tients with small renal masses.
especially the case when there is associated CKD,
hypertension, and/or other medically challenging
comorbidities. Renal masses are classied as large EPIDEMIOLOGY
and small. Small renal masses, dened as T1a (#4
cm) with no metastases or contralateral kidney in- Individuals with small renal tumors are older (average
volvement, have a 5-year survival rate approaching age of 60 years), predominantly white men (1214)
100% in most studies. Therefore, as the vast major- with notable comorbidity (13,14). In the Medicare-
ity of these patients are cured of their kidney cancer, linked US Surveillance Epidemiology and End Re-
the maintenance of renal function is becoming the sults (SEER) database, .10,000 individuals with
major determinant of clinical outcomes. In this era small tumors #7 cm surveyed had an average tumor
of almost indiscriminate use of diagnostic imaging, size of 4 cm, had a higher median age of 74, were
.50% of renal masses are incidentally discovered predominantly white, 8.7% were African American,
(1,2) and 16%23% of these are benign (3,4). Given and 65% were male. In this population with high
the rise of incidental tumors, nephron-sparing pro- burden of comorbid diseases, almost half had dia-
cedures (partial nephrectomy, cryoablation, radio- betes mellitus (DM) or chronic obstructive pulmo-
frequency ablation, or thermal ablation) are increasingly nary disease, a third had cerebrovascular disease,
replacing traditional radical nephrectomy (RN), and approximately 15% had peripheral vascular dis-
and even diagnostic renal biopsies are a viable option ease or preexisting CKD (14). The study population
as the concept of tumor seeding along the needle of the sole prospective randomized controlled mul-
track has been largely unfounded (5). ticenter trial, European Organization for Research
Cancer-specic survival and overall survival and Treatment of Cancer (EORTC) trial, which in-
between radical nephrectomy and nephron-sparing cluded small renal masses (#5cm), had substantial
surgery are comparable (6). For poor or nonsurgical comorbid disease (36%), which was largely cardio-
candidates, greater consideration is being given to vascular disease (8).
ablative therapies or even active surveillance, given Overlapping risk factors between renal cell carci-
that small renal masses grow very slowly at an av- noma and CKD may account for the high prevalence
erage rate of 1.3 mm/yr (7). This population (with for CKD and cardiovascular disease in this popula-
small renal masses) who remain renal cancer free, tion (Figure 1). Risk factors for renal cell carcinoma
are dying of other causes, most frequently due to in the general population have traditionally
cardiovascular events (8). With improving survival,
morbidity related to CKD from nephron mass loss,
as well comorbid diseaseinduced complications, Correspondence: Susie L. Hu, Rhode Island Hospital, 593 Eddy
has become more relevant and ultimately impacts St., Providence, Rhode Island 02903. Email: shu@lifespan.org

survival (with increased risk for cardiovascular Copyright 2016 by the American Society of Nephrology

American Society of Nephrology Onco-Nephrology Curriculum 1


HTN, which are reective of the predominant features dening
the renal mass cohort, and also include lower baseline estimated
GFR and larger tumor size (10,11,20). Hypoalbuminemia (19)
and postoperative AKI (29) are other likely determinants of GFR
decline (Table 1). With tumor resection, CKD prevalence in-
creased anywhere from 10%24% to 16%52% after treatment
(1820). Others reported a mean GFR decrease of 13 mL/min
per 1.73 m2, corresponding to a 30% drop in GFR after partial
nephrectomy (PN), and renal volume reduction seemed to be a
prognostic factor for GFR decline (30). Furthermore, among
diabetics, 60% developed CKD compared with only 43% of
the entire cohort; the 2-year probability of absence of CKD
Figure 1. Overlapping risk factors for renal cell carcinoma and was poor among patients with diabetes (47%) in contrast to
CKD in the general population and among those with small those without DM (76%, P 5 0.006) (20). Incidence of ESRD
renal masses. DM, diabetes mellitus; HLD, hyperlipidemia; HTN, was 5.6 times greater among renal cell carcinoma patients
hypertension; Neph, nephrectomy. (4.05%) than for a comparable control group (0.68%) (10). In
the US Renal Data System (USRDS), renal cell carcinoma has
included tobacco exposure, obesity, DM, and hypertension been reported as a cause for ESRD in 0.5% of the 360,000 pa-
(HTN) (15). Cystic disease and ESRD patients have a greater tients, with a higher mortality compared with other causes of
predisposition to renal tumors (16). This risk is 100-fold for ESRD (28).
ESRD patients (16) and a lesser but signicant risk has been
identied for stage 3 and 4 CKD (17). Therefore, the risk fac-
MANAGEMENT
tors for CKD seem to also predispose one to the development
of renal cell carcinoma.
Preoperative evaluation
With extension of survival, quality-of-life issues shaped by
Historically, small renal mass identication by imaging studies
postoperative chronic disease burden, primarily CKD, and
nearly always led to surgical intervention with the possible
cardiovascular disease have increasingly become an important
exception of an angiomyolipoma, which can be suspected when
factor in the care of these patients.
there is a signicant component of adipose tissue. However, active
surveillance and percutaneous kidney biopsies are viable options
Preoperative CKD and comorbidity
that are increasingly utilized, as up to 23% of patients will have
Prevalence of CKD seems to vary, considerably ranging from
benign small renal masses (oncocytoma or angiomyolipoma)
10% to 52% (1821) for those with small renal masses, similar
and can be spared any additional surgery (3,4). The diagnostic
to that of all-comers with all sizes of renal masses (11%32%)
rate of kidney biopsies approaches 80% in experienced centers
depending on the cutoff GFR and age of the general popula-
(31), and the concordance rate approaches 100% compared with
tion (2224). When examining older subgroups, CKD preva-
the surgical resection specimen (32).
lence nearly doubled (19,23), consistent with the nding that
Preoperative evaluation of potentially modiable risk
increasing age raises CKD risk. Of known CKD risk factors,
factors including DM, HTN, and CKD may play a role in
the most important two, DM and HTN, were also highly prev-
the preservation of renal function. Optimizing glycemic and BP
alent in this population, where 9%22% had diabetes and
control, as well as estimation of GFR and prevention of AKI,
23%59% were hypertensive (18,20,2426). The extent of
may minimize risk for deterioration of GFR postoperatively.
DM and HTN was greater not only among those with a di-
Prevention of AKI can be achieved through proper medical
agnosis of preexisting CKD, but also among those with renal
management to avoid nephrotoxic exposure and renal
cell carcinoma. Not surprisingly, preexisting CKD patients
had more DM (26%) and HTN (60%) in a Korean cohort Table 1. Risk factors for CKD
with small renal masses (n 5 1,928) than those without
Older age (.65 years)
CKD (DM, 12.7%; HTN, 32%) (19). In a case-matched Taiwan-
Male sex
ese cohort of 26,460 patients, those diagnosed with renal cell Tobacco use
carcinoma had greater burden of DM (19.6%) and HTN Comorbid diseases
(30.6%) compared with case-matched controls (DM, 7.7%; Diabetes mellitus
HTN, 14%) without renal cell carcinoma (10). Hypertension
Obesity
CKD risk factors Lower baseline GFR
Predictive factors for new-onset CKD or progression of CKD Larger tumor size
after therapeutic intervention include older age, male sex (6), Surgical procedure: radical nephrectomy
Postoperative AKI
tobacco use (27), obesity (27,28), and concomitant DM or

2 Onco-Nephrology Curriculum American Society of Nephrology


hypoperfusion (33). Renal nuclear scintigraphy has been used advantage in addition to GFR preservation with nephron-sparing
to help determine proportional GFR of each kidney to better surgery (37), which is clearly limited by its retrospective design.
assess the potential impact of renal resection (partial or radical The increased mortality seen with CKD has been attributed
nephrectomy). Preoperative proportional GFR assessment to high cardiovascular disease risk typically associated with
used to calculate the expected postoperative GFR tended to advanced GFR (9). Among 1,004 case-matched patients with
underestimate the actual postoperative GFR by 12% in one relatively small renal mass size (T1b, 47 cm), an association
study (34), presumably due to compensatory hyperltration of greater cardiovascular death and GFR decline was observed.
and hypertrophy. Additionally, one study examining pre- and GFR loss was measured using the difference between extra-
postoperative differential function with nuclear scintigraphy polated values of preoperative and (.3 weeks) postoperative
found that postsurgical differential function inversely corre- GFRs. The average GFR loss was less for those who had partial
lated best with ischemia time and tumor size, which may be nephrectomy (16.6 mL/min) as opposed to radical nephrec-
more predictive of intraoperative renal damage (35). Evaluation of tomy (23.5 mL/min, P ,0.0001). GFR decline generally oc-
differential function remains a useful tool in assessing operative curred within 3 weeks and then stabilized thereafter. Each
risk for CKD, providing we recognize these limitations of total excess loss of GFR of 7 mL/min per 1.73 m2 resulted in a
GFR underestimation (due to hyperltration by the preserved 17% increase risk of death as well as a 25% greater cardiovas-
kidney) and relative GFR overestimation in the surgical kidney. cular disease risk. Cancer-specic survival was not different,
but overall survival was better for partial nephrectomy (85%
Treatment of small renal masses and outcomes versus RN 78%, P 5 0.01) (38).
RN has been the mainstay of therapy for generations and The hard end point of ESRD has also been examined in a
remains an essential therapy for those with larger renal masses Taiwanese incident cohort with 10-year follow-up in a newly
or with lesions extending beyond the affected kidney. Small diagnosed renal cell carcinoma group (n 5 2940) and a control
renal masses have favorable prognosis and survival, which group (n 5 23,520). Progression to ESRD occurred in 4.05%
do not necessitate radical nephrectomy. Partial nephrectomy of the renal cell carcinoma group compared with only 0.68%
has equivalent/comparable oncologic and overall survival and of the control group (HR, 5.63; 95% CI, 4.377.24) with the
greater renal preservation (6,36,37). same risk factors (DM, preexisting CKD) for CKD progression
Innumerable studies have emerged examining these out- (10). In the USRDS, the higher mortality noted with ESRD
comes over the last couple of decades with similar ndings and from renal cell carcinoma compared with other causes was not
may be best illustrated in a meta-analysis performed by Kim observed for those who had undergone nephrectomy, sug-
et al. Risk reduction with nephron-sparing surgery assessed gesting that even though progression to ESRD from renal
from 36 studies (40,000 patients; 31,000 RN and 9,300 PN) cell carcinoma could not be prevented, nephrectomy may still
was 19% for all-cause mortality, 29% for cancer specic mor- confer a lower risk for mortality (28).
tality, and 61% for CKD (37). On the contrary, the EORTC Alternative therapy for small renal masses in individuals
study, which was a clinical trial of 541 patients with solitary with high operative risk includes nonsurgical ablative therapies
unilateral small renal masses (#5 cm), found that overall such as radiofrequency ablation and cryoablation, which
10-year survival was slightly higher for RN (81.1%) than are currently the two most common approaches used. Older
nephron-sparing surgery (75.7%), with a hazard ratio (HR) patients were more likely to receive radiofrequency ablation
of 1.5 (95% CI, 1.032.16). The small difference in mortality (RFA) with fewer major complications (RFA 3.1 % versus PN
was no longer present when only patients diagnosed with 7.2%7.9%, P ,0.001), but with higher local progression of
renal cell carcinoma were considered. Progression of disease disease (RFA 4.6% versus PN 1.2%1.9%, P ,0.001) than
and renal cell carcinoma death were no different between the seen with partial nephrectomy (39). In one series, however,
two therapies (8). oncologic outcomes were no different when excluding those
In the same EORTC study, risk of CKD was retrospectively with high risk for recurrence (40). Cryoablation was also uti-
examined and demonstrated that partial nephrectomy (PN) lized more frequently for older patients with higher operative
preserved GFR (baseline creatinine , 1.25 times the upper risk. In addition to fewer procedural complications, shorter
normal range), particularly early on where the difference of length of hospital stay was noted; however, this was also asso-
patients reaching GFR ,60 mL/min per 1.73 m2 between RN ciated with higher local (relative risk [RR], 9.39; P ,0.0001)
(85.7%) and PN (64.7%) was 21%. However, over time, this and metastatic progression of disease (RR, 4.68; P 5 0.01)
difference of progressive decline to lower GFRs ,30 (RN 10%, after cryoablation compared with partial nephrectomy (both
PN 6.3%) and ,15 mL/min per 1.73 m2 (RN 1.5%, PN 1.6%) performed laparoscopically) (41). Last, active surveillance
became insignicant. The steeper GFR decline observed ini- with judicious monitoring of tumor size and sometimes
tially with RN was not associated with an increase in mortality with delayed surgical intervention resulted in acceptable out-
(36), potentially suggesting that GFR loss from nephrectomy comes particularly among those age .75, which was no worse
did not confer the same risk of death usually seen with GFR than surgical resection for select populations (14). Although
decline from traditional causes of CKD such as DM or HTN oncologic outcomes for ablative therapies may not be as favor-
(9). The cohort studies, however, generally showed a survival able as surgical resection, they provide viable therapeutic options

American Society of Nephrology Onco-Nephrology Curriculum 3


with less complications and likely greater renal parenchymal c Older age, male sex, comorbid diseases including diabetes mellitus,
hypertension, preexisting CKD, and larger tumor size increase risk for
(thus also GFR) preservation for nonoperative candidates.
postoperative CKD.
c Nephron-sparing surgery (partial nephrectomy) among patients with
PATHOLOGIC EVALUATION OF TUMOR small renal masses has equivalent cancer-specic survival, practically
similar overall survival, and better GFR preservation compared with
NEPHRECTOMY
radical nephrectomy.

The evaluation of tumor nephrectomy specimens has always


centered around the renal mass, but careful assessment of the REFERENCES
nonneoplastic kidney parenchyma reveals the presence of
common yet undiagnosed nonneoplastic renal diseases. 1. Patard JJ, Shvarts O, Lam JS, Pantuck AJ, Kim HL, Ficarra V, Cindolo L,
Therefore, the synoptic reports established by the College of Han KR, De La Taille A, Tostain J, Artibani W, Abbou CC, Lobel B,
American Pathologist required in 2010 that the nonneoplastic Chopin DK, Figlin RA, Mulders PF, Belldegrun AS. Safety and efcacy of
parenchyma should be evaluated and reported for every renal partial nephrectomy for all T1 tumors based on an international multi-
center experience. J Urol 171: 21812185, quiz 2435, 2004
malignancy (42). Also, the Accreditation Council for Graduate
2. Chow WH, Devesa SS, Warren JL, Fraumeni JF Jr. Rising incidence of
Medical Education will require that nephropathology be part renal cell cancer in the United States. JAMA 281: 16281631, 1999
of the curriculum for all anatomic pathology residents effec- 3. McKiernan J, Yossepowitch O, Kattan MW, Simmons R, Motzer RJ,
tive July 1, 2015, as the vast majority of pathologists do not Reuter VE, Russo P. Partial nephrectomy for renal cortical tumors: path-
receive any exposure to this subspecialty. Several large studies ologic ndings and impact on outcome. Urology 60: 10031009, 2002
4. Kutikov A, Fossett LK, Ramchandani P, Tomaszewski JE, Siegelman ES,
found that diabetic nephropathy and hypertensive nephropa-
Banner MP, Van Arsdalen KN, Wein AJ, Malkowicz SB. Incidence of
thy (or arterionephrosclerosis) can be identied in 8%20% benign pathologic ndings at partial nephrectomy for solitary renal
and 3%14% of specimens, respectively (4345), and 60% mass presumed to be renal cell carcinoma on preoperative imaging.
88% of these diagnoses were not identied during the initial Urology 68: 737740, 2006
evaluation. With nearly 350,000 kidney cancer survivors in the 5. Volpe A, Kachura JR, Geddie WR, Evans AJ, Gharajeh A, Saravanan A,
Jewett MA. Techniques, safety and accuracy of sampling of renal tu-
United States, the CKD burden will only increase, especially as
mors by ne needle aspiration and core biopsy. J Urol 178: 379386,
more attention is given to the nonneoplastic parenchyma ex- 2007
amined by pathologists. In addition, there still remains much 6. Li L, Lau WL, Rhee CM, Harley K, Kovesdy CP, Sim JJ, Jacobsen S,
room for improvement regarding the coordination of urologists, Chang A, Landman J, Kalantar-Zadeh K. Risk of chronic kidney disease
pathologists, and especially nephrologists in the preoperative after cancer nephrectomy. Nat Rev Nephrol 10: 135145, 2014
7. Jewett MAS, Mattar K, Basiuk J, Morash CG, Pautler SE, Siemens DR,
and postoperative management of kidney cancer patients.
Tanguay S, Rendon RA, Gleave ME, Drachenberg DE, Chow R, Chung H,
Chin JL, Fleshner NE, Evans AJ, Gallie BL, Haider MA, Kachura JR,
Kurban G, Fernandes K, Finelli A. Active surveillance of small renal
CONCLUSIONS
masses: progression patterns of early stage kidney cancer. Eur Urol 60:
3944, 2011
The majority of renal tumors are small renal tumors discovered 8. Van Poppel H, Da Pozzo L, Albrecht W, Matveev V, Bono A, Borkowski
on routine imaging with excellent oncologic and overall sur- A, Colombel M, Klotz L, Skinner E, Keane T, Marreaud S, Collette S,
vival. The prolonged survival with earlier discovery has Sylvester R. A prospective, randomised EORTC intergroup phase 3
study comparing the oncologic outcome of elective nephron-sparing
resulted in higher likelihood of nononcologic death, where
surgery and radical nephrectomy for low-stage renal cell carcinoma. Eur
patients are saddled instead with CKD and associated increased Urol 59: 543552, 2011
cardiovascular morbidity and mortality. Minimally invasive 9. Go AS, Chertow GM, Fan D, McCulloch CE, Hsu CY. Chronic kidney
techniques for diagnosis and nephron-sparing surgery have disease and the risks of death, cardiovascular events, and hospitaliza-
minimized nephron mass and functional loss. Recognizing and tion. N Engl J Med 351: 12961305, 2004
10. Hung PH, Tsai HB, Hung KY, Muo CH, Chung MC, Chang CH, Chung
assessing modiable risk factors for CKD such as HTN, DM,
CJ. Increased risk of end-stage renal disease in patients with renal cell
and cardiovascular disease may potentially allow for greater carcinoma: A 12-year nationwide follow-up study. Medicine (Baltimore)
preservation of GFR and reduction of cardiovascular disease 93: e52, 2014
related death. To achieve this goal, communication and co- 11. Jeon HG, Choo SH, Sung HH, Jeong BC, Seo SI, Jeon SS, Choi HY, Lee
ordination of management is essential within the specialty HM. Small tumour size is associated with new-onset chronic kidney
disease after radical nephrectomy in patients with renal cell carcinoma.
care team, which is comprised of nephrologists, oncologists,
Eur J Cancer 50: 6469, 2014
urologists, and pathologists. 12. Crpel M, Jeldres C, Sun M, Lughezzani G, Isbarn H, Alasker A,
Capitanio U, Shariat SF, Arjane P, Widmer H, Graefen M, Montorsi F,
Perrotte P, Karakiewicz PI. A population-based comparison of cancer-
TAKE HOME POINTS control rates between radical and partial nephrectomy for T1A renal
cell carcinoma. Urology 76: 883888, 2010
c Early diagnosis of small renal tumors is rising due to incidental discovery 13. Lane BR, Campbell SC, Gill IS. 10-year oncologic outcomes after lap-
with favorable prognosis. aroscopic and open partial nephrectomy. J Urol 190: 4449, 2013
c New-onset CKD is fairly common after nephrectomy with overlapping 14. Sun M, Becker A, Tian Z, Roghmann F, Abdollah F, Larouche A,
risk factors for CKD and renal cell carcinoma. Karakiewicz PI, Trinh QD. Management of localized kidney cancer:

4 Onco-Nephrology Curriculum American Society of Nephrology


Calculating cancer-specic mortality and competing risks of death for 31. Volpe A, Finelli A, Gill IS, Jewett MA, Martignoni G, Polascik TJ, Remzi
surgery and nonsurgical management. Eur Urol 65: 235241, 2014 M, Uzzo RG. Rationale for percutaneous biopsy and histologic char-
15. Chow W-H, Dong LM, Devesa SS. Epidemiology and risk factors for acterisation of renal tumours. Eur Urol 62: 491504, 2012
kidney cancer. Nat Rev Urol 7: 245257, 2010 32. Volpe A, Mattar K, Finelli A, Kachura JR, Evans AJ, Geddie WR, Jewett
16. Denton MD, Magee CC, Ovuworie C, Mauiyyedi S, Pascual M, Colvin MA. Contemporary results of percutaneous biopsy of 100 small renal
RB, Cosimi AB, Tolkoff-Rubin N. Prevalence of renal cell carcinoma in masses: A single center experience. J Urol 180: 23332337, 2008
patients with ESRD pre-transplantation: A pathologic analysis. Kidney 33. Thadhani R, Pascual M, Bonventre JV. Acute renal failure. N Engl J Med
Int 61: 22012209, 2002 334: 14481460, 1996
17. Lowrance WT, Ordoez J, Udaltsova N, Russo P, Go AS. CKD and the 34. Bachrach L, Negron E, Liu JS, Su YK, Paparello JJ, Eggener S, Kundu
risk of incident cancer. J Am Soc Nephrol 25: 23272334, 2014 SD. Preoperative nuclear renal scan underestimates renal function after
18. Barlow LJ, Korets R, Laudano M, Benson M, McKiernan J. Predicting radical nephrectomy. Urology 84: 14021406, 2014
renal functional outcomes after surgery for renal cortical tumours: A 35. Sankin A, Sfakianos JP, Schiff J, Sjoberg D, Coleman JA. Assessing
multifactorial analysis. BJU Int 106: 489492, 2010 renal function after partial nephrectomy using renal nuclear scintigra-
19. Kim SH, Lee SE, Hong SK, Jeong CW, Park YH, Kim YJ, Kang SH, Hong phy and estimated glomerular ltration rate. Urology 80: 343346,
SH, Choi WS, Byun SS. Incidence and risk factors of chronic kidney 2012
disease in Korean patients with t1a renal cell carcinoma before and after 36. Scosyrev E, Messing EM, Sylvester R, Campbell S, Van Poppel H. Renal
radical or partial nephrectomy. Jpn J Clin Oncol 43: 12431248, 2013 function after nephron-sparing surgery versus radical nephrectomy:
20. Jeon HG, Jeong IG, Lee JW, Lee SE, Lee E. Prognostic factors for Results from EORTC randomized trial 30904. Eur Urol 65: 372377,
chronic kidney disease after curative surgery in patients with small renal 2014
tumors. Urology 74: 10641068, 2009 37. Kim SP, Thompson RH, Boorjian SA, Weight CJ, Han LC, Murad MH,
21. Huang WC, Levey AS, Serio AM, Snyder M, Vickers AJ, Raj GV, Scardino Shippee ND, Erwin PJ, Costello BA, Chow GK, Leibovich BC. Com-
PT, Russo P. Chronic kidney disease after nephrectomy in patients with parative effectiveness for survival and renal function of partial and
renal cortical tumours: A retrospective cohort study. Lancet Oncol 7: radical nephrectomy for localized renal tumors: A systematic review
735740, 2006 and meta-analysis. J Urol 188: 5157, 2012
22. Kaushik D, Kim SP, Childs MA, Lohse CM, Costello BA, Cheville JC, 38. Weight CJ, Larson BT, Fergany AF, Gao T, Lane BR, Campbell SC,
Boorjian SA, Leibovich BC, Thompson RH. Overall survival and devel- Kaouk JH, Klein EA, Novick AC. Nephrectomy induced chronic renal
opment of stage IV chronic kidney disease in patients undergoing insufciency is associated with increased risk of cardiovascular death
partial and radical nephrectomy for benign renal tumors. Eur Urol 64: and death from any cause in patients with localized cT1b renal masses.
600606, 2013 J Urol 183: 13171323, 2010
23. Canter D, Kutikov A, Sirohi M, Street R, Viterbo R, Chen DY, Greenberg 39. Wang S, Qin C, Peng Z, Cao Q, Li P, Shao P, Ju X, Meng X, Lu Q, Li J,
RE, Uzzo RG. Prevalence of baseline chronic kidney disease in patients Wang M, Zhang Z, Gu M, Zhang W, Yin C. Radiofrequency ablation
presenting with solid renal tumors. Urology 77: 781785, 2011 versus partial nephrectomy for the treatment of clinical stage 1 renal
24. Clark MA, Shikanov S, Raman JD, Smith B, Kaag M, Russo P, Wheat JC, masses: A systematic review and meta-analysis. Chin Med J (Engl) 127:
Wolf JS Jr, Matin SF, Huang WC, Shalhav AL, Eggener SE. Chronic 24972503, 2014
kidney disease before and after partial nephrectomy. J Urol 185: 4348, 40. Olweny EO, Park SK, Tan YK, Best SL, Trimmer C, Cadeddu JA. Ra-
2011 diofrequency ablation versus partial nephrectomy in patients with sol-
25. Choi YS, Park YH, Kim YJ, Kang SH, Byun SS, Hong SH. Predictive itary clinical T1a renal cell carcinoma: Comparable oncologic outcomes
factors for the development of chronic renal insufciency after renal at a minimum of 5 years of follow-up. Eur Urol 61: 11561161, 2012
surgery: A multicenter study. Int Urol Nephrol 46: 681686, 2014 41. Klatte T, Shariat SF, Remzi M. Systematic review and meta-analysis of
26. Takagi T, Kondo T, Iizuka J, Kobayashi H, Hashimoto Y, Nakazawa H, Ito F, perioperative and oncologic outcomes of laparoscopic cryoablation
Tanabe K. Postoperative renal function after partial nephrectomy for renal versus laparoscopic partial nephrectomy for the treatment of small re-
cell carcinoma in patients with pre-existing chronic kidney disease: A nal tumors. J Urol 191: 12091217, 2014
comparison with radical nephrectomy. Int J Urol 18: 472476, 2011 42. Srigley JR, Amin MB, Delahunt B, Campbell SC, Chang A, Grignon DJ,
27. Malcolm JB, Bagrodia A, Derweesh IH, Mehrazin R, Diblasio CJ, Wake Humphrey PA, Leibovich BC, Montironi R, Renshaw AA, Reuter VE;
RW, Wan JY, Patterson AL. Comparison of rates and risk factors for Members of the Cancer Committee, College of American Pathologists.
developing chronic renal insufciency, proteinuria and metabolic aci- Protocol for the examination of specimens from patients with invasive
dosis after radical or partial nephrectomy. BJU Int 104: 476481, 2009 carcinoma of renal tubular origin. Arch Pathol Lab Med 134: e25e30,
28. Stiles KP, Moffatt MJ, Agodoa LY, Swanson SJ, Abbott KC. Renal cell 2010
carcinoma as a cause of end-stage renal disease in the United States: 43. Bijol V, Mendez GP, Hurwitz S, Rennke HG, Nos V. Evaluation of the
Patient characteristics and survival. Kidney Int 64: 247253, 2003 nonneoplastic pathology in tumor nephrectomy specimens: predicting
29. Cho A, Lee JE, Kwon G-Y, Huh W, Lee HM, Kim YG, Kim DJ, Oh HY, the risk of progressive renal failure. Am J Surg Pathol 30: 575584, 2006
Choi HY. Post-operative acute kidney injury in patients with renal cell 44. Salvatore SP, Cha EK, Rosoff JS, Seshan SV. Nonneoplastic renal cor-
carcinoma is a potent risk factor for new-onset chronic kidney disease tical scarring at tumor nephrectomy predicts decline in kidney function.
after radical nephrectomy. Nephrol Dial Transplant 26: 34963501, Arch Pathol Lab Med 137: 531540, 2013
2011 45. Henriksen KJ, Meehan SM, Chang A. Non-neoplastic renal diseases are
30. Song C, Bang JK, Park HK, Ahn H. Factors inuencing renal function re- often unrecognized in adult tumor nephrectomy specimens: A review
duction after partial nephrectomy. J Urol 181: 4853, discussion 5354, 2009 of 246 cases. Am J Surg Pathol 31: 17031708, 2007

American Society of Nephrology Onco-Nephrology Curriculum 5


REVIEW QUESTIONS nephropathy occurs in ,2% of specimens. Amyloidosis oc-
curred in 3% of specimens according to a study in the late
1. What are the common risk factors for both CKD and renal 1980s, but is much less common due to the signicant stage
cell carcinoma (RCC)? migration that has occurred toward smaller neoplasms as a
a. Obesity result of early detection. Membranous nephropathy and
b. Diabetes mellitus pauci-immune crescentic glomerulonephritis rarely occur in
c. Hypertension the setting of kidney cancer, with only case reports being avail-
d. All of the above able in the literature.

Answer: d is correct. Diabetes, hypertension, and obesity 3. Which of the following statements is true regarding out-
are independent risk factors for renal cell carcinoma. Diabetes comes in the management of small renal masses?
and hypertension can be found in 25% and up to 60% of a. Cancer-specic survival and overall survival is superior
kidney cancer patients, respectively. These are also the two with radical nephrectomy, which should be considered
most common causes of ESRD. Given the link between CKD primarily in this population
and RCC, it is not surprising to nd these common overlap- b. Partial nephrectomy is associated with GFR preservation
ping risk factors. Obesity is a lesser but also signicant risk and comparable cancer-specic survival and overall sur-
factor common to both CKD and RCC. vival to that of radical nephrectomy
c. Ablative therapies are associated with increased treatment-
2. Which of the following is the most common pathologic
related complication rates but less disease progression
nding in tumor nephrectomy specimens? d. Active surveillance is not indicated for management of
a. Diabetic nephropathy small renal masses
b. Membranous nephropathy
c. IgA nephropathy Answer: b is correct. Cancer-specic survival and overall
d. Focal segmental glomerulosclerosis survival are equivalent between partial and radical nephrec-
e. Minimal change disease tomy. GFR preservation is greater with partial nephrectomy.
For small renal masses, nephron-sparing procedure should
Answer: a is correct. Diabetes is an independent risk factor be considered rst. Ablative therapies have worse oncologic
for RCC and is found in up to 25% of kidney cancer patients. outcomes but lower treatment-related complications. Ac-
Therefore, diabetic nephropathy is common and can be iden- tive surveillance in select populations such as older poor
tied in 8%20% of tumor nephrectomy specimens (depend- operative candidates have been shown to have comparable
ing on the denition of diabetic nephropathy). IgA outcomes.

6 Onco-Nephrology Curriculum American Society of Nephrology


Chapter 16: Cancer in Solid Organ Transplantation
Mona D. Doshi, MD
Department of Medicine, Wayne State University, Detroit, Michigan

INTRODUCTION lymphoma (NHL; Epstein-Barr virus [EBV]), Ka-


posi sarcoma (human herpes virus-8 [HHV8]),
Solid organ transplantation provides lifesaving ther- liver (hepatitis C virus [HCV] and hepatitis B virus
apy for patients with end-organ disease. The Scien- [HBV]), and anogenital cancers (human papilloma
tic Registry of Transplant Recipients (SRTR) report virus [HPV]). The increased risk of these cancers is
announced that 17,654 kidney, 6455 liver, 1946 believed to be related to impaired immune control
lung, 2554 heart, and 109 intestinal transplants were of these oncogenic viruses, which can be present in
performed in 2013, superseding the number of the recipient prior to transplant or transmitted at
transplants from prior years (1). It also reports con- time of transplant via the donor organ. This notion
tinual improvement in death-censored graft sur- is supported by similar amplied risks of these
vival (2,3). Success of eld of transplantation is cancers in individuals with compromised immune
reected in the rising number and longevity of the systems, i.e., HIV/AIDS, and reversal of this height-
transplant organs. However, the same report also ened risk on withdrawal of immunosuppression,
noted an increase in recipient death with graft func- i.e., kidney graft failure and reinitiating of dialysis
tion, primarily due to increased infections and can- (7). Independent of immunosuppressive effects,
cers associated with chronic immunosuppression. the antirejection medications such as azathioprine
Malignancy is the second leading cause of death and cyclosporine directly enhance the carcinogenic
with graft function (4). The risk of cancer is two- effects of ultraviolet (UV) radiation via inhibiting
to four-fold higher in transplant recipients than DNA repair and resulting in apoptosis of keratino-
age-, sex-, and race-matched individuals from similar cyte (8,9). Certain malignancies with no obvious
geographic areas (5,6). Not only are cancers com- infectious causes are also reported to be elevated,
mon, but they tend to be more aggressive and are i.e., colon, bladder, lip, kidney, and thyroid cancer.
associated with increased mortality among transplant These cancers have been reported to occur at a
recipients than in the general population. The relative greater frequency in patients with kidney disease
risk varies by age. Children have the highest increase prior to receiving a kidney transplant but do not
(1530 times), and older individuals (i.e., .65 years occur frequently in people with HIV/AIDS, conrm-
of age) experience a two-fold increase. The magni- ing the lack of a role of the immune system. Instead,
tude of increase in risk for all cancer types is similar they may be occurring due to underlying renal
across organ recipients; the incidence of specic or bladder disease, loss of kidney function, and/
cancer varies by transplanted organ. Knowledge or malignant transformation of acquired cystic kid-
of cancer types, including the magnitude of in- ney disease (common in individuals with renal fail-
creased risk and its clinical course, can help de- ure) (6). Traditional risk factors such as smoking
velop prevention and early detection protocols and alcohol intake leading to organ failure continue
and prompt management (including adjustment of to play a role in development of cancer, i.e., cancer
immunosuppression) to minimize cancer related in the native lung for those receiving a solitary lung
deaths. transplant for chronic obstructive pulmonary dis-
ease, liver cancer in alcoholics, and colon cancer in

COMMON CANCER TYPES IN


TRANSPLANT RECIPIENTS Correspondence: Department of Medicine, Wayne State Uni-
versity, Harper Professional Ofce Building, 4160 John R, Suite
908, Detroit, Michigan 48201.
The incidence of cancer is highest for malignancies
related to viral infections, including non-Hodgkin Copyright 2016 by the American Society of Nephrology

American Society of Nephrology Onco-Nephrology Curriculum 1


patients with ulcerative colitis requiring a liver transplant due appearance and distribution of cancer depends on recipient age.
to primary sclerosing cholangitis. There are a few case reports The older recipients are likely to have their rst lesion within 3
of malignancy transmitted via donor organ. To summarize, both years of transplant and it will typically develop on their head,
immunosuppression and host-related factors play an important whereas in younger folks, it occurs later (typically after 8 years),
role in the increased risk of cancer in transplant recipients. Table and lesions are located on dorsum of their hands. Squamous
1 lists the names of common malignancies with their incidence cell cancers in transplant recipients are also more aggressive,
rates after transplantation. especially when poorly differentiated on histology. The treat-
Of note, the cancer risk of nasopharynx, cervix, prostate, ment of skin cancers depends on the type of lesion and its extent.
breast, brain, and chronic lymphocytic leukemia are reported Supercial lesions can be managed with cryotherapy while deeper
to be lower in transplant recipients than in the general pop- lesions require excision with clean margins. Last, changing
ulation (5). Similar observations have been made in people immunosuppression to a mammalian target of rapamycin inhib-
with HIV infection, together suggesting that the immune sys- itor (mTORi)-based regimen has been shown to decrease the risk
tem may not be primarily controlling the development and of recurrent cancers (11).
growth of these cancers (10). Alternatively, this observation
can also be explained by aggressive screening for cervical, Kaposi sarcoma
prostate, and breast cancer in the transplant population, lead- Kaposi sarcoma occurs 80500 times more frequently in
ing to prompt treatment of precancerous lesions prior to or after transplant recipients than in non-immunosuppressed popu-
transplant. lations. In addition, it tends to be more aggressive and multi-
centric with visceral involvement. Most cases occur due to
infection with HHV-8, and therefore, are commonly seen in
CLINICAL COURSE OF COMMON CANCERS recipients from a high sero-prevalence area, i.e., Mediterranean
and African regions. It is also more common in men, with a
Non-melanoma skin cancer male to female ratio of 3:1, and often occurs within the rst year
Non-melanoma skin cancer is the most common malignancy of transplant. Ninety percent of them present as cutaneous
in adult white solid organ transplant recipients. Squamous and lesions on the legs or mucosal angiomatous lesions. Visceral
basal cell carcinoma account for .90% of all skin cancers. involvement commonly occurs in heart and liver transplant
Unlike the general population, squamous cell carcinoma is recipients. The mainstay of treatment is reduction of immu-
the most common skin cancer. It occurs 250 times as fre- nosuppression, but this may lead to graft dysfunction (12).
quently as that seen in the general population, whereas the
risk of basal cell cancer is increased by 10-fold. Thus, the ratio Non-Hodgkin lymphoma
of squamous cell to basal cell cancer in patients without trans- NHL, which is more commonly referred to as post-transplant
plant (1:4) is reversed in transplant patients (4:1). A third of lymphoproliferative disorders (PTLD), occurs seven to eight
patients will have both types of skin cancer. times more frequently than in the general population (stan-
The risk factors for developing skin cancer are as follows: 1) dardized incidence ratio [SIR], 7.54; 95% CI, 7.177.93). It
recipient related: history of skin cancer prior to transplant, the occurs more commonly in young (034 years) and older ($50
presence of premalignant skin lesions (warts or keratosis), years) male recipients. The incidence is highest in lung and
exposure to solar UV rays, location of residence (highest heart recipients and lowest in kidney transplant recipients,
incidence in Australia), older age, male sex, and fair skin possibly due to varying transfer of lymphoid tissue during
phenotype; 2) immunosuppression related: duration and type organ transplant and intensity of immunosuppression. Its oc-
(mainly azathioprine and cyclosporine); and 3) infection currence is associated with the use of T celldepleting agents
related: keratinocytes of transplant recipients are more likely and a mycophenolic acidbased antirejection regimen. Data
to be infected with HPV than nontransplant people. The on the risk of PTLD with use of tacrolimus are equivocal. Use
of cyclosporine and azathioprine is not associated with in-
creased risk of lymphoma. It commonly presents within the
Table 1. Common malignancies and incidence rates after
rst year of transplant or 5 years after transplant. Early-onset
transplant in United States
lymphoma is related to primary EBV infection, and late-onset
Incidence/10,000
Cancer type SIR (95% CI) lymphoma is independent of infection. Its pathology ranges
person-years
from benign hyperplasia to lymphoid malignancy. PTLD dif-
Skin cancer 23.7 13.85 (11.9216.00) fers from lymphoma in the general population not only in
Kaposi sarcoma 15.5 61.46 (50.9573.49)
histopathologic ndings, but it is also associated with in-
PTLD 194.0 7.54 (7.177.93)
creased extranodal involvement, predominant occurrence in
Lung 173.4 1.97 (1.862.08)
Liver 120 11.56 (10.8312.33) the transplanted organ, an aggressive clinical course, and poor
Kidney 97 4.65 (4324.99) outcomes. The 5-year survival was 41% and did not vary based
Data were obtained from reference 5. SIR, standardized incidence ratio (observed/ on time of presentation. The mortality was higher in heart
expected cases). transplant recipients than in kidney transplant recipients

2 Onco-Nephrology Curriculum American Society of Nephrology


due to inability to withdraw/cease immunosuppression. In for routine age-appropriate screening, as that in general pop-
addition to conventional therapy, the mainstay of treatment ulation, is recommended for all (Table 2). Annual instead of
includes reduction in immunosuppression, especially anti- biannual pap testing is recommended to detect precancerous
proliferative agents, and use of antiviral agents in those with lesions that may progress faster to cancer under inuence of
primary EBV infections (13,14). immunosuppression. There are no data on vaccinating trans-
plant recipients who are HPV nave. Annual mammograms
Lung cancer are also recommended for all women over age 50. The patients
Risk of lung cancer in transplant recipients is moderately should be counseled about higher incidence of false-positive
increased compared with that in the general population (SIR, ndings (calcication and increased density of breast with
6.13; 95% CI, 5.187.21). It is common in older male transplant chronic steroid use), resulting in increased interventions. In
recipients. It is more common in lung transplant recipients, with addition, recipients should also be screened for colorectal can-
the highest risk occurring in the rst 6 months of transplant cers with yearly fecal occult blood testing and ex sigmoidos-
(SIR, 11.17; 95% CI, 7.4816.04) and falling to a ve-fold greater copy or colonoscopy every 5 years. Of note, most of these
risk thereafter. The elevated early risk of cancer may be due to practices have not been validated in a transplant cohort (19).
cancer in the explanted lung (15). A novel study of single-lung In the absence of evidence, an individualized approach to
versus bilateral-lung transplant recipients matched for underly- screening should be used based on the individuals cancer
ing disease, smoking history, and age reported a ve-fold in- risk, existing comorbidities, overall life expectancy, and pref-
crease in cancer among those with single-lung transplants, erence for screening.
where the primary cancer was noted in the lung of the recipient Skin cancer may be prevented by using sunscreen (SPF 115)
(16). In addition to smoking and chronic immunosuppression, and sun hats, avoiding sun peak hours, and covering up the
chronic inammation and repeated infections may be playing a exposed skin with long sleeves. Annual follow-up with an ex-
role in development of lung cancer in native lung (17). Recipi- perienced dermatologist for total body skin examination is
ents of other organs had smaller elevations in their risk, and its also advocated for those at high risk. Systemic retinoid should
occurrence increased with time. be avoided, and topical retinoid treatments can be tried to
treat dysplastic lesions but with caution due to fear of in-
Liver cancer creased risk of rejection. Those with repeated precancerous
Risk of liver cancer was strongly elevated in liver transplant skin lesions can be counseled to switch to an mTOR
recipients compared with the general population (SIR, 43.83; inhibitor-based immunosuppressive regimen. Routine
95% CI, 40.9046.91). Ninety-ve percent of liver cancers were screening for renal cancer is not recommended (19).
diagnosed within the rst 6 months after transplant. Like the In addition to traditional therapy, reduction in immuno-
lung, the increased incidence of cancer within 6 months of trans- suppression is often recommended. The underlying idea is that
plant may be due to delayed recognition of cancer in the explan- this allows immune reconstitution and control of the malig-
ted liver. Thereafter, the risk of liver cancer was two-fold higher nancy by the recipients recovering immune system. If
than the general population. These late-onset liver cancers may
be due to recurrent disease related to HCV or HBV. Liver cancer Table 2. Common cancers and recommendations for
risk is not increased among other organ recipients (5). screening
Cancer type Recommendations for screening
Kidney cancer
Breast Annual or biennial mammography for all women
The risk of kidney cancer was highest in kidney transplant older than 50 years; for women between 40 and 49
recipients (SIR, 6.66; 95% CI, 1.573.04), but was also elevated years, no evidence for or against screening
in liver and heart recipients. Among all recipients, kidney can- Colorectal Annual fecal occult blood testing and/or 5-year
cer occurs mainly in older men and had a bimodal pattern of exible sigmoidoscopy or colonoscopy for
presentation. It occurred within the rst 6 months, and a sec- individuals .50 years
ond peak was seen 415 years after transplant. Some of the Cervical Annual pap and pelvic examination once sexually
early cases can be explained by malignant transformation of active
the cysts that develop in patients with ESRD prior to trans- Prostate Annual digital rectal examination and PSA in all
males after age 50 years
plant (6). As mentioned previously, the risk of renal cancer is
Liver a-Fetoprotein and liver ultrasound every 6 months in
already high among patients with CKD and continues to re-
high-risk individuals, i.e., HBV or HCV infection,
main high after transplant (18).
but no rm data
Skin Monthly self-examination and total body skin
examination every 612 months by an expert skin
SURVEILLANCE AND MANAGEMENT physician
Kidney No rm recommendation, but some have suggested
In view of the higher cancer incidence and poorer prognosis, regular ultrasound of native kidneys
prevention and screening play an important role. Surveillance PSA, prostate-specic antigen. Adapted from reference 24.

American Society of Nephrology Onco-Nephrology Curriculum 3


immunosuppression is stopped or lowered, particularly early and other risk factors for lip cancer after kidney transplantation. Cancer
after transplantation, graft monitoring at short intervals is Epidemiol Biomarkers Prev 18: 561569, 2009
8. Perrett CM, Walker SL, ODonovan P, Warwick J, Harwood CA, Karran P,
necessary. Successful reduction or cessation of immunosup- McGregor JM. Azathioprine treatment photosensitizes human skin to
pression was reported in transplanted patients who developed ultraviolet A radiation. Br J Dermatol 159: 198204, 2008
NHL and Kaposi sarcoma (20). Use of mTORi has been shown 9. Yarosh DB, Pena AV, Nay SL, Canning MT, Brown DA. Calcineurin in-
to reduce risk of new squamous cell cancer in patients with a hibitors decrease DNA repair and apoptosis in human keratinocytes
prior history of skin cancer (11,21) and are also very effective following ultraviolet B irradiation. J Invest Dermatol 125: 10201025,
2005
in treating Kaposi sarcoma (22). However, tolerability of 10. Grulich AE, van Leeuwen MT, Falster MO, Vajdic CM. Incidence of
mTORi is poor and is associated with a 35% discontinuation cancers in people with HIV/AIDS compared with immunosuppressed
rate. Although there are strong data favoring the use of an transplant recipients: A meta-analysis. Lancet 370: 5967, 2007
mTORi-based regimen in those with skin cancers and Kaposi 11. Campbell SB, Walker R, Tai SS, Jiang Q, Russ GR. Randomized con-
sarcoma, there are insufcient data for solid organ cancers (23). trolled trial of sirolimus for renal transplant recipients at high risk for
nonmelanoma skin cancer. Am J Transplant 12: 11461156, 2012
In summary, cancer remains a leading cause of morbidity 12. Penn I. Kaposis sarcoma in transplant recipients. Transplantation 64:
and mortality in transplant recipients. Routine surveillance 669673, 1997
and early detection with prompt intervention directed at 13. Parker A, Bowles K, Bradley JA, Emery V, Featherstone C, Gupte G,
cancer and immunosuppression are recommended to improve Marcus R, Parameshwar J, Ramsay A, Newstead C; Haemato-oncology
the life of the recipient and their transplant organ. Task Force of the British Committee for Standards in Haematology and
British Transplantation Society. Management of post-transplant lym-
phoproliferative disorder in adult solid organ transplant recipients
BCSH and BTS Guidelines. Br J Haematol 149: 693705, 2010
TAKE HOME POINTS
14. Pirsch JD, Stratta RJ, Sollinger HW, Hafez GR, DAlessandro AM,
Kalayoglu M, Belzer FO. Treatment of severe Epstein-Barr virus-
c Cancer risk is increased by two- to four-fold in transplant recipients and induced lymphoproliferative syndrome with ganciclovir: Two cases
tends to be more aggressive than age-, sex-, and race-matched indi- after solid organ transplantation. Am J Med 86: 241244, 1989
viduals from the general population. 15. Ritchie AJ, Mussa S, Sivasothy P, Stewart S. Single-lung transplant
c Skin cancer is the most common cancer, followed by PTLD and cancer of complicated by unexpected explant carcinoma: A management di-
lemma. J Heart Lung Transplant 26: 12061208, 2007
the transplanted organ.
16. Dickson RP, Davis RD, Rea JB, Palmer SM. High frequency of bron-
c Emphasis should be placed on adherence to recommended cancer chogenic carcinoma after single-lung transplantation. J Heart Lung
surveillance protocols for early detection and prompt management. Transplant 25: 12971301, 2006
c Management of cancer developing after transplant includes reduction 17. Engels EA. Inammation in the development of lung cancer: Epide-
of immunosuppression and switching to an mTOR inhibitorbased miological evidence. Expert Rev Anticancer Ther 8: 605615, 2008
18. Vajdic CM, McDonald SP, McCredie MR, van Leeuwen MT, Stewart JH,
regimen for those with skin cancers.
Law M, Chapman JR, Webster AC, Kaldor JM, Grulich AE. Cancer in-
cidence before and after kidney transplantation. JAMA 296: 2823
2831, 2006
REFERENCES 19. Kasiske BL, Vazquez MA, Harmon WE, Brown RS, Danovitch GM,
Gaston RS, Roth D, Scandling JD, Singer GG; American Society of
1. OPTN/SRTR. 2012 annual data report. Introduction. Am J Transplant Transplantation. Recommendations for the outpatient surveillance of
14(Suppl 1): 810, 2014 renal transplant recipients. J Am Soc Nephrol 11(Suppl 15): S1S86,
2. Matas AJ, Smith JM, Skeans MA, Thompson B, Gustafson SK, Stewart 2000
DE, Cherikh WS, Wainright JL, Boyle G, Snyder JJ, Israni AK, Kasiske BL. 20. Penn I. Incidence and treatment of neoplasia after transplantation. J
OPTN/SRTR 2013 annual data report: Kidney. Am J Transplant 15 Heart Lung Transplant 12: S328S336, 1993
(Suppl 2): 134, 2015 21. Euvrard S, Morelon E, Rostaing L, Gofn E, Brocard A, Tromme I,
3. Kim WR, Lake JR, Smith JM, Skeans MA, Schladt DP, Edwards EB, Broeders N, del Marmol V, Chatelet V, Dompmartin A, Kessler M, Serra
Harper AM, Wainright JL, Snyder JJ, Israni AK, Kasiske BL. OPTN/SRTR AL, Hofbauer GF, Pouteil-Noble C, Campistol JM, Kanitakis J, Roux AS,
2013 Annual Data Report: Liver. Am J Transplant 15(Suppl 2): 128, 2015 Decullier E, Dantal J; TUMORAPA Study Group. Sirolimus and sec-
4. Watt KD, Pedersen RA, Kremers WK, Heimbach JK, Charlton MR. Evolution ondary skin-cancer prevention in kidney transplantation. N Engl J Med
of causes and risk factors for mortality post-liver transplant: Results of the 367: 329339, 2012
NIDDK long-term follow-up study. Am J Transplant 10: 14201427, 2010 22. Stallone G, Schena A, Infante B, Di Paolo S, Loverre A, Maggio G,
5. Engels EA, Pfeiffer RM, Fraumeni JF Jr, Kasiske BL, Israni AK, Snyder JJ, Ranieri E, Gesualdo L, Schena FP, Grandaliano G. Sirolimus for Kaposis
Wolfe RA, Goodrich NP, Bayakly AR, Clarke CA, Copeland G, Finch JL, sarcoma in renal-transplant recipients. N Engl J Med 352: 13171323,
Fleissner ML, Goodman MT, Kahn A, Koch L, Lynch CF, Madeleine MM, 2005
Pawlish K, Rao C, Williams MA, Castenson D, Curry M, Parsons R, Fant 23. Salgo R, Gossmann J, Schfer H, Kachel HG, Kuck J, Geiger H,
G, Lin M. Spectrum of cancer risk among US solid organ transplant Kaufmann R, Scheuermann EH. Switch to a sirolimus-based immuno-
recipients. JAMA 306: 18911901, 2011 suppression in long-term renal transplant recipients: Reduced rate of
6. Maisonneuve P, Agodoa L, Gellert R, Stewart JH, Buccianti G, (pre-)malignancies and nonmelanoma skin cancer in a prospective,
Lowenfels AB, Wolfe RA, Jones E, Disney AP, Briggs D, McCredie M, randomized, assessor-blinded, controlled clinical trial. Am J Transplant
Boyle P. Cancer in patients on dialysis for end-stage renal disease: An 10: 13851393, 2010
international collaborative study. Lancet 354: 9399, 1999 24. Wong G, Chapman JR, Craig JC. Cancer screening in renal transplant
7. van Leeuwen MT, Grulich AE, McDonald SP, McCredie MR, Amin J, recipients: What is the evidence? Clin J Am Soc Nephrol 3(Suppl 2):
Stewart JH, Webster AC, Chapman JR, Vajdic CM. Immunosuppression S87S100, 2008

4 Onco-Nephrology Curriculum American Society of Nephrology


REVIEW QUESTIONS 3. A 12-year-old boy underwent kidney transplant 6 months
ago. He received thymoglobulin for induction and was main-
1. A 50-year-old white woman, status postkidney transplant tained on a triple immunosuppressive regimen including
10 years ago, was recently diagnosed with squamous cell mycophenolic acid derivatives. He was EBV negative at the
skin cancer on her nose. What should you advise her? time of transplant. He now presents with low-grade fever
a. Continue regular follow-up with dermatology and pain over the allograft. The biopsy reveals dense lym-
b. Discontinue calcineurin inhibitor and switch to mTOR phocytic inltrate with minimal tubulitis. The lymphocytes
inhibitorbased regimen stained positive for CD3 and CD20. SV-40 stain is negative.
c. Apply sun screen What is the most likely diagnosis in this patient?
d. Avoid peak hours of sun exposure a. Rejection
e. All of the above b. Acute interstitial nephritis
c. Post-transplant lymphoproliferative disorder (PTLD)
Answer: e is correct. The patient should follow-up with d. BK virus nephropathy
dermatology for a complete skin examination as the risk of a
second skin cancer is high. Exposure to UV light is one of Answer: c is correct. The boy has developed PTLD as sug-
the main risk factors for development of cancer, and there- gested by a mixed lymphocytic population in the biopsy.
fore, avoiding sunlight and applying sunscreen may prevent CD31 indicates the presence of T cells, and CD201 indicates
development of new skin cancers. Last, studies have shown the presence of B cells. Rejection will have only CD31 or
that switching to an mTOR inhibitorbased regimen re- T-lymphocytes. Lack of SV-40 staining rules out BK virus ne-
duces the risk of additional skin cancers. phropathy. The presence of abundant polymorphic lympho-
cytes with minimal tubulitis should be a clue for PTLD. He had
2. Which of these factors lead to increased risk of cancer in the several risk factors for development of PTLD including young
transplant recipient? age, EBV nave, use of a T celldepleting agent for induction,
a. Viral infections and use of mycophenolic acid derivatives.
b. Immunosuppression
c. Chronic Infections 4. How will you manage this patient?
d. Smoking a. Increase immunosuppression
e. All of the above b. Decrease immunosuppression
c. Discontinue bactrim
Answer: e is correct. The majority of the cancers in trans- d. Start cidofovir
plant recipients are due to viral infections such as HPV, HCV,
HBV, EBV, and HHV-8. However, smoking, chronic infec- Answer: a is correct. Treatment includes reduction of im-
tions, and certain immunosuppressants are also reported to munosuppression in this patient. Increasing immunosuppres-
increase risk of cancer. sion to treat possible rejection may be harmful.

American Society of Nephrology Onco-Nephrology Curriculum 5


Chapter 17: Cancer Screening in ESRD
Jean L. Holley, MD
Department of Medicine, University of Illinois, Urbana-Champaign and Carle Physician Group, Urbana, Illinois

INTRODUCTION should receive the HPV vaccine according to the


recommendations in the general population (9).
The American Cancer Society recommends specic Bladder cancer is also more common in ESRD pa-
age-related screening examinations for colorectal, tients, likely in part due to medications associated
breast, and cervical cancer and suggests that indi- with the development or treatment of kidney dis-
viduals discuss their risk factors and screening for ease (oral cyclophosphamide use, analgesic use
prostate and lung cancer with their primary care leading to chronic tubulointerstitial disease). These
physician (Table 1) (1). Such recommendations are cancers, as well as liver cancer, are more common
incorporated into guidelines for periodic adult among Asian ESRD patients (6). Because of the de-
health care for the general population. Cancer velopment of acquired cystic disease in ESRD, renal
screening for any individual is predicated on the cell carcinoma is also more common among dialy-
risk of developing cancer and the likelihood that sis patients, albeit with a relatively low incidence in
the screening test will detect the cancer. An individ- most studies (5,7) (Table 2). Routine screening for
uals expected survival is also an integral factor in renal cell carcinoma in chronic dialysis patients re-
cancer screening. If expected survival is low, then mains somewhat controversial but most advocate
the cost-effectiveness of routine cancer screening in for individual patient-directed screening based on
average-risk individuals argues against screening cost-effectiveness (10,11). The relatively low in-
because the patient will probably die before cancer cidence of renal cell carcinoma in the setting of
develops and is detected. In the ESRD population, acquired cystic disease and the low expected patient
therefore, when considering routine cancer screen- survival with ESRD argues against routine screen-
ing, it is important to ask the following: 1) is there ing. However, for patients on transplant waiting
an increased risk of cancer in this patient group?; 2) lists, screening may be advisable and required.
are screening tests accurate in this population?; and
3) will the patient live long enough for cancer
screening to detect a life-threatening disease that EFFICACY OF CANCER SCREENING TESTS
can be cured? These issues will be discussed to dem- IN ESRD
onstrate that, because of the high mortality with
ESRD, routine cancer screening is not indicated Cancer screening is primarily based on imaging
for most patients. techniques or laboratory and histopathologic ex-
aminations (Table 1). For most of these evaluations,
the positive and negative predictive value of the test
CANCER RISK IN ESRD has not been assessed in ESRD patients. Due to the
presence of vascular calcications, mammography
Table 2 shows a summary of the published literature interpretation in women with ESRD may be more
cancer incidence among ESRD patients. The stan- difcult (12,13). The higher rates of gastrointesti-
dardized incidence ratio (SIR) is typically used to nal bleeding in ESRD may result in higher fecal
assess cancer frequency. Viral-mediated cancers occult blood tests than the general population
like human papilloma virus (HPV)-associated cer- (14). This may actually lead to higher rates of
vical, uterine, and tongue cancer and hepatitis C
and Bassociated liver cancer are more common in
ESRD patients (28). Although there are no clinical Correspondence: Jean L. Holley, Nephrology, S2S2, 611 West
Park Street, Urbana, Illinois 61802.
data, the SIR for cervical cancer in ESRD patients
suggests that young women (and men) with ESRD Copyright 2016 by the American Society of Nephrology

American Society of Nephrology Onco-Nephrology Curriculum 1


Table 1. American Cancer Society Recommendations for Tumor markers are sometimes used as cancer screening
Routine Cancer Screening tools and may be affected by ESRD. Total prostate specic
Cancer Recommended screening antigen (PSA) is probably valid in ESRD patients (1719), but
Breast Yearly mammogram beginning at age 40, continuing as free PSA and free/total PSA ratios are less useful, as free PSA
long as in good health rises with hemoconcentration and high-ux dialysis mem-
Clinical breast examination every 3 years from age 20 to branes affect its clearance (18,19). For unclear reasons, pros-
39 and then yearly for age .40 tate cancer is the only tumor diagnosed at a later stage in ESRD
MRI for high-risk women patients compared with the general population (15). Prostate
Colorectal Beginning age 50: Flexible sigmoidoscopy every 5 cancer has generally not been more common in ESRD patients
years or colonoscopy every 10 years or double (27). However, a recent study found an SIR of 1.06 for pros-
contrast barium enema every 5 years or CT
tate cancer (8), raising the issue of an increasing incidence of
colonography (virtual colonoscopy) every 5 years,
this cancer among ESRD patients. Controversy continues
with yearly fecal occult blood test or fecal
immunochemical test or stool DNA test done every 3
about screening for prostate cancer in the general population
years (Table 1). Most tumor markers are unreliable in ESRD pa-
Cervical Begin screening at age 21: 2129 years: Pap every 3 tients; they are generally glycoproteins with high molecular
years; no HPV unless Pap is abnormal; 3065 years: weight that are rarely removed by dialysis and rise with hemo-
Pap 1 HPV every 5 years or Pap alone every 3 years; concentration, yielding false-positive results in ESRD. For
.65 years: no screening example, cancer antigen 125 (CA-125), a tumor marker for
Prostate Age 50, discuss pros and cons with MD; age 45 if African ovarian cancer, is produced by mesothelial cells, and patients
American or father or brother with prostate cancer with any serosal uid (pleural effusion, ascites) will have
before the age of 65 elevated levels. This is especially applicable to patients on
Lung No recommendation
peritoneal dialysis, making CA-125 less useful in all ESRD pa-
High risk: consider screening age 5574 in fairly good
tients, particularly those on peritoneal dialysis. b-human cho-
health with at least 30pack-years smoking history
and either still smoking or quit within the last 15 years
rionic gonadotropin and a-fetoprotein, as well as total PSA, are
Individuals with risk factors for specic cancers may need alternate screening
probably reliable in ESRD patients.
protocols and should discuss with their physicians. Adapted from the The recently published clinical trial on the cost-effectiveness
American Cancer Society website. of computed tomography (CT) screening for lung cancer in
high-risk individuals reported a 20% reduction in mortality
over a 4-year period in patients undergoing three annual CT
colonoscopies in ESRD patients, perhaps resulting in earlier
exams at a cost of $81,000 per quality-adjusted life-year and
detection of colorectal cancer in ESRD patients compared with
incremental cost-effectiveness ratios of $52,000 per life-year
the general population (15). There is no information on the gained (20). This study led the US Preventive Services Task
reliability of fecal immunochemical or stool DNA testing in Force to assign a B rating to the recommendation that annual
ESRD patients. An ongoing study on the performance of fecal
low-dose CT scanning be performed as a screen for lung can-
occult blood testing in CKD may help to clarify these issues (16).
cer in adults 5580 years of age with a 30pack-year smoking
history. However, remaining questions about the overall ef-
Table 2. Cancer incidence in ESRD: Literature summary cacy of this screening method prompted the American Cancer
Cancer SIR Risk factors in ESRD References Society (Table 1) to avoid endorsing CT scanning as a cancer
Renal cell 3.624.1 Acquired cystic disease 28 screen. The Centers for Medicare and Medicaid, despite initial
Bladder and 1.516.4 Analgesic abuse, 2,3,5,6,8 misgivings (21), have now endorsed lung cancer screening
ureter Balkan nephropathy, with CT scans. Lung cancer has not traditionally been more
oral cyclophosphamide common in ESRD patients (26), and ESRD patients reduced
Tongue 1.21.9 Human papilloma virus 2,3,6 survival argues against the adoption of lung cancer screening
Cervical and 0.9 Human papilloma virus 6 in this population. Recently, a 1.28 SIR for lung cancer in
uterine 2.74.31 2,3 ESRD patients (8), along with the benets of CT screening
Liver 1.44.5 Hepatitis B and C 2,3,5,6 in at-risk individuals (20), suggests additional study may be
Thyroid and 2.22.3 2,3,6
needed.
other
Endocrine
organs
Breast (women) 0.81.42 3,6,8 IS CANCER SCREENING APPROPRIATE IN ESRD
Lung/bronchus 0.51.28 3,6,8 GIVEN EXPECTED SURVIVAL?
Colon/rectum 1.01.27 3,6,8
Pancreas 1.08 8 Although survival in ESRD may be improving slightly, it
Prostate 0.51.08 3,6,8 remains poor (22). As noted above, a patients expected sur-
Adapted from reference 25 with additional data and references. vival is an important factor to consider when weighing the

2 Onco-Nephrology Curriculum American Society of Nephrology


benets of cancer screening. Hypothetical modeling done in Table 3. Suggested cancer screening in ESRD patients:
the 1990s suggested cancer screening in a dialysis patient Individualized, considering expected survival, risk factors,
would on average provide a net gain of 5 days of survival and transplant status
(23). This model was biased toward cancer screening, exam- Cancer Recommended screening
ining Papanicolaou smears for detecting cervical cancer, Breast -Yearly mammogram beginning age 40 and on
mammography as a screen for breast cancer, exible sigmoido- transplant list
scopy for colorectal cancer, and digital examination with PSA Clinical breast examination every 3 years for ages 2039
testing for prostate cancer assuming screening tests were per- and yearly for age .40
fectly sensitive and specic and that each detected cancer was Colorectal Beginning age 50: Yearly FIT or FOBT for those on
instantaneously treated and cured (23). Using this model, the transplant lists and exible sigmoidoscopy,
colonoscopy, double contrast barium enema, or
costs per unit of survival benet provided by cancer screening
virtual colonoscopy per transplant evaluation
were 1.619.3 times higher among ESRD patients (23). An-
protocols
other study focusing on breast cancer screening in dialysis Positive FIT or FOBT will require additional evaluation
patients found an absolute reduction in breast cancer mortal- Cervical Begin screening at age 21: 2165, yearly Pap for those
ity of 0.1% with a net gain in life expectancy of 1.3 days (24). on transplant list; consider HPV DNA and HPV
Even focusing on the dialysis patient with the best predicted vaccine in transplant candidates
survival (a young black woman without diabetes mellitus) and Prostate Age 50, annual PSA and digital rectal examination for
multiple risk factors for breast cancer, only 250 days of life men on transplant list
were estimated to be saved by screening with mammography Age 45 if African American or father or brother had
in another study (25). Such investigations led to the recom- prostate cancer before the age of 65
mendation to perform cancer screening only on dialysis pa- Renal cell Yearly CT or MRI in patients on dialysis .3 years and on
transplant list
tients assumed to benet; cancer screening in ESRD should be
For all the above cancers, consider screening in high-risk patients with long
based on the individual, considering his or her risk factors for expected survival. FIT, fecal immunochemical test; FOBT, fecal occult blood
cancer, as well as expected survival with ESRD (2330). test. Adapted from references 2229.
Transplant candidacy also needs to be considered when
contemplating cancer screening in ESRD patients. The eval- TAKE HOME POINTS
uative process for kidney transplantation includes age- and
sex- appropriate cancer screening such as mammography, c Viral-associated cancers like hepatitis B and Cassociated liver cancer
Papanicolaou smears, and PSA testing. Thus, cancer screen- and human papilloma virusassociated tongue and cervical cancer are
ing for transplant candidates is generally required. However, more common in ESRD patients.
ESRD patients will need to be assessed on an individual basis, c Because of acquired cystic disease, renal cell carcinoma is more common in
considering cancer risk factors, transplant status, and, impor- ESRD patients, and exposure to analgesic abuse and oral cyclophosphamide
tantly, expected survival to proceed with cancer screening in a result in an increased incidence of bladder cancer in ESRD patients.
cost-effective manner. Table 3 suggests an outline for cancer c Due to poor expected survival with ESRD, cancer screening is not
screening in ESRD patients based on these factors. appropriate for most dialysis patients. Patients with long expected
survival, those on transplant waiting lists, and those with increased
cancer risk factors are appropriate candidates for cancer screening.
SUMMARY AND CONCLUSIONS

Although bladder cancer and viral-mediated cancers like HPV- REFERENCES


associated cervical cancer and hepatitis C and Bassociated liver
1. American Cancer Society: American Cancer Society Guidelines for the
cancer are more common in ESRD patients, general routine
Early Detection of Cancer, 2015. Available at: http://www.cancer.org/
cancer screening in ESRD patients is not recommended. ESRD healthy/ndcancerearly/cancerscreeningguidelines/american-cancer-
patients in whom cancer screening should be considered are society-guidelines-for-the-early-detection-of-cancer. Accessed Janu-
those with good expected survival, candidates for kidney trans- ary 21, 2015
plantation, and certain individuals with a high cancer risk and 2. Buccianti G, Maisonneuve P, Ravasi B, Cresseri D, Locatelli F, Boyle P.
Cancer among patients on renal replacement therapy: a population-
good expected survival. Although acquired cystic kidney disease
based survey in Lombardy, Italy. Int J Cancer 66: 591593, 1996
is associated with an increased risk of renal cell carcinoma, the 3. Maisonneuve P, Agodoa L, Gellert R, Stewart JH, Buccianti G,
same general rules apply; routine screening is not recommended Lowenfels AB, Wolfe RA, Jones E, Disney APS, Briggs D, McCredie M,
for most patients. The tendency may be to implement routine Boyle P. Cancer in patients on dialysis for end-stage renal disease: An
cancer screening protocols in dialysis units, but individualized international collaborative study. Lancet 354: 9399, 1999
4. Heidland A, Bahner U, Vamvakas S. Incidence and spectrum of dialysis-
patient assessment is required for appropriate cancer screening.
associated cancer in three continents. Am J Kidney Dis 35: 347351,
The emerging model of personalized cancer screening for the discussion 352353, 2000
general population is being discussed (31) and seems clearly 5. Chen K-S, Lai M-K, Huang C-C, Chu S-H, Leu M-L. Urologic cancers in
appropriate for those on dialysis. uremic patients. Am J Kidney Dis 25: 694700, 1995

American Society of Nephrology Onco-Nephrology Curriculum 3


6. Lin H-F, Li Y-H, Wang C-H, Chou C-L, Kuo D-J, Fang TC. Increased risk compromised in patients receiving long-term dialysis? Urology 53:
of cancer in chronic dialysis patients: A population-based cohort study 11691174, 1999
in Taiwan. Nephrol Dial Transplant 27: 15851590, 2012 19. Bruun L, Bjrk T, Lilja H, Becker C, Gustafsson O, Christensson A.
7. Farivar-Mohseni H, Perlmutter AE, Wilson S, Shingleton WB, Bigler SA, Percent-free prostate specic antigen is elevated in men on haemo-
Fowler JE Jr. Renal cell carcinoma and end stage renal disease. J Urol dialysis or peritoneal dialysis treatment. Nephrol Dial Transplant 18:
175: 20182020, discussion 2021, 2006 598603, 2003
8. Butler AM, Olshan AF, Kshirsagar AV, Edwards JK, Nielsen ME, Wheeler 20. Black WC, Gareen IF, Soneji SS, Sicks JD, Keeler EB, Aberle DR, Naeim
SB, Brookhart MA. Cancer incidence among US Medicare ESRD patients A, Church TR, Silvestri GA, Gorelick J, Gatsonis C; National Lung
receiving hemodialysis, 1996-2009. Am J Kidney Dis 65: 763772, 2015 Screening Trial Research Team. Cost-effectiveness of CT screening in
9. Centers for Disease Control and Prevention: Immunization Schedules, the National Lung Screening Trial. N Engl J Med 371: 17931802, 2014
2015. Available at: http://www.cdc.gov/vaccines/schedules/. Accessed 21. Bindman A. JAMA Forum: Lung cancer screening and evidence-based
January 21, 2015 policy. JAMA 313: 1718, 2015
10. Sarasin FP, Wong JB, Levey AS, Meyer KB. Screening for acquired 22. US Renal Data System. USRDS 2014 Annual Data Report: Atlas of End-
cystic kidney disease: A decision analytic perspective. Kidney Int 48: Stage Renal Disease in the United States, Bethesda, MD, National In-
207219, 1995 stitutes of Health, National Institute of Diabetes and Digestive and
11. Brown EA. Renal tumours in dialysis patients: Who should we screen? Kidney Diseases, 2014
Nephron Clin Pract 97: c3c4, 2004 23. Chertow GM, Paltiel AD, Owen WF Jr, Lazarus JM. Cost-effectiveness
12. Evans AJ, Cohen ME, Cohen GF. Patterns of breast calcication in of cancer screening in end-stage renal disease. Arch Intern Med 156:
patients on renal dialysis. Clin Radiol 45: 343344, 1992 13451350, 1996
13. Castellanos M, Varma S, Ahern K, Grosso SJ, Buchbinder S, DAngelo 24. Wong G, Howard K, Chapman JR, Craig JC. Cost-effectiveness of breast
D, Raia C, Kleiner M, Elsayegh S. Increased breast calcications in cancer screening in women on dialysis. Am J Kidney Dis 52: 916929, 2008
women with ESRD on dialysis: Implications for breast cancer screening. 25. LeBrun CJ, Diehl LF, Abbott KC, Welch PG, Yuan CM. Life expectancy
Am J Kidney Dis 48: 301306, 2006 benets of cancer screening in the end-stage renal disease population.
14. Akmal M, Sawelson S, Karubian F, Gadallah M: The prevalence and Am J Kidney Dis 35: 237243, 2000
signicance of occult blood loss in patients with predialysis advanced 26. Holley JL. Screening, diagnosis, and treatment of cancer in long-term
chronic renal failure (CRF), or receiving dialytic therapy. Clin Nephrol dialysis patients. Clin J Am Soc Nephrol 2: 604610, 2007
42: 198202, 1994 27. Kajbaf S, Nichol G, Zimmerman D. Cancer screening and life expec-
15. Taneja S, Mandayam S, Kayani ZZ, Kuo Y-F, Shahinian VB. Comparison tancy of Canadian patients with kidney failure. Nephrol Dial Transplant
of stage at diagnosis of cancer in patients who are on dialysis versus 17: 17861789, 2002
the general population. Clin J Am Soc Nephrol 2: 10081013, 2007 28. Holley JL. Preventive medical screening is not appropriate for many
16. Wong G, Howard K, Chapman JR, Tong A, Bourke MJ, Hayen A, chronic dialysis patients. Semin Dial 13: 369371, 2000
Macaskill P, Hope RL, Williams N, Kieu A, Allen R, Chadban S, 29. Williams AW, Dwyer AC, Eddy AA, Fink JC, Jaber BL, Linas SL, Michael
Pollock C, Webster A, Roger SD, Craig JC. Test performance of B, OHare AM, Schaefer HM, Shaffer RN, Trachtman H, Weiner DE, Falk
faecal occult blood testing for the detection of bowel cancer in people AR; American Society of Nephrology Quality, and Patient Safety Task
with chronic kidney disease (DETECT) protocol. BMC Public Health 11: Force. Critical and honest conversations: the evidence behind the
516522, 2011 Choosing Wisely campaign recommendations by the American
17. Morton JJ, Howe SF, Lowell JA, Stratta RJ, Taylor RJ. Inuence of end- Society of Nephrology. Clin J Am Soc Nephrol 7: 16641672, 2012
stage renal disease and renal transplantation on serum prostate- 30. Walter LC, Lindquist K, OHare AM, Johansen KL. Targeting screening
specic antigen. Br J Urol 75: 498501, 1995 mammography according to life expectancy among women un-
18. Djavan B, Shariat S, Ghawidel K, Gven-Marberger K, Remzi M, Kovarik dergoing dialysis. Arch Intern Med 166: 12031208, 2006
J, Hoerl WH, Marberger M. Impact of chronic dialysis on serum PSA, 31. Saini SD, van Hees F, Vijan S. Smarter screening for cancer: Possibilities
free PSA, and free/total PSA ratio: Is prostate cancer detection and challenges of personalization. JAMA 312: 22112212, 2014

4 Onco-Nephrology Curriculum American Society of Nephrology


REVIEW QUESTIONS cancer screening. Screening is predicated on the patient living
long enough to develop a specic cancer and the sensitivity
1. Is the cancer incidence in ESRD patients higher than in the and specicity of the screening test to detect that cancer at a
general population? stage when cure is possible.

Answer: Yes, for certain cancers. Virus-associated cancers 3. Should routine screening protocols be in place in dialysis
(liver cancer, cervical cancer, and tongue cancer) and renal cell units?
and bladder cancer (because or risk factors) are more common
in ESRD patients. Breast, colorectal, and lung cancer are not Answer: No, routine cancer screening is not cost-effective
more common in ESRD patients. for most dialysis patients because their expected survival is
short. An individualized approach to cancer screening is
2. What factors affect the efcacy or cost-effectiveness of most appropriate for ESRD patients, considering the patients
cancer screening in general? specic risk factors for cancer development, transplant status,
and expected survival.
Answer: The cancer risk, the effectiveness of the screening
test, and the patients expected survival all affect the efcacy of

American Society of Nephrology Onco-Nephrology Curriculum 5


Chapter 18: Ethics of RRT, Initiation, and Withdrawal in
Cancer Patients
Michael J. Germain, MD
Department of Medicine, Tufts University, Springeld, Massachusetts

Malignancies are common in CKD patients, and the kidney by the malignancy, or surgical removal of the
incidence is higher than in the general age-matched kidney to remove the malignancy); and 4) a renal
population. Because cardiovascular disease and in- transplant patient with a terminal malignancy.
fection are so prevalent in CKD, especially ESRD In the rst scenario, withdrawal of dialysis (911)
patients, the mortality rate from cancer in ESRD is often the ethical question. In the second and third
patients is lower than the age-matched general pop- scenarios, withholding of dialysis or withdrawal may
ulation due to these competing inuences. Thus, the be the ethical issue. An important ethical aspect is the
relative risk of mortality from cancer is increased in ethical imperative of the clinician to rst, do no
the younger ESRD population and then declines with harm. The clinician has the right and duty not to
age (1,2). order a treatment that will do more harm than good.
Patients with cancer and a need for RRT present Nephrologists often nd themselves in the position of
very difcult scenarios for making clinical decisions, being asked to provide dialysis, by a patient, family, or
and an approach grounded in medical ethical prin- other clinicians, when dialysis may not be in the pa-
ciples can be helpful (37). Medical ethics reect the tients best interest. Many clinicians feel they are re-
culture and time that we are living in and also quired to provide dialysis treatment when the patient
include a religious perspective. This chapter will or health care provider (HCP) requests it. The SDMG
focus on a US perspective that reects the generally (recommendations 5 and 6) clearly state that the cli-
accepted values of our society at the present time. The nician has no such obligation. The clinician should
United States has a wide representation of cultural document these discussions and make it clear that the
and religious values, with many patients who are patient/HCP has the right to transfer care to another
new immigrants from many countries. A discussion clinician. Clinicians should not fear medicallegal
of the different medical ethical approaches from these concerns in this scenario; in reality, these rarely, if
societies is beyond the scope of this discussion, but ever, occur, especially if the SDMG is followed.
the clinician should always inquire from the patient Instead, shared decision-making is the preferred
and family how they want prognosis, goals of care, process where the clinician/care team (SDMG rec-
and end-of-life issues discussed with them. This dis- ommendation 1) and the patient/family/HCP
cussion will rely heavily on the national clinical make a care plan for the patient. The rst step in
practice guideline Shared Decision-Making in the Ap- this process is for the care team to ask, listen, and
propriate Initiation of and Withdrawal from Dialysis, understand the patients understanding of his or her
2nd Ed. (SDMG), in particular the section Ethical condition and values and goals in life. With the pa-
Considerations in Dialysis Decision-Making (8). Six tients explicit permission, the care team then ex-
ethical principles should be strongly considered for plains from their expert perspective the patients
patients with cancer when discussing RRT (Table 1). condition, prognosis, and the risks and benets of
Conicts between respect for patient autonomy the treatment options. Recent qualitative studies
and benecence/nonmalecence often can occur have shown that CKD patients want to know their
with these patients. There are four scenarios where prognosis. However, our experience is such that pa-
the ethical issues of cancer and RRT intersect: 1) tients often do not want a numerical estimate, such
patients with ESRD who develop a terminal malig-
nancy; 2) patients with a terminal malignancy who Correspondence: Michael J. Germain, Department of Medicine,
develop ESRD; 3) patients with a terminal malignancy Tufts University, 100 Wason Avenue, Suite 200, Springeld,
Massachusetts 01107.
who develop AKI (AKI can be caused by the treatment
of the malignancy, obstruction or invasion of the Copyright 2016 by the American Society of Nephrology

American Society of Nephrology Onco-Nephrology Curriculum 1


Table 1. Six medical ethics principles (http://en.wikipedia.
org/wiki/Medical_ethics#Values_in_medical_ethics) Box 1. Suggested steps for implementing
1) Respect for autonomy: The patient has the right to refuse or choose
recommendation 8 (reproduced with permission)
their treatment (voluntas aegroti suprema lex). Extended conversation
2) Benecence: A practitioner should act in the best interest of the 2 Why does the patient or legal agent desire dialysis when it is not
patient (salus aegroti suprema lex). recommended by the renal care team?
3) Nonmalecence: First, do no harm (primum non nocere). 2 Why does the patient or legal agent refuse dialysis when it is
4) Justice: Concerns the distribution of scarce health resources and the recommended by the renal care team?
2 Does the patient or legal agent misunderstand the diagnosis,
decision of who gets what treatment (fairness and equality).
prognosis, and treatment alternatives?
5) Respect for persons: The patient (and the person treating the patient)
2 Does the nephrologist misunderstand the patients or legal
has the right to be treated with dignity.
agents reasons for requesting dialysis?
6) Truthfulness and honesty: The concept of informed consent
2 Does the nephrologist understand the psychosocial, cultural,
or spiritual concerns and values the patient or legal agent have?
2 Has the nephrologist consulted a psychologist, social worker, or
as how many months of life they may have remaining. Rather, chaplain for assistance in fully understanding the concerns of the
they prefer a general statement about overall prognosis (SDMG patient or legal agent family?
recommendations 2 and 3). Consultation with other physicians

Through the process of consensus building, a shared 2 Do other physicians agree or disagree with the attending
decision and treatment plan is agreed on (SDMG recommen- physicians recommendation to withhold or withdraw
dation 4). In a consensus, each party may not get the plan they dialysis?
2 Is the request for dialysis by the patient or legal agent
originally favored, but they may be convinced by hearing the
medically appropriate?
perspectives put forth by others that a different plan is pre- Consultation with ethics committee or ethics consultants.
ferred. Sometimes the party may not like the consensus plan 2 Has the patient or legal agent been informed that the purpose of
but agrees to accept it. I have seen this in situations where the the ethics consult is to clarify issues of disagreement, and ideally,
HCP may want to initiate or continue dialysis in a patient who to enable resolution?
clearly is getting no benet (or even suffering harm), but the 2 Has the patient or legal agent met with the ethics committee or
nephrologist has decided that he or she cannot ethically order ethics consultants to explain their perspective and reasoning
behind their request for dialysis?
the dialysis treatment. After offering to transfer care to another
2 Can the ethics committee identify the reasons why the patient or
nephrologist, invariably the HCP will accept the clinicians legal agent is resistant to the physicians recommendation to
decision. For this to work, the clinician must show respect forgo dialysis?
for the alternative point of view, listen carefully, and validate 2 Can the ethics committee identify the reasons why the health
the persons views. If the views are based on religious objec- care provider is resistant to the patients or legal agents desire to
tions, then it is sensible to involve a clergy person who may begin or continue dialysis?
help explain the religions positions. A time-limited trial of 2 Has the ethics committee explained in understandable terms to
the patient or legal agent its conclusions and the reasoning
RRT (7) can be undertaken in certain dened situations
behind them?
such as when the benet to be achieved by dialysis is uncertain 2 Can the impasse be resolved with accommodation, negotiation,
or a consensus about the benet of dialysis cannot be reached or mediation?
(SDMG recommendation 7). Documentation

When consensus cannot be reached, the SDMG suggests 2 The physician must document the medical facts and his/her
conict resolution (SDMG recommendation 8; resolving reasons for the recommendation to forgo dialysis and the
conicts about what dialysis decision to make; Box 1 and decision not to agree to the request by the patient or legal agent.
2 The consultants should also document their assessment of the
Figure 1). The SDMG suggests a practical ethical approach
patients diagnosis, prognosis, and their recommendations in
to decision-making. The patients case is analyzed from these the chart.
perspectives (Table 2). An attempt to transfer the patients care
Each perspective is viewed through the six ethical principles in 2 If reconciliation is not achieved through the above procedure
Table 1. The SDMG then recommends the following process for and the physician in good conscience cannot agree to the
ethical decision-making (Table 3). Although the SDMG recom- patient or legal agents request, the physician is ethically and
mends that patients with a terminal prognosis (,6 months) legally obligated to attempt to transfer the care of the patient to
another physician.
should not receive dialysis, the guidelines recognize that palli-
2 Another physician and/or institution may not be found who is
ative dialysis (12) is an option for those who require more time willing to accept the patient under the terms of the familys
to nish their life goals. Such goals include activities for signif- request. Physicians and institutions that refuse to accept the
icant events like a wedding, birth, or graduation. Palliative di- patient in transfer and their reasons should also be documented
alysis allows the patient to transition to a more comfort-oriented in the medical record.
care. The patient may shorten their dialysis treatment time, re- 2 Consider consultation with a mediator, extramural ethics
strict further hospitalizations or procedures, and, when appro- committee, or the ESRD Network in the region.
priate, receive hospice services (SDMG recommendation 9).

2 Onco-Nephrology Curriculum American Society of Nephrology


Table 2. Perspectives to consider in ethical decision-
making*
1) Medical indications, the diagnosis, prognosis, and treatment
2) Patient preferences
3) Quality of life
4) Contextual features (social, economic, legal, and administrative)
*Adapted from reference 8 with permission from the Renal Physicians As-
sociation

Table 3. The seven-step process of ethical decision-making


in patient care*
1) What are the ethical questions
2) What are the clinically relevant facts
3) What are the values at stake
4) List options (what could you do)
5) What should you do (choice the best option from the ethical point of
view balancing all the above factors
6) Justify your choice based on the ethical principles
7) How could this ethical issue have been prevented
*Adapted from reference 8 with permission from the Renal Physicians As-
sociation.

Table 4. Systems approach to American College of


Physicians in nephrology practice and the dialysis unit
1) Normalize the conversation: start discussions of EOL issues early in
the patients interaction with the nephrology team.
2) Involve all members of the care team. In the ofce, this depends on
human resources available. In the dialysis unit, train and utilize the
dietician, technicians, social worker, and nurses.
3) Have a champion. Without this, likely there will be little buy-in or
progress. Although the nephrologist does not have to be the
champion, the nephrologist leader (i.e., medical director in the
dialysis unit) needs to show strong support.
4) Teach all staff members simple communication techniques.
5) Integrate ACP into the workow.
6) Do continuous quality improvement on the process.
7) There are resources available to learn from established successful
programs .

result in a shortened survival (20). Patients are not harmed, and


they appreciate honest communication of bad news (21). It is
important to recognize that our patients want to know their
Figure 1. Systematic approach to resolving conict between
prognosis, and there are validated tools available for the clinician
patient and renal care team.
to utilize when having this discussion (22,23).
In the end, the goal of the communication between the
Finally, an effective process depends on excellent clinician patient (and family or other preferred surrogate decision-
patient/family communication (1315) (SDMG recommen- maker) and the kidney care team is shared decision-making.
dation 10). To have these discussions, appropriate systems Shared decision-making is the recognized preferred model
must be in place in the nephrology practice and dialysis units for medical decision-making because it addresses the ethical
to facilitate the process (Table 4,) (1618). need to fully inform patients about the risks and benets of
There are excellent resources to help the health care team to treatments, as well as the need to ensure that patients values
accomplish these goals and tasks. Offering meticulous end-of-life and preferences play a prominent role (8). Shared decision-
care, including hospice, is mandatory for all of our patients with a making has been referred to as the pinnacle of patient-centered
,6-month prognosis (1719). It is important for patients and care (24). Patient-centered care has been one of the six spe-
families to understand that palliative care and hospice do not cic aims for improvement for health care since the Institute

American Society of Nephrology Onco-Nephrology Curriculum 3


Table 5. List of resources REFERENCES
1) RPA SDM Toolkit: https://itunes.apple.com/us/app/rpa-sdm-toolkit/
id843971920?mt58 1. Available at: http://www.usrds.org/2014/view/Default.aspx. Accessed
2) ACP DECISIONS: http://www.acpdecisions.org March 24, 2015
2. Oneschuk D, Fainsinger R. Medical and ethical dilemmas when an
3) Coalition for Kidney Supportive Care: http://www.
advanced cancer patient discontinues dialysis. J Palliat Care 18: 123
kidneysupportivecare.org/Home.aspx
126, 2002
4) Supportive Care for the Renal Patient, edited by Chambers, Brown, 3. Del Vecchio L, Locatelli F. Ethical issues in the elderly with renal disease.
Germain, 2nd Ed., London, UK, Oxford Press, 2010 Clin Geriatr Med 25: 543553, 2009
5) RPA SDM Guidelines, 2nd Ed., 2010: http://www.renalmd.org/ 4. Davison SN. The ethics of end-of-life care for patients with ESRD. Clin J
catalogue-item.aspx?id5682 Am Soc Nephrol 7: 20492057, 2012
6) Five Wishes: https://www.agingwithdignity.org 5. Skold A, Lesandrini J, Gorbatkin S. Ethics and health policy of
7) Albertas Conversations Matter: http://www.albertahealthservices. dialyzing a patient in a persistent vegetative state. Clin J Am Soc
ca/9098.asp Nephrol 9: 366370, 2014
8) ASN Geriatric Curriculum: http://asn-online.org/education/ 6. Moss AH. Ethical principles and processes guiding dialysis decision-
distancelearning/curricula/geriatrics/ making. Clin J Am Soc Nephrol 6: 23132317, 2011
7. Rinehart A. Beyond the futility argument: the fair process approach and
9) KDIGO Renal Supportive Care Initiative: http://kdigo.org/home/
time-limited trials for managing dialysis conict. Clin J Am Soc Nephrol
conferences/supportivecare/
8: 20002006, 2013
10) Vital Talk communication techniques: http://vitaltalk.org/sites/ 8. Renal Physicians Association. Shared Decision Making in the Appro-
default/les/quick-guides/NURSEforVitaltalkV1.0.pdf priate Initiation of and Withdrawal from Dialysis, 2nd Ed., Rockville,
11) https://www.prepareforyourcare.org/ MD, Renal Physicians Association, 2010
12) http://www.ihi.org/Engage/Initiatives/ConversationProject/ 9. Akbar S, Moss AH. The ethics of offering dialysis for AKI to the older pa-
Pages/default.aspx tient: Time to re-evaluate? Clin J Am Soc Nephrol 9: 16521656, 2014
13) http://www.gundersenhealth.org/respecting-choices 10. Brown EA. Non-dialysis therapy: A better policy than dialysis followed
14) Six-month mortality predictor: Under nephrology/HD, http://www. by withdrawal? Semin Dial 25: 2627, 2012
qxmd.com/apps/calculate-by-qxmd 11. Murtagh F, Cohen LM, Germain MJ. Dialysis discontinuation: Quo va-
15) Breaking Bad News SPIKE: https://depts.washington.edu/bioethx/ dis? Adv Chronic Kidney Dis 14: 379401, 2007
12. Grubbs V, Moss AH, Cohen LM, Fischer MJ, Germain MJ, Jassal SV, Perl
topics/badnws.html
J, Weiner DE, Mehrotra R; Dialysis Advisory Group of the American
Society of Nephrology. A palliative approach to dialysis care: A patient-
centered transition to the end of life. Clin J Am Soc Nephrol 9: 2203
2209, 2014
of Medicine (IOM) issued its 2001 report, Crossing the Quality 13. Schell JO, Cohen RA. A communication framework for dialysis
Chasm: A New Health System for the 21st Century (25). The decision-making for frail elderly patients. Clin J Am Soc Nephrol 9:
20142021, 2014
IOM noted that the US health care delivery system does not 14. Schell JO, Green JA, Tulsky JA, Arnold RM. Communication skills
provide consistent, high-quality medical care to all people. The training for dialysis decision-making and end-of-life care in nephrology.
IOM dened patient-centered as providing care that is re- Clin J Am Soc Nephrol 8: 675680, 2013
spectful of and responsive to individual patient preferences, 15. Tamura MK, Tan JC, OHare AM. Optimizing renal replacement ther-
needs, and values, and ensuring that patient values guide all apy in older adults: A framework for making individualized decisions.
Kidney Int 82: 261269, 2012
clinical decisions. Since the publication of the IOM report,
16. Da Silva-Gane M, Cohen LM. Planning a supportive care programme
there has been growing national interest in more individualized and its components. In: Supportive Care for the Renal Patient, edited
patient-centered models of care. These models focus on what by Chambers J, Brown E, Germain MJ, 2nd Ed., Oxford, UK, Oxford
matters most to individual patients and less on what might University Press, 2010, 3948
matter to providers or health systems (26). A recent qualitative 17. Germain MJ, Kurella Tamura M, Davison SN. Palliative care in CKD: The
earlier the better. Am J Kidney Dis 57: 378380, 2011
study suggests that patients want to discuss ACP with the ne-
18. Wong SP, Kreuter W, OHare AM. Treatment intensity at the end of life
phrologist (27). When effectively done, it can increase use of in older adults receiving long-term dialysis. Arch Intern Med 172: 661
hospice and provide a good death (28). A recent review em- 663, discussion 663664, 2012
phasizes the ethical principles involved in these discussion with 19. Farrington K, Chambers JE. Death and end-of-life care in advanced
the elderly CKD patient (29). This chapter has sought to explain kidney disease. In: Supportive Care for the Renal Patient, edited by
how high-quality, ethical care can be delivered to patients with Chambers JE Brown E, Germain MJ, 2nd Ed., Oxford, UK, Oxford
University Press, 2010
advanced kidney disease and cancer. 20. Temel JS, Greer JA, Muzikansky A, Gallagher ER, Admane S, Jackson
VA, Dahlin CM, Blinderman CD, Jacobsen J, Pirl WF, Billings JA, Lynch
TJ. Early palliative care for patients with metastatic non-small-cell lung
TAKE HOME POINTS cancer. N Engl J Med 363: 733742, 2010
21. Wachterman MW, Marcantonio ER, Davis RB, Cohen RA, Waikar SS,
c The ethics of RRT in cancer balances the principles of respect for patient Phillips RS, McCarthy EP. Relationship between the prognostic ex-
autonomy with nonmalecence. pectations of seriously ill patients undergoing hemodialysis and their
c In some cases, palliative dialysis may be an option for these patients. nephrologists. JAMA Intern Med 173: 12061214, 2013
c Good communication skills are the key to shared decision-making and 22. Germain MJ. How to integrate predictions in outcomes in planning
patient-centered care. clinical care. Blood Purif 39: 6569, 2015

4 Onco-Nephrology Curriculum American Society of Nephrology


23. Cohen LM, Ruthazer R, Moss AH, Germain MJ. Predicting six-month 27. Goff SL, Eneanya ND, Feinberg R, Germain MJ, Marr L, Berzoff J,
mortality for patients who are on maintenance hemodialysis. Clin J Am Cohen LM, Unruh M. Advance care planning: A qualitative study of
Soc Nephrol 5: 7279, 2010 dialysis patients and families. Clin J Am Soc Nephrol 10: 390400, 2015
24. Barry MJ, Edgman-Levitan S. Shared decision making: Pinnacle of 28. Cohen LM, Ruthazer R, Germain MJ. Increasing hospice services for
patient-centered care. N Engl J Med 366: 780781, 2012 elderly patients maintained with hemodialysis. J Palliat Med 13: 847
25. Institute of Medicine. Crossing the Quality Chasm: A New Health System 854, 2010
for the 21st Century, Washington, DC, National Academies Press, 2001 29. Thorsteinsdottir B, Swetz KM, Albright RC. The ethics of chronic dialysis
26. Tinetti ME, Fried T. The end of the disease era. Am J Med 116: 179 for the older patient: Time to reevaluate the norms. Clin J Am Soc
185, 2004 Nephrol 10: 20942099, 2015

American Society of Nephrology Onco-Nephrology Curriculum 5


REVIEW QUESTIONS these discussions take place with a family member or HCP
instead of with them; this is a common scenario for some
1. A long-term dialysis patient presents with metastatic sar- cultures in the United States (Native American, some Asian
coma that is not treatable, and the prognosis is poor. What cultures).
are the relevant medical ethical principles to consider in
this patient? 3. The patients health deteriorates rapidly, and she is in
pain whenever she is moved, such as transportation to
a. Autonomy
and from dialysis. She is very lethargic and not commu-
b. Nonmalecence
nicative. The clinician feels that dialysis is doing more
c. Benecence
harm than good for the patient. When this is discussed
d. Respect for person
with the family, they insist that dialysis be continued.
e. Truth and honesty
They believe that in their religion withdrawing dialysis
f. All of the above
is a sin. Attempts at shared decision making and involving
g. None of the above
their pastor have not resulted in a resolution of the
conict. The correct approach is to:
Answer: f is correct. Patient autonomy, nonmalecence
(avoiding the harms of RRT), benecence, respect for person,
a. Continue dialysis
truth, and honesty.
b. Seek a court order to withdraw dialysis
c. Explain to the family that you understand and respect their
2. The family requests that you withhold the cancer di-
point of view; explain that you have an ethical duty to do no
agnosis/prognosis information from this patient. What is
harm by the treatments that you order for your patients,
the ethical principle that would guide your decision?
and at this point, dialysis is doing more harm than good;
a. Nonmalecence: The information would be harmful to the and you will be discontinuing your order for dialysis and
patient the family can seek another clinician to take over care if
b. Truth and honesty. they wish
c. Benecence
Answer: c is correct. After following an shared decision
Answer: b is correct. In our society, it is not ethical to with- making process and conict resolution, if there is still no con-
hold this information. In some cultures, it is left to the doctor sensus, then the clinician has the right and ethical duty to not
to decide if it would be harmful to the patient to give them order RRT if the principle of nonmalecence and justice out-
bad news. If a family asks that you not give bad news to the weighs the principle of autonomy. The patient has the right to
patient, it is acceptable to ask the patient if they prefer that refuse a treatment but not to demand a treatment.

6 Onco-Nephrology Curriculum American Society of Nephrology


Chapter 19: Palliative Care in Patients with Kidney
Disease and Cancer
Alvin H. Moss, MD, FACP, FAAHPM
Department of Medicine, Sections of Nephrology and Supportive Care, West Virginia University School of Medicine,
Morgantown, West Virginia

INTRODUCTION RELEVANCE OF PALLIATIVE CARE

Many, if not most, cancer and kidney disease patients There is an increasing recognition that skills in
have two things in common: they have a shortened life palliative and end-of-life care are required for physi-
expectancy and a high symptom burden. Both cians, nurses, and others who treat patients who have
populations benet from early palliative care inter- CKD and ESRD (Table 1). The principal reasons are
ventions. The goal of palliative care is to relieve as follows: rst, they have a signicantly shortened
suffering and to support the best possible quality of life life expectancy; just over half of dialysis patients
for patients and their families, regardless of their stage (52%) are still alive 3 years after the start of RRT (1).
of disease or the need for other therapies, in accor- Second, patients with CKD and ESRD have multiple
dance with their values and preferences. By Medicare comorbidities and consequently many symptoms
regulation, hospice care is limited to patients esti- such as pain, fatigue, itching, and difculty with
mated to be in their last 6 months of life if their disease sleep. In one study, the symptoms of CKD and
follows the normal course. In patients with kidney ESRD patients were found to be comparable (mean
disease and cancer (hereafter kidney-cancer patients) of 10.7) and severity (2). Similarly, cancer patients
who have a higher symptom burden than patients have been found to have a high symptom burden
with either disease alone, the need for meticulous pain compared with age-matched controls, and pain,
and symptom management is even more important to anxiety and depression, and insomnia were noted
maintain quality of life. In addition, there is a unique as most prevalent in a population-based study of
need for advance care planning for these patients, 1,904 cancer survivors (3). An interaction between
most of whom have two life-limiting illnesses. As in cancer status and comorbidity was found, resulting
other populations, for kidney-cancer patients, pain is in a higher symptom burden for patients with co-
one of the most common and severe symptoms. morbidities such as CKD. Thus, it is reasonable to
Multiple studies in kidney patients show that pain is conclude the CKD or ESRD patients with cancer
undertreated. Treatment of pain in patients with stage will have a higher symptom burden than patients
4 and 5 CKD and ESRD is more challenging because with either cancer or kidney disease alone (3).
of the failure of renal excretion of active metabolites Third, the dialysis population has been growing
from some commonly used opioids, which leads to progressively older. The incidence rates of ESRD are
opioid neurotoxicity. The nephrology community has highest in patients 75 years old and older, and they
developed a clinical practice guideline that endorses continue to rise in this group (1). Older patients sur-
the process of shared decision-making in reaching vive the shortest period of time on dialysis, and they
decisions about who should undergo dialysis. It withdraw from dialysis signicantly more often than
recognizes that the burdens of dialysis may sub- younger patients.
stantially outweigh the benets in some patients and In consideration of the high symptom burden
notes that nephrologists may want to recommend and the low survival rate for dialysis patients, the
forgoing dialysis to kidney-cancer patients who are
terminally ill from their cancer.
Correspondence: Alvin H. Moss, Department of Medicine,
This chapter describes the growing interest in the Sections of Nephrology and Supportive Care, West Virginia
nephrology and oncology communities in incor- University School of Medicine, 1195 Health Sciences North,
Morgantown, West Virginia 26506-9022
porating palliative care into the standard treatment
of patients with CKD, ESRD, and cancer. Copyright 2016 by the American Society of Nephrology

American Society of Nephrology Onco-Nephrology Curriculum 1


Table 1. Palliative care for CKD/ESRD patients: Need for a Table 2. Components of a dialysis facility palliative care
systematic approach program
Pain and symptom assessment/management (11,21) 1. Palliative care focus
Shared decision-making for informed consent a. Educational activities, including dialysis unit in-service trainings
Patient-specic estimates of prognosis using the surprise question b. Quality improvement activities, including morbidity and mortality
Timely discussions prompted by prognosis conferences
Inclusion of family/legal agent in discussions c. Use of the Would you be surprised if this patient died within the next
Completion of advance directives year? question to identify patients appropriate for palliative care
Completion of physician orders for life-sustaining treatment (POLST) d. Collaboration with local hospice programs to coordinate a smooth
paradigm form as appropriate transition to end-of-life care
Immediately actionable medical orders 2. Pain and symptom assessment and management protocols
Transferrable throughout health care setting 3. Systematized advanced care planning
Referral to hospice when indicated 4. Psychosocial and spiritual support to patients and families, including
Adapted from reference 5. the use of peer counselors
5. Terminal care protocols that include hospice referral
6. Bereavement programs for families that include memorial services
American Society of Nephrology (ASN) and the Renal Phy- Adapted from reference 5.
sicians Association (RPA) have recommended that dialysis
facilities incorporate palliative care into their treatment of
patients (4,5). Nephrologists have been encouraged to obtain Morphine is the best studied of the opioids used for pain
education and skills in palliative care, and dialysis facilities management, and its most common metabolites (including
have been urged to developed protocols, policies, and pro- morphine-3-glucuronide, morphine-6-glucuronide, and nor-
grams to ensure that palliative care is provided to their patients morphine) are excreted by the kidneys. The clearance of these
(Table 2) (5). Also, dialysis units have been urged to develop a metabolites is particularly problematic in stage 4 and 5 CKD
working relationship with local hospice programs, so patients and ESRD. Morphine-6-glucuronide is an active metabolite
with ESRD who stop dialysis or patients undergoing dialysis with analgesic properties; it crosses the bloodbrain barrier
with a nonrenal terminal diagnosis may be referred for hos- and may have prolonged central nervous system effects. A
pice. Similarly, the American Society of Clinical Oncology has comprehensive review recommended that morphine not be
issued a provisional clinical opinion that early involvement of used in patients with kidney disease because it is so difcult
palliative care when combined with standard cancer care leads to manage the complicated adverse effects of the morphine
to better patient and caregiver outcomes, including improve- metabolites (12).
ment in symptoms, quality of life, and patient satisfaction and Studies of codeine pharmacokinetics suggest that codeine
reduced caregiver burden (6). metabolites would accumulate to toxic levels in a majority of
patients undergoing hemodialysis. Codeine use is not recom-
mended because serious adverse effects have been reported in
SYMPTOM MANAGEMENT patients with CKD (12).
Hydromorphone is metabolized in the liver largely to
Pain hydromorphone-3-glucuronide. This metabolite accumulates
As in other patient populations, the burden of symptoms for in patients with kidney disease and can cause opioid neuro-
patients undergoing dialysis is inversely associated with their toxicity. Some studies suggest that hydromorphone is removed
reported quality of life (7). Pain is one of the most common with dialysis. It is recommended that hydromorphone be used
symptoms reported by patients undergoing dialysis, and sev- cautiously, if at all, in patients stopping dialysis (12).
eral studies have found that approximately 50% of these pa- Use of oxycodone in patients with kidney disease has not
tients report pain. For most patients undergoing dialysis, the been well studied. The elimination half-life of oxycodone is
pain is musculoskeletal in origin. Smaller numbers of patients
have pain related to the dialysis procedure, peripheral neurop-
athy, peripheral vascular disease, or carpal tunnel syndrome.
Table 3. Pain medications for use in advanced kidney failure
Three studies have found that pain is undertreated in 75% of
patients undergoing dialysis (810). As in cancer patients, use Recommended Use with caution Do not use
of the World Health Organization (WHO) three-step analge- Fentanyl Tramadol Morphine
sic ladder has been found to be effective in the treatment of Methadone Hydrocodone/oxycodone Codeine
pain in dialysis patients (9). Because the metabolites of some Hydromorphone Desipramine/nortriptyline Meperidine
Acetaminophen Propoxyphene
of the opioids on the analgesic ladder are renally excreted and
Gabapentin
active, these opioids, morphine, codeine, meperidine, and
Adapted from reference 11. Seehttp://www.kidneysupportivecare.org/Files/
propoxyphene, are not recommended for use in patients PainBrochure9-09.aspx for recommendations for use, dosage reductions,
with advanced kidney disease (Table 3) (11). and cautions in advanced kidney failure.

2 Onco-Nephrology Curriculum American Society of Nephrology


lengthened in dialysis patients, and excretion of metabolites is neuropathic processes, and low-grade hypersensitivity to
impaired but almost all are inactive. Oxycodone can be used products used in the dialysis procedure have all been identied
with caution in patients with advanced CKD and ESRD (12). as possible contributory factors. In addition to careful man-
The WHO analgesic ladder recommends the use of fentanyl agement of all these factors, the following interventions have
for severe pain. Fentanyl is metabolized in the liver primarily been tried for pruritus with some success: emollient skin
to norfentanyl. There is no evidence that any fentanyl metab- creams, phototherapy with ultraviolet B light three times
olites are active. Several studies have found that fentanyl can be weekly, intravenous lidocaine during dialysis for refractory
used safely in patients with CKD and ESRD. It has negligible itching, gabapentin, naltrexone, and thalidomide (14).
dialyzability. Fentanyl use is deemed to be one of the safest
opioids to use in patients with advanced CKD (12).
The WHO analgesic ladder recommends methadone for ADVANCE CARE PLANNING
severe pain. Studies in anuric patients have found that nearly all
of methadone and its metabolites doses are excreted in the Advance care planning is a process of communication among
feces. No dose adjustments are recommended for patients patients, families, health care providers, and other important
undergoing dialysis. The use of methadone appears safe in individuals about the patients preferred decision-maker and
patients with advanced CKD and ESRD (12). appropriate future medical care if and when a patient is unable
Opioids are also often used to treat dyspnea at the end of life in to make his or her own decisions. Advance care planning has
patients with CKD, ESRD, and cancer. In the setting of wors- been recommended as a central tenet of CKD, ESRD, and cancer
ening renal function or withdrawal of dialysis, the clinician may patient care (15). It is especially appropriate because of the life-
be challenged to distinguish uremic encephalopathy from opioid limiting nature of these diseases. The surprise question
neurotoxicity. Both can cause sedation, hallucinations, and would I be surprised if this patient died in the next year?has
myoclonus. If respiratory depression is also present, it is advisable been validated in both the ESRD and cancer patient populations
to stop the opioid until the respiratory depression subsides. If as a reliable trigger to identify patients who are at increased risk
the patients respiratory rate is not compromised, the opioid can of death within 1 year and for whom palliative care consultation
usually be continued, and a benzodiazepine such as lorazepam including advance care planning is appropriate (16,17).
is added to control the myoclonus. Occasionally, a lorazepam Researchers have developed an evidence-based robust in-
continuous intravenous infusion at 1 or 2 mg/h is necessary to tegrated prognostic model with a C-statistic of 0.8 to estimate
control the myoclonus. dialysis patients 6- and 12-month survival (18), which is avail-
Although nonsteroidal anti-inammatory drugs are rec- able online at www.touchcalc.com/calculators/sq. The American
ommended for use in step 1 on the WHO analgesic ladder, the Society of Nephrology and Renal Physicians Association have
use of these drugs in patients with CKD is contraindicated recommended that advance care planning for CKD and ESRD
because of their nephrotoxicity and in dialysis patients because patients including a patient-specic estimate of prognosis and
of the increased risk of upper gastrointestinal bleeding. shared decision-making occur prior to the initiation of dialysis
The Mid-Atlantic Renal Coalition and the Coalition for (5,19). Nephrologists are responsible for advance care planning,
Supportive Care of Kidney Patients assembled a panel of although aspects of it can be delegated to other nephrology per-
international experts on pain management in CKD and de- sonnel (15). Advance care planning is important for kidney-
veloped an evidence-based algorithm for treating pain in dialysis cancer patients because it can ensure that patients wishes for
patients that is accessible online (11). end-of-life care are respected, that unwanted interventions are
avoided, and that patients and their families are satised with the
Other symptom management care provided (15). Although nephrologists are expected to pos-
Because of their comorbid illnesses, patients undergoing dialysis sess primary palliative care skills, they are encouraged to consult
are among the most symptomatic of any population with chronic palliative care physicians for more complex cases (20). Table 2
disease. In one study (13) of 162 patients undergoing dialysis presents a comprehensive approach to incorporating palliative
from three different dialysis units, the median number of symp- care into dialysis facility patient care.
toms reported by patients was 9.0. Pain, dyspnea, dry skin, and
fatigue were each reported by .50% of the patients. Of the 30
different symptoms reported by the patients, the 6 most both- CONCLUSIONS
ersome (starting with the most severe rst) were as follows: chest
pain, bone or joint pain, difculty becoming sexually aroused, There is a growing commitment among the leadership in the
trouble falling asleep, muscle cramps, and itching. nephrology and oncology communities to enhance palliative
Pruritus, or itching, is one of the most frustrating symptoms care for advanced kidney disease and cancer patients. It is
experienced by CKD and ESRD patients. Secondary hyper- highly likely that palliative care for these patients will be sig-
parathyroidism, increased calciumphosphate deposition in nificantly improved over the next decade, as nephrologists,
the skin, dry skin, inadequate dialysis, anemia, iron deciency, oncologists, and palliative care consultants apply the knowl-
chronic inammation, imbalance in endogenous opioids, edge and skills discussed in this chapter.

American Society of Nephrology Onco-Nephrology Curriculum 3


TAKE HOME POINTS 8. Davison SN. Pain in hemodialysis patients: Prevalence, cause, severity,
and management. Am J Kidney Dis 42: 12391247, 2003
9. Barakzoy AS, Moss AH. Efcacy of the world health organization anal-
c Early palliative care intervention is becoming the standard of care for
gesic ladder to treat pain in end-stage renal disease. J Am Soc Nephrol
kidney-cancer patients. It improves patients quality of life and respects
17: 31983203, 2006
their treatment wishes. 10. Bailie GR, Mason NA, Bragg-Gresham JL, Gillespie BW, Young EW.
c CKD, ESRD, and cancer patients have shortened life expectancy. The 5- Analgesic prescription patterns among hemodialysis patients in the
year survival rate for incident dialysis patients is 40%, which is 30% less DOPPS: Potential for underprescription. Kidney Int 65: 24192425,
2004
than that of incident cancer patients (66%).
11. Coalition M-AR. Clinical algorithm and preferred medications to treat pain
c Because they do not have active metabolites excreted by the kidneys,
in dialysis patients. Available at: http://www.kidneysupportivecare.org/
fentanyl and methadone are the safest drugs to use for severe noci- Files/PainBrochure9-09.aspx. Accessed March 25, 2015
ceptive pain in patients with advanced kidney disease. 12. Dean M. Opioids in renal failure and dialysis patients. J Pain Symptom
c Palliative care consultation can help with complex pain and symptom Manage 28: 497504, 2004
13. Weisbord SD, Fried LF, Arnold RM, Fine MJ, Levenson DJ, Peterson RA,
management and advance care planning, including shared decision-
Switzer GE. Prevalence, severity, and importance of physical and
making about the goals of care. Collaboration with hospices can help
emotional symptoms in chronic hemodialysis patients. J Am Soc
dialysis units implement a palliative care program and appropriately Nephrol 16: 24872494, 2005
refer patients for hospice care at the end of life. 14. Murtagh F, Weisbord S. Symptoms in renal disease: Their epidemiol-
ogy, assessment, and management. In: Supportive Care for the Renal
Patient, edited by Chambers EJ, Germain M, Brown E, 2nd Ed., Oxford,
UK, Oxford University Press, 2010, pp 103138
REFERENCES 15. Holley JL, Davison SN. Advance care planning for patients with ad-
vanced CKD: A need to move forward. Clin J Am Soc Nephrol 10: 344
1. United States Renal Data System. 2013 USRDS Annual Data Report: 346, 2015
Atlas of Chronic Kidney Disease and End-Stage Renal Disease in the 16. Moss AH, Ganjoo J, Sharma S, Gansor J, Senft S, Weaner B, Dalton C,
United States, Bethesda, MD, National Institutes of Health, National MacKay K, Pellegrino B, Anantharaman P, Schmidt R. Utility of the
Institutes of Diabetes and Digestive and Kidney Diseases, 2013 surprise question to identify dialysis patients with high mortality. Clin
2. Abdel-Kader K, Unruh ML, Weisbord SD. Symptom burden, de- J Am Soc Nephrol 3: 13791384, 2008
pression, and quality of life in chronic and end-stage kidney disease. 17. Moss AH, Lunney JR, Culp S, Auber M, Kurian S, Rogers J, Dower J,
Clin J Am Soc Nephrol 4: 10571064, 2009 Abraham J. Prognostic signicance of the surprise question in cancer
3. Mao JJ, Armstrong K, Bowman MA, Xie SX, Kadakia R, Farrar JT. patients. J Palliat Med 13: 837840, 2010
Symptom burden among cancer survivors: impact of age and co- 18. Cohen LM, Ruthazer R, Moss AH, Germain MJ. Predicting six-month
morbidity. J Am Board Fam Med 20: 434443, 2007 mortality for patients who are on maintenance hemodialysis. Clin J Am
4. Renal Physicians Association and American Society of Nephrology. Soc Nephrol 5: 7279, 2010
Shared Decision-Making in the Appropriate Initiation of and Withdrawal 19. Williams AW, Dwyer AC, Eddy AA, Fink JC, Jaber BL, Linas SL, Michael
from Dialysis, Washington, DC, Renal Physicians Association, 2000 B, OHare AM, Schaefer HM, Shaffer RN, Trachtman H, Weiner DE,
5. Renal Physicians Association. Shared Decision-Making in the Appropriate Falk AR; American Society of Nephrology Quality, and Patient
Initiation of and Withdrawal from Dialysis, 2nd ed., Rockville, MD: Renal Safety Task Force. Critical and honest conversations: The evidence
Physicians Association, 2010 behind the Choosing Wisely campaign recommendations by the
6. Smith TJ, Temin S, Alesi ER, Abernethy AP, Balboni TA, Basch EM, American Society of Nephrology. Clin J Am Soc Nephrol 7: 1664
Ferrell BR, Loscalzo M, Meier DE, Paice JA, Peppercorn JM, Somereld 1672, 2012
M, Stovall E, Von Roenn JH. American Society of Clinical Oncology 20. Quill TE, Abernethy AP. Generalist plus specialist palliative care:
provisional clinical opinion: The integration of palliative care into Creating a more sustainable model. N Engl J Med 368: 11731175, 2013
standard oncology care. J Clin Oncol 30: 880887, 2012 21. Davison SN, Jhangri GS, Johnson JA. Cross-sectional validity of a
7. Kimmel PL, Emont SL, Newmann JM, Danko H, Moss AH. ESRD patient modied Edmonton symptom assessment system in dialysis patients: A
quality of life: Symptoms, spiritual beliefs, psychosocial factors, and simple assessment of symptom burden. Kidney Int 69: 16211625,
ethnicity. Am J Kidney Dis 42: 713721, 2003 2006

4 Onco-Nephrology Curriculum American Society of Nephrology


REVIEW QUESTIONS Fentanyl does not have active metabolites excreted by the kid-
neys. Morphine and codeine are contraindicated in advanced
1. Which one of the following is an advantage of the physician kidney failure because of the toxic accumulation of active me-
orders for life-sustaining treatment (POLST) form com- tabolites in kidney failure. Acetaminophen is not appropriate
pared with an advance directive for a patient with stage 5 for severe pain.
kidney disease, terminal cancer, and loss of decision-
making capacity? Reference
a. It is legal in all fty states
b. It is an immediately actionable medical order Dean M. Opioids in renal failure and dialysis patients. J Pain
c. Checklist format prevents contradictory orders from being Symptom Manage 28: 497504, 2004
issued
d. It is appropriate for patients in all stages of CKD 3. Which one of the following statements best summarizes
the role of shared decision-making for patients with ad-
Answer: b is correct. The POLST form or variant is en- vanced kidney disease and cancer approaching the need for
dorsed in 22 states at present and being developed in another dialysis?
23. Contradictory orders could be written between Sections A a. Shared decision-making is an outmoded concept from the
and B such that the patient is to receive CPR in Section A and 1980s
comfort measures in Section B of the form. The form is only b. Shared decision-making ts well with a disease-oriented
appropriate for patients who are seriously ill and for whom the approach to CKD patient treatment
physician would not be surprised if the patient died in the next c. Shared decision-making for CKD patients defaults to di-
year. alysis modality choices
d. Shared decision-making is the recognized preferred model
Reference for medical decision-making

Citko J, Moss AH, Carley M, Tolle SW. The National POLST Answer: d is correct. Shared decision-making was intro-
Paradigm Initiative, 2nd ed. FAST FACTS AND CONCEPTS. duced in the 1980s as a process to promote informed consent
Available at: http://www.mypcnow.org/#!blank/k2dh9. Accessed and decisions that adequately take account of patients pref-
January 28, 2016. See www.polst.org for more information. erences. It ts well with an individualized, patient-centered
approach to decision-making and not a disease-oriented ap-
2. Which one of the following medications would be the proach. Shared decision-making for CKD patients encom-
preferred, recommended medication for a patient with passes decisions about whether to start or stop dialysis and
stage 5 CKD and lung cancer with painful metastases to the not just dialysis modality.
bone who describes his pain as aching and 10/10?
a. Morphine References
b. Acetaminophen
c. Codeine Barry MJ, Edgman-Levitan S. Shared decision making: Pin-
d. Fentanyl nacle of patient-centered care. N Engl J Med 366: 780781, 2012
Renal Physicians Association. Shared Decision-Making in the Ap-
Answer: d is correct. Fentanyl is appropriate for severe no- propriate Initiation of and Withdrawal from Dialysis, 2nd Ed.,
ciceptive pain in patients with advanced kidney failure. Rockville, MD, Renal Physicians Association, 2010

American Society of Nephrology Onco-Nephrology Curriculum 5

You might also like